Вы находитесь на странице: 1из 336

THE

THEORY

OF

THOUGHT

TREATISE

ON

DEDUCTIVE

LOGIC

Jt. -Davis
"

'O \iiv

ovv

Kara

TO

Trpay/m
Q.
"

Gewptiv
ARISTOTLE,

ra

KOIVU

NEW
HARPER "

YORK
PUBLISHERS
SQUARE

BROTHERS,
FRANKLIN

Entered

according

to

Act

of

Congress,

in

the year

1880,

by

HARPER

"

BROTHERS,

In

the

Office

of

the

Librarian

of

Congress,

at

Washington.

PREFACE.

LITTLE
in

preface
sense

is

needed. the
believe

The

treatise
within

is not the

elementary
grasp
of and mature imno

the

of

bringing
This
is made.

subject
to

minds.

be

impracticable,
in

such

profession
at

It is

elementary
the Its
in

the

sense

that have such the

it
no

begins previous
one

the

beginning,
of the

supposing subject.
will be

reader
extent

to

knowledge
masters

is

that nical tech-

who

its contents

possession of
with
its

details

of

the

science, acquainted
to

established interest of the have vanced adold for if ject, sub-

doctrines,

and

prepared
It

study

with
a

profit

and

treatises.

is, in general,
the
its modern

reproduction
student
may

Logic.

Whatever
he
must

scorn

antiquity,
he would
even

know

doctrine the
recent

respecting
literature for
of it is

Thought
of

read that

intelligently
in

the

sympathy
and

with the

him,

permeated
ancient

with

the

terminology

doctrines

the

gicians. lo-

The
of mind.

treatise
his
"

reverts
as

to

Aristotle, and

is

largely
through
has been
is

ment restate-

theory
his

colored

by

filtration

mediaeval taken able ed concludsumed conno

Since

day,"
also
and A up

says
to

Kant,
this
to

"Logic
it has

backward
take
no

step, and step

time
all

to

forward,

thus,

appearance,
ages
to

and

perfected."
it
nor

fiery

trial

for

has remain

neither

refined

it, and
the
sum

it is

likely
human aimed
at

perpetually
Hence,
and

an

accepted treating

part
the

of

of
I have than

knowledge.
at

in

old

Logic,
rather

clear, correct,
modification.

complete
late which
a

statement

any been

Of
some

years
are

many examined is

innovations
and

have

proposed,
in
as

of

criticised.

Whenever
indicated

the

treatise

new

view

offered,

it is

distinctly

such.

IV

PREFACE.

great
and

number

and

variety

of

examples,
the have

both

for

tion illustraof the

for
seemed

praxis,
to
me

mitigating
desirable.

somewhat

severity
been

subject,
from

They
some are

collected

every
are

available

source

ancient,

some

modern,

many

newly
used
at

invented.
with hand

have

great

freedom

standard

authors, Hamilton,
and
a

keeping
Mansel,
or more

constantly
Thomson,
school-book their
views

Arnauld, Mill,
and

Whately, Bain,
their

De

Morgan,
writers,
and

dozen
and

profited

by

research,
it seemed

adopted ous. advantage-

phraseology
references

whenever
to

Abundant

them,

together
be deemed with

with

this

eral genI
ter, mat-

acknowledgment,
have
not

will,
to

hope,
the

sufficient. recondite
accessible

sought
have
to

embellish

margin
to

but

added lead

many reader been in

references
into

other

works,

hoping
The there
it is

the has blunders

yet

wider
with be
too

fields.
much

treatise

prepared
it

pains.
to

That but

are

no

would
all its

much

hope,
its

sent

to

its

account

with is

imperfections
it

on

head.

If,
if

on

the
it

whole,
will

it

good

book,
the

will

live

and

be

useful

not,

die,

the

sooner

better.

K.
UNIVERSITY
OF

DAVIS.

VIRGIMA.

CONTENTS.

PART

FIRST."

INTRODUCTORY.
OF LOGIC.
Paga 1 2

I.

DEFINITION

" " " " " " "

1. 2. 3. 4. 5. 6.

Definition,
Science.
"

and

tb

word
an

Logic
Art of

Logic
the

not

Thought
Forms of

Object-matter

Logic

4 5

Thought
Forms.
"

Necessary Theory
Free of

Psychology

Distinguished

Thought Adopted

1
8

7.

Treatment

II.

PRIMARY

LAWS. 9 10
11

" " "

1. 2. 3.

Their The The The These

Origin
Law Law Law Laws
a

and

General

Character

of of of

Identity
Contradiction Excluded
are

"4. " " " "


5.

Middle and Co-ordinate The Absolute

13 14 15 16

Complementary
Criterion Sufficient of

6.

Only

Negative
of of

Reality.

"

7. The
8.

Principle
Postulate

Reason

The

Logic

17

PART

SECOND."
I. THE

OF

CONCEPTS.

TERM.
19 19

" " "


"

1.

General

Divisions Kinds and

of

Logic
of Marks

2.
3.

Abstraction." Generalization

Specialization
and General

20 22
24

4.
5. 6.

Conception,
Realization These Acts

Individual of

" " " "

Concepts
Each Other

Imply

25 25

7.
8.

Abstractions

Language.

"

Symbolic

Thinking

27

VI

CONTENTS.

II. QUALITY.
Page

" " " " " " "

1. The
2. 3.

Four

Views

to be

Taken of

of

Concepts

30 30 31 31 32 33 34

The

Leibnitzian and and

Analysis

Knowledge

Obscure

Clear Distinct

4. Confused
6. 6.

Inadequate
Intuitive and

and

Adequate

Symbolic

7. Perfect

Knowledge

III. QUANTITY.

" " " "

1. Intension 2. The
3. 4.

and

Extension

35 36

Law

of these of
an

Quantities
Abstraction

The The

Quantity

37
38

Coexistence

of the

Quantities

IV.

RELATION.
40 40 42 42

" " " " " " " "

1. In 2.

Intension,
"

Identical

and

Different Conflictive

Congruent, Incongruent, and


and Co-ordinate

3. Involved
4.

Relations

in Extension
"

6. Subordination.
6.

Genera

and

Species

43 45 45

Correspondencies
Terms Second Intentions

7. Correlative
8. First and

46

V.
.

DEFINITION.
48 49 50 50 52 54

" " " " " "

1. 2. 3.

The The

Intensive Scholastic

View View

Intersection

of

Concepts

4. Kinds 5. Rules
6.

of Definitions

Praxis

VI.

DIVISION.
50 50

" " " " " "

1. 2. 3. 4. 5. 6.

Definition Two Kinds

and

Division

Contrasted

of Wholes
"

Co-ordination. The

Dichotomy

57
58 59 60 62

Principleof Division Polytomy

Trichotomy and
Canon and Rules

$ 7. Praxis

CONTENTS.

Vll

VII.

COMPLETE

SYSTEM.
Paga 64 65
._

" " " " " " " " "

1. 2. 3.

Scheme Tree of

of the

Two

Quantities

Porphyry
Genus.
"

Sumraum

The

Categories
Individual

66 68 69

4. Infima
5. 6.

Species." The
of the and Series

Extent

Definitions

Divisions

Convertible

70 71 72 74

7. The
8. 9.

Logic Logic

of Common of Scientific Laws

Systems Systems

The The

Primary

Applied

PART

THIRD."
I. THE

OF

JUDGMENTS.

PROPOSITION.

" " " " " " " " " " " " " "

1.

Judgment

Defined.

"

Return

75

2. Parts
3. 4. 5. 6.

of the

Proposition." The

Subject

76 77 78 79

The The The

Copula
Strict

Logical Form

Predicates Intensive and Extensive

Judgments,

80 82 82 85

7. Categoricaland
8. 9.

Conditional

Total

and

Partial and of

Positive

Negative Quantity and Quality


and

10.
1 1. 12. 13.

Symbols

88

Simple,Complex, Propositions, Judgments, Analytic and Judgments


Praxis of

Compound

89 93 94

Synthetic

Degree

14.

100

II. INFERENCES.

" " " " " " "


"

1. Divisions.
2. 3. 4. 5. 6.

"

Immediate

Inference

102
103 104 104

Implied Judgment
A

Discriminated.

General and

Rule Passive and

Active Added

Determinants

Complex Conceptions

104 105 106 108

Infinitation

7. Conversion
8.

Opposition
Praxis..,
.

89.

113

Yin

CONTENTS.

III. INNOVATIONS.
Page

" " " " " "

1. 2.

Many
The

Proposed
Semi-definite "Some" Predicate.
"

115 115

3. Quantification of the 4. The


5.

Table

116

New

Forms,
to be

their

Occurrence

118
120 123

Proved

Compounded
in Character

6. Mathematical

PART

FOURTH."
I. THE

OF

REASONINGS.

SYLLOGISM.
125
'

" " " " "

1. Its Definition 2. 3.

Its

Parts,and

their Order
,

130 133

Its Various Canon

Kinds

4. The
5.

137
139

General

Rules

II. FIGURE

AND

MOOD.
1-14

" " " " " " " " "

1. 2. 3. 4. 5. 6.

Conspectus Reasonings
The Names Moods

of

Figure
Second and Third

in the Evolved

Figures

145 148 148 150 154

of the Moods

Reduction

Equivalent Moods
Fourth

7. The
8.

Figure.
.

-.

156 158 165

The

Syllogism Vindicated

9. Praxis

III. QUANT1TATIVES.

" " " " " " "

1. 2. 3. 4. 5. 6.

of Equivalence Syllogisms Mathematical Reduction


to

170
173

Demonstration

Qualitatives

175
176

Syllogisms of Comparison
Hamilton's The Causal

Unfigured Syllogism Syllogism

178 179
180

7. Praxis

IV.

COMPOUND

AND

DISGUISED

FORMS.
183 186

" "

1.
2.

The Enthymeme Irregularities.


"

The

Epichirema

CONTENTS.

IX

Pag.

" " " " " "

3. 4. 5. 6.

The

Sorites of

187

Resolution

Arguments
Premises

180 194 190 398 200

Syllogisms having Compound

Syllogisms having IrregularPremises


of

7. Modes
8. Praxis

Arguing

Named

V.

CONDITIONALS.
200 .' 200

"

1. Divisions 2. 3. 4. 5. G.

" " " " " " " "

Conjunctives Disjunctives

207
209 211 213 215

Conjunctive-disjunctives
Conjunctive Syllogisms DisjunctiveSyllogisms Syllogisms Conjunctive-disjunctive
The Dilemma

7.
8. 9.

217 220

Praxis

VI.

ANALYSIS Order

OF

CONDITIONALS.
225 226

" " " " " " " " " "

1. The 2. Real
3. 4. 5. 0.

Question,and
and Ideal

of Discussion

Thought
of

First Second

PrepositionalUse PrepositionalUse
Founded
on

Contingent Hypothetical

229 232

Reasonings

these

Uses Use

235

Reasonings Implied
Unreal

in the

Second

237
242

7. The
8. 9.

Hypothetical
not

Conjunctive Syllogisms are


Other Conditional

Inferences
not

245 249

Syllogisms are

Inferences

10.

Summary

of Doctrine

250

PART

FIFTH."

OF

FALLACIES.

I. DISTRIBUTION.

" "
"

1. 2. 3.

Treatment Bacon's

of Fallacies Idols

Justified

253 254
..................................................

Mill's Classification

"
"

4.

Whately's

Classification Classification

255
............................................. .......................................
......

5. Aristotle's 0.

256
258

"

Paralogisms

.........................................................

II. SOPHISMS

IN

DICTION.
261

" "

1. Their
2. Of

Common

Fault

................

=.

...............................

Equivocation

.......................

.............................

261

CONTENTS.

Page

" " " "

3.

Of

Amphiboly. Composition
Accent of and Division

266

4.

Of

267

5.

Of

268

6.

Of

Figure

Speech.

270

III.

SOPHISMS

IX

MATTER.

" " " " " " "

1.

Of

Accident

272 and the Limited

2.

Of

Absolute

Terms

273 276
280

3.

Of

Ignoring
Antecedent

Refutation

4.

Of

and

Consequent

5.

Of

Begging
False

the

Question

282

6.

Of

Cause

290

7.

Of

Many

Questions

294

IV.

EXAMPLES.

" " " " " " " "

1.

Inexplicable
The Achilles

Fallacies

290

2.

296 Cronus

3.

The

Diodorus

298

4.

The

Litigiosus
Mentions

299

5.

The

300

6.

The

Sorites

'

301

7.
8.

The

Ignava

Ratio

302

Praxis

305

LOGIC,
OR

THE

THEORY

OF

THOUGHT.

PAET

FIRST."

INTRODUCTORY.

I. DEFINITION

OF

LOGIC.
forms of

thought. The word Logic is derived from the Greek Xoyto/,an adjective stood. of study)underor Trjoay^m'a (matter qualifying "7rtorr//t^ (science) "
1.

Logic is
"

the science
"

of the

necessary

The The

meaning
latter is

of

Xoyto/ and

of its

ous. original, Xoyoc, is ambigu-

to both the ratio and the oratio of the equivalent rivative, into the deto thoughtand to speech. This ambiguity passed Latins, and has affected the views of many to the objectas logicians matter

of the

some science,

holding that

it treats of words

or

language

rather than

of

thought.1
not

Aristotle did
doctrine he

first

by the term Xoyuv/ the science whose designate Analytic, Apodeictic, fullydeveloped. The terms
3.

See

Hamilton's

Logic, p.

It may

be well to note

at the outset

that

logicians

are

divided be the

into three

to

subject of
and the

Phenomenal,

or conceptions schools, according as they hold words,things, the Verbal, the respectively, Logic ; and these are entitled, The first is representedby many Conceptional Schools.

scholastics, by Hobbes, Whately, and Helvetius, Comte, J. S. Mill,and Bain


of the third is

De

Morgan.

The

second

numbers At

Bacon,
the head

among

its chief the

expositors.
recent

Kant,

followed and

by Krug, Esser, and


Mansel French with

German and

logicians English pupils


The

and generally,
; to whom

by
may the

Hamilton be added

their train of Scotch

most

Arnauld. writers,following of

present

treatise takes
as

in

view. thought. But Logic treats Kantian, or conceptualist, and is expressed in words,Logic cannot proceed thoughtis always about things, See entire disregardof these, but should constantlykeep them subordinate.

Cretiens's

Method, ch. Logical

v.

Oxford, 1848.
1

INTRODUCTORY.

and including to Dialectic, latter equivalent Sophistic) Topic (the of Logic. names were by which he denoted partsor applications special the whole science. Plato used to designate He used no term one than Logic proper includes, while the term Dialectic to denote more and it is usually Aristotle used it to denote less, regardedas the most the designation for Logic. With whom ancient synonym nated Logicorigiand does not appear ; but it is ancient, being used by Cicero, is attributed by Boethius to the early Peripatetics. and

in pointof having, complementof cognitions, acter form, the character of logical perfection ; in pointof matter, the charof a branch' of knowledge of real truth."3 The logical perfection and exhibiting its objectis attained by systematically arranging and in harmonious connection. matter, clearly, distinctly, completely, classification. Again,the object-matter of a science must This implies

"

2.

"

Science is

otherwise be real truth,


or

it cannot

be said to be known of
a

; what

is unreal

false cannot be

be

constituent

science.8 Hence
:

the definition
a

may

abbreviated conveniently

thus

Science

is

perfected tem sys-

of real branches of

truths; or
have far

thus:

Science

is classified knowledge. Few

knowledge have

reached
so.

this ideal But since

mathematics,none

done

perfection ; if not the in many departments

knowledge has
"

and made great outgone its crude beginnings, called sciences. branches s uch towards this are ideal, properly progress that science is a body The distinction between science and art is, of
a

and deductions to explain some object-matter ; an principles of for the completion with practical skill, body of precepts, A that science teaches
us

art is
some

work.

to

know,

an

art to do ; the former


causes

clares de-

with somethingexists,

the laws and

which be

its

the latter teaches how existence;


ut sciamus

somethingmust
ut

to belong produced."4

In science scimus

; in art scimus

producamus.

Science

discovers laws ; art

This /i"j.6 sciences pure speculative leads


so

ov ITTHTTI]TO givesrules. Uepiyeveaiv Ti\rr), TTEJOI distinction holds good, in reference to the extremes, as to

and

mere

manual

arts.

But

science often

pendent into art,and art, except in its rudest forms, is so dedirectly that usually on science, they cannot be set clearly apart.

Logic, p. 335. of axiomatic Scientific knowledge (TO iiritsTavQai), principles except when of and of the conviction the truth edge a given proposition, a knowlrequires (VOEIV), of its reason Aristotle's Anal. Post, i, or cause. 2, 1.
3
"

Hamilton's

Thomson's

Outline

" of the Laws of Thought,

6.

Aristotle.

DEFINITION

OF

LOGIC.

Economies, sciences, Moreover,there is a body of practical e.g. Ethics,


that occupy etc., others
e. again,

intermediate g.

called arts; ground,and yet are never Rhetoric, Grammar, etc.,are commonly viewed as

arts.8
have viewed Logicas an art,and entitled it The art logicians of thinking (Aldrich) (Arnauld 7) ; The right ; The art of reasoning it to be both, thus : Others pronounce of reason etc. use (Watts), Ars artium, et scientia scientiarum (Duns Scotus,13th century);* art of thinking, The art and science of reasoning (Whately) ; The Some which
means,

of

correct

and thinking,

the science view of

of

the conditions
art

of correct

The thinking(Mill0).
us

extreme

is that it teaches
course course

how

to
as

think. needful them view

in

Logic is
see.

about

Logic as an absurd. This is evidently thinkers for making men


a course

A
as a

in Ethics them

is to make A modified

or virtuous,

in

to Optics

make think how and

is that

how to avoid correctly, or, negatively, to test the validity of given arguments. be set aside
as

Logic teaches us how to that it teaches or fallacy,


If such is the
no

scope

of Logic, it may object of


a

of littleor
never use

sisting convalue,

system of rules which the initiated


miss. Such views have

and

the

initiated un-

never

as was just Chemistry great discredit, of the alchemists.10 pretensions extravagant

historically brought Logicinto by the broughtinto disrepute

It is art. nor an even primarily, secondarily, how we do think and how we a science ; the science teaching strictly think if we think correctly. It is the theoryof reasoning must ; or, it is the theoryof thought. The difference between better, Logic and an Art of Thinkingis similar to that between Anatomy and Surgery. The value of Logic is such as belongs to pure science, which, in this day, needs no demonstration. It is somethingof profoundcst
not

But

Logic is

interest to know of and thinking,

what

are

the mental
matter

of such

act processes in the intellectual Is it the liberal mind asks primarily,

true?

not, Is it useful?

Knowledge 'is power,

but

we

have

to

do

fi

See Hamilton's
I}Art de

Sfefapkysic*, pp. 81-84.


that
most

"

Penser,1662, Port-Royal Logic."


8

admirable

work, known

commonly

as

the

See Hamilton's See Locke's

Logic, p.

26.

Ex. of Hamilton's

Phil. vol. ii, p. 149.

iv,ch. xvii. Also opinion " in Faust,pt. i,speech of Mephistopheles der Schuler." It may be to Goethe's, that this is merely the mocking gibe of Mephistopheles; but cf. in Wahr. objected und Dicht. pt. i, bk. iv.
contemptuous
of

10

Logic,Essay, bk.

INTRODUCTORY.

with

it here the

as

not knowledge,

mastered

there is a science, of his reasoning powers ; and, moreover, he


can

Where, power. result in a practical


as

however, one

has

cultivation special

whatever

it is manifest understands,

more

process one clearly and efficiently clearly form.11 per-

The

Greek

subdivided
utens.

and after these the scholastic Aristotelians, Aristotelians, Logic into what the latter called Logica docens and Logica former is
as explained an

The

abstract

guce

tradit proecepta ; the latter as a concrete of these rules to use ing quce utitur prceceptis.Hamilton, follow"

theoryof thought as an tion applicapractice,


"

the latter Logic," Concrete Logic." The former onlyis Logic; the latter, or Special called "Applied Logic,"and treating of the quiteproperly chiefly sciences should be logically methods by which particular developed, the science of of is no whatever of cordingly Logic, Logic proper, and acpart
or
"

Kant, calls the former

"

General

Abstract

will be

in disregarded

the

present treatise.13
Thus
own

"

3. The

of Logic is thought. object-matter other each of which sciences,


treats of the stars ; has

guished it is distin-

from matter.

its

special object-

Astronomy

of geology,

the earth's crust ;

of its flora ; mathematics,of quantity of its fauna ; botany, ; zoology, of principles of mind ; of God ; philosophy, ; psychology, theology, of morals,etc. ; so Logic treats of thought. Thought denotes ethics, from perception, as distinguished only the acts of the understanding, of whose exercises Logic takes no account. volition, desire, feeling, memory, of one tion noThought may be simplydefined as the cognition in
or

or

under

another.
a

Hence

in this act

understand
of

be

conscious it is.
"

the is

thing. E. g., A book the thing makes impression


mere

said to hend comprelies before me. I may


we are

without

cognizing
it

what and

This
a

say,

It is

book," I
we

perception.But have brought it


call
"

if I cognizewhat under This


a

is,
or

certain class

Now thought.13 think about all conceivable things, but all of these are to Logic we indifferent except one, that is, thought itself. In Logic we perfectly think about thought. What Logic evolves.14 thoughtinvolves,

concept of thingswhich

book."

is

11 "

See Hamilton's Hamilton's See

Logic, pp. 7,8

and

McCosh's

Logic, " 80. pt. iii,


13

Logic, p. 38, and

p. 42.

Id. pp.

9, 10.

Elench. ix. Sciences and demonstrations, De Soph. says he,are Aristotle, The dialectician and would infinite, requireomniscience to treat them. possibly of all has to discover only the principles to common spheres thought.

"

DEFINITION

OF

LOGIC.

"
with

4. W^e

then,that Logic does observe,

not

at

all

concern

itself

what

thoughtconsiders. things

It treats of

thoughtregardless

It is usual to express this by sayingthat Logictreats of its content. its matter. The i. e. excluding of the forms of thought abstractly, form thus: of from thoughtas distinguished I think
"

its matter
me

When

that the book of

book
"

before

be exemplified may is a folio, the matter


"

of this

thought is
The forms

into which
or as as a

very various

the form of it, is a folio," ;" and ment." judgas thought may be represented empty shells, of thought; the enter content matter may as different substances may

mere

to which outlines,

statue

marble.

of whether the same be formally may So the science Morphology treats of the forms

conform, like wood, metal, or


of

abstract and Crystalology, an animals, of the forms


no

and plants treats only science, geometrical and the form

of minerals. No
no

The

matter

have,of
under

course,
some

separateexistence.
of

form

thought;
we

and

consciousness

unless

some
can

cogitable except have any form of thoughtcan be thought under it. object
these of

objectis

existence in But

by

lytic ana-

abstraction the

consider the
manner

apart;

we

can

consider
can

either

or thought, object

the matter

from

the form

of

thinkingit ; we thought. Now it is the

distinguish
of

form

thought,

its matter, that Logic considers. arc logicians abstracting stood is extralogical. This might be underfond of sayingthat all matter ter to represent Logic as a science without a content, without mathas its own like every other science, But Logic, of its own. special is extralogical. Its object-matter is thought; all other matter content. Its form of is thoughtdischarged object-matter of its matter
; i. e., it is the

Modern

thought. an Logic,then,is properly

abstract

one science,

from abstracting
to all ;

each and allthe i. e. the formal its

and considering sciences, onlywhat is common

thought to which all are Hence Logic is in a object-matter.


fundamental is therefore the
to

all sciences, and of

all.

and making that subjected, relation to similar and equal is the science Now philosophy science in the
sense

and principles,

the fundamental

that its

is object-matter

primary truths

that underlie

edge. all knowl-

not at all. Hence or Logic logically proceeds philosophy exhibits the that it is fundamental in the sense to philosophy even well as every as necessary processes of thoughtwhich bind philosophy to proceedlogically, other science. Moreover, Logic is itself bound

But

and

can

become

science

only by conforming to
and explicate exhibit.

those

processes

which

it is its province to

INTRODUCTORY.

supposedthat Logic treats of thoughtonly in scientific pursuits.It treats of thought as exercised and displayed Thought as exhibited in all kinds of literature and universally. in silent meditation ; all our common in common conversation, speech, about the most trivial things and at every instant, thinking, every-day is is formally all of the same nature, proceedsin the same manner, if correct. trations illuslaws,is logical by the same Consequently, governed of Logicare to be drawn not merelyfrom any of the principles wherein but from any kind of knowledge, of the sciences, anything of thought. Logic teaches or explains becomes whatever an object mind rightly thinks at any time about anything. how any human

Let it not, however, be

"
not

5. To

define

Logic as

the science of the forms


not

of

thoughtwould

clearly is inter alia a science of formal thought, its character. Psychology from Logic. Psychology is an empirand needs to be distinguished ical is evolved from It therefore is tive, inducscience ; it an experience. It systematizes the conscious natural science, a posteriori. one In dealing mental activities, and pointsout their laws. with the faculty the modes in which we of thought, it explains think,teaching
how
we

be sufficient to set it entirely apart,would

discriminate

do

think,and refers for the


the of every individual. other hand, if taken
a

test of its doctrine

ive to the reflect-

consciousness

Logic,on
all
an

in its strictest sense, is not

at

It accepts science. or theoretical, speculative obtains by the analysis of given products of from or Psychology, certain primary laws ; from these it deduces secondary laws thought, and thus proceeds the necessary processes to demonstrate of thought, follow in thinkingcorrectly. It is therethose we fore of thought, must but empirical,
a

deductive science, one purely


a

do

think,as
but to

matter

priori. It teaches but how of fact, must we think,as


a

not
a

how

we

matter

of

be consequent. if the thinking necessity,

It

not appeals,

to consciousness,

in support of its truthfulness. demonstration, of thought; Logicis the then,is the natural history Psychology, considers thoughtas an theoryof thought. Psychology operation ; of Logic considers it as a product. Psychologytreats conceiving, judgments, reasonings. reasoning judging, ; Logic treats of concepts, it must be. as treats of thought as it is ; Logic of thought Psychology Psychologyteaches how we do think,Logic teaches how we must of thought merely as actual, think. the The one treats the forms other proves them necessary. Like

demathematics,Logicis purely

INTRODUCTORY.

Theory
reduced.

denotes It is
into
a

the

most

general
of the
a

laws

to

winch

certain drawn from

facts

can

be
and facts the

collection

inferences

facts
of of

compressed demonstrably
laws their the that

principles Logic

; it is

systematized
a

explanation
collection

true.

is and

such it

systematized
occupied
It with

govern

thinking,
certain

is

demonstrating

validity from Theory


of

axioms.

is, therefore, properly called

Thought.

"
pure of

Y.

It is evident would be

that very
no

work abstract

strictlylimited
and difficult. since
a

by

the

definition
a

of

Logic

Being
pure

discussion form
can

forms,
be

it could realized

offer in Even

examples

; for

abstract much

cannot

consciousness in
are

apart from

matter,

less

it the

be

expressed.

general expressions by algebraic symbols,


a

symbols
Again,

themselves treatise

species

of

matter

that from It

is

extralogical. Psychology,
it

if the of the
no

be

kept strictlyapart
to

will

admit of its how

reference mind does

actual

thinking.
in

will

tell
to act

us

ing noth-

actually proceed
of the
nature

its efforts

tize systemaas

knowledge,
the

nothing

of the the shorn

thinking
of

giving

rise

to

logical product, nothing


Thus
our

of be have the

phenomena
of

illegitimate

thinking.

science
if any,

would writers In

its rays. themselves


to
we

Consequently, few,
bound pure, of shall

allowed

be

idly rig-

by

the

definition. its

present

treatise, while

make ory Thewe

abstract

Logic
and

basis, while
this

developing systematically the


prime
of the

Thought,

keeping
the

object constantly
definition how the whenever mind

in

view,
it
seems

freely transgress
We it does and
to
as

limits
not

desirable. but and how

shall think.

consider We actual

merely

must

think,

shall

give copious
of

concrete

illustrations,
to

analyze
and

exhibit the

exercises of

thought, appealing
to

servation obthe filment ful-

experience

consciousness
to

corroborate observe the

theory, just
of

the laws

astronomer

turns

the

stars

to

the

of

the

Mecanique

Celeste.

PRIMARY

LAWS.

II. PRIMARY

LAWS. find that by subjective analysis

"

1. In the
are

of Psychology we study of

is necessitated the mind to which intelligence constitution. Among by virtue of its essential and original necessitated by the others are certain forms of thought determined or itself. The chief of these necessary of the thinkingsubject nature forms the judgment,the reasoning. By sayingthat are, the concept, think that the mind cannot these forms are necessary is meant truly

there

certain modes

except
must

in

them.

But

since

be

both universal,

in the
sense

human

mind, and in the


on some

necessary, they in every found sense they are that all the thoughts of each mind are that it cannot others. be that
a

they are

native

and

always determined

in them.

For
on

form

is be

sary neces-

occasions and not

If so, it would of

merely
the

which contradicts our notion contingent, forms being necessary and universal, we may

necessity.Now,
them
as

view

governed
general
a

by

These necessary laws. abstract principles common will analysis, enunciated, they are
or

laws will be
to the

an

of expression

the

forms,and, as the result of


axiomatic. When the

plete com-

be

ultimate
as

and

evolved

and

known of

as or LogicalPrinciples,

Primary

Fundamental

Laws

to pure Again,if, preliminary

embodied as objectively they are found to

Thought.1 productsof thought viewed Logic, in language ysis, to a critical analare subjected
or general

exhibit

universal forms. of

In

other

words, if from
literature
we

the various abstract the


a

manifestations
matter
common

and

thought in speechand all differences characterizing


a all, mode, a

them, we

discover

residuum

to

manner,

ing havThese

certain forms

that

all. that interpenetrate belongto all,

considered as governedby laws ; and these are forms,beinguniversal, those obtained by found to be the same as laws,when enunciated, are Thus subjective analysis. the two processes
are

rative. mutually corrobo-

complement of laws is assumed by Logic as its punctum from them as and it proceedsto demonstrate saliens, synthetically
This
1

For the

of history

these

laws,see

Hamilton's

Logic, pp.

62-68.

10

INTRODUCTORY.

axioms the

the judgment, laws of the concept, secondaryor special The whole of pure Logic is onlyan articulate development reasoning. in which they of these Primary Laws, and of the various modes the necessary, is to say that the ble, It is not that they are inviola-

are

applied.
To
say that these self-evident laws
are

of contradictory
not

each is inconceivable.
is constrained

to obey them, as a necessity constrained to obey the laws of gravitation, inertia, planetis blindly or sciously unconetc. They are violable in the sense that we may wilfully or, inconsequence; them; but the result is fallacy, disregard null and void. the mental process is then suicidal or absolutely rather, be legitimate All consequent thinking must forms con; i. e., it necessarily to these laws,advertently or inadvertently. They are the primary

that the mind

of

conditions The obvious been

of the
not
mere

of valid thought.2 possibility be offended


to

reader must
as

find these axiomatic

laws

so

to

seem

truisms.

When

stated, they appear

to have

perienced always known, beingimpliedin every thoughtwe have ever exor observed, though until stated we are as unconscious of them of the laws that govern our as we breathing.Being the are widest generalities, and, indeed,governing penetrating every science, that comprehends anything, theyseem of all every mental movement that thingsthe most familiar and trite. Their very truth requires Standingrelated to the axioms of geomthey contain nothingnew. etry first elaborate at related these are to as they propositions, appear frivolous. But if and even singularly meagre, barren of significance, the code by which all human these laws are really thought is actually then their study is not futile ; so far from being barren, regulated, of principles productive ; so far from they are the most wonderfully

theyhave beingfrivolous, "


2. The

the

significance. profoundest

three. The firstis the Law are Primary Laws of thought affirmation. It is variously of all logical of Identity.It is the principle Omne est ens ;3 Everyenunciated: e. g., Whatever ens or is, is, thing is equalto itself;* Every objectof thoughtis conceived as which agree can be united itself; A thingis what it is ; Conceptions
6 6

in

or thought,

affirmed of the
;
or

same

at subject

the

same

time.7

The

formula
3

is A=A

A=a'+a"
s

+ a'"
form.
4

Scholastic Logic, p. 56. Hansel's ProlegomenaLogica, p. 167. 'Thomson's " 114. Outline, See Hamilton's

Hamilton's Bain's

Logic, p. 16. Logic, p.

57

PRIMARY

LAWS.

11

"2 + 2 4;" "4=2x2;" examples: "4 The Idea is equal 3 + 1," etc.; "Accordingto Plato, to 2X2;" "4 itself;" Man's a man for a' that ;" is nitrate of potash;" Saltpetre
are following
= = = " " "

The

Francis Bacon of inductive

was

Baron

Verulam
"

;"

"

Francis

Bacon

was

the father

;" philosophy ;"


"

Man

is rational and

animal
"

;"
A

"

Man habit

is the last creation


is
a

Man

is the

;" onlybeing that laughs

habit

;"

"

What

I have

I have written." written,

Hamilton
or

extends this law to include the relation of in which


a

partial identity

sameness

concept stands
formula.

to
"

each of its

in expressed
notion
"

the second
"

Man
;
"

is E. g., Man " rational " comprehends the notion


"

as constituents, i. e. my rational," as one

of its constituents
"

Man similarly,

is animal."

We

may

go

further,

to the
poreal, cor-

part of
the

part. E. g., The


"

notion animal Man


is
a

the comprehends

notion

and

we

may

say,

corporeal."In
part of what
to be

this extension
in implicated

of

is only law, the predicate

is

the

subject.
To affirm that
a

thing is itself seems


Nulla

solemn

and trifling,

is

ridiculed

propoxitioest verier ilia in qua idem de seipso.6 ject When, however, we consider that every obprccdicatur
of

by

Locke.

thoughthas

definite characteristics all others


as

and

from distinguished

by which it is marked off it beingitself and nothingelse,

is evident that every concept may be viewed in relation to these characteristic and that these two several aspectsmust be affirmed of each other. and The law then declares the
as

of thinking the concept necessity


same.

its constituent characters

the

better

of expression

it

This perhapsbe : A notion and its constituents are the same. is a more of the axiom : A whole is equalto the general expression of its parts. In the predicate, the whole is contained explicitly sum which in the subject is contained implicitly. It is obvious that this law enjoins it is self-consistency ; or, rather, in the necessity for self. in thought that is formulated -consistency be the aspects this law. Whatever of a thing, whatever be the modes the thoughtunof statement derlying it, concerning they must be equivalent; each
must

would

be the

same.

"

3. The of all

second

is the

Law

of Contradiction.
are:

It is the The
same

ple princisub-

logical negation.
at

Enunciations affirmed and

attribute
same

cannot

be

the

same

time

denied

of the

Boethius.

See Hamilton's

Logic, p.

507.

12

INTRODUCTORY.

No subject can ject;9

that contradicts it;10\Vhat is predicate No object is unthinkable ; be thoughtunder concan tradictory contradictory attributes ; The same thingcannot be A and non-A : this the absence cannot be both hot and cold.13 As this law enjoins room it ought condition of thought, of contradiction as the indispensable
a
" ia

have

rather to be called the Law A is not A=0


; or, A
are

of Non-contradiction.14 non-A. is

The

formula

is:

is not
"

Exampleswhich, if taken
infancyin
"

ally, liter-

violate the law hand is most


"

Dotage
"

dexterous
;
"

;"

The

blind see, the

old age ;" His left deaf hear,the dumb


was

etc. speak,"

However

the choice, he unwilling risen

to compelled
"

volunteer

;"

Since

the war, all values have

;"

Two
"

kinds

of

individuals prepare in this life is true


want

fops and fools ;" Nothing extempore speeches, of the elements ;" We ;" The decomposition of the foreand but littleof that." Each going nothingbut silence,
" "

a examplesis a logical paradox,

self-contradiction ; each violates

the

law, and
a

is

felo de

se.

By

fundamental

law of

mind, which

Bain

calls the Law

of Relativity,

oppositeor counter every notion has an the other be conceived. virtue of the one can
there is

and only by notion, To line straight line ; to good is


a

the

crooked or line, opposed the not-straight and a knowledge of good is impossible to opposedevil, evil. Hence Now the old scholastic maxim
:

mind

not

knowing

Contrariorum

eadem

est scientia.

and of

cannot their union is contradiction, consist, opposites formulated in the Law as consistency, thorough-going
we

these

forbids it. Thus, when Contradiction,


must
not

affirm that

this is

we line, straight

we our

think

an

act

good,we

also say that it is a crooked line ; when may not also think it evil. Our assertions,

must not contradict each other. If to be consistent, thoughts, it destroys itself. Having made an sertion, asthey do, the thought is null, Affirmations not self-consistent are to abide by that. are we

unintelligible. of But the principle


affirmation

contradiction

carries

us

one

step further.

An

tradictory, beingmade, it not merely forbids us to affirm also its conbut it authorizes us, or requires the contradictory us, to pronounce its contradicfalse ; i.e., to deny,of an object of thought,

who Aristotle,

says this is by nature

the

of principle

all other axioms.

"

Metapli.

10

Kant's

Critique of Pare
Logic, p.
58.

Reason.
"

11

Hamilton's Bain's

13

Logic, p. 16.

transl. p. 115. Meiklejohn's Hansel's Prolegomena Logica, p. 167. followed 18 Krug's Logik, " by Hamilton. ; See

PRIMARY

LAWS.

13

the principle be enunciated thus : Of two tory. Accordingly, may be false. E. g., This straight must line is not one contradictories, crooked ;" This good act is not evil ;" No chastisement is joyous ;"
" " " "

Francis

Bacon

was

not

Roger Bacon;"
are

"A

dishonest
to

man

is not
some

If trustworthy."
or

all diamonds
are

then precious,

say that
is

is false. precious be to a notion must opposite, contradictory, any


not

diamonds

Whatever denied
are

repugnant,
Neither

of it.

The
can

Laws

of

and Identity
as a

Contradiction the other


as

co-ordinate.

be deduced

the evolved

first. In every such attempt is unavoidably which is pesecondary presupposed, second from
two

The titio principii.1* eminent

have, however, been identified by many ilton as Leibnitz, Wolf, Kant, Herbart. And Hamphilosophers,

" says, The two laws form." and negative

in the statement

All

essentially onlyby a posone, differing itive the be two summed Perhaps fairly may be self-consistent. must thought
are
J

"

4. The

third is the Law

of Excluded

Middle.

Its logical cance signifithe laws,

is that it limits the thinkable determines


or

in relation to affirmation ; for it the first two No the middle other


must
one

that of the two

forms
as

givenin
or

the

other must

be affirmed

necessary.

no ground,

third Hence
ria /

affirmation, one being possible,


the
names :

be

true.

Lex

exclusi medii

aut

tertiiinter duo

contradicto:

Wre Principiumcontradictionis affirmativum.


one contradictories,

enunciate it thus
a

Of
must two

two

must

be true. is

Either
no

given judgment
course.17 Of
formula be

be true,or

its

: there contradictory

middle

one contradictories,

must

exist in every
one

The subject.18

is: X

is either A

or

non-A;

the being snblated,


"

other must

posited.
A few will examples suffice: Either it is true
"

that God
a

or exists,

it is true that he does not his acts


"

exist ;"
"

Man
not

must to

be

free

agent,unless

are

necessitated

;"

To

be

or

;" be,that is the question

Infinite mercy offers salvation to all." In this last example the opposition " in denied nite," infiis bounds no-bounds bounds and is between ; and hence no-bounds
must

be

which affirmed,

is done ad

in

"

offers is
an

salvation to all." of application hence

The

argument called Eeductio


Of
two

absurdum
one

this law.

alternatives it shows
one

to

be absurd,

the other must

be true ; for

being false, proposition

15 17

Shown

in Hoffbauer's

Logik, %

23.

16 "

Thomson's

Outline, " 114.

Logic, p. 59. Hansel's App. to Aldrich, p. 241.

14

INTRODUCTORY.

we

are

authorized

or

by required
and

this law to pronounce Excluded Middle

its contradictory

true.

The

Laws united

of Contradiction in
one

may

be

iently convenname

statement, to which
is the
of principle

might
strict
:

be

given the

Duality."It We disjunction.18 may


Law of
one

"

division logical
two

and

enunciate

it thus

Of

contradictories,
firmed either af-

must
or

be true, the other false ; of every false.30 This compound of Excluded denied

Every predicate may be Every assertion must subject;


form is often mistaken So Goclenius: He Middle.

be

either

true

or

by logical
Oportetde
the

writers for the Law omni Law


re

aut affirmare

negare*1 Hamilton
:

also.

givesfor

firm afwe can attributions, contradictory the other affirmed, onlyone of a thing; and if one be explicitly is denied." is implicitly This is the compound ; the latter member the principle of contradiction. His subsequentexposition, however, is correct. Bain clearly makes the mistake.23 So also Herbert Spencer. Middle Middle is : The appearance of Excluded says the principle without excluding of any positive mode of consciousness cannot occur mode cannot mode ; and the negative a correlative negative occur He without the excluding correlative

of Excluded

Of

mode.2* positive

and Excluded Middle arc Contradiction, " 5. The Laws of Identity, complementary. "The objectwhich I conceive is by the mutually Law and by the Law of Identity discerned as beingthat which it is, But it is not. of Contradiction is distinguished from that which these two them made correlatives must also be between regardedas constituting the universe of all that is conceivable ; for the distinction above is not between
two

definite

of thought, but objects

between

the

of object

which

I think and all those of which

I do not think.

Non-A

the implies

exclusion of A

and only,

of

end nothingelse,

thus denotes

19

The Mill

of the diaiperiRov 'A^ititfia

Greeks.
Tie says Logic, p. 205. the unmeaning : possibility,
"

20

the absolute truth of this axiom. questions


the
true

that
e.

between
"

and
a

the false there is

third

g.,
meaning un-

Acracadabra

is

second

is neither intention,"

true

nor

false.

But

is

an

If unmeanis a vacuum. ing, proposition any it means then,is nothing ; or, nothing, says nothing. The third possibility, Examination ilton, Hamalso See the true and the false. there is nothing between of

assertion at all ?

Its content

ch. xxi.
21

Lex.

P/iilosoph. p. 136.

"
34

Logic, p.

59.
5.

83

Logic, p. 17.

No. Fortnightly Review,

16

INTRODUCTORY.

i. e., possible in thought; and this is logically possible, in fact. Pure mathematics true of many thingsthat are impossible with mere That the stars may deals exclusively logical possibilities. the earth is physically fall on impossible ; that revenge may be a But both are conceivable ; they may be duty is ethically impossible. in thought; they are logically represented possible.I may think but this conceivability Waterloo is a failure, a fiction, or Christianity evidence that they are so. no While, then,these laws are the highest of an object, criterion of the non-reality they are no criterion at and all of its reality ; and theythus stand to existence in a negative, relation. Says Kant, The principle tion of contradicnot in a positive, is a universal but purely criterion of all truth." And negative this holds equallyof all the proximate and special of applications of the whole of Logic. Our science, these laws ; that is, then,in its is not a positive criterion of truth ; it can relation to other sciences, with thoughtsand not criterion, only be a negative beingconversant with the possibility and not the reality with things, of existence. is said to be
" "

We eminent

have

referred to the

Law psychological

of

Some Relativity. mind


no

German
to the

competent
and This

have held that the human philosophers of the absolute, has or that which cognition extensive
can

is tion, rela-

have

elaborated thereon
Absolute

systems of philosophy.

or proceedonly upon a more the primary laws of thought. Fichte and less admit the Law of Identity, but deny the two others, the Schelling between the Ego and the Non-egobeingmerged empirical antagonism in the identity of the absolute Ego." Hegel regards all the laws as but only for the finite Understanding, valid, theybeinginapplicable The eclecticism of Cousin atto the higher tempts processes of the Reason.
"

of the Philosophy completedenial of

the

of cognition

the absolute without


at once

the repudiating

laws of

Logic.
from

It is therefore there is
no

involved

in undeniable

contradictions

which

escape.

"

7. The

of principle
as

Sufficient Reason,

or

Determinant

Reason,
It is

has been

laid down
:

fourth primary law of

Every judgmentmust have a first distinctly It was enounced of Identity with the principles and Contradiction, the together Contradiction as the of his Logic. Kant adopted it, regarding
thus
87

ciated enunthought. sufficient ground for its assertion. who made by Leibnitz, it,

basis crite-

Critique of Pure Reason,p.

115.

PRIMARY

LAWS.

17

and Sufficient Reason the criterion of as possibility, logical But logical and logical are one. logical reality. possibility reality rion of followed Fries and Hamilton, in his lectures,

Krug

in

admitting the
"

in Logic,but subsequently he gave to this high position principle it extralogical." Mansel says, The relation it up, and pronounced to the act of judgment is merelynegative of this principle ; it forbids
us

in certain

cases

to
a

reason logical thought; and


own

for

and it does no more. judge at all, thought of any kind is its relation
case

The

only
other

to

some

this relation will in each


one or more

be

proper

law,"i.c., by
that
2

of the three is therefore


no

by its givenPrimaryLaws.
law of

determined

"The

of Sufficient Reason principle statement


or

only the
some
as

law

other."

but thought, be governed by every act of thought must the principle, Adopting this view,we reject of

forming no

element positive

Logic.

"
state
to

8. In
an

connection

with

the of

Primary Laws,
Logic.

it is
:

importantPostulate
to

It is this

to appropriate Logic postulates


tained conimplicitly

be allowed in the

state

in language all that explicitly

is

the According to Aristotle, thought.90 with the

doctrine of the

of reasoning in ordinary guage, lanexpression but with the internal reasoningof the mind itself. Logic, all the propositions has alwayspresented of a syllogism, though altherefore, of them is usually in actual argument one left unexor more pressed. and highly But, since all speechis very elliptical rhetorical, be allowed to Logicin general, the postulate be furand must ther must the accurate extended to include not only and complete rendition however into language, and awkward its expresof the thought prolix sion rhetorical for but also the be, Logic disregards elegance, may
not deals, syllogism

of of metaphors, and,indeed, transmuting


most

allrhetorical

forms,into the

providing only that the practicable, thought itself be not changed. For as Logic deals only with the of the accidents of expression. it must be independent thought,
literaland direct statement Hence when
one a

deals logician
"

with

an

abbreviated than
meets

or

pression, exfigurative

wherein

more

is meant

the

for much ear,"

he at once and metaphors, intimations, thoughtis conveyed in hints, He then proceeds, What is the full and true meaning of this? asks, and must be allowed to strip off all ornament, to supply all Iacuna3,

38

Logic, p. 62, note

; and

p. 251.
30

29

Prolegomena Logica, p. 182.

Hamilton's

Logic, p. 81.

18

INTRODUCTORY.

and
to

to

exhibit

the

thought
so

naked and

and

entire. and
to

This

is

often

difficult

do,

thought

being
He

subtle be

evasive,
too,

language
make the and called much

so

meagre
in

and

inaccurate. for

must

allowed, provided
Such and

changes
thought entirely
"

seology phrais
not

mere

convenience,
modified.

always
alternative

thereby

essentially
having
a

similar

propositions, Propositions,"
Mill
to assert

equal
for
matter

power which thus in

reach,
shall
"

are

Equipollent
in be the allowed
quel. se-

term

we

have

use

states

the

Logic

postulates
which will for

to

the the

same

meaning
of it."

any
a

words

express any
is

it;
that

we

require

liberty
with that

exchanging
The deals

proposition
of

other self-evident

is

equipollent
the

justice
not

the

Postulate but with

on

ground

Logic

with

words,

thoughts.

PAET

SECOND."

OF

CONCEPTS.

I. THE

TERM.

"
the

1.

Thought
the

viewed

as

product
the
as

of

intellect

exhibits when

three

forms,
in

Concept,

Judgment,

Reasoning, which,
the

expressed
the and of

language, severally appear


The may
or more

Term,
in

the

Proposition,

gism. Syllosonings rea-

three be

are

not to

different

kind, for both


A

concepts
is the into result these

reduced of
a

judgments.
may

concept
be

one

prior

acts

judgment', and
of But and the each

analyzed
of
two

again.

reasoning
relations distinct

is
to

judgment
a

relation of these
a

things through
of

their for

third.

forms

thought
of

calls mentary eletheir the

consideration,
Under their the

constitutes

general
us

division

Logic.

Concepts, then,
constituent
or

let

consider

first

Origin, they
common

and

elements Term.

being comprised by

title of

N6tion,

the

"

2.

An

account

of the
cannot

genesis of concepts
be

belongs
here.

more

to strictly

Psychology,
may First When
as

but

entirely omitted

Three

momenta

be of

distinguished, viz., Abstraction,


Abstraction. the mind in is attracted
sense,
as

Generalization,

Conception.

to

some

object,either

an

external
memory,

thing
it apprehends

presented
it
are

or

mental of
a

image presented
number other of
or

in

only

possessed
as

of and

qualities.
If then

These mind

ities qualperiencing ex-

apprehended
what is

unlike
"

each shock

several, the

called

the the

difference." the

attention while pass


out

is the of

now

fixed

on

one

quality,as

color,

weight,
or

other

qualitiesconsequently
one on

become

obscure,
attention the

perhaps
is

consciousness, this
into vivid
of

which

is fixed
not
one

thereby

drawn

consciousness,
Thus
or

becoming by

chief, if
to

the

ject exclusive, obmind

cognition.
abstracted view

attention away from

this

quality the
In of the this

has

been

drawn is the

all others. correlative

logical psycho-

Abstraction

negative

positive act

20

OF

CONCEPTS.

of qualities, in being plurality But this one be considered as concentrated on one. quality may from all others. In this logical abstracted or drawn view Abstraction away is a positive act by which we cognizeone quality apart. It that we obtain a clear and distinct notion is thus by abstraction of the qualities, ject, attributes, characters, features, properties, etc.,of an oball of which terms and are included in are nearlysynonymous, Logic under the one term, marks.
to
a

the mind of attention,

being denied

It may

be at
"

once

noted

that marks

are

of several sorts

or

kinds.

They
a

are,

1st. Positive

or

as negative;

rational

is

and imperfect positive,

mark negative,

of

man.

2d. Essential, necessary,


and essential, learned
an

accidental, contingent ; mark of man. accidental,


or

as

rational

is

an

a calculating original, of his rationality. mark of man, this being a consequence derivative, 4th. Simple or complex ; as conscious is a simple mark (i. e., one animal of and not susceptible analysis) a complex mark of man, the and sentient. So red is a simple latter beingcompounded of organized mark of one kind of rose, and vegetable a complex mark. mortal 5th. Common is a mark to man or as common peculiar; with other animals, risible a peculiar mark, found in no other being. A peculiar is called mark and to to this, as a property," belonging

3d.

or Original

derivative

as

rational

is

an

"

no

other, yet
A
as

not

considered

essential ; thus
is
one

mobile
a

is

property of

body.
alone ; The

mark particular the mark


set upon

belongingto
be discerned

individual single

Cain. may
in any

number

of marks

which

great. It would
all the marks
of grain
corn.

be

to impossible

enumerate

which

might

be discerned

in

so

simple an

definite objectis inexhaustively objectas a

"
and

3.

to us in sense are or presented objects complex. In our apprehendingthem, very

But

in memory many

many of their marks

as

produce the
but The
some

shock

of

or difference, produce dissimilar impressions;

givkthe

shock
an

of repetition

or produce similar impressions. similarity, is precisely what excites attention, or impression

of

determines
are

the direction of reflective consciousness.


to

When

some

jects ob-

agree, while others wholly disagree in certain respects, those which partially consciousness is concentrated naturally on

found

agree ; and

then,in them,

on

those marks

in

or

through which

they

THE

TERM.

21

So far this is mere abstraction. agree. We observe a number of animals; our

To

givea

crude illustration:
is attracted to
a

attention

horse,an

ox,

from greatly goat, a dog, etc.,differing

the

birds, tiles, pep--

etc.,that may
then observe
more

but agreeingin some present, respects. We and is/otwv that each has a hairy hide, particularly

be

in which footed, Similar

marks
are

they agree.
which stand in similar relation to
our gans or-

marks

those

the similarity is complete, cognition.When the effects which they produce in us are indiscernible. But what we is to us virtually the same cannot distinguish ; i. e., they are subjectively if they were identical. The same, to us as identical, objectively consider them to be, though really in different objects. we accordingly,

and

faculties of

This

act,to think the similar the


so we a

same,

is the

essence

of all

Generalization ;
the
same.

It is would

is to think the similar may say that to generalize fiction of thought,1 but one without which our be- unable
of objects grasp the multiplicity think that each of the animals named in the
to

limited powers

We to us. presented above example has the same to either of now applicable to unityin thought. Such notion.

is g., four-footed. This mark the objects.A plurality has been reduced
e. a

mark,

mark generalized

is

stract abgeneral simple

that generalization is classification. by anticipation a They are but different aspects of the same operation.By thinking to several individuals, mark as common we therebygroup them, we constitute a class containing Thus the animals above named them. belongto the class or group quadruped. remark that when we Also we of ana number imals speak of observing have already Their we together, thought them as one group. marks and therebywe have already have been generalized, common constituted the total of the objects considered into the class already We may animal.
of that, havingaffirmed the mark four-footed of these objects, a class, some we, in the same constituting thereby act,under the shock of dissimilarity, deny this mark of the rest. The class;i.e., are birds, reptiles, etc., therebyconstituted into a negative

observe

Now

let it be noted

the Overlooking
same,
a

fictitiouscharacter

of this act, and doctrine

the thinking

similar to be

the really

times,that
to

gave rise to the erroneous universal objectively exists

of the Realists of mediaeval

all

Thomson's

things of one kind. Outline, " 62 ;

of things, and is common independently See Whately's chapter on Realism, Logic, p. 305 sq. ; and Ucberweg's History of Philosophy, " 91 sq.

22

OF

CONCEPTS.

one

characterized by the

in groups are dissimilar The the other does not. the equals

These two mark, non-four-footed. negative and that one possesses the mark four-footed,
sum

of the two

groups, the A

and

non-A,

total of the

animal. universe,

Further,if

we

contemplate

the

we special group quadruped, and of dissimilarity. The ox and the goat each have horns ; beingand call them horned we similar, quadrupeds. The horso generalize horns ; being similar in this negative and the dog have no respect, of -horned them into a negative non we quadrupeds. generalize group But, at the same time, the two groups being dissimilar in respectof having and not having horns,we think them different or diverse. classifiedinto two co-ordinate kinds,the or They are thus specialized, and the non -horned,subordinate to quadruped, which is their horned the necessary correlative to gensum. This,then,is Specialization, eralizatio is to think the dissimilar and we may say that to specialize or diverse,

ity the shock of similaragainexperience

distinct. or different, distinct from

It also is classification. It is
occurs

not

the

for neither generalization, other; theymutuallyimply each other.


a

process

without

"

4. The

third moment
To

of

its product the thought is Conception, grasp When together.3


may
a

Concept.
number

conceive have

is to (con-capere) been

of marks
one

abstracted, they

be

collected

by
a a

thought into
concept is
a

notion,and this referred to substance concept. A concept, then,is a union of marks in one
bundle
of marks

constitutes
notion
some

; or,

thought of
the mind many

as

in inhering
an

thing.
our

Every
marks.

to thingpresented

has

indefinite
to

of plurality

Observation

can

make

known

us, but

edge, knowl-

in increasing though constantly


never

fulness and

can complexity,

exhaust take
in

them.
at
once

Moreover, the limited powers


all those marks when whose
we

of the mind
is known.

not can-

presence
to

A
prehend com-

becomes representation
in
we one

confused than
a

attempt

grasp

or

more
a

form

tempt, Giving up the atvery few of them. few embracing comparatively concept of the thing,
a

of its ascertained distinctive and the

marks, making
The

selection of those
or

which

are

most

essential.

concept

involves therefore, notion,

of a part onlyof the marks of which an individual representation affords only a one-sided and inis the sum, and consequently object

So the German

Hence

begrafm, cognate to our words for "concept." the German Begriff\"

"

grip," grab,1' group,"etc.

"

"

24

OF

CONCEPTS.

making a unityof

complex notion for each of many things, is therefore general, and


stand for many
or

I form my plurality, concept of star. This is applicable a host of distinct to,or contains under it, individuals has all these marks. the word
"

this

The

notion

express

this bundle To
a

which has been adopted to star," of marks, is the common of name of


or

individuals. A

this form combination marks


a

concept Hansel's definition applies:


reduction

concept is

of the similar
are

or qualities

of the

unityin thought objects thought upon, which


to

therebyconstituted
We
may
now more

into

class. the act in which think


is to
ceive, con-

the mind
"

knows is to form

define thought as adequately of concepts. To thingsby means

concepts.
that the

"
of

5.

It is obvious

between similarity
are

concept expresses merely the relation it denotes, the things of course, that implying,
But
a wre

there

also differences.
to

mere
can

relation cannot

be realized in

cognizea relation of things, is contradictory and absurd. and yet not the thingsrelated, Or, an an act of cognition necessarily implies object cognized ; but a relation be an object, of its terms cannot since there is nothingin it stripped is not cognizto the thinking subject.A concept,therefore, able opposite it affords no absolute or irrespective of that is, in itself; object knowledge. A concept can be realized in consciousness only by applying of the related similar in those it to one more as or spects. reobjects When we attempt to represent by an image any abstract find it impossible.We absolute object, thus we can an as generality individual and determinate object. realize it only as attached to some is found to consist in this : that though we Its whole generality must individual of the class, realize it in thoughtas comprisedin some we The generality of a concept,then,is potential, may do it under any.
suppose
not

consciousness; for

actual.2
For

example:
This
term

I have
is

sides.

notion triangle, of three a figure general to several species, others to applicable among the Now
at the

the

and to the scalene. equilateral be it must in its generality, triangle scalene. But herein is
a

should
same

attempt to realize

time both

equilateral

and

image must have its sides all equaland yet unequal. Hence such an image is impossible. and scalene, be both equilateral it must while the image cannot Still,
a

contradiction ; the

See Hamilton's

Logic, pp. 91, 96.

THE

TERM.

25

an image,then, or else draw with my pencil, of its sides, the equality and equilateral triangle ; and by disregarding all particular marks this individual figure, I can template concharacterizing alone the notion trilateral figurewhich it comprises. Thus realized in consciousness. only is the concept triangle one or

be

the other.

"

6.

It must
are

not

be

understood

that the three momenta

now

scribed de-

They are actually separateand successive in the mind. in reality but are not distinct and independentacts, guished only so distinand stated in order to enable us to comprehend and speak of indivisible operation. It is merely the several aspects of an actually of a mark the generalization cannot a logical analysis.For instance, without a grouping of several without a classification occur ; that is, is essentially in one which stractio abclass, conception objects ; and, again, is analysis, that there was which implies already by the mind a though it may have been very obscure,of marks, from synthesis, into clear consciousness; is drawn but. a synthesis which one among is conception. of marks, however obscure, Moreover, a mark and a concept are commutable. Every mark is A conand every concept potentially a mark. a concept, cept potentially is expressed by a substantive or substantive phrase by ; a mark and of the transmutation these an or adjective phrase; adjective to the change in the aspect of grammatical forms, corresponding
is thought,
a

familiar fact.
is used

Thus
a

"

Man

is

an as

animal,or
a

is animal."

Here

animal

first as
use

next concept,

mark.

The

tion distinc-

consists in the

made
is
a

of the notion
if used
"

the notion
is
a

concept ;
is
an

notative by thought. If used dethe notion connotatively, that


; but
man
"

mark. of

Thus

"

Man

animal

means

is

one

of the
r'

kinds
means

thingsdenoted
man now
"

by

the class animal

Man*

is animal

that

has the attributes connoted observed is used


are so

Let it be word mark.


one
"

that

here,and

by the mark animal. the throughoutthis treatise,


either
a

notion The

genetically ; it

means

concept or

two

into the other in

so changingthe convertible, constantly freely to exterm need a common press that we thought,
"

either.

For this the word

notion

"

is most

suitable.

"
very

7. In
common

this

connection

may

be

noticed

another

very

subtile but

only the play of thought. A mark, which is strictly this relation being obscured by attribute of something, or is, quality itself a thing. Inoften thought of as though it were abstraction,

26

OF

CONCEPTS.

stead of

being referred
Marks

to its

severed
existence.

therefrom
so

it is, it were, comas pletely substance, original and established in an indepenby thought, dent treated are specifically called abstractions,"
"

and

are

by expressed
of

abstract terms, of which Thus:


blue is
a

great number
of

have

the termination
ocean,

"-ness."

mark

etc. ; but fineness is thought of as sapphire, of these things, and spoken of as though it had a real existence but justice is extolled apart by itself. Again, Aristides is just, In the one is thought as concrete from the mark case any person. inherent in something; in the as (con-crescere=to together), grow other case abstract. These are proper oppoit is thought as entirely is concrete is A concrete sites. Human term ; humanity,abstract. of an of an the name attribute. object;an abstract term the name An abstract term, then,is the name of a mark thought as a thing.3 of sucli terms in every refined language The uncounted multitude shows what familiar use is made by human thought of this fiction. and the abstract are and different regionsof thought, concrete The and constantly ing the difference should be clearly as a confusobserved,

sky, of the pendent somethinginde-

the

of

one

with

the

other

often

leads to

the

grossest absurdities.

the factitious and fictitious not Plato,in the Sopkistet, recognizing of abstracts, exist which are nature real; incorpoargues that thingsmay and for justice and wisdom, says he, are incorporeal; justice and
a thing something. By something he means in and by itself, and not merely as the quality of capableof existing other thing. From this source some grew the Platonic doctrine of that abstracts are independent entities. The Aristotelian Ideas, teaching

wisdom

must

be

"

"

doctrine idle

of

substantial forms

and

second

and substances,

all the

stractio TO eV,TO vpotot', and similar abrespecting speculations ov, TO have the same origin. Many of the gross blunders of of the abstract attributable to this confusion modern are metaphysics If the student of philosophy would always, and concrete. at least or of importance, guage in cases adopt the rule of throwingthe abstract lancouched into a concrete it is so frequently in which form, in with aid the obscurities and he would find it a powerful dealing He would of metaphysical then see clearly speculation. perplexities which constitute what of nothings the character of the immense mass passes for philosophy."
" 4

"

See Mill's

ch, ii, bk i, Logic, "

4. See remarks of

Letters on Bailey's

tiicMind, vol. ii, p. 139,

Bain,Logic, p. 52

sq.

THE

TERM.

27
to language

"
and
means

8. It is

to important

consider the relation of

our

cepts. con-

concept would
from
to

immediatelyfall
it has been it permanent.
and

back

into the
were

infinitude
some~

confusion

which render

called out This


a

there not

by

which

words. of which

The it

concept is fixed
can

ratified in

accomplished by verbal sign, by means


consciousness. guage, Lanof things. cognitions

is

at

then,is the
It is the

any time be recalled into attribution of signs to our

of thought. Many thoughtsare valuable either not register and are dismissed. at all or only for the moment, Any one of high is preserved value and needed for further use by a sign; we give it a
"

name.

Nomina of
a

sunt

notionum

notce."
is
a common noun.

The
noun,

name

notion general
a

Every common
to belonging

therefore, expresses

fasciculus of attributes
a

each

of several

objects. It
to
a

stands for

unit that
an

be

used

in further

and is a factitious productof thought, have already marked rethought. We


a a

by concept is expressed
an

and that adjective noun, considered as a thing. We and also that many action,6
of
"

substantive noun, abstract noun is the name

mark
a

by

of

mark

add that a verb is the name of an may notions are registered in phrasesinstead

of
a as

words ; e. g., we have no single word to express our notion single to only one like a rainyday." A noun singular applies object,

but then it is singular only in its presentapplication proper name, ; "a song,""this world," "my horse," "the king." It is evident

that the The


"

singular meaning is
means a

obtained
"

by adding some
or

word. limiting
as

indefinite article
us

some

any

one

of the class ;"

in

Give

song."
a

The

definite
a
"

etc.. possessives, to belonging

indicates class ;
as

with demonstratives, article, together it as individual, particular yet designates


"

king comes," Cesar's army." All such names are connotative, they implyattributes or marks, and when used to denominate a subject they carry these marks into the subject is non-connotative.6 and attribute them to it. A proper name strictly It denotes an individual, but does not indicate or imply any attributes of that individual. of a quality It is not the name or qualities ; it is but an in our minds unmeaning mark or sign which we connect with an object, that when this signmeets our so may eyes or ears we
6

The

J. C.

traced Scaliger

the distinction between


noun

the

noun

and

the verb to
a

ence differ-

of time ; for the and


6

the verb representsa permanent thing,

temporary

state. transitory

See

Mill's

bk. i, ch. ii, Logic, " 5, for


and

an

able discussion

of the distinction between

connotative

non-connotative

terms.

28

OF

CONCEPTS.

think

of that

individual ; it does
nor individual, name

not

of itself connote

of quality This is true

the

convey

any

information
as

of the proper

considered
mere

the But

name

imply any it. respecting or signof an


or

individual

to object presented

intuition.
is

if it stands

for

or

a evidently complement of in the example given indefinite plurality, an as marks, and connotes above ("4) of the notion Socrates. When Euclides, having heard of from the fame of Socrates, went Megara to Athens to see him, and That person is Socrates, then cersome one tainly pointedhim out, saying,

expresses my

notion

of

an

it individual,

the proper name, thus attributed to the person, connoted and carried with it the marks which constituted Euclides' notion of rates, Socand identified this While

concept with that person.

language is
have

not

thought must

been

absolutely necessary its to prior name, it is


it
we

to

thought,for
to

the

necessary

progress. in the scale of lowest degrees

Without

could A

never

rise above

siderable any conthe very

necessary to give to our intellectual progress, to establish each step in advance stability for our advance, to another beyond. Without as a new starting-point there language

thought.

sign is

knowledge realized of the essential properties of things, and all ascent from the sphereof sense to the sphereof moral and religious is without it impossible, or intelligence possible only to a very low degree.7 In thinking without language, it follows from what was said in " 5
no

could be

that at every

step in the process each motion


the

must

be realized in

sciousness con-

example. It is obvious that this is a clumsy and very restricted procedure.By the device of languagethe is emancipated from mind of continuous the necessity realization. Instead of this intuitive thinking, this to call it, prefer or, as I would aged, imeither perceived we or thinking by example, may think by signs, which is called symbolicthinking.8 As Berkeleyremarks,9 It is not in the strictest reasonings, that significant necessary, even stand for notions, which should, every time they are used, names, for. the ideas they are made to stand excite in the understanding In reading and discoursing, names are used,for the most part,as symbolic in which, though a particular be letters are in algebra, quantity that marked it is not requisite by each letter, yet,to proceedright, image
an
" T

by

of

See Hamilton's Eori fitv ovv Int. ch. i. See Minute


TCI

admirable

of exposition iv ry

iv ry

(fiwpyT"V

Logic, points, pp. 98, 99. ovf.ij3o\a. Aristotle, "fyv\y TraQrjfJ-aTwv


these
"

De
'

Dialoguevii, " Philosopher,

8.

THE

TERM.

29

in

every

step each

letter should
was

suggest
to

to

your for."

thoughts that
By
this
means

ticular parthe

quantity it
and facility

appointed thought
are

stand

range

of

vastlyincreased.
in this
use

There

is

danger,however, peculiar
for

of words that

as

ry tempora-

substitutes and
write
as we nonsense

thoughts. Campbell
persons
are

shows

by

it many
and
even

cious judito

well-informed without

sometimes
one

led to talk

knowing it.10 Thus,


of "a

say,

speak thoughtlessly,
"

bilinear
is cold

might trippingly, or, tary involunor figure," an


"

donation,"
corrected
was

or

say
"

the I

weather
my

as

blazes."
men are

The
liars

ist Psalm-

himself
this And

said, in

haste, all
and of

;" which
very

well, for
lie.
are

saying included
this

him,
us

if true, was therefore,

likely a
"

reminds
of wise

the but
at

saying
the money
end

of

Hobbes, that
of

words

the

counters

men,

fools." process the

It of

is

consequently needful,
the
test

says

Mansel,
ascertain

the

of

thought,and
to

at occasionally

intermediate

to submit stages,

result of

of

an

example, and
a

the

possiblecoexistence
The
the individual intuitive its

the
of is
a

attributes class is

in

corresponding object of
if the

intuition.
of

existence members

possibleonly
its

existence

possible;hence
or

symbolicalcognition supposes
condition, and
a

cognition
it.

actual The which

possibleas
of

derives

validityfrom
of
an

test

thought, then, as
in the

is an possibility,

image

example,
We

is

possible only
that the

absence them

of self-contradiction.

must,

then, envisage our them,


is done insure

notions, look

in the

face,and, thus

realizing
This

they

do

not

involve
case,
or

attributes. contradictory
an

by

intuition which

of

example, called, by
an

the

mans, Ger-

Anschauung,
to

may

well be translated that in which


an

envisaging.11
with the attributes members
of

Symbolic conception,then, is
sense or

arbitrary sign present


all the

imaged by

the

mind, and

associated

of

general notion, is regarded as employed


as a

of significant

the class. As in
be defined symbolic thinking,the concept may now and representof attributes united under a sign, ing

collection

objectsof possible

intuition.

10

bk. ii, ch. Philosophy of Rhetoric,

vii. p. 106. See also Hamilton's ure Logic,Lect-

11

Prolegomena Logica, p.
X.

35

sq. and

30

OF

CONCEPTS.

II.

QUALITY.
in four ways. with reference First,

"

1.

Conceptsmay

be viewed

to the

and in which, which theyrepresent, the external objects things, or indirectly, they originate, they are considered as arising directly of from them as their source or qualities ; as constituted of the marks to one to many the things : this is or common thing, ; as applicable ject, subwith reference to the mind, or thinking their Origin. Secondly, towards perfection ; as they are considered as having gradations less clear, etc. : this is their Quality. Thirdly, or distinct, beingmore with reference to their contents, they are considered as comprehending : this is their Quantity. Fourthly, marks, or as extendingto things tions relawith reference to each other, they are considered in reciprocal co-ordinate as or as or contained, as the 'same different, containing Their Origin having been subordinate : this is their Relation. or considered under the previous topic, to examine, secondly, we come now their Quality.

of concepts. His quality in a famous littletract On Knowledge,Truth, views were expressed In it he pointedout the distinction, and Ideas." examined, already between intuitive and symbolic to Hamilton, which, according thinking, in Germany the whole controversybetween Nominalism and superseded that agitated France and England for several centuries. Conceptualism have quality less perfectly as or according they more Concepts resent repin consciousness their objects.The following scheme marks the degrees : by which knowledge approaches perfection

"

discussed the 2. Leibnitz first thoroughly

"

( Obscure

Knowledge

is...."" Clear. '(


. . .

( Confused2

Inadequate

(Distinct....-]
(
{

( "j Adequate
( Intuitive

[-Perfect.
)

Symbolic
of the
tract

In

Ada

Eruditorum,
be

1684.

See

translation

appended
then

to

Baynes's edition of the Port-RoyalLogic.


2

It would

if this better, perhaps,


as a

were

named
and

and Indistinct, Indistinct.

Confused

might

be taken

genus

to

include Obscure

32

OF

CONCEPTS.

it is thingsthat lie nearest to it, declaring -what it is. and then proceedto render it distinct by stating not these, etc.3 it is righteousness, is not pardon, E. g., Justification We think a concept distinctly Distinctness of thought has two modes. from each the marks which it connotes when we distinguish when we can other, this is its distinctness in intension ; and, again, each other, this is its the thingswhich it denotes from distinguish Intensive distinctness is obtained by definition, distinctness in extension.
of especially ing it,

those

"

"

the

enumeration

of the marks. discovers the

Extensive

distinctness is obtained
under
can

which by division,

concept. Thus, a chemist's notion of and its marks, i. e., give its intension, name
there i.e., give its extension. be),

things contained gold is distinct : he


name

the both

its varieties of

rieties va(if

My notion thought was from and imagination it clearly obscure until I separated perception, memory, its characters and it is now becoming distinct by studying indistinct ; but intensively and kinds. Our notion of red is very clear, it from blue. the particulars cannot name we by which we distinguish somewhat It is, more as we can name distinct, however, extensively A primitive etc. the varieties scarlet, notion, such as crimson,pink, be analyzed, is without marks, is therefore cannot identity, being ultimate, be cognizedonlyper se. and can Though perfectly indefinable,
it has clear,
no

either intensive distinctness,

or

extensive.

think a then adequate. We knowledge is inadequate, and importance distinct concept adequatelywhen the relative number sufficient to correctly of the marks which it connotes are representthe When everything," thingswhich it denotes. says Leibnitz, that enters into a distinct notion is distinctly known, when the last analysis

"

5. Distinct

"

"

tinguish must we keep in mind that he does not disLeibnitz, and that these graduateinsensibly into but degrees, kinds of knowledge, has which another I discern in an objectsome one each other. When quality not, the is to declare that they are not be sufficient ground for me one this may two, be sufficient and so far my the other, knowledge is clear. But this would not, perhaps, it and cannot what I tell the for to describe is, me yet object ; my ground
3

If

we

would

understand

But as I discern more and still indistinct or confused. therefore, knowledge is, from what we call merely more marks, my knowledge of the objectgradually passes I then describe and it. When can distinct into but clear knowledge, indistinct, I can but not until define it inadequately, the distinctness becomes more complete, tinctive i . enumerate I all its disit define all its marks discerned can adequately, e., are to perfection consists in a discerning marks. The whole process from obscurity of
more

and

more

marks.

QUALITY.

33

of which I scarcely know then the knowledge is adequate, reached, the knowledge of numbers^ whether a perfect example can be offered; have a nearly to it." Perhaps we near however, approaches adequate of so few marks, its definition consisting knowledge of a chess-board, ultimate and simple and they so nearly : a square composed of sixtycolors. Dr. Thomson four equal opposite says, squares of alternately carries the which We analysis may consider any knowledge adequate equate far for the purpose in view." E. g., A machinist has an adsufficiently and knowledge of the machines he has invented, constructed, But this is practical, not logical, used. adequacy. The great bulk of our inadequate. knowledge is logically

is

"

"

6.
a

Distinct
distinct

knowledge

is also either intuitive


we

or

dividual, inimage an in it all the marks connoted by the concept,and containing denoted by the concept. Notions itselfcontained under the class of things those of visual objects, not arc very complex,and especially in an we exemplified readily image ; but when one is very complex, able to image it completely. Thus could not image a not we are Even before the eye, we could not such figures were some chiliagon. the difference between one of 1000 sides and one of 1001 sides. perceive at least to or When, however," says Leibnitz, we are able wholly, this image,I call the knowledgeintuitive." to form a great extent, in longeranalyhe continues, especially But, for the most part," ses, do not behold at a glance the whole nature of a thing, which we would be intuitive knowledge, but we employ signsinstead of things. We commonly omit,for the sake of expedition, of these any explication in presentthought, that we have such explisigns cation knowing or believing in our of when I think a polygon of a Thus, chiliagon, power. thousand equalsides, I do not alwaysexpressly consider the nature of of of but I employ these words in the a side,' thousand,' equality,' them." This is symbolic placeof the ideas which I have concerning of numbers, such as those which state the velocity thinking. All large miles per second), the distance of the sun (91millions of (186,000 light and also all such very complex notions as religion, civilization, miles), known to us onlysymbolically. the Englishconstitution, are war, etc., is readily while Our knowledgeof primitive intuitive, as unit, notions, for the most our part,symbolical.4 knowledgeof compositeones is,

think

when concept intuitively

symbolic. an example,

We

"

**

"

'

'

'

Leibnitz

was

not

the

as first,

Hamilton

and

Thomson

who intimate,

re-

34

OF

eoxc^rrs,

"
it
is

be at the V. If knowle'clg'e

same

time both

and intuitive, adequate

it is clear, when perfect.We think a concept perfectly distinct, in a nd individualized It intuitive is evident an adequate, example. is hardly, that knowledge logically or tainable perfect only in rare cases, atBut we are too easily with obscure content by the human mind. and thus our indistinct knowledge, or thoughtsare often vague, and absurd, without our or even of self-contradictory becoming aware it. Then we believe that we blind. we are really see, when But should our still our edge knowlconcepts become logically perfect, would be very far from absolute perfection. Truth," savs Cudworth, "is biggerthan our minds, and we are not the s-ime with it, but have a lower participation only of the intellectual nature, and arc than comprehenders rather apprehendcrs thereof. This is, indeed,one of have not a perfectly our state,that we creaturely badge hensive compresuch as is adequate and commensurate nor even to knowledge, Yet it is the ability of things." the essence to form concepts of things, the many in one, to classify to comprehend and understand and arrange of knowledge, in order of relations the objects that is above all other?, of intellect, the gloryof the human the great power mind, and that which constitutes its immeasurable the brute. over But, superiority of the other hand, it is the necessity forming concepts at all, the on of resorting to a fiction of unity in plurality, the necessity necessity of making a minute perfectio part stand for a vast whole, that marks the immind. However and finite character of the human ly perfectof a logical this may be done, it is merely the perfection device, of knowledge. To know not the perfection thingsin any measure,
"

the human

mind

must

think

them, and this constitutes

its immeasurable

does not think at all, to the divine mind, which but inferiority all things, knows, by the immediate intuition of the things themselves, and totality. in their real plurality at once

marked
he

this distinction between


on

intuitive and

impressedit

modern

philosophy. I find the

in De Soph. Elencli. ch. i, as by Aristotle,

though certainly symbolicthinking, distinction clearly same implied Not being able to point out follows :
"

the
as

in
to

instead of the realities about,we use names thingsthemselves that we reason in the names to be consequences their symbols. Then the consequences appear in the counters to the calculator the realities, just as the consequences appear tion, the counters. in the objects be the consequences by As, in calcularepresented the counters in manipulating deceived by those unskilled are those who are
are are

who
names

skilled, so,
deceived

those who are unacquainted reasoning, See by paralogisms." supra, pp. 28, 29. in

with

the power

of

QUANTITY.

35

III. QUANTITY.

"

1. We
a

are

next

to

consider

concepts with reference

to

their

tents, con-

of Logic than the within the province strictly lyingmore which belong rather to Psychology. That a concept two preceding, since it consists of a variable is manifest, be viewed as a quantity may of things. to a variable plurality of marks, and is applicable plurality view

And

this indicates
or an

that the

is quantity

twofold.

It is either

an

tensive in-

is said quantity. A concept viewed intensively its marks, which are reduced to unityin thoughtby being to connote in one substance ; viewed extensively, the conconceived as inhering cept is said to denote which are reducid to unity its objects or things, in thoughtby being conceived class or group, as one constituting of the class possessing Its marks, then, each member all the marks. constitute the deconstitute the connotation of a concept; its objects notation extensive

of The

concept.1
of
a

intension

termined or depth,is deconcept,or its comprehension by the greateror smaller number of marks contained in it,
it is the
sum.

the concept man is composed example, of the marks all thought as existing, sentient, rational, living, in one substance. of the connotation of the This explication inhering is its determination is a being, definition ; thus,Man or concept man and rational. sentient, living, of a concept,or its sphere The extension or breadth,is determined of objects of specific or by the greater or smaller number concepts, contained under it. For example, contains under it the concept man the specific etc. mechanic, Again,the chemist, artist, concepts logician,

and

of which

For

This

important distinction, though


acuteness

taken

in

general terms
remained It
was

caped esby Aristotle,

the marvellous
until the

of the schoolmen, and

of the publication modern

Port-RoyalLogic,1662.

taken and philosophy, of the subsequent works on doubtless again original.It passed thence into most but in England nothing beyond Logic. In Germany the doctrine was developed, Arnauld's expositionwas borrowing attempted until Hamilton expounded and appliedit, as from Krug,Esser,and other German an writers, integral largely part of the science. That he overestimated its consequences
will be seen

time

in

overlooked totally f or therein, the first it. was applied by Arnauld, with whom

in the

sequel.

36

OF

CONCEPTS.

contains under it the objects Aristotle, Porphyry, concept logician and the This rest. of the denotation Boethius,Arnauld, Hamilton, explication
of
a

division. or concept is its specification


are quantities

Observe viewed

that while both

said to

is said to contain in it, to or intensively it is said to contain under but viewed extensively things.

a contain, concept comprehend,marks,

it other

concepts,or

"

2. It is evident that if the

number extend
marks

of marks
to
are a

of

concept be few, it may


the other

the content constituting of things great number ;


and

and, on

hand, if the
be

many

the distinctive,

of onlya small number of things. predicated Thus the concept bird has only a few marks, such as existing, living, it is sentient, tains winged,biped, etc.; but feathered, applicable to, or conof things number undcl' it, the cona great variety or cept ; whereas dudf\i9" more or marks, such as ivcb-footcd, etc.,and the variety denoted is Hence of less. number have the genwe thingsthereby eral

concept can

include and

law

that the greater the the

the intension,

smaller

the extension ; and


; or, in other

the

smaller

the greater the intension,

extension

in inverse ratio. arc quantities ing ThinkConceptsare modified in thoughtby changingtheir content. in marks, we and vice versa. In theoretical strictthink out things, ness, the thinking in one mark is the thinking class or thing, out one and vice versa, and the ratio is exact; but in actual thought, owing to the incompleteness of our concepts,the ratio is very far from exact, and the law applies The theoretical statement, sense. general only in a loose, however, should be limited to the essential and original marks,

words, the

two

and

does

not

refer

to

the

accidental

and

the

derivative.

Original

marks

carry The

their derivatives

along with
not

do therefore, latter,
it
more

tion. necessary implicaincrease the intension, but really


of intension is the

them

by

only render
It follows maximum is
a

explicit.
the above A that the minimum

from

of extension. is
one

concept in which
a

the intension marks


can no

minimum

in which

of plurality

depth longerbe
or

mark. i. e., it has but one Such a concept is being or distinguished, which connotes thing, only the mark existing. It is called a simple concept as opposed to complex or compound. Now the extension or breadth of a simple concept is at a maximum. Thus the concept extends to, everything that exists, or beingor thingcontains under it, in the universe. everything

QUANTITY.

37 of extension
or

On

the other A

hand, the minimum

is the maximum

of

intension.
is
one

concept in which
in which

i.e. it includes
a

mum sphereis at a miniguished, of objects no can a plurality longerbe distinin its sphere to but one or object.Such applies the extension
the sJcy^ or or tomb, or Virginia,
day's to-

Hadrian's or concept is Aristotle,


lecture.

Each

of these denotes it cannot


an

because individual,
or

be

and is called an only one object, divided. Now the intension logically
a

comprehensionof
Aristotle

individual is at

maximum.

Thus

the

cept con-

an

in it, or or contains, contains, conceivably comprehends, of marks, so numerous indefinite plurality to defy all computaas tion ;
a

number

is equal to the number which, theoretically,

of the

in things

the universe.

is said that an abstract term previoustopicit was the name of a mark thought as a thing. This is a device of thought, into a form which enables us to make them mere bringing qualities the subjects of judgments. A quality, beingthus treated as a concept, be thought as itself possessing must the two quantities intension and
a

"

3. Under

extension ; that is to say,

an

abstraction

is both

connotative

and

notative. de-

compound quality thought as


E. g.,
"

an

abstraction is from

connotes

its

ponents. com-

The

wisdom

that (abstract)

above is first pure,

then

and good full of mercy peaceable, gentle, easy to be entreated, without partiality, and without hypocrisy." Here both posifruits; tive and negative tributed atelements, which,taken together, are wise, compose to wisdom its intension or connotation;they are its marks. as Now is wisdom from we above,and thus convey into may say, Charity it (connotare), attribute to it, all these marks. tion or Again,an abstracdenotes its several kinds. The wisdom just described is one

kind. scendeth
are,

But
not

we

arc

told there is

and that "This another,

wisdom
a

de-

then,at
or

There devilish." above, but is earthly, sensual, least two kinds of wisdom, and these constitute its extension It is evident
that the

from

denotation.
are

kinds

denoted

by

an

straction aban

themselves be

abstract notions; whence

it follows

that

abstraction can

as Charityis predicated only of another,

wisdom.

these Evidently
concrete

marks The

of the abstraction may

be

attributed to the
"

notion.

above

marks

may

be affirmed of

The

spirit-

15 Epistleof Jamce, iii,

and

17.

See

also 1 Cor. from

i,17

to

ii, 16,where

St. Paul discusses several kinds

of wisdom.

38

OF

CONCEPTS.

wise man." Bat an abstract lias qualicarnally ties the E. wisdom that "The concrete. that do not belong also to g., than from to be desired above is more cometh more rubies, precious of is a defence better than strength, better than weapons than gold, wise man." The be said of "The stractio abwar." This cannot spiritually

wise,or ually

of the

then,connotes
It does not

new

series of marks.

What

is this series? of

consist of the component, derivative

marks, but
a

original

marks, attributable to the


is

merely as quality
rare

: e. quality

g., AVisdom

and desirable, ennobling,


rare so

; that

is,it

is

desirable, ennobling,
so plete, com-

and

With quality.3
as absolute, a

the firstseries the abstraction is not the

with

second, wherein
more

the mark

is viewed

more

be can therefore, things, said of the abstract than of the concrete is notion,which, perhaps, of the favor shown it by thinkers. A one reason notion, primitive without such as single, when taken abstractly, havingno components, is, the first series. We can or unity only those things say of singleness to the second series. belonging

thoroughlyas

thing. Many

"
as a

4. It has been mark


one or we

The

thinkingin
the

either predicate as a class ; as, Facts are stubborn, things. or, arc stubborn call thinking in the intensive quantity ; the other, may plies imthe extensive quantity. It is true that one quantity
we

stated already

that

may

think

and other,

we

do

not

think the

one

without

at the

same

time

thinkingthe

of two is in vivid one thinking ordinary while the other, is comconsciousness, paratively, though within consciousness, and it may be very, obscure. either phase of thinkNow ing become one more habitual, considering attentively may person of a thing, another the qualities it as a member of a class. regarding I am inclined from observation to believe that thinking in intension

other.

But

in

bk. i, ch. ii, Logic, " 5, says : A non-connotative term is one which a signifies subjectonly (e. name), or an attribute only. Whiteness, g., a proper attribute only. None of these names, virtue,signify an are conlength, therefore, nctative." But is not prudence a virtue ? He afterwards modifies this statement, in some only of attributes, saying: Even abstract names, though the names may instances be justly considered as connotative ; for attributes themselves have may
in Mill,
" "

his

attributes ascribed attribute of those


"

to

them

; and

word

which

denotes is the

attributes may
"

connote

an

attributes."
"

His is
a

example
name

word
to

fault ;" equivalent to


and attributes,
notes cona

quality." This word attribute an hurtfulness,' horse,is a fault. This means


'

hurtful

common

many

of those various attributes."

in E. g., Slowness, which

that the

name

is

hurtful

or

undesirable

qualityin quality.

horse

receives this

40

OF

CONCEPTS.

IV.

RELATION.
will view different.

"
them Of

1. In

relations of concepts, the reciprocal we considering firstintensively.1 identical or Notions thus viewed are

for unless there are identical strictly absolutely none; and are fore therethere be some difference, they cannot be distinguished, taken strictly, one. Indeed, the phrase two thingsidentical," is a contradiction in terms. Yet in Logic we tions, speak of identical nodiffer meaning those which,having reference to the same object, mind only in being conceived by different minds, or by the same at different times,these slight differences being considered not as whose differences belonging to the notion itself. Notions proper but only extrinsic or accidental, not intrinsic or essential, tively relaarc are
"

notions

identical.

Such

notions

are

also called

or similar, cognate; and

the essential attributes


or

they are called reciprocating being all common, different languages, convertible. Thus signstaken from passion Comas and sympathy,""Conspectusand synopsis," and "Achromatic
"

stand often for similar or cognate notions colorless," of a definition, is unmerited "Grace as favor,"are for each

; and

the terms
tions, no-

convertible

the same essential marks. comprises Notions are absolutely different when there is no similarity. ly Strictis loosely there are in extreme none cases ; but the term applied the similarity is very slight and unimportant, when in "Blue and as different heavy,"or "Money and memory." Notions are relatively when and verse; dione they have at least one importantmark common
thus

"Saint"
from

differs from

"Sinner," "Wise

"from

"Unwise,"

"A

brightday"

"A

dark

day."

are Again, notions viewed intensively congruent, incongruent, and conflictive. Congruent notions are such as agree, or may be connected in thought. All identical notions also are congruent; many that are not identical ; as "Learning and virtue," "Beauty and riches," for though in themselves very different, "Magnanimity and stature;"

"

2.

The

doctrine Sec

Drobisch.

in general, Hamilton's,drawn is, Hamilton's Logic, pp. 150-158.

mainly from Esser,Krug, and

RELATION.

41

they can

be easily

combined.

notions Incongruent
"

are
"

such

as

not can-

unite in the Aristotle this there


an

same

object ; as
answer,

loud

ache." circle," A brighttooth-

To "Is happiness praiseworthy?" puts the question proper


for it has
are

is

no

incongruous jumble.
one

Notions

proper conflictive when"

no

meaning.

It is is

the difference

Virtue arid vice," negationof the other ; as " Beauty and deformity," Wealth and poverty." Such notions are said to be in opposition. and contrary. of two is principally kinds,contradictory Opposition denies or Contradictories are only two ; and to affirm or deny either, affirms the other ; both cannot cal be,but one must be ; they are reciproand not-blue," "Bine as "Walking and not-walking," negatives; and Gentile," "Jew "Simple and complex,""One and another,""A there are, by In case of contradictory and non-A," etc. opposition, of Excluded the principle ceivable. Middle, only two conflictive notions conContraries also are only two; These are disjunct notions. both be that neither exists; it may but while they cannot coexist, tertium be denied through the affirmation of something else, a may White and black," Running and lying," traries. etc.,are conquid. Thus ciich that involves
a
"

"

"

A be neither

color may

be

neither white

nor

black,but gray.

I may

but sitting. lying, firstdefine disparate In order to define contraries more must AVC exactly, notions. notions or contradictories, cannot These,like disjunct notion ; they exclude, be associated in one they theydeny each other, conflictive. contradictories contraries differ from were are as They said to do ; i.e., it may be that neither of two exists. But disparate

running nor

notions
notions

are

more

than

two.

They

constitute

a
"

series of co-ordinate

two between extremes White, graj, black ;" graduating ; as "Running, walking,standing, sitting, lying;" "Old, middle-aged, of a disparate Day, twilight, night." Now the two extremes young ;" "Wise and scries are contrary notions; e. g., "Day and night," foolish," Tall and short," Love and hate," Infinitely great and in the Categories, small." Aristotle, vi,14, says: Contraries infinitely
" "
"

"

are

those which It must be

in the

same

genus

are

most

distant from

each other.

and from

nothing of disparates Logic knows When abstract we involve matter. as contraries, they necessarily the matter of a notion, and consider only its form, it is impossible
observed that pure
to know

that

one

notion

of the other, A as negative between notions except contradiction. no opposition

unless one opposes another, and non-A. Therefore,pure

is the

mere

Logicknows

42

OF

CONCEPTS.

"

3.

We

note

one

other

distinction between

concepts viewed
or a

tensive in-

As

they are comprehended,


when

either involved
the

co-ordinate.

One
sum

concept involves another

latter forms

part

of the

of the former. the comprehension constituting and both Two concepts are co-ordinate when they are coexclusive, lower concept. in the same immediately comprehended These and Athenian. For example: Socrates involves both famous of the marks
arc

co-ordinate. and

But

Athenian

further involves

Greek;
"

and

Greek,

It is evident that these latter notions European,human. that not are proximate and immediate in Socrates," equally and that they are to each other some are given only through others, Thus in the relation of part and whole. thought evolves the simple of knowledge consists in this out of the complex ; and the perfecting the intension into clear and distinct consciousness unfolding progressive obscure and confused. of notions originally and of marks In speakingof concepts as involving, as parts of a The sense. whole, these words are used in a peculiar parts are not

European ;

partes

extra

partes,for each mark


when
I think

permeates and
as

informs and

the whole the brittle,

concept. Thus
whiteness

of chalk
arc

both

white

and the brittleness

thoughtto

coexist

throughout.
conceptsin

"
the

4. We

now

pass to

consideration

of the relations of

which, however, be it constantly kept in quantityof extension, thing. These relations are mind, is but a different aspect of the same
of three

and exclusion. intersection, inclusion, sorts, Inclusion.


extent

1st. Of

One
one

concept
coincides
cases

is included

in

another

when

the

sphere or

of the

with,or

is contained
:
or are

under,that

of the other.

There

are as

two

of inclusion

Coextension ; (a.) Subordination (b.)


a

when
as a

the
one or

coincide spheres
is contained
as an

common.

when genus,

under

the
a

as other,

under species

individual

under

species. spheres

2d. Of have
a

Intersection.

Two

concepts intersect when


a

their

common

part,and each
One

part not

common.

3d. Of their

Exclusion.
no

concept

is excluded

from

another

when
:

of exclusion There arc two cases part common. Co-ordination ; as when, though mutuallyexclusive, both arc (a.) have spheres

contained immediately Non-co-ordination (6.)


are

under

the

same

concept.
under the

as

when, while mutually exclusive, they


same

not

both

contained immediately

concept.

RELATION.

Let

us

now

restate

the

above, and

by symbolize

Euler's circular

in which the sphereof a concept is represented notation,2 by a circle ; linear notation,3 in which the extent of a con-" and also by Hamilton's by a horizontal line ; the relation of two or more, cept is represented under the other, and by their comparativeone ly by such lines standing by a vertical line greater or less extent ; affirmation beingexpressed horizontal ones two nection. joining by the absence of such con; negation,

Globe

( Coextension
Sphere

Inclusion.
Animal

Subordination

Protestants

Intersection,
Irish

Wonpon

Co-ordination Exclusion

Swortl

Spear

Evolution

Non-co-ordination

[ E

}(

0
Chance

Of
"

these relations there

are

only three

that call for

remark, special
will
ordination co-

and subordination, intersection,


once

co-ordination. the

Subordination
and

be treated at

; intersection under

topicDefinition ;

under

Division.

"

5.

When

one

concept is subordinate
the

to

or

contained

under

other, anmore

it differs from

higher concept by comprehending


of

relations of concepts the logical sensualizing of it in his Lettrcs d une made attributed to Euler, who use by circles is usually PrinccKse d'Allemagne, in a 1768. It is found, however, posthumous work of Christian Weise, Rector of Zittau, who died in 1708. Ploucquet employed the the triangle, instead of the circle. Drobisch's Logic, " 84 ; see square, and Haass
2

The

invention

of this method

"

also Thomson's
3

Outline, " 104;


modification
and

and
an

Hamilton's

Logic, pp.

133

and

180.

improvement of Lambert's linear notation as found to the circular notation. Organon,1764. It is to be preferred Both and therefore, those in intension, not represent only relations in extension, are though convenient and helpful, inadequate. See Hamilton's Logic, p. 670 sq.
is
a

This

in his Neucs

44

OF

CONCEPTS.

by extendingto fewer individuals. It is called a species. Thus sword is a species of weapon is a species of animal. ; man is contained Sword under iveapon ; it comprehends more marks, but it extends to fewer notion. The superior things ; it is the narrower is the more concept,sinc3 it contains under it more things, general
notion,and
sword
;

marks

and

hence

is called the genus. of


man.

Thus The

weapon

is the genus

of
to

animal

is the genus

notion notion.

animal

extends

merely relative terms ; for the genus may be contained under some ative higherconcept, and then relto this highergenus it is a species. is a species Thus weapon of the genus instrument. Of course contain the species under it may and then become lower concept, the genus of that lower species. some sword is a genus Thus under it the lower species sabre, containing A etc. relative to lower a genus rapier, concept that is alternately relative to some concepts,and a species higher concept,is called a
subaltern genus. A genus is a universal notion,or
of from
a

besides men ; it is the broader many things It is manifest that genus and species are

because universe,
This is the

it binds

parts into the unity of

whole.

plurality direct logical,


a

ad unum A versus. etymological, meaning of universe, E pluribus unum. It is called, then, means, strictly, universe, by way of eminence,the Logical A species, Whole.3 since it is but a part of this whole,is a particular notion. We should distinguish between the usual meaning of universe, that unlimited highest as prises genus which comall thingsin one, and universe considered as a limited genus which unites only some things. A universe or genus is usually present to the mind of a speaker, within which his thoughts revolve, and under which, often without in his statements. If we naming it,he is bringing sumed apprehendhis asand follow understand his thoughts we universe, may ; if not, confusion is inevitable from the ambiguities of language. Thus the word civil has many to meanings; it is opposed to natural," and to if to Now ecclesiastical," discourteous," so on. military," civil service be spoken of,and it is apprehendedthat the talk is under the tacitly implieduniverse of "the departments of government," it is intended then we understand that to exclude military In rude parlance and and confusion is avoided. we ecclesiastical,"
" " " " " " " " " " "

the

"

Universale

totum

quoddam

est ;

tur

totum

munus logicum, quia logicce

ut partcs. Diciquippe multa complectitur est de universis disputare." Burgersdjck.


"

RELATION.

45

know one what, in general, say, we must understand his particular statements. Both genera
and
are species

is

in about, talking

order to

called

and the arrangement of classes,

is called classification. The and species to genera thingsaccording has been somewhat pated anticipsychological classify process by which we and specialization, which in the account given of generalization and specification. with generification AVhen we terms are synonymous form all the think the similar to be the same, we a genus including and brute we similar things. Thus in contemplating man experience each what is similar ; we abstract from the shock of similarity ; we think it the same, or common to both ; we give it a name, and thus and the genus, animal, containing under it man establish the class, think the dissimilar to brute as species.On the other hand, when we of the thingsconform be diverse, a portion sidered. a we excluding species, animals the shock of Thus in contemplating we experience the quality which marks abstract from man rational, dissimilarity ; we Thus we affirm it of man and deny it of the rest. the diversity ; we of animals,the rational and the irrational, establish two or species
men

and

brutes.

These the species as Finally, partsmake up the genus as a whole. do marks, but are as are paries extra partes,for they do not coexist, and rubies are actually separable groups of things; c. g., diamonds it is possible of jewels. Consequently, to symbolize species rically, geometthe relations of concepts viewed in extension, by circles or lines, which is not practicable when they are viewed in intension.

"

6. It should

be

observed

that subordination

in the

of quantity

of intension. Also to involution in the quantity corresponds the term is applicable while the term generalization to either quantity, to the intensive relates to extension, and corresponds specification is a thinking For determination term determination. in, a synthesis, concretion of marks, and this, since it throws out things, specifies a a fying concept. Determination,then,restricts the denotation by ampliextension the and terminates only in individualization. connotation,

"

7.

Many concepts are


to

related to each other

as

correlatives.

cording Ac-

the

Law

things. We
out
an

know

of the dark ; up

knowledge always includes two Relativity, heat by transition from cold; lightby passing There is no such thingas by contrast to down.
of

absolute

knowledgeof

any

one

; property

we

could not

know

mo-

46

OF

CONCEPTS.

knowing rest ; our firstparents had no knowledge of good until it was bought dear by knowing ill." We of one member be thinking of the couplethan of the other, more may of the heat rather than of the cold, of the straight line rather than of the crooked ; but if either exists, the other always coexists with it in consciousness. is the explicit, The one the other the implicit, subject of the thought. This would seem double names to occasion verse throughoutall the uniof things, and if complete, would contain no language, single but consist of couples. Accordinglywe find a great many .names, Correlative Terms," in each of which,if called couples, specifically either member be expressed, the other is implied;as "Parent and and effect," "Ruler and subject," "Cause child," "Heavy and light," and species," "Rich and poor," "Genus "Positive and negative." The last example, Positive and negative," To affirm and to and the generalization of all or deny,"is probablythe basis, origin, the rest. of the two less of a negative has usually One more or where names have not been adopted for both character ; and in cases for every exists in thought as a negative.Hence one correlatives, itive posframed be 'concrete name a as relative corcorresponding negative may such the prefixes to it by attaching a as negativeparticle,
tion if
we were

debarred

from

"

"

"

"

un-,

and the suffix -less; in-, and


"

"

as

Conscious

and
"

unconscious," Temperate
and

"

Godly intemperate,"
Another mode
in which

and

godless," A

non-A."

"

8.

concepts are
is
a

related is

expressed by
and

the old and

almost

disused

terms logical
or

First Intention

Second

thingsformed of the mind to the object. It denotes by the first or direct application things. The concepts which we have been usingas illustrations Socrates is regardedby the mind The object all firstintentions. are as Greek,man, animal,body, etc. A mental state may be thought as
concept
a

Intention.6 A firstnotion

intention

of

consciousness. a feeling, a a sensation, smell, A second notion


or

All these

are

firstintentions

intention is

from concept generalized


or

firstintentions. is the
as

It denotes

first intentions the mind

concepts

of

It tilings.

conceptionunder

which Thus

regardsits

first intentions
to man,

related to each other.

the relation of animal

and

of

4 6

See Bain's In-tendere.

gentiam animae

Logic, p. 2 and p. 55. "Ego dico intentionem nil aliud esse quam De in alicujus rei consideratione." Zabarella,
"

attentionem

ac

dili-

Rcb. Nat. p. 871.

48

OF

CONCEPTS.

V. DEFINITION.

"

1. In

order

to

give to

them into a systematize must consider what we think,i. e., what is comwe prehended operation. First, consider in thought. Secondly, of what and must we how many i.e., to what and how many the thingswe think, objects The thought extends. comprehension of thought is developedby its extension, Definition; by Division. Our thoughtsby this means rendered distinct, the internal or intensive distinctness being seare cured by definition ; the external or extensive distinctness, by division. of thought (ii, Thus we approximateperfection " 4). It has already been stated that definition is the explication of the marks of a thought or concept (iii, essential and original " 1). Thus, Man is defined as rational, to repeatthe example, existing. sentient, living, It is

and to thoughts scientific precision, scientific whole, we must perform a double


our

however, that this mode manifest,


cases

of statement

is aAvk-

ward, and
animal
common

in most

impracticable. then,that Observing,


other
"

the that this

tion noarc tion, no-

involves
man

all the marks sentient, organized, existing,


we concepts,

to

with

substitute for them animal."

and

define

summarily,
in this

Man

is rational and is

The

mark

not included rational, to


man

summation,
that connote

as distinctive, belonging

alone of all the notions

animal.

then,consists
one

of two, and and

only two,

essential and

definition, logical marks, original

of which

is

common

the other distinctive.

the marks which the implicitly it follows that they are reciprocating definition contains explicitly, or convertible concepts (iv, " 1), Either may be substituted for the is a polygon of three sides," and other. A polyThus, A triangle gon of three sides is a triangle." Or, as "Every rectilineal figure so having a common point," Every may be divided into triangles rectilineal figure a may be divided into polygonsof three sides having common point." of marks,are no as plurality containing incapable Simple notions, for of definition. The notion being, example, having only one mark, is an indefinable, and no differential or distinctive element, an existing, indefinite notion. It is distinguishable a mere only from nothing,
" " "

Since

the

notion

defined contains

DEFINITION.

49

Indeed,a simplenotion, having having no content. empty negation but one mark, cannot in strictness be called a concept. On the other since practically \ve defined, hand, an individual cannot be logically
cannot

form

notion
common

which

it has in
can

marks all the essentialand original comprising vidual with any other notion or thing. An indi-

only be described.
that
to the above definition, according

"

2. It is obvious
to primarily

account,

relates

the intension

of

concept. The

gicians, scholastic lo-

however,viewed
and

it in the extensive usual with


us.

nomenclature

are

most

consists of the The

proximategenus
is that class under

and their view quantity, nition Accordingto them, a defiand the specific difference.

which the notion defined is proximategenus is the proximategenus to the Thus animal contained. immediately the difference is that which distinguishes The specific concept man. Thus rational is of that genus. notion defined from all other species difference that distinguishes the specific from all other species man Let it be recontained under animal, as beasts, etc. marked birds, fishes, since it distinguishes that rational is also the genericdifference, the notion
man

from
et

the genus

animal.

Such

lastic is the scho-

"Snow Other examples are: differentiam. is frozen (^specific mist" ( proximategenus); "Logic difference) is the science (=p. g.)of the necessary forms of thought" (rrs. d.) ; the of is men's conduct "Eloquence influencing (=p. g.)by power of speech" ( s. d.). means These two elements, the proximate difference, genus and the specific for the proximate make up the whole intension of every notion, genus all the marks common connotes to the several species.But to make it is further necessary to define the genus. the explication complete, We This done, the same necessity again appears, and is met. ceed proin this manner until we notion as the highest and reach a simple final genus, which cannot be defined. For example: definition per genus
== =

A
A

A
An An

mammal (=g.). ( d.) flesh-eating its young is a vertebrate (=g.) suckling mammal ( d.). vertebrate is an animal (=g.) havingan internal skeleton (=d.). animal is a sentient ( d.)organism( ".)" organismis a living (=d.)being( g.)carnivore is a
"

"

"

Here

connotation, A carnivore is flesh-eating, internal-skeletoned, sentient, living, suck-giving, existing."


we

have

the whole

"

50

0?

CONCEPTS.

"
common

3.

Concepts often
part,and
each

intersect ; that
a

two is,

concepts often

have

not
are

There are Irish Protestants part not common. and there are Protestants Irish not Protestants, ; also there are Irish. Some blacls. not; some heavy things thingsare heavy,some is which is conThe not. a common tained black,some species part under each
a or

either of the total conceptsas

words, whenever
of two Now may
to

certain group of thingsmay the group being a common part. genera, these genera intersect, the and the of the two portions a definition, genus difference,
as a

In other genus. be referred to either


a

be each viewed

concept in extension.
to be the

If so,

they will

be

seen

and the notion defined intersect,

common

part. Thus

the

notion
^_^^_^^

rational intersects the notion


common

animal
are

man,

being both, is
that
are are

the
ra-

part; while there


are as

animals

not

/
7?

A
A

tional,as the beasts of the field ; and

there

rational

I Aft

\) j
as a

beingsthat
think of
man

not
an we

we animals,as angels. Ordinarily, him under this notion animal, bringing use

proximate genus
to

; and

the mark
him

rational

as

specific
animals. the
is
a

difference But it is

characterize

him,
as

to to

mark

off from
to

other

perfectly competent
to
use

refer him

rational
;

being as
"

genus, and
rational the

animal

the differential mark

thus, Man

being ("p. g.)having animal nature" (" s. d.). Therefore of a definition are convertible in thought, and it depends two portions wholly upon the use made of them in thought as to which
which the difference.

should be called the genus, and


is
a

So, if

watch

it may be thought either as a sort of portportable timepiece, able of thing or as a kind of timepiece ; if a concept is a bundle marks, it may be thought either as a kind of bundle or as meaning that kind of marks which are bundled together. Aristotle observes that specific difference is of the nature of genus.

"

4. Since

definition is the

of explication

all the connotation

of

of its definitions is the perfection of a science. the perfection thought, In studyinga preparedscience, we begin with the definitions ; but in constructing end with the definitions. True, in its we a science, of provisional, make constant we use early necessarily imperfect stages, of scisubstitutes ; and so it was that Socrates, at the birth ence,1 presiding for and analyzing definitions. spent his whole life in searching

The

mother method

of of

his this,

was Phaenarete, Socrates, /mta, a truth by questioning was eliciting

midwife

and, in allusion

to

called the maieutic method.

DEFINITION.

51

But

as

improved, science progresses, its definitions are modified, gradually the science real ; and when they are finally and made perfected,

perfected. definition three kinds of logical This givesoccasion to distinguish and the genetic.This the nominal,the real, per genus et differentiam, it treats of the distinction is grounded on the matter ; pure Logic, as kinds of definition. Consequently, if we form, only, does not know each of these three kinds of definition exhibits consider the form only, difference. When look into the we the proximategenus and specific the discover such variations and imperfections as justify matter, we
is

above

distribution. definitions express the meaning of a word as all the marks and used, not explicating understood much requires
are

Nominal

it is popularly
common (since

usage

less than

exact

and freely science), ing employ-

or derivative, Thus, "A pension accidental, peculiar. "A violin is a musical instrument is an allowance for past services;" with a bow ;" The east is where the and played having four strings

those that

"

sun

rises." The
mere

definitions givenby the dictionaries are


"

mostlynominal.
a or rule, decree,

of synonyms, heaping-together

as

Law
"

is

Centaur" "bullas means or statute," merelygivingthe etymology,' nitions defiare no goader," though often called nominal definitions, obviously at all. The imperfect, definitions, provisional spokenof above in order to prepare the way for real ones, are nominal as preliminary,

definitions.8
Real definition is concerned essential marks with the real nature
of

things ;

it

folds un-

form, and these only, original defined. It is that are and adds none not implied in the subject ence. Such are the perfected definitions of a scitherefore strictly analytic. actness An example can hardlybe found. The exunexceptionable of mathematical thought gives approximations.Thus, A from is everywhere whose periphery circle is a planefigure equidistant
all the
"

in their

the centre." real definition

In

the distinction between practice descried. cannot alwaysbe clearly

the nominal

and

the

into They graduate

in which there is no dence eviis one to Aristotle, definition, according taur. of the existence of the object to which the definition is applicable ; as a centotle Aristhe recent ones, differ widelyfrom Subsequent logicians, especially it from and the its meaning and from each other in stating distinguishing of the best authorities, and real. The statement in the text agrees with some It is a pointof littleimportance. On to accord best with popular usage. seems the whole subject, note C. Hansel's Appendix to Aldrich, see
2

The

nominal

52

OF

CONCEPTS.

each and

other.

The

that requisite

the latter shall consist of the essential

relates marks, which constitutes the distinction, evidently original to matter, not at all to form. Hence, as said, exclusively pure Logic knows nothingof this distinction. itself with the rise or proA geneticor causal definition concerns duction but as becoming. Thus, of a thing; considers it, not as being, is a solid generated A cone by the revolution of an angleabout one but made, this defiof its sides.7' The notion defined not beinggiven, nition is synthetic. called definitions definitions are sometimes,though improperly, Logical definitions a posteriori. A them from to distinguish a priori, the consequences the conditions, definition a posteriori or generalizes the marks connoted but the things of a concept, or explicates, not
"

denoted. which

E. g., " Malaria is that which induces fever ;" " Mind desires and wills." Obviouslythese knows and feels, and hence all,
are

is that
are

not

definitions at second

also

called

The pseudo-definitions.

tivity, example,which merely unfolds the denotation of mental acdivision. a logical is,of course, strictly of the marks. evolves only some An An unqualified, Explication, A Description is a series of explications. givesmarks or Exposition in the thing. It deals, characteristics as concrete therefore, only with of its marks, the selection being the individual, giving any number governedmerelyby the purpose.

"

5. A

few

RULES, practical

some

of them

deduced

from

the above here.

and useful in forming good definitions, are principles, be definition A good must If the genus is not proximate, the 1st. Adequate.
"

admissible

definition is too
of the

wide.
it is too
a

If the
narrow.

difference is not E. g.,


" "

common

to

all members

class,

Man

is

rational

(toonarrow). A praying animal (" 1). convertibility but 2d. Not negative. A definition ought to tell what a thing is, E. g., Parallels are lines that do not tell merelywhat it is not. some meet ments opposed to pain." Negative state;" Pleasure is the feeling
" "

being" (too wide) ; or, "is convenient test of adequacy is

serve

to render

notion
not

and clear,
a

are

valuable
distinct

as

precursory

to

but they do definition, the notion

render is

notion

ever, how(ii, " 3). If,

defined

then gentile, want, etc., defined


an

as shadow, freedom, essentially negative, E. g., Cuvier, its definition is properly negative. as

invertebrate

"An

animal destitute of

an

internal skeleton."

DEFINITION.

53
,

3d. Not

tautological. A definition must


a

not

contain the

name

of the

nor thing defined,


i

derivative
a

nor

thingby in a circle or reciprocally, a\or, by the ancients, diallelon ("m, It is a sort of logical of the E. g., Life is the sum seesaw. X//Aa"'). vital functions ;" A cause is the concurrence that produces an fect." ef" " " "

term, for this is to define

correlative a nor synonymous itself. This vice is called defining

Here board
is
a

the fault is immediate. and plank,"


of
a
"

It may
a

be mediate.

thin

plank is
"A

thin

board
who

;"

"

E. g., " A Law is the

will expressed

and ruler,"

ruler is

one

gives laws."

There

is

similar vice in

4th. Precise. E. g., " Oats is a but in Scotland

called by the same names. reasoning, It must contain nothing unessential or superfluous. grainwhich in England is generally given to horses, supports the people" (Dr. Johnson). This specific

difference is unessential.
does
not
nor fail,

So,

"

Man

is

risible animal."

This

nition defi-

but it offends strictly logical purity, scientific system or arrangement of thoughts. Again, A triangle against is a figure having three sides and three angles."Here is superfluity. Derivatives should be excluded as superfluous, for they are contained in their originals. E. g., The circumference of a already circle is a curved line returning etc. ing Every line returnupon itself," curved is superfluous. upon itself is a curved line ; hence 5th. Perspicuous. It should be intelligible, and brief. We literal, define only to make distinct ; hence terms more a thought more fused conthan the one defined violate perspicuity. is E. g., Net-work with interstices anythingreticulated or decussated at equaldistances, between the intersections" (Dr. Johnson). "The soul is the firstenlife potenor telechy body possessing tially energy of a natural organized is This Leibnitz. all obscure, (Aristotle). tive Again, figurasays should d o cate indibe excluded. for not language instance, Tropes, what a thing is, but onlysomething similar. E. g., The Divine Nature is a circle whose centre is everywhereand the circumference have ceased nowhere." metaphorical Many terms, however, originally in to be so. These may must be used, and sometimes be, especially but extreme ity brevmental is certainly science. Finally, a merit, brevity
" " " " " " "

violate

may

leave

matter

more

obscure

than needless

prolixity.3

See Hamilton's

Logic, pp.
121-123.

341-349.

His treatment
notes

is borrowed

almost

entirely

Krug, Logic, "" Appendix, note C. Aristotle


ch.
'

from

See also Mansel's

in

Aldrich, pp. 38-43, and


Post. bk. ii. See especially

discusses Definition in Anal

x.

54:

OF

CONCEPTS.

"
1. 2. 3. 4. 5.

6.

V.

and criticisethe following : Analyze, classify, line is without breadth. Euclid. A length Science is classified knowledge. is a machine for raising water. A pump A beggaris a person who asks alms. Motion is the translation of matter through space. Words are signsof thoughts. is formed ameter. A spheroid about its diby the revolution of an ellipse
6. Praxis.
"

8.

is the Philosophy
sun

science

of

9. The

is the orb

givingthe

principles. of day. light


lines to each other. mathematical
ideas
as

10. An 11.

angleis the inclination of two of Philosophyis the recognition


the world.
"

ing constitut-

OJcen.

12.

The
and

soul is the

by principle
"

which

we

live, feel, move,

perceive,

understand.
is

Aristotle.
is the conscious

13. Mind 14. Mind


15.

substance;or, spiritual
"

subject.

is the unextended.

Bain. concentrated.

Attention

is consciousness

16.

17.
18. 19. 20.
21.

22.
23.

24. 25. 26.

ternal Perceptionis the faculty by which we immediatelycognizeexobjects. A dragon is a serpentbreathing flame. is a conspectusof the chief points. A synopsis Logic is the art of reasoning. of the understanding Logic is the light-house (pharusintellectus) A pension is an allowance made to any one without an equivalent. In England it is generally understood to mean pay given to a for treason to his country. Dr. Johnson. state-hireling Green is a color compounded of blue and yellow. in the wrong Dirt is matter place. Lord Palmerston. Truth is the agreement of a cognition with its object. of dog. A spaniel is a species A whale is a fish inhabiting oil as the polarseas, and furnishing
" "

an

article of

commerce.

27. Animal
28. 29. 30. 31.

etc. fish, birds, reptiles, denotingmen, beasts, Wealth is thingsuseful, necessary, and agreeable. Pain is a disagreeable affection of mind or body. A feeling is a mental affection involving cither pleasure or pain. in recognizing we Beauty is the feeling unity amidst experience variety.

is the genus

56

OF

CONCEPTS.

VI. DIVISION.

" 1. The correlative of Definition is Division. As to the intension, or depth,of a concept,so primarily
to primarily

definition relates division relates

evolves or explicates marks ; a division explicates evolves subordinate conceptsor things. or nition the comprehension the sphere. By defiThe one develops ; the other, the denotation. the connotation is analyzed By ; by division, distinct definition the notion is rendered internally or intensively ; by Thus division the notion is rendered externally distinct. or extensively is defined by unfoldingits connoted the notion man parts,rational its denoted and animal ; it is divided by unfolding as logician parts, be assured and non-logician.Only by division, can we says Aristotle, that nothinghas been omitted from the definition of a thing.
its
or breadth. extension,

A definition

"
in
are or

2. As

it is preliminary, which
:

needful to thinks

under

the mind

kinds two distinguish the objects to presented

of wholes it.

They

as

follows

1st. The

Whole. This is of two sorts: Qualitative or Logical The comprehensive, intensive whole, whose or characteristic, (a.) parts are marks evolved by Definition. extensive The or whole, whose parts are universal, generic, (b.) or thingsevolved by Division. species Mathematical
or

2d. The

Whole. Quantitative

Of two

sorts

The integral whole. (a.) The collective whole.1 (b.)

whole,with which we are at present more particularly logical creation of thought. It is qualitaa concerned, is purelysubjective, tive of a bundle of qualities or marks, ; i.e., it is the concept consisting and containing other concepts. These its parts are separable onlyby The the There two mental abstraction. are first, intensive species. whose in the old Logic a metaphysical whole (called whole), parts are
The

we

also Logic commonly distinguishes shall find need only for the above.

the

others Whole, and some Physical See Hamilton's Logic, pp. 142-144.

; but

DIVISION.

57

marks,has
extensive

been

considered

under the

whole, whose

parts are
is
an

the previous topic. The second, kinds or thingsunfolded by logical


us.

is more division,

before especially whole

mathematical viewed
as a

either objective or individual,


of consisting

tive, subjec-

and quantity,

These

can

be

evolved
must

which partition, Such

only by be clearly from distinguished


nor one

the whole

separable. parts actually cut asunder,i.e., by being

parts are
be

neither marks
is

the integral whole First, may

division. logical kinds. This whole is of two species. in which its parts originate. They

homogeneous,as a polygon severed into similar triangles; or the human of head,trunk, and limbs. as heterogeneous, body,consisting of dissection. A sword,which divides Anatomy is a science of partition, into sabre,rapier, ondly, Secetc. etc.,is parted into hilt and blade, the collective whole is an of similar parts, one aggregation by the parts. Such are the notions of an army, a forest, originated a These are formed by the repetition town. of of the notions a soldier,
a

tree,a house.
class

We

must

not

confound

the notion

army,

which

is

generalor
taken
as a

notion,a logical whole, with the notion an army collective notion, individual thingformed by a collection an

of other individual

things.

ordinate subform we alreadyseen how by specialization Since pure Logic considers onlythe form, groups, or species. each genus or universal whole can contain under it only two species, marked with A and non-A. For A beinga generic i. e., a difference,
mark
not

"

3. It has

been

found
we

in the genus
a

or

divisum,but found

in

some

of its

members,

know

without priori,
are

that thought,

the members

of any research into the matter exclusive of each other and exhaustive

of the divisum.

This is division

by dichotomy,and
are

the members and

are

contradictories.
or are

For illustration: animals

rational

irrational,

vertebrate and invertebrate ;

and

anglesare rightand oblique j the oblique Jews acute and obtuse; the ancients were Greeks and barbarians, or ly, and free. The process viewed intensiveor bond Gentiles,
in marks, is thinking

as

called determination

; viewed

is called specification. as extensively, establishing species, In relation to each other, as the two species are co-ordinate, being of equal rank in respectof the divisnm ; but we remark that either may be of indefinitely greater breadth than the other. The negative sence of the dichotomy is characterized by the abmember of the mark A, mark non-A. or, in other words, by the negative

58

OF

CONCEPTS.

Hence
or

we

have

class peculiar
some

infinitated almost

concepts. In

and everything, E. g., Ungodly,unhappy,apathy, blindness, senseless, dark, positive. freedom, shadow, atheist, idle, sober,dead, etc. cold,infinite,

privative, concepts called negative, their sphereis very wide, denoting cases almost nothing connotingvery little,
of

division, that pursued beyond a single is regarded and subdivided into when a species as a subaltern genus is, to select some lower species,then it is requisite at the outset one erence in refof the original mark divisum as a ground or principle on or This generic to which the several divisions shall be made. mark chosen is called the ground of division, so fundamcntum diviMankind select his religious sionis. For example,in dividing we character or creed as the ground of division, and, subdividing upon the same thus : obtain a logical we series, principle,

"

4. When

the process

is

"

"

Mankind
I

Theists
I

Atheists

Monotheists

Polytheists
Antichristians

Christians

Papists
Jesuits

Protestants

Non-Jesuits

[etc.
The number
of distinct forms

etc.]
in which this of mark, the principle

thingsto be divided will determine the extent has respectto the matter of the series. This procedure obviously of thought, add that, and is not strictly if it is propure Logic. We posed the i.e., one to establish a real division, of true nature unfolding the thingscontained under the divisum, scientific, one rigidly or, in short, of division an essential and it is requisite to select as a principle of the divisum,and to adhere to it throughout. So mark original in fact, but it is, in an logical perfection requires, rarely practicable
division, appears
in the extended And series. this

suggeststhat the distinction made

between

nominal

and

real definition may well be carried out relative to division. A nominal artificial division would be one or made for some transient purpose

DIVISION.

59

or

to attain
or

division ;

end ; or one tentative and precursory practical such as the and useful, one popularly accepted observed
A
on or

to

real

hundreds of"

that may be conversation.

real

every page, and in every few scientificdivision would be one

minutes

to proposing*

divide notions and

to their true and essential nature, thingsaccording in order to attain correct objective knowledge of thingsas they are. Such division developsnatural kinds,and is to be looked for in the
more

refined sciences. and

The

Linna?an

artificial divisions of flora

were

precursory and more

tentative ; those

of Jussieu's natural

system

are

real

scientific. rigidly

"

5.

In divisions not

but having respect to purelylogical,

the matter,

than dichotomous happens that we have those more ; we have a trichotomy(rp/%a, threefold ; -ip"Eir, to cut), or a may poE. "Doctrines arises hurtful." This are harmless, lytomy. helpful, g., from two Either it is an abbreviation by which series of causes. a subordinate species is condensed co-ordinate into one statement, as, Mankind and obtuse ;"or, are "Angles are acute, right, Christians, and atheists ;" or, Plants are Jews, Mohammedans, polytheists, endogens,exogens, acrogens." Or it arises from the lack of a sharp
" "

it often

definition of

our

concepts. There

is between

very

many

of

our

to assignclearly thoughts a wide border-land which it is impossible to either, tertium quid, third species which it is neca a essary constituting to

insert in order to exhaust

the divisum.

Thus

we

have

our

hours divided, with twenty-four

reference to their
"

light, into day, twilight,


"

and

night. So ;"
"

want

petence, White, gray, black ;" Riches,comof these old,"etc. For many Young, middle-aged, we

have

mediate
and weak

we species

have

no

names,

as

between

side and

well ;

strong

We A

etc. ; long and short ; wise and foolish, have remarked that in a strictly division the two logical

bers, mem-

of that universe non-A, are contradictories ; no member be both, nor the be neither. In a trichotomy can can or a polytorny members parate notions. river,are disare Thus, brook, creek, disparate

and

notions
two extremes

contained
of such
a

under

the

genus
brook

streams

of
may

water.

The

as division,

and

are river,

traries. conlogical

A it may

thing of

this genus

cannot

be

both, but

be neither ;

be the tertium

quid.
seems

Let it be also noticed that in which a notion cases many have been originally of its co-ordinate notion a mere negative into thought it a
so character, positive

to

has had

that either may

be

now

thought

CO

OF

CONCEPTS.

negative ; or perhapsboth are really positive, white and exists. and no mere Thus, black, the mere negative is dark. So true and untrue or false; happy and negative unhappy ; honor and dishonor ; temperate and intemperate, which last has become So protestant. So also pleasureand pain. Plato taught inverted. is merely the absence that pleasure nists or negationof pain; the Hedoboth arc positive. taughtthe reverse Also, ; but unquestionably it was that evil is the mere taughtanciently negationof good ; and to-daythere are those who hold that good is the absence of evil ; but both good and evil are positive, and in this case there is no intermediate ferent ground. Actions are either good or bad ; there are no indifas

and" positive

the other

as

"

actions.
of one, and only one, strictly a polytomous division admits Finally, notion. or Thus, ** Some men lend,some borrow, privative negative do both, others do neither;" "Plants some are monocotyledonous, flowerless." and The intermediate or dicotyledonous, acotyledonous the undefined or indifferent part, often takes this ground,well named Men are negativecharacter ; as positively lazy, very industrious,
"

and

neither the

one

nor

the other."

"

6. The

importanceof
well be

the correlative processes

of definition and

respectively in the balance of which of analysis and synthesis, lies the perfection of knowledge.3 Such is the excellency of definition and distribution," old logician, that almost they alone do suffice for the absolute says an of any art ; therefore, in Phcedro the wise Socrates, putting-down saith that if he find any man who can he Platonis, cunninglydivide, will follow his stepsand admire him for a god." shall do well, We directions. then,to observe the following practical first present for forming divisions this the account From we given, instances of the objectsdenoted CANON : Assemble representative mark as a principle by the divisum,and, having fixed upon a generic this principle for a select a mark of division, immediatelyinvolving the specific difference ; then divide the denotation by affirming specific which it determines, difference of the species denying it of all other divisions pursue a similar course, contained objects. In subsequent
are
" "

division cannot

overrated.

They

the reflex

and thoroughlydivided, have attained we defined, adequately this and a process exhausts its complete knowledge of its characters and kinds, See Kant's Logik," 98. content.
8

When

notion is

DIVISION.

61

in involving

each

new

and, of
To this

course,
we

difference the one preceding, immediately specific of division. the original principle
now

canon

RULES, useful append the following


:

as

ther fur-

guide to
1st. Each
same

correct

division

division

throughouta
should be
an

series should essential and

be

which principle,

governedby importantmark

the
of

the firstdivisum. The rise to intervention the


of
a

different groundof division in the series called "Cross


and the

gives
are

fault logical

division." This

Thus:
is
an

"Men

Americans, negroes, Europeans, series in which the ground of

pagans."

abbreviated

firstdivision is members

geographical;the
cross evidently or

second,color
common

the

third,religion.The
man

each other; a overlap when vice,


more

may

bo

all of the last three. and

This

very

is detected concealed, is to

the division tested

rect, trichotomyor polytomy,if cormember be reduced to a dichotomy by taking any one as may and including all the others under its negative. If this can positive be done with each member, without cutting any one, the division is mineral." sound. Thus, "Physical substance is animal, vegetable,

by dichotomy.

That

say, any

Tested:

"P.

S. is A
or

and

non-A

(=A
the
"

M) ;"

is "M

and non-M

(=V+M);" (=A+V)."

or

is "V

and

non-V

will clearly demonstrate following The religious sects of Great Britain and Dissenting." if we The principle selected must be essential, would attain scientific knowledge. It must be important, determining many
we

to applied that it is logically vicious: are Catholic, Calvinist, pal, Episcotest

This

to

real,
other The

if attributes,

would

evolve

an

extended

and

valuable series.

division is made determines its ground. purpose with which an artificial into horsemen In civilaffairs it would be useless and absurd to divide men and

in a gramaffairs it is important.Words military mar in phabeti aldivided relations to a are dictionary, according syntactical ; manual will divide plants Medical botany and the florist's to and both deviate from Jussieu. We sort our books by size, differently, for show. fit our shelves ; by subjects, for handy reference ; by binding,

footmen;

but in

2d.

members Dividing
one

must,

as

the whole divisum. equal parts,


"

No

must

exhaust

the divisum

; as,

Mankind

are

rational beings
"

and

more

ments politicians." Togetherthey must exhaust it ; as, GovernThis is insufficient and democracies." are monarchies,oligarchies, not ; there are other forms of government. Togetherthey must than exhaust it ; as, Vertebrates are quadrumana, bimana,quad"

62

OF

CONCEPTS.

in man. Leibnitz overlap communicant writers callsthis last fault So, Imaginative species." and writers of tales." Again, Sciences are deductive are dramatists, poets, and inductive." These species ter are communicant, since the latof deduction. There is no science non-deductive. makes large use
" " "

and bipeds."Bipeds and rnpeds,

bimana

3d. Divisio Each

ne

sal turn. faciat

from its own must directly proximategenus. species emerge and immediate overlook its not must overleap Thought parts and from the divisum This the theoryrequires to remote species. spring ; for the sake of brevity, such a saltus is allowed, but practically, error. thought passingthrough intermediate steps to guard against Thus
of
we

may

say that "Mathematics

treats

of

as well infinitesimals,
"

as

This lastmember equals nonquantity." magnitudesof assignable is far from being infinitesimals." The genus "mathematical subjects" to these species.2 proximate

"
out

7. Praxis.

Are

the If

neither? or Divisions, the defects.


If

sixteen examples Partitions or following If not, point are Divisions, they correct?
to dichotomous

reduce correct,

statement.

1.
2. 3.

4.

and negative. are affirmative, Propositions hypothetical, or Thought is by conception, by judgment,or by reasoning. The mental faculties are sensation, ory, memperception, imagination, and judgment. Is the year or are the seasons divided into spring, tumn, ausummer, and winter ? flower consists of

5. A

6.

and the corolla, calyx, stamens, and pistil; and stigma. consists of ovary, style, pistil Literature consists of writings sical, clashistorical, poetical, religious,
and
current.

7. Matter is
8.

and aeriform. What is the principle ? solid, liquid, and Turanian. are Aryan, Semitic, Languages
are figures more

9. Rectilineal

and triangles, rectangles, parallelograms,

of figures

than four

sides.

See Hamilton's 134-137.


was

Lect. Logic, See


a

xxv.

His doctrine is drawn

mostlyfrom
Drobisch's

Esser's

Logic, ""

also Thomson's method

Outline, "
bk.

55 ; and

Logic,
of Definitions,

"

119.

Division which

favorite
censures

with Plato for the demonstration

ch. v), and teaches that its ii, obtained. chief use is to test definitions when the Among the later Peripatetics have drawn chiefly from Bcethius's Modern method was esteemed. more logicians work De Divisione. Cf. Cic. Top. 10. See Kant's Logic, and Quintil. also ch. vi, v, " 113; and Trend,Elem. " 58. Aristotle

Post. (Anal.

64

OF

CONCEPTS.

VII.

COMPLETE

SYSTEM.
of

" 1. In concludingthis generaldivision it is needful to gatherup into one Concept,


and

of Logic treating

the

some a

of the results obtained,

this will
of
a

giveoccasion topic.
We

to

remark

few

additional

The

notion

series of related concepts has

been

points. pecially esanticipated,

of the form proceed to examine such a series when it is evolved into a completesystem. will present As preliminary, and at the risk of some we repetition, scheme of the two quantities and remark : upon the following

under

the last

"i f Existing g
2. ?

I Existing, living
sentient living, j Existing,

Men. Minerals, Plants, ~j p Brutes, I "" Men. Plants,Brutes, Men. Brutes, j%

i. rational sentient, living, Existing,


most

Men.

The
as

obvious

pointhere
one

illustrated is the law of


and diminishes, vice
versa

thoughtthat
; that the

intension

extension increases,

maximum

of either

is the minimum

of the other ; that these two

of thought are in quantities In ascending the series we

inverse ratio. think


out

marks

and

think

in

the Now

same

act.

For

this

act,on

concept is this thinkingout marks, is stractio the intensive side, about,


a we

each

mark

thrown

thingsin brought in.

; for in it abstract

these from

away a complement of marks, and thus at least one other which passes out of consciousness. from sentient, existing, living,
rational
"

draw

Thus,
then

we

first abstract

from existing living. On the side of extension there is, for each abstraction, a generalization. In thinkingout rational, think i. the marks in brntes, e., we existing, are as sentient, living, belongingto brutes in common generalized

from living, existing,

sentient ; then

with

men;

and

these two

classes of

are things

united

in the

more

class which we or term animal. Hence, generalization general generic is also generification. It follows that abstraction and generalization what might be called directly correlatives ; directly are parallel, parallel in the different quantities. direction as moving in the same In descending think in marks and think out things the series, we

in the

same

act.

This

act, on

the intensive

is determination, side,

COMPLETE

SYSTEM.

05

because the

down and fixes spein a mark, while it narrows bringing cifically of of a lso tains atsmaller class the limit a or things, definitely which in Determination,, fuller, deeperknowledge of them.

the scheme

is the inverse correlative of abstraction, which descends, there is for each determination a On the side of extension, ascends. think In thinking in sentient, into existing, we living, specialization.
out

by and we a subordinate, special, vegetation, excluding is also specification. animal. Hence or class, specialization specific is the inverse correlative of generalizawhich descends, tion, Specialization, determination and specialization which ascends. Finally, may correlatives. be called also parallel parallel, directly or, simply, the side abstraction is analyIt should also be observed that on one sis, the other side, the order is and determination synthesis on ; while, is analysis, is synthesis. and generalization reversed, specialization that is analysis in one of thought, is Hence the movement quantity in the other. The neglectof this distinction by logical synthesis
plants,i. c., the
notion

into organism is specialized

animal

have

established

authors has led to much

confusion

in the

use

of these terms.

of Aristotle, of Porphyryto the Categories ten writisagogue in the third century,was of the as a detailed explanation designed its doctrine subsequent relations of genera and species.From cians logiconstructed a scheme which, because of the form it presented, called by the Latins the tree of Porphyry (arbor was Porpyriana), and by the Greeks the ladder (xA/juaQ.1 It exhibits a hierarchy of concepts scheme presents a complete representing system. The following the device in a modified form, with the same used: matter already

"

2. The

as Aquinas,see Mansel's Aldrich, p. it, given by Thomas 32. The isagogue will be found appended to Owen's translation of the Organon of Aristotle (Bonn'sed.) to St. Hilaire's LogiqueD'Aristote, also prefixed ; and traduite (Paris, from the 1844). The doctrine of the isagogueis drawn largely of Plato. and the sometimes is in almost writings, expressed very words,

For

delineation of

66

OF

CONCEPTS.

"
and

3. It is evident

that the

from mind, rising

individuals to

classes,

generalizations forming wider and wider classes or the marks connoted, at last must genera, at each step diminishing of widest reach a notion generality, connotingbut one mark, above rise ; and the process necessarily ceases. which, of course, it cannot notion is called fined the This highest, widest summum genus,"and is dethe genus that cannot become a as species.It is represented in it only the noin the above scheme by Being or Thing, containing tion and containing under it all things. This is a simple existing, be defined, notion and cannot not being referable to a genus. fixed by The Aristotelian logicians consider the summa as genera to the ten Categories or nature, and ten in number, corresponding of Aristotle.2 By the Categories, Aristotle means, metaphysically, Predicaments of objective of the modes a posteriori or real a classification classification of the most a existence;logically, generalterms that of any subject whatever. trated illusbe predicated can They are as follows, : examples by his own by
successive
"

1.
2.

Substance; it is a
"

man,

etc. horse,

3.

Quantity ; Quality ;
" "

4. Relation ;
"

5. Action

;
" "

three cubits, etc. long, it is white, etc. grammatical, it is double, half as large, etc. greater, it cuts,burns, etc. it is two

cubits

6. Passion ;

it is cut, is

burned,etc.

7. Place ;
"

8.

9. 10.

Agora,in the Lyceum, etc. last year, etc. was Time; it is to-day, yesterday, etc, Posture ; it is recliningj seated, Possession ; it is having shoes, armor, etc.
it is in the
"

"

"

that Everything
or

can

be

spokenof

or

thought of

comes

under

one

be a can Categories ; in other words, whatever is in one or the other of these Predicaments. subjectof predication of a series of notions, Each is, the highest therefore, generalization in his logical each a summum whatever Aristotle, writings, genus. intends the Categoevidently ries place they may hold in his metaphysics, of the widest notions signified enumeration to be an by single been sharply cised, crititerms. They have excited a world of discussion, and as often recalled to banished to metaphysics, repeatedly is not made the analysis to them : 1st. That Logic. Kant objects the enumeration is incomplete on principle ; 3d. ; 2d. That any one the other of these

ch. iy. Categories,

See also

9 \, Topica,

; and

7. iv, Metaphysica,

COMPLETE

SYSTEM.

That among

notions are intruded among empirical that the original. Hamilton objects but included absolute,
:

the pure, and the summum


one

derivative
genus of

each series is not

under

higher.3

lie redistributes the series thus

Being,ens.

in different deassumed partment ma sum Practically, particular genera are that notion is accounted the sumnium of thought. Usually, or genus which is characterized by the mark selected as the principle of the ground of the division. This summum genus is the subject

science.
mum

science, Plant" will be the actual sumsubgenus ; in zoology, Animal ;" in chemistry, Compound stance," "Wealth in political common;" and so in more economy, Thus,
in botanical
" "

"

See

Logic, pp.
der
r.

139-141

; and

his note

in Reid's
45 sq. ; and

Works,p.

687.

See also Kant's

Kritik Note
A

V. p. 65 ; Mill's

Logic, p.
see

Hansel's

B.

For historical matter,

Gcschichte der Trendelenburg's

was Categories which, as a mnemonic, we quote as follows : pupilin Martinus Scriblerus, man, Cornelius was forced to giveMartin sensible images. So,calling up "he coachanswered : I at the bear-garden. The man he asked him what he had seen the for a prize two was a fair man, a saw men sergeant in the guards, fight ; one blue butcher the other black,a butcher ; the sergeant had red breeches, ; they

concrete popular,

illustration of the

Aldrich, Appendix, KatcgorienleKre. given by Cornelius to his

"

the butcher in fought on a stage about four o'clock ; and the sergeant wounded the leg. Mark, quoth Cornelius, how the fellow runs through the Predicaments : Men (substantia}, black fair and two sergeant and butcher (qualitas), (quantitas), four o'clock (quando), et passio), a stage (ubi), wounded the other (actio on (relatio), blue and red breeches (Jtabitus)" fighting (situs), Another mnemonic is
124 as

follows

: 3 5 6

Arbor
789

sex

servos

fervore
10

ustos, refrigerat
ero.

Ruri These
two

eras

stabo,nee
to

tunicatus

mnemonics

will also serve

illustrate the statement

that

Logicis

an

not merely of scientific, but analysis

of the most

common-place thinking.

68

OF

CONCEPTS.

placematters.
mum

See But

genus.

is the sumexample invi,"4, where "Mankind" of the words the frequentuse thing," being,"
" "

etc.,shows
mum

what

constant

mental
we

reference

is had

to

the

true

suman

genus.

Indeed,whenever

do not do not
"

know
care

the
to be

proximateor
we exact,
"

or approximate genus of an object, eagles' wings,and call it up on curious thing." Also,whenever we

mount

thing."

Thus

: an

comet

is

relative object call it a thing, thus mark we to some or one exclusively, especially, all others by a direct reference of it to the summum omitting genus ; wish is a hurtful thing, when Wine we because,"etc. So, also, as, mark ; as, Cruelty is a hateful thing." to emphasizesome one
" "

wish to consider

"

4. On

the other and

hand, when

the mind

descends

in

it must reach a finally things, rejecting mum class that connotes that contains under it only individuals, a maxia of common marks, and denotes a minimum plurality plurality division into kinds must of things. Here the process of logical cease. fined class is called the This deepest, narrowest ;" and is deinfima species It is represented in become that cannot the species a genus. as in it many the scheme by Man, containing common taining marks, and conunder it only individual human beings. also as fixed consider the infima species The Aristotelic logicians such in terms like man, horse, etc. Classes, by nature, and expressed to be as not, by them, be admitted negroes, mustangs, etc.,would but only accidental varieties. But the whole question at all, species of natural kinds belongsentirely and with it Logic to the naturalist, Pure Logic cannot essential has nothingto do. discriminate between The logician and accidental marks. nature gets nothingfrom objective

marks

ing addthought, class of things

"

but

and elaborates individuals, the

from

them of
no

his

system

without

any

other

restriction than into

primary laws
until The

thought.
mark,
common

Hence
to

the
even

division
two

kinds logical remains.

continues

dividual in-

that comprehends all the common species for that onlycannot be made the infima species, marks is theoretically a genus by further division. The individual then,not beinga kind,is not a logical not part,i.e.,canbe obtained by division. The constituents of the infima species be counted, and hence may may, however, be estimated numerically, But the individual, under it individuals. it is spokenof as containing
as

the word

is also described indicates,

as

that which
to

cannot

be divided. individual

What, then,is

the

difference

by

which

the distinguish

-*.^

COMPLETE

SYSTEM.

^^^Ss

69

from

the infima

? species

It is

while that,

the infima

consists species

at least marks, the individual possesses, besides these, only of common in the scheme one mark, represented particular by Father of Logic. mark This particular determines only a numerical, and not a specific difference ; therefore, the individual cannot but only debe defined, scribed. Such is the logical individual. The actual, individual or real,

possesses also

distinct existence in space


can

or

time.

It

can

be

ered sev-

and onlyby partition, external


or

be

discriminated differences
are

only in perception,
endless.

internal.

Its numerical

and brief, obviously very meagre senting prethan is of the to fication. classino more requisite exemplify principles of series but The extent is, incalculable, theoretically, any of thought, the upper and lower and in view of the matter practically, reached. If the characters which afford the principle limits are soon of such a division are only external and contingent, there is a division in the.wider sense; if they are internal and constant, there is a division in a stricter sense but also essential ; if they are not onlyinternal, and original, there is a division in the strictest sense. with Starting

"

5. The

scheme

before

us

is

summum any assumed terminate in

an

genus, infima

even

the wider

divisions must

soon

tically prac-

in the

botanical

natural

through some
characters far
an as as

hundreds accidental

as species, though the strictest divisions, extend system, may, treated by dichotomy, of steps. But pure Logic takes no account of or

the laws of of

as essential, congruent or repugnant. As in to unite, thought are concerned,it is permitted

act

conception, any
The number

attributes which

are

each

other.

of attributes in the

of contradictory universe not thus logically


not

with incompatible finds


no

each

is infinite, and the mind, therefore, other,

limit to its downward

progress

in the formation

of subordinate

notions. the summum Hence, theoretically, both unattainable reach them.


This
are genus and the infima species but never We may approximate,

exceptper

saltum.

follows : in two laws, is expressed as impossibility lar 1st. The law of homogeneity: Any two notions the most dissimibe similar. Consequently must, in some they can always respect, be subordinated to some higherconcept. 2d. The law of heterogeneity:Any two notions the most similar furnishes the be dissimilar. This dissimilarity must, in some respect, which process, therefore, ground for a new division, may be continued ad infinitum.
"

"

70 6. Before
to the

OF

CONCEPTS.

"

the dismissing

tree of

Porphyry,attention
division.

must

be

called re-

relations of definition and When


a

the scale ;

down. division, We

Definition looks up is to be fully we subject treated,

which sets it apart difference, give the specific and then the proximate from co-ordinate notions, next genus, the one above,which involves all the marks of the preceding genera, including the highest. Thus the definition comprises all the scale lyingabove its subject. Next we proceedto divide and subdivide until we reach and include the lowest species.Thus division, hausts moving downward, exthe scale. The system then is complete, the work is thoroughly is and the treatment exhaustive. done, scientifically expansive is It not necessary that this order should be rigidly observed. In the progress of and one
a

first define it.

treatise the form the other will

of definition may

often

division, replace

to the point preponderate according in the scale at which a beginning is made, or according to the inclination of the writer or the nature of the subject. In Plato's Kepublic, of the noblest examples of logical one method, successive definitions of justice until a satisfactory are one broughtto the test and rejected is obtained. in the enumeration Then division preponderates, of the
or

and of the classes in a State that answers soul, powers of the human to them ; as well as of the declinations throughwhich the perfect ity, polif it could

be

would constructed,

have

to

pass.

The

whole

is

fused

in such a manner and adorned together by a dramatic element, the finest work to render this dialogue of heathen philosophy. as In the Nicomachean Ethics of Aristotle, definition predominates, but with considerable aid from division. Thus he enumerates the opinions of about the good," and rejects all but the rightone. men Defining that under the name of he is led on to define the parts happiness,"
" "

of his firstdefinition ; and, in the he does virtues,


of both
not

case

of the

moral

and

intellectual
a

consider his

explanation completewithout
convertible

vision di-

classes.4
are

a scientific correlatives, either in tabulated divisions, in a or entirely system may be expressed series of definitions. These are, mutatis mutandis, the same thing. We with the exhaust the summum may begin genus, and, descending, scale by a series of divisions. Or, we may begin with the infima exhaust the scale with a series of definitions. and, ascending, species,

Since definition and division

it is requisite to Any specific concept beingdefined,

define the

proxi-

See Thomson's

Outline^ " 128.

OF

CONCEPTS.

in one,

but

in many and

series.

It is true

that in most

minds

there

is

much

confusion

disorder in this fabric of very

evinced words.
in

by

the

indefinite and

an thought, ambiguous character

entangling
of
common

the greater part Still,

of the humblest

mental

life is

occupied
recting cor-

and generalizing

in systematically and specializing, arranging of

the arrangement When


savage

thoughts.
a cow

Captain Cook
natives

landed

in the
:

South
a

Sea
of

the Islands,

exclaimed

in astonishment

It is
to

kind

goat !

The

goat being the only horned


this mark. their definition
A

animal

known

them, they generalized


difference be

They specialized by
was

thinkingin the largegoat.


of this

large;

so

cow

is the

It may

hoped

that
in

they
form

have their

now

corrected
at once

matter

but classification,

perfect. If I should speak of a button, What It beingby no do you mean a child by button? I may the means easy to define this familiar thing, escape, and satisfy the different kinds of buttons ; or, querist by naming and describing is a button, show it a specimen, : This still, perhapsmore easily saying which will do prettywell, to the scholastic aphorism, since, according
was logic might ask:
"

omnis

intuitiva
to think

notitia est

This definitio. throw

is

more

cline easy, for in it I deof

the matter, and

the burden

Every book, whose author has well the point. In turning the leaves, find the whole divided into we the general divisions being so called by way of eminence ; the parts, these into sections; these into parts are subdivided into chapters; formal partition paragraphs responding cor; these into sentences ; this external, to the internal logical division of the subject-matter. So it is, in matters small and great, we are governed, though for the most law ; and whoever his notions of part unconsciously, by logical adjusts in systematic to their true relations, thingsaccording order,each clear
of others and he is

child.

thinkingit on the his subject, trates illusdigested

distinct in

his itself, thinker.

is the

well-stored cultivated,

lect, intel-

the eminently

in the usus inaccuracy loquendiof familiar words requires that they should be largely set aside in building Hence up a science. fined, denearlyevery science has many unusual,technical terms, sharply and located in its system : such words not as are to be likely drawn into vulgaruse, and have their edgesworn off by the attrition of every-day handling.In these technicalities a science arranges its classificationsin obedience to the logical have discussed, we principles and when its system is complete, it then has attained that logical per-

"

8. The

COMPLETE

SYSTEM.

73
of science
to according'

fection which

is

characteristic specially

its

ideal definition.

Owing to names peculiar


of

the

divisions multiplied

in

sciences, many

have

adopted-

also for the several subaltern genera, in order to mark the relative placeof each stepin the ascending and descending series grees and conveniently the various declearly of generalization. Thus the system of Zoology, as given by is as follows : modified from Cuvier, Agassiz, slightly
out
Second
Intentions. First Intentions.

and thus mark classes,

Kingdom,
Branch. Class. Order.
. . . .

Family
Genus
.

Species. Variety.
The any student
of

Mineral. Animal, Vegetable, Radiates. Articulates, Vertebrates, Mollusks, Fishes,etc. Mammals, Birds,Reptiles, etc. Ilerbivora, Bimana, Quadrumana, Carnivora, etc. W easels, Bears,Seals, Cats, Dogs,Civets, Felis (thetrue Cat), Lynxes,etc. etc. Panthers, Leopards, Lions, Tigers, Indian Lions. Nubian,Arabian,Persian,

Logic would

do well to make

visit to thoughtful

a logical History. It presents On entering universe. The summum genus is material productof nature. of suche finds this universe logically cession divided,on the principle ancient in time,perhapsinto two floors ; the lower presenting subdivided into Zoology Geology; the upper, recent products, products, in opposition to the extinct life in the lower and Botany extant life, floors is, sort of these two division. Between a a gallery, perhaps, the enter tertium quid,devoted to Lithologyand Mineralogy. We into two lower apartment,the Hall of Geology. It is subdivided the other for Structural Geology. The for Paleontology, one halls, the ftmdamentum dico-ordinate subdivisions, first of these has many to be visionis beingagainhistorical. At first glance the ground seems the the open floor, on size^ largespecimensbeing grouped centrally small being in side cases. But these largespecimens mostly belong

Museum well-arranged

of Natural

"

to the

ever the fault is not serious. Whatgeologic age, and hence be pardoned as pracit must however,is involved, tically logical offence,
same

unavoidable.
a

The
;

side cases,
one

we

observe,are

we

is the Silurian age ; If the Carboniferous ; and so on in the order of time. ; another, approach the last named, we find it subdivided into fossiliferous

geologic age

each replabelled, resenting the Devonian another,

fauna

and
to

flora.

On

the side of the flora

we

find

one

set of shelves

devoted

the tribe of

to Phosnogams ; another,

that of Calamites

74

OF

CONCEPTS.

another, to that
we

of

Cryptogams.
the
is

Looking
Ferns,

on

the the

shelves

of the
In

latter,
some

find
a

the

Lepidodendrons,
the individual In this in time
a new

and

Equisetse.
and species,
so

cases

single shelf
come

subdivided, giving infima

then
of

at

last
other

we

to

specimens.
is abandoned

And

with

each

the the
terior inis
a

departments.
succession
case,

it will be observed distribution, when division It


we come

that
to

of principle of the

the

and

ground
to
cross

of

adopted.

This

fault,and logical

gives rise

divisions.

is,however, justified

by

the

results. practical collected furnish

If these

objects were
amusement,

arranged merely
but
not to

to

please the

eye, It is

they might
this

scientific

instruction. natural
museum

logical arrangement

according
that As
a

important
of

affinities,

evolving a complete system,


difference,scientific. specific

distinguishesthis product
of in the

by
it offers
a

the this

thought,
it

peculiaradvantage

to

the
in

student

Logic, that

presents

logical

not system displayed,

words,

but

things themselves.
of

"

9.

We

now

close

this

general
and
to

division

Logic.
its the second

In

it

we

have
most

considered

how

thought
We
are

does

must enter

elaborate upon

highest and
Before

complete
however,
to

results.
is

about

part, which,

only
new

another

aspect

of

the

details.
to

ing proceedapplication
to

the

view,

attention

is recalled in every
we

the

three
Their

fundamental

laws
at

which

govern has

pure
so

thought

aspect.
have

each

step
A

been

obvious, that
may its

felt it needless If any genus, the Law

point
is divided

it out.

general example
into both affirmed
of

be here

given.
and

X,

by dichotomy
must

A species,

non-A, then
turn

genus
of

be
e.

these

species in
non-A

by

the

ty Identibe

g.,

Every

is

X, and

Every

is X. ;

The
e.

speciesmust
A be affirmed
;
e.

denied One
the

of each

other

by

the Law
of
a

of Contradiction

g., No

is non-A.

species being
Law of is not in the

denied

thing, the being


Such

other
no

must

by
g.,

Excluded non-A progress


are

Middle, there
is A.
of

middle

ground
be should

Whatever made

applicationsshould
The Laws
root

ly constantnever

the very

subject.

be whole

forgotten,as

they

the

corner-stone, the

of

the

Theory

of

Thought.

PAKT

THIRD."

OF

JUDGMENTS.

I. THE

PROPOSITION.

"

1. To
as a

judge

is to of

bring

one

thing
is
are

in

or

under
or

another.
of

judg*

raent,
Two

product
or

thought, compared

the

issue

result
as

comparison.
or as

things
and

notions

apprehended
that

similar
or

similar, disagree. dis-

the
virtue
a

judgment
of this

pronounces declared
two terms
as

they

agree

that

they
notions
are

By
reduced
to

relation, the

duality
in

of the relation other.


is then

is fied. uniboth

unity

; the
one

being thought determining


since

Necessarily
cannot

is

thought

the there be

For

be

thought
On
since

as

merely determining,
other

nothing
as

determined.

the there the

hand,

both

cannot

thought
Hence,
other. the

merely
one

determined,
be
one

is then

nothing determining.
the
one

must

determining,
is

other

determined,
or

of

the
in

Therefore,
is is

thought

as

an

attribute
;
or

mark

contained
as a

other, which
the other

thereby
contained Before

determined

else

it is

thought

class which

under, and

thereby
it will
are

determined. be well
are

proceeding,
which
we

to

reiterate
not
an

that

the

tions considerathose
of
to

upon

entering
not to

advance the

beyond
arrangement
We

just

concluded. into
over a

We

are

advance,
is

since

thoughts
pass
a

complete
a

system
of The the
same

logical perfection. ground,


but
to

are

again

portion
of

consider

it from
and

different

point
has

view. been
an

almost

complete identity of concept


A

judgment
;
a

already
is

remarked.

concept
E.
"
"

is

an

ment implicit judgis


a

judgment

explicit concept.
the and marks
animal
"

g.,
"

Man" animal

concept
the

that

involves implicitly
"

rational
"

and
from

;"

ment judgin

Man

is rational
or

differs its

the We
that
or

concept
are

only

that
upon

it
new

unfolds,

explicitly states,
It is

content.

not, then,
a

ground.

sufficiently apparent
time
or we

in

forming

rarchy hie-

of
at

concepts, every
step
now

subordinate
we

co-ordinate
a

notions,

every
is

of

division
is

definition,
consider

pronounce

judgment.
of

What

proposed

to

the

parts

and

kinds

these

76 and judgments,

OF

JUDGMENTS.

laws which regulate their formation or limiting the grounds upon which we do to investigate determine their validity, the relations of our and must judge in determining concepts. This is for oftentimes true not onlyof immediate judgment,but also of reasoning; cannot determine the relation between two we cepts, condirectly do it by comparing each with a third. but must Let us, then,keep in mind that in what follows wre are only improving our pletely knowledge of the modes by which the mind progresses towards comits thoughts. And let us also remember that systematizing every step is governedby the three primary laws,and, in pure Logic, by no others.

the

judgment expressedin language is called a proposition. is subjectively The What a a proposition. judgment is objectively is to sever firsttreatment it by partition into three portions. These said above: notion of some1st. The thing was are, accordingto what of something determined, called the Subject; 2d. The notion called the Predicate ; 3d. That which determining, expresses the two, called the Copula. These relation between this recognized exist the existence of,neither can each implies terms are correlatives, the other. In every express judgment somethingis spoken of without, that is the Predicate ; that is the Subject ; somethingis said of it is Pure ;" Snow that which says this" that is the Copula. Thus, "some Sentences is Pardoned;" "Sighs are are "Sin Prayers;" Stars are Planets;" or, to indicate merely the "some Propositions;" and predicate, The subject beingthe extreme parts form, S is P." It is not of the proposition. called the Terms in this partition, are words ; they should consist of single that these terms at all requisite relations. words in intricate grammatical may be composed of many struse in the study of an abencounter difficulties E. g., The very many we to be overcome science ("subject) are by persistent ("copula) effort stimulated by a desire to acquire knowledge" (^predicate).

"

2. A

"

"

"

"

"

"

With

taper light
to

To Is

seek the beauteous wasteful (=copula)

eye of heaven

garnish("-subject)
ShaJcs. (^predicate).
"

and

ridiculous excess"

The

meaning metaphysical
We
noun

of

be understood.1
a

observe

and substance is supposedto subject must that the logical alwaysbe subject consist of many
104

substantive

(which may

i. e., somewords),

See Hamilton's

Metaphysics, pp.

and

110.

THE

PROPOSITION.

77 Non-entis
or

thingthoughtof
The We may, the the

as

havingsubstance.

nulla sunt

predicata.

predicate may

be either substantive

however, take the view that,in


means subject

i. e., attributive. adjective, accordance with its etymology, under


or

that which

is thrown

contained

der un-

predicate.

includes the copula, and this is still predicate reckon the copulaas a the usage of grammarians. But logicians now distinct co-ordinate part. Since a judgment always expresses the in mind, the copulamust always presentrelation of two notions now

"

3. In Aristotle the

appear
its
own

as

the

presenttense

of the verb to be.

kingdom in which an eternal now The copula admits of only one negation. Hence qualification, the negative wherever it may sentence particle, negative occur,
as a

E. g., " For the mind does alwayslast."

is

in is
is
no

considered
not

part

of the

strained;""No
old

of mercy copula. E. g., The quality chastisement "Britannia is joyous;" needs


is not

"

bulwark,"i.e.,Britannia
The in order to This is the

needinga bulwark. held that the copulamay be otherwise modified logicians that attends the judgment. express the degreeof certainty Doctrine of Modality."Thus,"
"

( Problematic Modal

; as, A
can

may

be B.

judgments are
v

?"^\

as" A

be Bcannot

Impossible ; as, A Necessary;


as, A

be B. be B.

must

"Demonstrative." Recent or Apodeictic" the doctrine of modality, and account the modifiers as reject logicians be B." a part of the predicate ; thus, A is something that may above stated, that the copulacan be modified in no way as They hold, whatever except by the negative particle.8 it has quite The meaning of the copulais ambiguous,or, rather, a section it will be number of different significations. In a following is conthat it may be interpreted either as as or seen tained comprehends,"
are
" " "

The

latter two

called

"

under."

Thereafter

we

shall find that it sometimes

"

means

is

meaningswill appear as we progress. We need to remark here onlythat it requires Always,however, interpretation. it implies Aristotle the modified or limited by predicate. existence, not an The copula affirms merely a relative, existabsolute, says :3 equalto ;"
and other
"

See Hamilton's

Loyic, p.

181

sq.

DC

Elenchi, Sophist-lei v, 3.

78 ence." From

OF

JUDGMENTS.

"Ptolemy
or

is

dead,"we So,
"

cannot

infer that
ns as a

"

Ptolemy is"
man

exists ; but i. e., actually

only that

he exists to

dead
"

can,

Ptolemy is not alive denies his it in the other sense. but implies existence relative to life, the verb to be declares absolute In merely existential propositions I am," sum* it is both copula and predicate. Thus, existence, I am I am in such or means a being." The existing," predicate is the summum case So, Enoch simplemark. genus, or its single, not ;" "It is fine weather was elry to-day;""There was a sound of revthat doeth good, no not by night" (Byron); "There is none that practise i. e., some exist, self-denial," one;" "There are men be construed as existential or Some a propositions may very few. otherwise ; as, It is impossible to love and be wise " may be construed by
" " "
"

remembrance

tradition.

"

"

either "To
"

love and be wise So


is

be wise cannot

be"
"

i. e., cannot There


are

coexist,
six Rich-

or

To

love and

impossible."So,
"

monds

in the field."
"

also,
this old man's

That

I have

ta'en away

daughter,
her."
"

It is most

true

; true, I have married

Shaks.

That

speechthe copulais absorbed in verb forms word. and the whole proposition consist of a single or elided, may that twinkle ; E. g., Stars twinkle," Stars are things Cogito,"=I Ilium fuit" is raining am (existential) thinking;"Pluit"=It ; "Did he Troy is something which formerly existed (existential); is one who came come "Yes,"=He yesterday?" Ans. yesterday; He is who did "He love. All verbs one was or loved," loving, to be" their variety ing arisare perhaps fundamentally one, the verb of various attributive notions with this from the incorporation verbial simple verbal element,and its own past and future forms beingadwith the present tense. notions incorporated
in
common
" " = " =^
"

these is, Very often

facts exist ; or, these accusations

are

true.

"

"

4. In accordance

that all sitions with its postulate,4 Logic requires propoin the above examples, without shall be transformed, so as that, distortion of the
the thought, The severally appear.

or retrenchment, addition,

three

parts,
is be said

shall Subject, Copula,Predicate, sometimes

process

quitetroublesome and awkward, but nevertheless must has justbeen performed. E. g., "So he said" becomes "What
"

See Part

ii, " 1st,

8.

80

OP

JUDGMENTS.

of these four the ten

and is contained Predicables,

under

one

or

another

of

the number Categories.Porphyry and the Schoolmen enlarged of the Predicables to five, by substituting Species(aspredicable and Specific Difference for Definition. of individuals) This was the of improvement; for, as Aristotle himself had reverse remarked, each of these is of the nature of Genus, and interchangeable with it.' The doctrine of the Predicables, however, like that of the Categories, has ceased to play a prominentpart in Logic.7

"

6. Various

divisions

are

made

of

judgments

or

for propositions

As the logical purposes. is as a different principle


cross

in each case, and genus divided is the same used in each,it is evident that there will be

ative judgment may be either affirmbe either intensive or negative ; and an affirmative judgment may This is not, however, a logical fault here,since the extensive. or several divisions are not proposed as steps in a series, but are independent divisions.

Thus,

an

intensive

of each other.

judgments are intensive This distinction is grounded on the relation of suband extensive. ject and contained, and predicate, whole as as containing reciprocally and is the whole, or part. In the intensive judgment the subject is the part, minor term. or Thus, The major term ; the predicate
The
"

firstdivision to be considered

is that

earth is

Here spherical."
a

let

us

view the notion

"

earth

"

as

an.

sive inten-

to
as

We complement of marks. which thereby it the mark enters into, or spherical," a part of,this whole ; for it is only one mark out
of whole, consisting
"

then attribute is recognized

characterize It is

our

notion

"

earth."
:

This
"

is

an

of many that attributive judgment.

thus conventionally expressed On


the other

The

earth

comprehends spherical." part,or


minor
"

is hand, in the extensive judgment the predicate


term

the

whole, or
"

major
is
a

; the

is subject

the

term.
"

Thus,
8

The

earth

Here sphere."

let us view

the notion

sphere

Part See

2d,v, "
Hansel's

3.

Aldrich, Appendix,Note A, for a discussion of the Predicables. found in Topica, 8,where Aristotle says : Every predicate i, it expresses with its subject, does not. If it reciprocates, cither reciprocates or of in either the whole essence the former case or none; (TOri yv ilvai) the subject, it it If does it is called Definition ; in the latter, either a not, Property. expresses
The doctrine will be

part of the essence,


either Genus
or

or

none.

In the former

case

it must
case

be

tion, part of the defini-

Difference. is neither

In the latter definition


nor

it is

for evidently Accident,


nor

accident is that which

property

genus,

and

yet is

present with

thing.

THE

PROPOSITION.

81

as

such as the things, the geometother planets, the sun, all globular their satellites, fruits, rical 'under included all which etc., things are sphere, rain-drops, notion our sphere." Now in the givenjudgment we declare that the earth is one of these things, a part of the great complement of thus: thingsdenoted by "sphere." It is conventionally expressed The earth is contained under sphere." This means : My notion notion of sphere." For another of the earth is contained under my the mark attributing example: Men are mortal ;" this is intensive, the major term. to men, Again, Men are mortals ;" this mortality is extensive, mortals is the major term, a genus embracing also is a species contained under this and "men" "brutes" and "plants," how of it. Let it be remarked the which is predicated genus and replacedby "comprehends" and copula is here interpreted
an

extensive

whole,constituted of

greatmany

"

"

"

"

"

"

"

"

is contained

under."

but most there is frequently only is the copulaambiguous, to show which of the two quantities nothingin the entire proposition mind readily view to the is thought. And, indeed, passes from one and any proposition of being interpreted whatever is easily other, capable While this is of logical cither intensively or ment, moextensively. in a theoryof thought, it is not of the smallest pracimportant tical One volume a viewingevery person might peruse consequence. read another the volume same viewingevery intensively, proposition and the knowledge acquired by each would extensively, proposition It is a fault of the old Logic, not, for that reason, differ appreciably. Not

however, that all


reference to the

of

its nomenclature

and

treatment

has

exclusive

of extension. quantity while a logical the proper place to observe that, This seems tion proposihave individual it cannot individual have an an subject, may in extension is For the predicate of a logical proposition predicate.
a one mere

genus
nor

; in

is intension, We

mark.
say

An
"

individual

can

neither

be the
a

the other.

may

Great
is the

is Diana

;"

but

this is

rhetorical
"

"Diana" inversion; may


a

is

great." Again,we
in the

Aristotle ;" but this is not


one, not

and the predicate subject, The favorite pupil of Plato was say but an equivalent proposition ; logical,
"

but in qualitative,

the

whole. quantitative

les attributs qu'elle enferme de 1'idee, J'appelle comprehension convient." Port' soi. J'appelleetcndue de Tidee, les sujets a qui cette idee en ch. vi. RoyalLogic, pt.i,
8

Says Arnauld

"

"

82

OF

JUDGMENTS.

"
are

7. The

second The

division of

is into categorical and propositions

ditional. con-

of

since they grammaticalforms of sentences or clauses, of mental states, with the generic faculties correspond expressions mind, thus,
"

Sentences

are

\ Interrogative, Cognition. Conditional, "" expressing ) Categorical, "" Exclamatory, Feeling. Desire. Optative, Will. I Imperative,
" " "

is real and not merely interrogative sentence, if the question shows that a comparisonis being made which has not yet rhetorical, reached an issue in judgment. It is the search after ground for judgment. Conditional sentences or propositions express a comparison so completethat only certain grounds or premisesremain in nearly is stated as a condition. matter question. Such doubtful or contingent and disjunctive. E. g., "If They are of two sorts,conjunctive it ;" and, This view is either can this, explain you you understand inaccurate." Conditional propositions junctive, subaccurate or arc indicative, A categorical or potential. proposition son expresses a comparii t is not reference absolute. to completed; making any condition, and potential It also appears in the indicative, moods. subjunctive, forms ; Logic is concerned only with the conditional and categorical of the other four forms for these only are propositions, none ing expressThe consideration of-the conditional a declaration. judgment is gorical postponed until we shall have finished our examination of the catethen,the words judgment,propjudgment. For the present, osition, will be understood to mean the categorical. etc., unqualified, it means The term categoricalis originally legal, accusing." In Logic it means attribution a downright statement, a predication or and hence simpleor absolute. A categoriby condition, unqualified cal is unconditionally then, is one in which the predicate proposition,
"

An

"

"

"

affirmed

or

denied

of the

subject.8
judgments is
"

"
9

8.

The

third division of

into total and

partial.It
"

the term tive," categoricalmeant originally by Aristotle, merely affirmahis successor in the Lyceum, as opposed to negative." By Theophrastus, in the it was sense a s ditional." conabsolute," simple," direct," opposed to employed it has continued to be employed by all subsequent In this signification See Hamilton's Logic, logicians. p. 207.
As

used

"

"

"

"

"

"

THE

PROPOSITION.

83

is called their is
is

judgment or proposition determined this of the subject, as solely by the quantity according total or partial. visions, diThe following exhibits the important scheme
a
"

of The quantity Quantity.10

f are Propositions

Total

or

definite

j Individual,e.
"

g., All the world's


men men men are

stage.

-j
( Partial
or

Universal,e. g., All


g., Some

players.
love.
seek

e. j Indefinite, particular

e. g., Some ( Semi-definite,

reputation.

and hence of the proposition, is indiof the subject, cated quantity It is often the case that all, by the predesignations some, etc. no sign of quantityis prefixed.A judgment always has quantity of the thinker and speaker, in the mind but the either total or partial hearer is frequently left to surmise the quantity intended from the

The

context,

or

from

the matter.

Thus,
"

"

Birds

i.e., all do, the breathe,"

Birds sing," i. e., some ter do, the matpredicate being of the essence ; Some class these as definite inbeing accidental or contingent. logicians a" doubtful propositions," grammariansspecify very like as some gender." But, as seen in the scheme, we have another and a better
"

use

for

the

word
"

"

and these undetermined,or, indefinite,"


"

as

ton Hamil-

stitute conproperly class. When undertake to reduce such a proposition a we to strict logical to designate the quantity form, it is needful, generally, of the subject by its sign. Individual propositions those in which a whole, the subject, is are indivisible unity. It may be a proper judged of or viewed as a single, Caesar is ambitious ;" or an objectdesignated vidual indias an noun, as, or by the definite article, by any demonstrative word or phrase ;

calls

do them, preindesignatepropositions,

not

"

"

as,

The

world

is round

;"

"

This

man
"

is

and the whole heart faint ;" sick,

is crazy ;" " The whole head All Jerusalem went out to meet
"

him."
"

It may

be

collective

whole,as,

The

senate

has

;" adjourned

tribes." was Apostles Universal propositions have subjects which are logical wholes. The total number of objects within a divisible but undivided class are i.e., all taken together judged of ; as, All men are players," ; Every
in the twelve typified
" "

The

of college

10

the

ent unfortunately ambiguous,being used to express two quite differrelations ; the quantity of thought or of concepts being intensive and extensive, of quantity judgments being total and partial.If not heeded, this various application of the term is liable to confuse. The quantity of a judgment has no reference whatever
to intension
or

This

term

is

extension.

84

OF

JUDGMENTS.

Such terms said to be are severally. of each object distributed; because what is said is said distributively All is ambiguous, in the class. It seems, then,that meaning either in individual propositions in all as a unity, as as ; or all as a plurality, The former is called the cumular meaning ; the universal propositions. usual meaning. The and is its most latter is called the exemplar, distribution are all, or every, each,both, signsof universality any, none,
man

is

i. c., all player,"

taken

"

"

of material substances, neither, always,never, whoever,wherever,etc. Names and universal are as gold, etc., singular water, flame, stone,salt, without
one

They predesignation.
of substance.
or

each denote

any

and

every

portionof

kind Partial

those in which we judge of a are propositions particular less than the whole denoted by the naked subject. number of objects, but onlyof some. The old logical That is, meaning we judge not of all, is is some of some at least, perhaps all ; hence it only may be De Morgan proposes to This is the indefinite some. particular." and instead indefinite ; instead of call it vague or particular also is ambiguhe proposes full." The word of some universal," ous Some at least, men love," perhapsall, as, ; either it is some haps per" " " " " " " "
" "

all do ;
not

or

it is

some

at

most,

not

all, as,
if
we

"

Some

men

seek reputation,"

all, which
"

is

true clearly

mean

which

is found

"

in the cannon's

mouth."

The

only that reputation definite first is the wholly in"

it excludes all." judgment; the second is the semi-definite, is indefinite or semi-definite, the predesignation Whether is gensome erally the but context to be determined or Hamilton, who matter, by portance which he considers of imintroduced into our Logic this distinction, insists that some in reasoning, is alwaysthought as semi-definite of the judgment is universal ; a rule that when the other term A subject is certainly objectionable.11 qualified by the article a or an and semi-definite ; as, A German when it means any) is particular (except If we did. i. e., some German invented printing," substitute one becomes total and the name for Faust,"the proposition a German The signsof partial individual. or are subjects particular some, not
" " " "

not all,

every,

few, there
the

are

"

a that, are

or

an, one,

times, sometwo, three, etc.,

somewhere,etc. but, being


most, almost
less than in
common

There

also

whole, are

that approximate a whole, signs if taken still, strictly, ular, particall ; as, many,

though

every

one,

to speechoften tantamount the large majority of, etc.

The

following

11

Sec

Appendix

to

his

Logic, p.

531.

THE

PROPOSITION.

85

total negatives : few, very few, hardly or nearly scarcely any, little, Few small, slight, are saved,"i.e., Nearly all rare, seldom, etc. ; e. g., are not, perhaps none ; hence indefinite. But a few is affirmative ;"" i. e., a small number all;hence are, perhaps e.g., "A few are saved," indefinite. Terms or qualified by such signs, merely thoughtas particular,
are
"

are

said to be undistributed.

and negative. judgments is into positive The positive that the subject affirms, proposition by the Law of Identity, in that of part in the relation of equivalence, and predicate are or and whole, contained and containing.The negative denies, proposition such relation, and by the Law of Contradiction, excluding subject the sphereor comprehensionof the other. each from predicate, By of Excluded the principle Middle,no third form of declaration is possible; the relation in question between and either subject predicate

"

9. The

fourth division of

does every

or

does

not

it is yea exist,

or
"

nay.

is of the form is not P." S is The proposition ground of this division of judgments is called their Quality.12 But let us examine the meaning of negation littlemore a larly. particuOftentimes a negativejudgment simply denies one thing of If we is not vapor," Smoke the meaning another,no more. say, probablyis that these two notions, though liable to be confounded, unlike that they should be set entirely are so essentially apart in It is simply a holding thought. There is no thought of a genus. if it is said,"Smoke is not fluid," back from So also, the error. is simply denied mark as no more. comprehended in the subject, Again,in other negativejudgments there is a thought of a genus, is not a gas;" i. e., the which is denied to the subject; as, "Smoke
" "

Hence, S P," or

as

has been

said,

genus

gas does not


from

contain

under

it smoke

as

one

of its kinds.

Smoke
no

is excluded
more. a

from it, simplyrejected may

the

sphereof

gases, but

there Or, thirdly,

be

mental

reference

of both notions to nied deare species, the thought is,


and
as brute,

containing genus,
of each

under
"

which Men
are

they,as
not

co-ordinate Here
man

other ; as,
to

brutes."

most

limited likely,

the universe animal, while of the


may be genus,
are

co-exclusive and

exhaustive
notions

the Lastly,

tories. thought as contradicor thought merely as disparate,

13

Another
or

unfortunate
no

confusion

of terms, for the whatever


to

has negative

reference

the

of judgments as posquality itive of concepts heretofore quality

discussed.

86

OF

JUDGMENTS.

and contrary, perhaps


a

as a

such

denied

of eacli other ; as,


"

"

Man

is not

for bruising ;" or, See Follywaltzing reptile In far from Wisdom's way," i.e., Folly's way is not Wisdom's way. there is a thought of a containing this case, also, genus or universe in common, to a narrow limitingthe notions, which .have much sphere. In a negative the copula, judgment the negativeparticle qualifies though it may not stand in connection with it. E. g., Not a drum is "No heard" (Wolfe);"Not man was every mistake is culpable;" shall inherit eternal wiser for his learning" drunkard (Selden) ; "No that doeth good,no, not one ;" That goodness life;" There is none is no name, dream" and happiness no judgment (Byron). A negative which is a taken strictly, is said to have a copula," phrase, negative the qualified contradiction in terms, but is used to designate copula. 'It is needful to observe that affirmative propositions often contain of the predicate, and should as a part either of the subject or negatives for negative E. g., Not to know not be mistaken me propositions. unknown" "lie that does not heed,stumbles (Milton); argues yourself

beast for

nor burdens,

"

"

"

"

"

;"
"

"

To

wonder

not

is

rare

art," Nil

"

admirari

prope

res

cst

una

(Horace).In

those

it is following

in the

is a part of the subject.In the negative in that extremity the general "Even predicate:
" "

the

cry was, No

surrender

not you, reject it." It should also be remarked

On iny bended knees (Macaulay) ; this bill (Brougham) We do cannot ;


"
"

I plicate supwithout

wherein negative, ;" He is perishes


"

no

arc propositions E. negative particle appears.


"

that

often
"

ly essentialThe land brute and

g.,
on

blind ;"

Darkness

and

silence fall

sea." may
as

These also be

also are, in

affirmatives. Negativethought form, logical affirmative forms

conveyedin

by

means

of such

phrases

the contrary, wanting or deficient on far from, the reverse of, beyond, and the like. in,devoid of, the negative the predicate, the judgment When particle qualifies but something is affirmed is affirmative;it is not mere a denial, is a negative notion. of the subject, We have though the predicate that many remarked have notions, originally already pure negatives, character.13 These are no in usage had thought into them a positive accompaniedby the thoughtof a narrow longerpure, and are generally in a pure negation which. is not the case or universe, ; e. g., helpgenus unwell, uneven, indirect, immortal, etc. Thus, if I Lss, unpleasant,

13

See supra, Part

2d,vi, "

5.

88
"

OF

JUDGMENTS,

tribution section that the dispredicate.We have shown in the previous of the subject is according of the judgment ; to the quantity do not distribute, that universals distribute, and particulars the subject. the distribution of the predicate, Now which takes place in thought without any verbal sign, of the judgment. depends on the quality

the

The Some
"All
some

RULE

is:

do Negatives distribute the predicate, affirmatives


to

not.

simple examples will suffice houses are some i.e., buildings,"


that buildings
are

illustrate this rule.

Thus,
are

buildings only,for
"

there

not

houses, as
or

etc. forts, bridges, ships,

is undistributed hence this predicate

Again, particular. since are no pyramids, pyramid can be called is distributed or house ; hence this predicate universal. Again, a Some houses are i. e., some for tents, dwellings," dwellingsonly, is particular. ; hence the predicate caves, and shipsalso are dwellings i. e., some houses,such as Again, Some houses are not dwellings," not any are churches, factories, shops, dwellings ; hence the predicate
No houses

pyramids;" i.e., not any

"

"

is here universal.

It is evident that this which

which rule,

comes as

from

the old

Logic,and
is that

Hamilton, as
we we affirm,

we

shall see, the

impugns
of

has altogether defective, Its view

exclusive reference when

to

extension

the terms.

in the therebyinclude the subject as by the predicate merely a part of it ; and that when from that class wholly. therebyexclude the subject

class denoted
we

deny, we

"

10.

In

order to facilitatethe statement

combine logicians result four


in the

forms,which

of the sylloanalysis gism, and quality the quantity of judgments. There exhibited as theysymbolize by vowel letters,15

and

following
TABLE
OF THE

PROPOSITIONAL
Formulae.

FORMS.
Examples. All oaks
are are

Quantity. Universal Universal

Quality. Symbols.

Affirmative,
"

"

All S is No S is S S

Negative, Negative,
to

"

E
"

"

Particular Affirmative, I Particular


"

"

Some Some

"

(some) P (any)P is (some) P is not (any) P.

(some)trees.

No

oaks
are are

Some ..Some

(any)vines. (some) evergreens. not (any)shrubs.

"

It is curious

Petrus logician, nego. We

first adopted by an old symbolicletters were vowels in and first two the the words affiwao HSspanus; theybeing add that the old logicians abounded in mnemonic cordingly, devices, and,acmay the said Petrus supplied the following stanza,
note

that

these

"

Asserit A, negat Asserit

E, sed universaliter ambae ambo. I,negat 0, sed particulariter

THE

PROPOSITION.

Individual considered

is a total, arc (" 8),since the subject propositions and symbolized as universal, by A and E.

ly usual-

rather than of judgments. propositions are Propositions Simple, Complex, and Compound. A Simple proposition consists of only one tains judgment; i. e., it conthan one and one not more subject predicate.It may, however, and skilfully consist of many elements ; as, grammatical Well-organized of happiness administered in their governments are productive subjects." A Complex proposition involves with the principal judgment one subordinate ment incidental judgments. This subordinate eleor more or incidental to the principal or subject cate. prediappears as a clause, is A who is learned E. g., man ;" Whoever is right respected
11. The

"

fifth division is of

"

"

"

is safe fire is

;"

"

Who

steals

my

purse,

steals trash

"

(Shaks.) ;
cannot

"

little

rivers trodden out,which,beingsuffered, quickly

quench"
first
is in

"Ill (Shaks.);

blows

the

wind

that

In profitsnobody" (Shaks.).

these the clause is in the the second partly, the predicate :


" *

subject, though the latter two are, the inverted. In the following the clause wholly,
am

monarch
"

of

all / survey
"

"

(Cowper) ;

"

The

cry is still They come."1 to choose the wrong the wit In the
:

is out

"

I ivas When ways a (Shaks.) boy, I used al; the age is in, side" (Johnson); When What I have written,I have written." (Shaks.) ;
"

"

there following They that are


"

are

incidental clauses in both


as

ivise shall shine


man,

the stars

and predsubject icate ;" Shylock, (shine)


"

who

was

hard-hearted such

exacted he
ivas

the payment
much disliked

of

the money
all

he

lent with

that severity

by

good men"

(Lamb).
A subdivision of incidental clauses may

be made

into

and

Restrictive. or Limitative, connoted


is

The

the marks

of

woman,

evade

the God
not
now

who is born by the notion of few days and full of trouble;" "Jonah sought to who is omnipresent" Explicative clauses express judgments renewed subordimade, but formerlymade, and now
or

clause Explicative it qualifies; as, "Man,

Explicative merelyunfolds

nately. Limitative
to
as
"

which restrictive clauses, clause


or

may

also be allowed

include the concessive the terms Men


who
are

limit indicate,
avaricious
a are

those which, are removing restriction, restrict the notion they qualify ; as,
This
"

discontented."

is not is well

said of all

men,

but is said of all in

limited class.
"

So,

He
mere

paid that
a

is well

satisfied" (Shaks.) Honesty,when ;

it is

is not policy,

90

OF

JUDGMENTS.

virtue."
a we
"

The

concession

in

"

I will trust So in
"

him Live

though he slay me
we

"

moves re-

conceivable
"

restriction.
"

how

we

can,

yet die

They strive that they may enter in" and limited by purpose ; are They take heed lest theyfall" the predicates tion, the restrictive is a condiWhen the other negatively. one positively, be converted into a conditional. the categorical easily proposition may
must

In (Shaks.).

Thus

the

example above

may

become

"

If

men

are

cious, avari-

they are
We
now

discontented."
garded these incidental clauses of all kinds being rethat,

observe

adverbial elements, the or adjective, merely as substantive, needful to is in Logic treated as simple. It was complex proposition clauses for be forewarned not to mistake discuss it only that we may to strict logical a proposition ; and, in reducing propositions principal to the principal form, that we may be careful to subordinate them in place is he is ing, rich, savor predicate.Thus, He, who, though subject share with him ivho is needy without lessening that can is one S is P. what is enjoyed tence Indeed,the complex sen;" here the form is, that is strictly reducible to one is often directly simple. Thus, A who is is respected," learned man the first example given above, A learned man, of learning," is respected." A man or reduces to is one that comprisestwo The or more Compound proposition for or co-ordinate, nearlyso ; and these, logical purposes, judgments, and stated independently.It is of two kinds, to be separated require less obvious. or accordingas the compounding elements are more has received are The first kind,wherein these elements quiteevident, and needs only the illustration of a few examples no name, specific ; and life is is "Art de(Longfellow) fleeting" long, ; "Every man as, old would be live but sireth to man no (Swift). long,
" " " " "
"

We

are
on

such
; and
"

stuff
our

As

dreams

are

made
a

littlelife

Is rounded
"

with
come,
on

sleep."
men

Shaks. go,

Men But

may
I go

and

may

forever."

"

Brook. Tennyson's

"

in three words. vici" contains three distinct propositions vidi, Veni, and Caesar were triumvirs;"here are three prop"Pompey, Crassus, ositions: "Crassus vir trium2 d. 1st. was a Pompey was a triumvir;'* triumvir." a If,however,we say ;" 3d. Caesar was Pompey,
" " "

and Cassar Crassus,


and for simple,

were

then the the triumvirs,"


are

the three

"Koses

and

liliescontend

for

is single proposition taken collectively whole. as one So, and in her cheek," is single home a

THE

PROPOSITION.

91

simple;but
there
are

in "Darkness

and

silence settle

on

land

and

on

sea,"

four
"

propositions.
cannot

Ho

! hearts, scribes, bards, poets tongues,figures, ! hoo write, Think,speak, sing, cast, number, His love to Antony." Shaks.
" " "

In this curious sentence


it not

there

are

six distinct
to its
own

that each

answers predicate

and propositions, we subject might

were

count

thirty-six.
of Compound propositions

the

second

class, having elements

less

and requiring for this reason called Exponibles. are obvious, analysis, These more than the others require since they are attention, special and in syllogizing with them it is often requisite that more intricate, resolved. We three species: 1st. Exclusives name they be distinctly and Exceptives and De'sitives. ; 2d. Comparatives ; and 3d. Inceptives 1st. EXCLUSIVES. Compounds of this species may be formulated

thus

AisB.
=

..

..=A ..=E

(Nonon-AisB
E. g, "Faith alone

justifies"=

\"th Josti^ faith does What


.

is

not

not

justify.

It is obvious made particle faith


=

that this
to

alone," 15
"

proposition may be inverted and the exclusive thus,"Justification is by appear in the predicate; is only A.
" " "

All but one in which were Exceptivesare exemplified saved," One was saved and not saved ;" I and O. means Nearly all were No useful rule can be given for the resolution of these two forms of exponibles.Generally, if not always, the elementaryjudgments differ in quality, is to be noted as direct and the other as indirect and one or implied. The distinction between the exclusive and exceptive is of no forms convertible, practical moment, as they are readily the only difference being that what is the direct judgment in the one of the exbecomes the indirect in the other. The following some clusive are and merely,sole, : only,alonet exclusively, particles exceptive the annexed to the subject solely, but,etc. These particles quantify i.e., He is all the wise. God alone is wise," predicate universally ; as, Annexed to the predicate to that predicate they merelylimit the subject
*'

; as,

"

The

sacraments

are

but

two," i.e., there

are

no

more. pression ex-

"Wo

give some
to

of their various- modes examples illustrating None facilitate the recognition of them hereafter.
" "

but

the brave

deserve the fair

(Dryden) ;

"

fool thinks

none

except

92

OF

JUDGMENTS.

Caesar,was ;" merely patriotic ;" Brutus, in killing satellite ;" or, is only our The moon Christ is the only Saviour ;" is our ;" Mercy but murders, pardoning those that only satellite "The kill" (Shaks.); paths of glorylead but to the grave" (Gray); i. e., we "God alone is worthy of being loved for his own sake," sake,and all other thingsfor God's ought to love God for his own ways sake ; Only those riches which you shall have given away will alabide with you," Quas dederis solas sender habebis opes tial, (Maris in the exclusive or exceptive particle Ep. v, 43). Sometimes the sense, but not expressed one Lord, one faith, (Thereis only) ; as,
himself
"

wise

"

"

"

"

"

"

"

"

baptism (Eph.iv,5). contain in which 2d. COMPARATIVES. we Propositions compare two judgments; for it is one to say that a thing is such,and one less so than another thing. Thus, other to say that it is more or affirms that Pain is the greatestof evils," of Epicurus, the maxim This is more and that it is the extreme that pain is an one. evil,
one
"

"

evident
two

when

we

consider Stoics The the

that the maxim

may

be

contradicted

in
no

ways. pain is an but

The evil. denied

denied

the first component,

saying that

why may not the Pain is a great evil ?" Because what is said here as predicate, other evils merely excludes other evils ; but in the above comparative included by what is said. are expressly When and DESITIVES. 3d. INCEPTIVES we say that a thing has
But
"

however, allowed the first compoPeripatetics, nent, is the evil. that vice extreme second, saying be said of any proposition same having a qualified

commenced,
the "I

or

ceased to

be,such,we

make

two

one statements,

about

thing as before,the other as begin to believe" affirms that


believe ; and Observe
"

the time indicated. Thus, after, I now and that heretofore believe,
traries. con-

I did not

I have

ceased to believe" affirms the two

simply "I believe" says nothingof the began to be marble," and, Augustus Home past. Again, "With These may fairly be With Augustus Ptome ceased to be of brick." and left it of brick, as "Augustus found Rome saying, interpreted and desitives are compounds becomes a little marble." That inceptivcs consider that a questionsuch as "Have evident when we more you 2" affirms the component that you have been drinking, quitdrinking and questions drinking. only the second,whether you are now that which classed not observed are It should be judgments many as compound,whose outward form is simply S is P," nevertheless imply in thought an indirect judgment. This is true of every semithat to say
" "

THE

PROPOSITION.

93

definite

judgment (" 8). Sometimes, on the other hand, we convey ing leavour merely insinuate," ; we thoughtsby indirections expressed real meaning,to be understood. the direct judgment,our Logic its to with the latter, postulate, and, according alwaysdeals primarily
"

givesit completeexpression.1"

"

12.

sixth division is of

rather judgments
:

than

of

propositions.

It is exhibited in the

scheme following

Analytic
Judgments
are

priori.
or empirical. Postcriori

Synthetic\
(

a a

priori or priori

pure. pure.

S as somethingwhich P belongs to the subject predicate in the concept S, the judgment is called is contained, though covertly, of the adds nothingto the conception Analytic. Since the predicate but only unfolds its constituent marks, which are thought subject, it is also called Explicative. in the subject, though confusedly, already, I need not go beyond the concept extended. E. g., All bodies are but need merely body in order to iind extension connected with it, in it. this predicate in order to discover to analyzethe conception The analytic judgment is a priori. It is not grounded on experience, to form because I need not go out of the sphereof my conceptions it, of i s and hence resort to the testimony experience quiteunnecessary. is not That extended bodies are an empirical judgment,but itself its necessity stands firm a priori. It is a necessary judgment, arising from the ground of identity.Analytic tant, judgments are highlyimporof distinctness but are so only because by them we attain the in the and extended synthesis which is requisite for a sure conception progress of knowledge. P lies completely out of the concept. When, however,the predicate the judgment is called Synthetic. Since S, though connected with it, of the subject increases the conception the predicate by something ered could have discovwhich contained in it, not which was no analysis this judgment is also called Augmentative or Ampliative. in it, is something totally E. g., All bodies are heavy. This predicate When the

derived mostly from Arnauld, of compound propositions, analysis for these supposed to be exis intended to serve haustive. logical purposes, and is not even and To the student of Logic it will be sufficient in most generally cases, illustrativeand helpful. The above

16

94

OF

JUDGMENTS.

different from

what

think necessarily

as

contained

in the

concept

body,and
those
a

adds

to the content
are

of that notion.

Syntheticjudgments
priori. The

subdivided
are

into

those

and posteriori

which as judgments from experience, such are alwayssynthetical. I cognizeby analysis the concept body But now I augment throughthe marks extension and impenetrability. of body, I find knowledge. Looking back on my experience my connected with the above marks ; so I amplify ception conweightalways my of it this additional mark, saying, by predicating Body is heavy. is the ground of this synthesis, because the notions body Experience and weight, though one is not contained in the other,stillbelong to another contingently one as parts of a whole of experience. But synthetic judgments a prioriare not grounded on experience, does experience E. g., Every nor help us at all in forming them. has a cause. event The concept event implies antecedent time,from which I could form of a an judgment. But the conception analytic lies quiteout of the concept event, and indicates a thing encause tirely different. This judgment, therefore is not analytic.Moreover, from which I derive the conception of event does not the experience of cause, and hence experience is not the ground include an experience of the judgment. Again,the judgment has a universality which experience that cannot of can never a come give, expressing necessity is essentially which contingent. Such a judgment is, experience, How is a a priori. What, then,is its ground ? therefore, altogether synthetic judgment a priori possible? This is the questionwhich in his Critique Kant undertook to answer portance Its imof Pure Reason. is inestimable, for upon this class of synthetic or ampliative of the whole judgmentsdepends speculative knowledge.17

former

13. "'

in

considered two kinds of wholes we topic18 previous under which mind its objects the logical, or or contemplates itative qualand the mathematical, whole. Under the presor quantitative, ent have thus far considered we topic judgment as in the former only, Under
a
"

Vernunft, " 4. The distinction of propositions bk. ch. followed (Logic, i, vi), by Bain the above famous bk. i, ch. ii, the same seems as tion distinc(Logic, substantially " 7), of by Kant judgments into Analyticand Synthetic. Those logicians reject, a nd all consider a however, the class of synthetic judgments priori, synthetic judgments real propositions to be a posteriori or or empirical. 18 Part 2d,vi, " 2. See also Hamilton's Locjic, pp. 379,380.
to

17

See Introduction into Verbal

Kritik der reincn Real


made

and

by Mill

96

OF

JUDGMENTS.

declared relation between reduced


to

two

notions

or

terms.

Can De

all relations

be

these

three?

Are

there not

others?

Morgan

sists in-

that relations
to

include X

them
has
a

"Every
We

and proposes distinct are very numerous, essentially thus: all in one generalized copula of relation," relation to some Y," embracing the above, and also
"

such connectives
shall of the for

as

in

"

controls

Y,"

"

X
no

causes

Y,"

and

many

ers. othtension ex-

see subsequently copuiar meaning;

there

is

need

for this

great

can mass

adding "is greater than" in Logic. It recognized be construed as compound.


greaterthan that
"

but there appears to be a necessity and its obverse to the meaning commonly
is true

that
"

judgment comparative
mass as as we

Thus,

The

of the earth

is

of the moon"
mass

means,

have much

seen as

in the
mass

precedingsection,
of the moon,

The

of the

earth is

the

somethingin addition." But if the comparison of the copula, then combe accepted parative as an interpretation expressed would to be in thought quite simple. seem propositions of two planets, I judge simply I mentally the masses When compare is greaterthan the other, without at all thinking and directly that one The and has somethingto spare. is as much the other, that one as construed ical, mathematas copulathus understood,and the proposition from law disappear.29 difficulties arising syllogistic many Both terms of judgments of degreeare always individuals viewed dividual inmathematical of designating wholes. There are various modes as such as by the definite article, by demonstrative and possessive
and has
" Thou art the man pronouns, etc. ; e. g., wholes. Collective wholes These are integral

;"

"

This

is

our

home."
"

often

occur

e.

g.,

legionis ten cohorts." Another mode is by a proper name, or by mark ; as, Aristotle is the Father of Logic." Every some particular whose is quantified, "all" or as "some," is predicate proposition
" 30

The

two

copulasabove
The

described

express

the relation of
and

degree between
as

two

individual wholes. is expressed by the


"An
arc

relation between

the whole
"

its parts, merely is part

such,

o/the ;" part of;" e. g., This is of circle." different a a quantitative o/the judgment part kind from but does not seem to require especial that of degree, exposition. that "comprehends" in the qualitative whole is similar It is worthy of note is greater than'1'1 in the quantitative is contained under" is strikingly to liko ; and is lessthan." real. AVe quantitativeBut there is another correspondence more ly is contained think the relation of whole and part, and well as qualitatively as under" corresponds to "is apart of." For example, teachers. The preachers contained under (or, are a class of) are section the teachers. The preachers of a rc a a are of) (or, part
is
a
" " " "
"

copula "is

The

thumb

hand

THE

PROPOSITION.

97

brought into thereby


are some

the mathematical
;
e.

whole, and

the

"

all" is not

but cumular distributive, birds." Here


=

" all reasoncrs are g., All men individual totals. both terms are

;"

"

Ducks

The

braic alge-

is a 2x3," and 'V" / (a;+y)(ar"y)," equation, as, "6 its two members individuals. are character, judgment of the same called quantitative, because primariAll such judgments are properly ly, and essentially they alwaysrelate to space or time, fundamentally, the science of quantity. the bases of mathematics, An individual may be known by the test that its partsare not kinds. in become in " 6 that an individual cannot We have seen a predicate In the mathematical whole the predicate, whole. well the logical as the individual predicate the subject, as is, beingalwaysan individual, the characteristic mark of a judgment of degree. therefore, of these propositions, and a test of their A consequent peculiarity is that they are all simply convertible. No special equivalency, symbol Since the subjects is needed. vidual are total, theyare treated like indiand symbolized with this (" 8), by A and E (" 10), propositions that whereas individual propositions marked tible inconverare difference, of degree is always and only simply " 7),the proposition (ii, the copula is greatconvertible. When the terms not equivalent, are er in conversion than" must be substituted by is less than" and. vice versa. When either term may be substituted they are equivalent,
=

"

"

wherever

the other

occurs.

be discriminated from individual, terms must with which Singular is a logical "A man" they are apt to be confounded. qualitative of member the class one whole, meaning man," and is a single That man," which is a mathematical, thought very different from The whole. first is singular; the second, individual. quantitative liable to be confused with equivalent are Singular propositions sitions, propoit is in both ; but surely of the terms because of the oneness there is generality animal evident enough that in A horse is an there is animal and no equivalence This horse is my ; whereas in and no generality. equivalence Likewise let us distinguish coextensive and equivalent notions. between Two coextensive logical wholes are aptlysymbolized by two concentric circles whose radii arc equal. But it should be kept in and hence are mind that these circles are mathematical quantities, rather equal. But coextcnsion belongsto the logical or equivalent, coextensive The following are whole, and is essentially qualitative. notions : ;" En;" Triangleand trilateral Honesty and probity
" " " " " " " " "

98

OF

JUDGMENTS.

;" dogensand monocotyledons


and "Double-refracting and to become extension
is neither
master

Acotyledonsand "To crystals;" polarizing


of one's self."

"

flowerless

;" plants

But

conquer one's passions when the fact of co-

ment thought of,i.e., whenever the judgis is the subject consuch terms in extension strued simple, containing the when in thought as contained under the predicate.And be replaced is thought, still the copula cannot coextension by the is equal to" but it should be read is coand read extensive signof equality
nor expressed
" "

with." Also
we

must

not

be embarrassed

by

the factitious

of generality

and doubt that the terms are individual propositions, many quantitative " A is equal to the quanA=B The quantity totals. ;" this means, tity
"

able, equalquantities, purelyas such,are indistinguishthe quanof A is the same The quantity as (isidentical with) tity of The strength Men are stronger than boys means of B." of boys." "Every diameter is a is greater than the strength men The lengthof every diameter is equal to the double radius means than The superior more move slowly planets lengthof two radii." is less than that of The the inferior means speed of the superior The specific Iron is not as heavy as lead the inferior." means old as Circus jokes arc of iron .is less than that of lead." gravity
B

;"

or, since

"

"

"

"

"

"

"

"

"

"

"

"

"

the hills "


"

"

means

The
"

Women The

love best

age of the one " Woman's means

is

equal to that

of the other." any

love is greaterthan

other."

"

color of her eyes is the color of the skies."


mere

It will be noticed vidual indi-

that

abstract

are qualities

i.e., as thought quantitatively,

and totals,
are less,

when

if indistinguishable as abstract, greater and is the


same

identified
to

by

"

as."

wholes and the mathematical logical convertible in thought, often readily such transference requiring are of few verbal changes or none to adapt the expression to the mode presses Mankind," which in the very form of the word exthought. Thus be replaced notion containing under it species, a general may and race," which is individual, having no species, by The human into sections. So there are kinds of army ; and can onlybe partitioned all there are wings of an army. Being or thingis general, including is not a general kinds of existing notion, things ; but the Universe the but a mathematical whole, a collection of all thingsinto a unit, only one not a part of any other,and is capableonly of dissection. be thought as Again, the term animal is general ; but animals may collective whole comprisingmany individuals similar in certain (isa It remains observe that the
" "

THE

PROPOSITION.

99

sential such
as

and this whole respects, the part saved The


in the

may

deluge.
in and

indefinite article
ways ;

either of two

by thoughtinto parts, ark, and the part destroyedby the be interpreted a qualifying predicate may Gold is a metal thus means cally logi" "

be severed

think it that gold is a kind of metal,but we strictly may that gold is a part of metals taken as a collective mathematically, notion may be thus transmuted whole. In short, perhapsany general reduced in thought to a mathematical collective whole or a quantity, its species of many similar individuals, ered becoming dissevconsisting members. and the essential difference between the two modes weightyfact, is the reason, I apprehend, of thought, not being recognized, why and a number of subsequent Hamilton have attemptedthe logicians and to equations, reduction of all propositions proposedtherebyto the old logical But such reduction is artificial. supersede system.21 It exhibits the processes of thinking, not as they really occur, but in forms into which less violence. or they may be construed by more of Logic is possible Such a presentation only because of the power which the mind has of transmutingits notions from logical wholes
and

This

partsinto mathematical wholes and parts. On the other hand, the old Logic was limited
would part. A Latin logician
a

to

the

whole logical

and

called

judgment of degreeis a case and would relation all such insist on belongingonly to the logical forms being construed in the logical whole. no Hence, perhaps, bol symto such propositions, was nor were assigned they otherwise recognized. But these propositions in constant abound, they are use, they stand in ics and mathematas frequently premises all kinds of reasonings,
" "

probablydeny that what we have of predication at all predication

consists

may, it is true, transmute but we then incur the most strictly logical,

of them.

We

them

into

ositions prop-

ments serious embarrass-

in the

this
with

again is

law. Moreover, attempt to bringthem under syllogistic for we not natural, not thought as it is, reason artificial,

It is without any such transference. propositions needful, then,to admit them to a prominent and importantposition in Logic if we would thinking. truly representhuman mathematical

and his Notably George Boole in his MatJiematical Analysisof Logic (1847), his the of Laws will A Investigation of principles of Thought. very good resume be found in Bain's Logic, make Jevons would Logic mechanical ! pp. 1 90-207. See his LogicalMachine,"facingthe titleof his Principles of Science.
"

3i

100

OF

JUDGMENTS.

is the form In each of the following propositions, " 14. Praxis. is it simple, what? or (" 7). If categorical, conditional, or categorical, duce or complex,or compound? (" 11). If simple or complex,rethe copula (" 6). form to strict logical (" 4),and interpret and quality solve Affix the symbol of quantity (" 10). If compound, re-

it into

its

elements, and

affix the

or express it as an equation and honor the king. to fear God 1. It is the duty of every man like home. disinterested. There is no place are 2. Very few patriots

symbol to inequality.

each.

If mathematical,

0. NothinoO

is harmless

that is mistaken No
news

for virtue.
news. me.

4. Men

are

all sinners.

is

good
"

5. All these claims

6. One 7. Not
8. Not

upon my time overpower is right. whatever is, truth is clear,


if any

Pope.

many

metals

are

without

lustre.

9.

beingrich is not alwaysan evil. Diogenes was no fool. but that which there is nothingbeautiful, -existent, Except the self
is not.
"

Rousseau.

10. 11. 12. 13. 14.


15.

into vice. passing if it is knowledge. Virtue is teachable, The rich are not therefore happy. All is not gold that glitters. civilization. of the highest None but Aryans are capable of Virginia. the father of the University Jefferson was

Hardly any

virtue is safe from

Ah The

! few shall part where


snow

many

meet,
"

shall be their

winding sheet.
far
as

Campbell.
flatters.
"

1C.

17.

Charityaffords relief as loves most He who truly


toucheth quarrel
his

possible.
he who
us

is not

18. The 19. After 20. 21.


22. 23. 24. 25. 26.

none,

but
and

alone.
were

Shaks.
no more.
"

death,resistance
my
serves

order

Gibbon.

I propose
Whereto

That

thou art

Burnet. thoughtsonly as conjectures. of the visage mercy, but to confront Milton. happy, owe to God.
" "

offence?

Arrows George Eliot is Mrs. Lewes. Though this be madness, yet there's
Those

are

swifter than
in it.
"

eagles.

method

Shaks.

here present constitute is


no one

the class in

Logic.

There But

howsoe'er fireside,
vacant

defended,
won

has

chair.
"

Longfellow.
the
"

27.
28.

is Saltpetre

nitrate of ask not

potassa. That horse


if thine eye

race.

There

are a

who

be

on

them.

Wordsworth.

29.

There's

that shapesour divinity them how


we

ends,
Shaks.

Rough

hew

will.
"

THE

PROPOSITION.

101

30.

The

time
"

has

been

my

senses

would

have

cooled

to

hear

night

shriek.
31.
32.

Shaks.
so

Nothing is
Some few books
to

easy
are

as

to

object.

He

is
to

as

wise

as

Solomon.
some

to

be and

tasted, others
"

be

swallowed, and
L.
do

be chewed
now are

digested. Bacon, Essay


There's few
grave
or none

33. 34.
35. 36.

Our Who The That And

revels lived

ended.
I could

know

me.

king,but

dig his
the

"

Shaks.

longer the day, the


he is mad, 'tis true

shorter
;

night. 'tis true, 'tis pity;


can

pity 'tis'tis true.


not
a

"

Shaks.

37. There's Like


38.
39. 40.

joy the
far

world
"

give,

that it takes away. alms


are

Byron.
his
means.

His

beyond

I will not The


Even

let thee

go, unless

thou

bless
was

me.

author
a

of Novum

Organum

not

the inventor

of Falstaff.

41. 42. 43. 44. 45. 46.

when fool,

he holdeth

his peace, is counted

wise.

It will The

hardlybe
is

sufficient to resolve
was generals

only a

few

of these

examples.

most

skilful of

Napoleon.
dishonest.
of forms

Every sly act Logic is


Not
one

than nothing less (orelse)

the science of the necessary that saith unto the will of my


me,

thought.
enter

every

Lord, Lord, shall

in,but

he that doeth 47.


48. 49.
50.

Father.

circle is the
duties
was are

Your He

of greatestarea. figure not another's. My


man

tasks have

are

all but

impossible.
many
errors.

too

impulsivea
is the

not

to

committed

Yonder He

forest

refuge of

outlaws.
"

51.
52. 53. 54.

first and

last will

reign sole king.


the Union.
women.

Milton.

for Congress legislates

Mankind God's The Le

are

all men

and

is to be word, exclusively,
most

testimonyis merelyprobable. received without question.


before
"

All

55.
56.

sublime

act

is to

put another
virtus.

thee.

salut des vaincus sola est

est

de n'en

point attcndrc.
"

Tr.from Virgil.
20. viii,

57. Nobilitas
58. 59. 60.

atque unica

Juv.
"

Sat.

Nullas Nemo Melior Latin That

habet

spes

Troja,si tales
a

habet.

Seneca.

nisi Ia3ditur,
est

seipso.
"

Id.

61. 62.

has

fortes. et vir prudens quam vires, sapientia quam been a dead language for five hundred years. survives is the fittest.

which

102

OF

JUDGMENTS.

II. INFERENCES.
of modes previous topic we have examined seven An eighthremains, so important dividing judgments or propositions.

"

1. Under

the

that each part calls for division is

separateand
the various
as

extended

consideration. which

This

grounded on

processes
:

by

judgments are

formed, and may

be stated
(

follows

Intuitions.

Judgments

are

"

( Inductive.

( Inferences.

-J
( Deductive,

( Im mediate.

-J

Mediate.

synthetic judgments of Kant These kind being empirical, the other pure. one from which all knowledge, the ultimate premises but They lie on the threshold of Logic, the science of principles. to Philosophy,
Intuitions
are

the

described, already the ground of are arise all other judgments.


longs their discussion be-

Inferences
from
we

are

defined by Aristotle to be
down follows and of

"

enunciations

in

which,
what

somethinglaid
have
laid down

admitted, somethingdistinct from


"

Locke says, To infer is necessity." laid down as true,to draw in nothingbut,by virtue of one proposition another as true." (inferre) Says Mill : It is the act of drawing a conclusion from to infer is to derive a premises."More generally, premisedjudgments. judgment from one or more inferences Inductive are synthetic.They are universal judgments and of empirical derived from cases intuition, particular furnishing Their importance is so great premises for subsequentdeduction. discussion of them will require that an adequate a distinct treatise. Deductive inferences are analytic. They are inferred judgments of than that of the premises. They are the subject equalor less generality and mediate. immediate and are of two kinds, of Deductive Logic,
"

When

two

notions

known

as

related are, in
intervention

modified
a

cluded form, con-

of each other without


medium of

the

of

third

notion

as

In this case the inference is immediate. one comparison, from another. There is but one premise, judgment is derived directly the the givenjudgment ; and the derived judgment merely represents givenmatter in a modified form.

104

OF

JUDGMENTS.

the Finally,
"

"

Immediate
and

Inference
is not

by
an

the Sum

of

several Predicates but merely all, Copper is red,


"

of Thomson

McCosb,
of the

inference at

compound judgment
"

obvious
"

sort.

Thus,

"

merely compounded of Copper is red," Copper is malleable," It is strange phrasing etc. to call it inference from these components. It is also quite remarkable an that McCosh includes under this head the bringingtogether the components
of
a

and malleable, ductile,

tenacious

is

definition.

"

3.

As

preparatoryto

an we

account

of

those

several kinds
we

mediate of imstate
a

inference for which

shall have

subsequent use,
of the
may
not

in the form to all deductions prohibition applicable RULE : The quantification We must not be increased. all to from all,
some

following
infer from
some

to some,

from

all to some, but is said

from
some

to

all.
no

It is

evident sufficiently
a

that what

only of

furnishes

ground for

statement

all. concerning

"
two

4. Active

and

Passive.

The

change from

active to

and passive, The

vice versa, is the firstform forms


are an

of immediate

inference to be noticed.

but they seem usually regardedas merely equipollent, the inference,


of
one
"

to be rather

from

the other.
is the

In

"

God

made

the

world,"something is said
In
"

God

;" he
the

of thought. subject
u

is The world,"and subject since the somethingis said of it. The inversion, too, is only partial, made is in the predicate Hence notion in both cases. I would prefer to consider this change as an immediate inference ; but it is a of littleimportance. question
was

The

world

made

by God,"

"

"

"

5. There

are

two

kinds

of immediate

inference

introduced

into
"

which,beingvery similar, Logicby Leibnitz, may


Added Determinants.

be stated

together,
terms

The
new

same

mark

may

be added

to both

of

judgment.
"

The
"

judgment thus
is

formed
"

is inferred from

the

cheap fuel ;" since Science is system," then A false science is a false system." is narrowed, is more The extent of both subject and predicate closely in a mark, going from genus to species. determined. This is thinking
Thus, since
Coal then fuel,"
"

other.

Cheap

coal is

We

add that the subtraction of the and


is predicate

same

determinant
not
an

from

both

ject sub-

also

but legitimate,

inference. the other.


same

Complex Conceptions.
two

This inference is be added


as

to parallel

The

terms

of

judgmentmay

marks

to the

concept.

INFERENCES.

105

then " A scientific arrangement is a Thus, since " Science is system," then The consumption systematic arrangement ;" and since Coal is fuel," of coal is the consumption of fuel." Two judgments may of one be amalgamated on this principle, the terms being added as
" "

marks
of

to the

terms

of the other.
"

Thus, since

"

museum

is is
a

tion collec-

A scientific museum then objects," interesting collection of interesting objects."

tized systema-

"
the

6. Inflnitation.'

This mode

of immediate

merely negativejudgment to the in the outer, infinite sphereof things, the subject (i, " 9). It places and limits it only by the subtraction of the predicate from that sphere. Thus, from "The soul is not mortal,"I immediatelyinfer The soul is non-mortal." These propositions that express different not identical, but merely similar. The thoughts. They are not equal,
"

inference passes from infinite judgment of Kant

inverse inference
of
an

is included

under
a

the
mere
"

same

name

tion ; i. e., the reduc-

infinite proposition to

called infinitation.
we

Thus, from
"

is also, for convenience, negative, Quakers are non-combatants,"

combatants." not Quakers are immediately infer that purelyaffirmative and doublynegative judgments are said to iinitated thus,since and
vice
versa.
"

Also be in-

Man

is

then mortal,"

"

No

man

is

non-mortal;!*

the by infinitation, RULE : Change the quality of the judgment and of the predicate. This is done, if the premisehas either a negative copulaor predicate, the negative from to the other; if one by simplytransferring particle both are negative, it from both ; if neither, by subtracting by adding it to both. Observe that, of the judgment is always though the quality the quantity remains unchanged. This process Bain calls changed, but he denies that it is properly Obversion," an inference, insisting that the two notions arc mutually under the law of Relativity. implied To avoid awkward compounds with non, we make use of a privative has been or as as suffix, in-, prefix although, -less, etc., un-, dis-, remarked, words so formed are often not pure negatives. repeatedly of the quality, For example, but they often mean, not the privation the existence of it in a low degree So uncom; as, unwise, careless. crooked. We pounded negativeterms are generally impure ; as, night, inference
"

Hence,

for immediate

Commonly
in
a sense

in

different

ever, J^quipollence.We use this word, howlogicians the to mean accordant with its etymology, same more only thought Part 8. See 1st, phraseology. ii, "

called by the old

106

OF

JUDGMENTS.

are,

then,to be

on

our

guard in using such


much.
"

lest we tion,

derive too

Under

express infinitathis precaution add some we


terms
to

illustrations as
Since
"

follows,
are

All metals No

fusible ;

then
"

No

metal

is infusible is

A E I 0

E yields
"

miser is
sins
men

"

Some Some

"

happy ; are pardonable ; not gentle are ;


a

Every miser
Some Some sins
men are

"

"

are

unhappy not unpardonable. ungentle


. .

A O I

"

"

We
as

may

in the

thought through a following example,


pursue
"

series of immediate

inferences,

Since Then

Some Some

Then Then

Some Some

thingsare not intangible ; invisible thingsare tangible ; (Convertsimply.) tangible thingsare invisible ; tangible thingsare not visible
invisible

0 I

I
0

cians, Morgan, followed by Thomson, Bowcn, McCosh, and other logifrom the first by a complex rule, and derives this last directly of infinitation ; but,as it obviously volves inclasses it as a second method modes confuses two of inference. to do so conversion, needlessly
De One
Since Then

other

example,
"

Then Then

Every unjustact is inexpedient ; No unjustact is expedient ; (Convertsimply.) No expedient act is unjust; Every expedientact is just
would contend

A E

"

E
A

Some

moralists who would

of this for the firstproposition

ries, se-

hesitate to admit

the last. But

the inference is necessary.

"

7. Conrersion. and

In

immediate

inference

the by conversion,

ject subare

with change places predicate

each other ; i. e., the terms

transposed.Besides
we

observingthe generalrule given above (" 3),


to

must

take

heed

make

total transfer ; i. e., the whole the whole naked

naked

made predicate is meant without its signof quanterm a term a naked tity, has been a boy," Thus, from all,some, etc. Every old man infer that cannot we Every boy has been an old man ;" but only old man." Some has been a boy is an who one Hence, to avoid to reduce the proposineedful before converting tion error, it is generally and predito its strict logical form, that in which subject, copula, cate of will three We consider kinds illative only distinctly appear.
must subject subject.By
" " "

be made

and predicate,

INFERENCES.

107 need subsequent


convert

and conversion,

these

onlyso
\ve

far

as

our

in

ing syllogiz-

which requires,

that is,

be able to

each

of the four

A, E, I,O. judgments
the terms without changing the Simple conversion transposes of the proposition. It may be applied the quality to E, or quantity
1st.

and to I.

Thus,
"

Since JS"oone Then Since Then No


true

without

poet is

sympathiesis a true poet without warm sympathies ;


warm are

E E
I

Some Some

good
poor

mathematicians financiers
are

poor

financiers;

good

mathematicians

by A or E, is always " 13), symbolized judgment of degree (i, and only simplyconvertible. of a proposition 2d. Conversion per accidens reduces the quantity but leaves its quality (hencealso called C. by limitation), unchanged. is I. Thus, It is applied to A, cud the converse
The
"

Since Then

All Some

rectilinear figures are planetriangles ; rectilinear figures are planetriangles

"

A
=

The

name

was

the universal
If
we

givenby Boethius,because it is not which per se, but only of a particular


hold to the rule that affirmatives do evident that the

conversion

of

the universal includes.


not

distribute the

of the convertend, linear "rectipredicate in becoming the subject does not change its quantity figures," of the converse. of the conthe subject vertend, But, for the same reason, of the affirmain becoming the predicate tive planetriangles," reduced. Also observe that our has its quantification converse, the I rule (" 3) forbids us to retrace this step to reconvert general

it is predicate,

"

"

into A. 3d.

E also may be converted per accidens. Conversion by contraposition changes the the
converse

of quantity

It proposition.
is I.

is

to applied

the

but not the quality remainingjudgment


we

O, and the
RULE
:

In order to then convert


wholesome air is pure

contrapone

have the following


"

and Infinitate
Some Some
course

simply. Thus,
;
=

Since Then

pure

air is not

0
I

unwholesome

This is of

compound process, and was devised to convert O, It has been also "which cannot be converted simply, or per accidens. called conversion by negation." obvious that the doctrine it is sufficiently Upon a slight inspection An intensive of conversion has respect to judgments in extension.
a
"

108

OF

JUDGMENTS.

time changing its judgment cannot be converted without at the same into a concept ; as, All men into a mark, and its predicate subject
"

are

mortal
in

"

converts

to

"

Some

mortals

are

human."

Otherwise

the

be changed to extension. must converting lows (i, " 6),it folAgain,since an individual cannot become a predicate The individual judgment (i, that no " 8) can be converted. be held inconwhen used to representit, vertible. must " 10), symbolA or E (i, and is pretty," We say "Venus may say, "Something prettyis Venus ;" but this apparent conversion per accidens is only a This gives of thought is stillVenus. rhetorical inversion ; the subject
view

occasion to remark

that

no

mere

inversion is

conversion. logical

"
each

8.

Opposition.
forms

and predicate given in subject


is in
to opposition

either

one

of in

the four

A, E, I,O,

the

same

matter

of the

other three forms.

The

is opposition

such

that if the

be taken as true, or as false, can we given proposition of the others. of at least some infer the truth or falsity exhibited upon a diagram, thus, kinds,usually
"

immediately
It is of four

All Salt is

Pure,

Contrary

No

Salt is Pure.

SQUARE

OF

;"

"

"*.

"^

OPPOSITION.

\S
Some Salt is

\J
Subcontrary
"

Pure,

!
"

Some

Salt is not

Pure.

1st.

exists between Contradictory opposition naked


or

the

same

and subject unquantified

propositions having but which differ predicate,

in both cannot

be

quantityand quality. Both cannot be true, and both false. of the laws of statement This is merely a specific
and Excluded Middle.
an

Contradiction
be

sublated
"

then by (denied), Some


we
"

E. g., If A, " All Salt is immediate inference we can If I, " Some


"

Pure,"

posit

O, (affirm)
it is true

Salt is not
can

Pure."

Salt is

Pure," be

then posited, that

that

"

Some

men as

such doctrine,

If immediatelysublate E, No Salt is Pure." then it cannot has a conscience," be said Every man have no conscience." Again,if you prove that A mind and body, is to be believed, the connection between
"

though it is not comprehensible," you


"No

have it is

shown thereby

that

doctrine

is to

be

disbelieved

because

incomprehensible."

INFERENCES.

109

Such

in are said, propositions Aristotle used the

common

to phrase,

be

diametrically posed. op-

for diagonal

the

of A contraryopposition
"

and

E, and
is

for

this reason,

the phrase diametrically perhaps, posed" op-

it being applied both to contraries and to contradictories.4 ambiguous, form of is the only perfect Contradictory opposition all others being more less imperfect. or opposition, Proof is direct and indirect. If we wish to refute an adversary, we

may

show

that his

arguments

are

do false,

not

sustain his

assertion,

and is fails. The result is merelynegative, which, beingunsupported, and prove his asserwish to go further, often sufficient. But we may tion false. positively the method If this is done
is direct.

by
if

an we

attack upon affirm the

his

own

sertion, as-

But

contradictory
his assertion

infer and, having established it, immediately proposition, the method false,
"

is indirect.
are

Thus, if

one

affirms with

Hobbes

that

All human

motives
"

to

Some one prove that If this be established, then the this O that his is,
A is false.

we selfish," alwaysultimately motive in some case single

may
was

undertake unselfish."
from

immediate, necessary inference

The

proof called

reductio ad
use

absurdum

is indirect and of

similar. quite

Euclid makes

much

of it.

Instead

he a proposition directly, demonstrating


is absurd

demonstrates

that its contradictory

and

thence infers its truth.


A

2d.

exists between Contrary opposition

and

E, universal propositions

in quality differing only. Both cannot be true, but both may be false. Between these propositions there is a tertium quid,namely I and O. If A, All S is P," be posited, E, No S is P," is sublated,
" "

and vice

versa.

But that

if either is

this does sublated, S is P"


"

not

the other, posit To

for it may "All Stars


are

be
are

"Only

some

I and

O.

deny
some

that Stars
are,

Planets"

does not

afford the inference that "No


case are

Planets
some

;" for it may


not.
wars are

be, and in this


sublate
"

true is,

that

and

are
"

To
are

No

wars some

evil

"

does not
some are

giveposition
not, then

to

All

evil ;" for if

are, and

both

the others

false.
or

When, however, the judgment


4

all disis individual, proposition

The

Aristotelic doctrine
is the

of

from differs considerably Opposition

the

one

here

which given, thus: L'opposition (TO.avriKiifj^va) peut


"

approved Scholastic form.

Saint-Hilaire represents the former


etre

de

quatre espeees.

II y

a:

1"

et de la possescelle des relatifs; 2" celle des contraires ; 3" celle de la privation sion la de Cette et the1'affirmation de i cal 4" enfin celle negation. (TiprjmQ t"ic) ;

joue un grand role dans le systeme d'Aristote." De la Looppositions ch. x. Tome 1838). See Aristotle's Categories, giqm D'Aristote, i, p. 172 sq. (Paris,
orie des
"

110

OF

JUDGMENTS.

tinctions in

or disappear, opposition which, in such case, simplenegative,


"

rather become is the true


a

merged into the E. g., contradictory.


either

Caliban
In

is

man,"

and

"

Caliban
often

is not take

man."

controversyopponents
establish
asserts
"

and contrary positions, other


an

to failing

his
"

own

gives to
men are

the

apparent victory.
Another opposes

E. g., One

that
are

All

to be trusted."

this with

No

men cases

to be
some

face of cited
in

of his

but being unable to prove it in trusted," who are to be trusted, leaves the question
in of possession

and confusion,
not

opponent
have

the field.

Indeed,

they have
is too

faced squarely

each

the indirect method, should

opposer, in adopting which undertaken,not the contrary, other. The


"

that Some men are much, but the diametrical contradictory, would insure an easy victory. not to be trusted," which in this case I and O, particular exists between 3d. Subcontrary opposition ositions propin quality differing only. Both

be false. If
not

Hamilton

calls these
as

may be true, but both cannot " subaltern contraries, compossible."

I, Some

"

S is P," be taken
true.

true,it may
that
"

be that
must
"

0,

"

Some If
"

S is Some

P," is also

But

if I is

then O false,
true

be true. Some

it may also be Sighs are Prayers," Prayers." But if it is false to say

that

Some

Sighs are not Sighs are Prayers,"


P," and
"

then it must Let it be is not

be true

that

"

Some

are

not."
"

in noticed, however, that if,


same

Some

S is

Some

P," the

compossible." In Logic,which takes it thus,knows the sphere of the Some usually


"

then the propositions are intended, and hence strictness they become contraries,
is
no
"

"Some"

"inpure

subcontrary opposition.But
one

in the
"

is different from

that

in the other.
are perhapsall)

Thus,

if I observe may

that

Some
"

metals

(some at least,

it fusible,"

be that
"

Some

know,
rule

are

infusible."

Here

the

Some

"

is

for aught I others, and our wholly indefinite,


"

holds

definite
"

if the Some good. But, further, (i, " 8),then our rule changes from

"

be

thought as
may

semi"

"

Both Some

be true

to

Both

must

be true."

Thus,

I know

that

"

flowers
"

(some at
flowers

most, not
are
"

not

are ;" then all) fragrant This Hamilton fragrant."

it must

be true that

Some

since the two calls "integration,"

constitute the whole. Somes," taken together,

4th. Subalternate
in

exists opposition

between

quantity only. If the is false, the particular the


credit in

universal is true, the universal is false.


may draw for

propositions differing is particular true ; if


$100
at my not

If I have
or a

bank, it is evident I
I cannot credit,

$5

$10.

If I have

$10

at my

draw

$100.

This is

specific application

112

OF

JUDGMENTS.
4

The summed

chief
as

not results,
"

the semi-definite including

meaning,may

be

follows,
"I

All S is P, Some No S is not P.

Contradictories }S is

"

One

must

be true,and

the other false.

P, J

Some

S is P.

'^ P'

Contraries

"

Both

cannot

be true, but both may

be false.

'

Some

is not

T.

P. )

Subcontraries

"

Both

may

be true, but both cannot

be false.

All S is Some No

P,

-]

S is P.

iS
P.

It

tes

"

^ "$

^e

uniyersal ig ^rue" *ne

is true particular

S is P S is not

is false, the universal is false. I If the particular

Some

\]
may be tabulated also
Contradictories.
r

The

same

matter

thus,
"

Contraries.

Subalterns,
"

If A
If E

is true, 0 is is true,
. "
.

false,
, A

E A

false,
false,

I true.

I is false,
_

0 true.

Umversals

""
_. .

If A
v.

is

0 false,

is

true.

The
)

otherg undetermined.

If E

I is true. is false,

If I is true, E
If 0
is

is false. )
is

The

otherg undeterminedi
A
. . .

Particulars

-j
i

true, A

false. ) 0 true, I true,


.

If I is

E is true, false,
A false,

false.

If 0 is

is true,

E false.

Hence others
"

by
are

the truth of determined

and by the falsity of particulars, all universals,

; otherwise

only the contradictory.6

v *

The

old Latin

said to correctly is trulythe same. in the


1.
same
"

rather needlessly that opposition cannot warn us be logicians the exist unless of both propositions predicate [and the subject] violate this precaution, We when do not predicate we say they,

Manner;

as, Hector

is and

is not

man;

i.e., he is

dead

man,

but not

2.

one. living Respect; as, Zoilus

is and

is not

he black; i.e.,

is

but black-haired,

red-

faced.
3.

Degree ; as, Socrates is and but not, compared Scipio,


; as, Xestor is and

is not with
an

long-haired compared ; i.e.,he is, Xenophon.


old
man

with

4. Time

is not

; i.e.,he

is not when

boy,but

is at the

siegeof

Troy.

INFERENCES.

113

"

9. Praxis.

Draw

an

immediate

inference from

each of the following

propositions by added
1. The

determinants

(" 5):
"

wages

of sin is death.
"

Use
2.

as

and just. determinants, inevitable,

Noveltyis pleasure.
as a

Use

the. determinant, the greater


"

3. War

is

an

evil.
with

Use,
"

unprovoked,welcomed

ardor,which

reaches

to

our

hearth-stones.

Infer from
4. The

following by complex conceptions ("5): ceremonious. are ignorant


concept,
"

the

"

Use

the

5. Heaven

from

age. all creatures

an

hides the book

of fate.
"

Pope.
V-

Use,
"

wisdom

and love.

Combine
C.

each of the

into pairs following

one

(" 5): proposition


"

Honesty deserves reward. whom meet is a neighbor. we Every man 7. The year is dying in the night. Tennyson. The swift runner is speedily exhausted.
"

Infinitate each of the


8. All 9. The 10. No

("6): following propositions


"

is useful. knowledge Chinese


are

industrious. feathers.
an

have reptiles
to

11. It is wrong 12. There 13. Some 14. In 15. No


are

put

innocent

man

to

death.

studies much hearts


are

vaunted, yet of littleutility.


not

men's

in the

right place.
old.

and jewels

brutes

are

cannot men gold, responsible.

grow

Convert
16.

each of the

the symbols(" 7): affixing following,

"

Life every man holds dear. 17. Two straight lines cannot enclose
1 8. None 19.
are man

free who

do not govern

space. themselves.

With

20. Few
21.

know

are many things themselves.

impossible.

'Tis

22.

to load a falling man. cruelty Fame is no plantthat grows on

mortal soil.

114

OF

JUDGMENTS.

23. 24.
25.

Whoso Each Fair There Full The

loveth mistake

instruction, loveth
is
are no

knowledge.

proof
often
on

of

ignorance.
to

promises
falls many
no

not

be

trusted.

26.

shadow gem of

his tomb.
serene,
ocean

27.

purest ray
caves

dark

unfathomed

of

bear.

"

Gray.

From the
28.

each

of

the

following premises obtain, by (""


happy
6

immediate

ences, infer-

annexed the

conclusion
are

and

7) :

"

All
.'.

righteous
is

Whoever human All

unhappy
are

is wicked.

29.

No
.".

virtues

perfect ;
are

perfect virtues possible cases


are

superhuman. improbable
are

30.

Some
.*.

Some
true

improbable patriots are


are

cases

not

impossible.

31.

Some
.'.

not not

popular;
always unpatriotic.

The

unpopular
is
a

32.

Certainty
/.

kind

of

light;

Darkness

is doubt.

If the and what


33. 34. 35. 36.

following propositions
false ?

are

true, what

opposites arc
Young.

also

true,

(" 8) :"
an

By
No Some Some

night
one

atheist

half

believes

God.

"

is

always happy.
are

democracies

unstable.
not

great

orators

are

statesmen.

If the and what

following
true

are

false, what
"

opposite propositions

arc

also

false,

(" 8) :

37. All
3". 39. 40.

self-confident honest
men

persons become

have

strong will.

No Some

bankrupt. public benefits.


seed.

private

vices
not

are

Some

plants do

produce

INNOVATIONS.

115

III. INNOVATIONS. which " 1. Since the revival in England of the study of Logic, of Whately'streatise, about by the publication there has brought manifested
from

was

been
herited in-

much

dissatisfaction with the Aristotelic doctrines


or

as

of the Middle logicians Ages. This body of doctrine we have spoken of as the old,or Latin Logic, that it is obsolete, not meaning to intimate thereby to even or likely it from recent doctrines. vanish away, but simplyto distinguish The from of dissatisfaction has arisen not so much a supposedinaccuracy the old doctrines as from their supposed inadequacy.Many important modifications and additions have been proposedby high authorities, such as Hamilton, De Morgan, Mansel, Boole, Thomson, Mill, but as yet few have been generally Bain,Jevons,and others, accepted, its the old holds and has been the chief Logic ground. Hamilton his views have been most and made innovator, widely discussed, the deepestimpression will give our attention eswe pecially ; and, therefore,

the scholastic

Latin

to them.

"
been

2. Hamilton's

doctrine

of

the semi-definite

"

Some"

has

already

whether it should be received questionable into Logic at all, even as a mere exception. Some," if not either a wholly and simply indefinite, probably always designates else a compound or wholly definite judgment imperfectly expressed, judgment whose two elements are each whollyindefinite. If we say members of this University Some this are now studyingLogic," a certain Some," i.e., judgment in our minds would be whollydefinite, "All the members of the Philosophy Class are now studyingLogic," of the University. without any thought whatever of other members But
" " "

stated/

it is very

In

" i,
of

8.

It may

be remarked

its if fully adopted, that,

consequence

to

the old

doctrine

Opposition (ii, enlargedby " 8),


The
student

the addition of four


statement

thing judgments,is some-

fearful.
to

is referred to the tabulated

in the

Appendix

out. the whole scheme Logic, p. 535, where with the of doctrine old new the one Opposition replacing entirely of "Incompossibility," and hence better, it would to seem simpler and sufficient, treat the cases of the semi-definite meaning as exceptions to the old rules. Instead of thus

Hamilton's

is

worked elaborately

116

OF

JUDGMENTS.

should be reduced to judgment then is A, and the proposition with the thought. Again,if we that form, in conformity Some say flowers are fragrant," then this implies at most, not all," meaning some the counter-thought that "Some flowers are not fragrant."If in a grammatically this double thought be expressed simplesentence, for the logician that it be expressed, then we have postulates Only flowers are fragrant."This is an exponible some compound proposition flowers which into how Some know not (I analyzes many) are flowers (I know and "Some how not (I), fragrant" many) are not fragrant" (0). Each of these elements considered in itself, entirely is for the indefinite I meaning of ; apart from the other, wholly know how not many" must in that case be "perhaps all." The semi-definite character does not at all appear unless one judgment is this is the case the other ; and when the judgas limiting recognized ment is not simple, but compound. Now to be n Logic, professing of thought, must not stop short of its simpleelements, thorough analysis the compound as co-ordinate with the simple, must not recognize and does not, cannot, undertake to formulate the compound modes of thought, which are legion, but evolving their elements formulates only these. Therefore the semi-definite judgment,being compound, forms of thought, be denied a position must the elementary among perfect imand if recognized take its place the abbreviated, at all must among of statement, subject modes to analysis at any moment and full discrete expression. The
" "
"

"

"

importantaddition to the old Logic proposed by is his doctrine of "The Hamilton Thorough-goingQuantificationof The old Logic teaches that negatives the Predicate."8 distribute the affirmatives do not teaches that in (i, " 9). Hamilton predicate, both affirmative and negative be either judgments the predicate may distributed or undistributed. Hence, to the four Aristotelic judgments four of the old Logic he has others, superadded commonly

"

3. The

most

the Novum

called bis great work Appendix,p. 509 sq. As Bacon Logic, in the Aristotelic to allusion Oiyanon, so Hamilton calls his Oryanum, in allusion to Aristotle's Analytics," New Analytic," treatment of these for.ms the and proposes cellent thereby to placethe keystonein the Aristotelic arch." For an exof Hamilton's statement An see warmly approved by himself, views, Essay the New on Analyticof logical Forms, by Thomas Spencer Baynes, an admiring from having been a prize The Essay is the more pupilof Hamilton's. interesting See Hamilton's
" " "

examination

paper.

INNOVATIONS.

117

called the Ilamiltonian table.

judgments. These

are

included

in the following-

TABLE

OF

THE

EIGHT

PROPOSITIONAL

FORMS.

BEST

1,u, afa,Toto-total, All

men

are

all reasoners

|mcn
lovers

C:

"-

:P

"

All ii, A, afi,Toto-partial,

men

are

some

lover?

[men
poets

":

Aff.
"

Some 3,Y, ifa,Parti-total,

men

are

all poets

[tm?n
singers

(^ C,

,^_

.p

Some I, ifi,Parti-partial, plv,

men

are

some

singers

|me"
s'ngers

"

,P

"owe 5, w, ini,Parti-partial,

men

are

not

some

singers.lnc"
poets
me"
. .

C,BT- ,P C, "K
:P

"

Some vi,0,ina,Parti-total,

men

are

not

any poets.

lovers
"

Not ani,Toto-partial, 7,?;,

any

men

are

some

lovers..

meu

":""-,P
C: H:P

brutes

WORST
"

E, viii,

ana,

Toto-total, Not any

men

are

any

brutes..

mea
.

is needed. to discussion, The table explanation, preparatory of symbols, and of examples. All the consists of six columns one the same thing. In.the first symbols in any one horizontal line mean Roman numerals t he column, the designate Aristotelic or Latin judgments ond judgments. In the sec; the Arabic numerals, the Hamiltonian column, the Hamiltonian judgments also are designated by vowel Some letters:
u

for

universal ; Y, and E.
In

as

correlatives of O
or

cognate to I ; the third column,


a

u"

and

77,

as

the Greek
a

stands for

sal univer-

undistributed term ; or particular for the negative stands for the affirmative copula f (affirmo) ; n (ncyo) in each of the first consonant copula f and n being respectively needs serve obthose words. The fourth column no explanation ; but we the affirmative that its symbols are defective in not distinguishing from the negative forms, and must therefore be supplemented by

distributed

term

; i for

"

the words

"affirmation"

or

"negation."The
that "men"
for
"

fifth column includes

is of

amples, ex-

in which

it is understood further that


"

both
"

males

and each

females ; and

birds"

are instance,

lovers" of

and also other,

are

described notation, already

singers" The sixth column under a previous topic.

is the linear

118

OF

JUDGMENTS.

vice deingenious As of Hamilton's, to which,however, he gave no name. specific will call it the "Graphic Notation" it is not properly we symbolic, is expressed The subject or by C or T (gamma),the third predicate of the Roman and Greek alphabets.These are letters respectively unconscious taken that they may be indifferent, no preference being which perhapsmight not be if they were successive given to cither, lettersfrom the same pressed alphabet. The distribution of either term is exAll C." next to it ; thus C: is read The by a colon standing is expressed next non-distribution of cither term to it ; by a comma thus ,F is read Some T." The positive copulais expressed by a pointed crossed (H-). The peculiar dash ("^) by the same ; the negative of the device is that it discriminately sion advantage expresses either extenthis copula indicates intension. or Pointing to the predicate, extensive contained an judgment, and should therefore be read The seventh column
more an
" " "

calls for

remark.

It is

under;" thus C:-^,F

is read

"All

is contained intensive
"

under

some

F."

it indicates an Pointingto the subject, " comprehends;" thus C: ,P is read Other examples are: ,C:=Some F,
" "

All C

judgment,beingread r." comprehends some


under
=

is contained any

some

C;
I1

C:-H:P=rNot does not The

any

is contained

under

T;

-H:C JT,

Some

comprehend any C. meaning of "Best" and


when of any.
we

"

Worst"

in
we

the

table is this: We when

declare "best"
we
"

affirm all of all ; Each


two

declare "worst"

deny
a worse

any

of the
terms

judgments in
than any that

the table declares stands above it."

relation between

The

no require remainingpoints

explanation.
us

"

4. The

before firstquestion
77, are

Whether is,
forms of them?

the four
and uses,

Y, w, and

not
never

such

as

the mind
some

judgments, u, even though it

may tions

rarelyor
all, some,

express

True

the

predesigna-

in the above table as quantifying the occurring in the old forms we not usually so are are expressed. Still, predicate, for the predicate.Thus in A, we said to think in a quantification think All are some ;" in O, Some are not any,"etc. Now, do we All are all," Some think are also sometimes not are all," Some any,
" " " "
"

not

some," and
The
seems all,"

any are some?" evidence in favor of the natural and


"

Not

common

use

of the

afa,

"

All

are

to

be

overwhelming.

If

we

into inquire

quantity
a

of the

shall find we predicate, is predicated; thus,"Man

that this is the form is risible"


means

whenever "Man

erty prop-

is all that is

120

OF

JUDGMENTS.

and predicate to be of any moment. Each ject ceases, it is claimed, it becomes indifferent which stands first, term quantified, ment every judgof two terms. being reduced to an equationor non-equation the old doctrine of Conversion is swept away, and we Consequently terms at will.4 Upon this we mark resimplytransposethe quantified may at once, that to claim that the distinction between the subject, that which we that which we are speakingof,and the predicate, say of it, has been reduced to naught,is absurd; for to nullify this distinction would require not a mere of technical forms,but remodelling of the forms of human a remodelling thought. will concede that the two But we affirmative Hamiltonian ments judgin thought, and occur (whatever may be said of the negative) however,does not entitle them appear in our reasonings.This alone, in Logic co-ordinate with the Aristotelic forms. to a position Before what decidingupon this claim it is needful to re-examine them all somemore closely.

"
Are

5.

The

second
can

to question,

be

decided
with

before affirmatively the Aristotelic

the

Hamiltonian

be

admitted

to

rank

to show they but compound, and hence are to be rejected. they are not simple, The two negative have been rejected forms,ani and ini, by nearly writers on various grounds. They at once all logical excite prejudice forms of speech. by being so awkward, and so unlike the common and not the power, Says Thomson, "They have the semblance only, of a denial." We exclusion ; and add, that a denial is essentially an if the quantity of the thing excluded be thought of at an exclusion, exclusion is meaningless, ex m or all, is, termini, of a total. A partial rather a self-contradictory phrase. The exclusion of a part of a thing that the total portion is totally has meaning,and it is, excluded. Let that a total exclusion (a tautological is differit be remarked ent phrase) from the exclusion of a total. Moreover, we may totally exclude, without any thoughtwhatand in simplejudgment do totally ever exclude, of the thingexcluded. of the quantity no Therefore, simple ative negpredicate. judgment can have a particular If it be said that the exclusion of a part implies the non-exclusion and that this is expressed of another part, by ni,we replythat such a of a simplenegative, is compound, consisting exproposition totally

simplejudgments?

We

undertake

forms,is, if logical, that,

Hamilton's

Logic, p.

525.

INNOVATIONS.

121

arid of a simple affirmative, eluding, including.Such compound judgments are admitted to be conceivable, they may be sometimes in in reasoning, useful, they may appear as premises they may occur forms. But, being compound, they cannot claim a place syllogistic less can in Logic, much "hey take rank with the simpleforms, to and to which they must themselves reducible, be rewhich they are duced in any completelogical analysis. It is at least The form ifa has been accepted by some logicians. When Some the simple we men are poets," say very questionable. Some contained under the class poets,' are men meaning is that In neither Some that men are poetical." or, changingto intension, does there seem to be any thought case, in this simplepredication, in the predicate.It is neither "all poets"nor whatever of quantity is indefinite in an absolute sense, i.e., some poets." The quantity it does not exist in the thought. If the question stantly think inwe arises, All poets are men," and compounding the two proposithat tions for the sake of brevity, all poets we men are may say, Some ("ifa). This,then,also appears to be a compound proposition. of an I maintain here that the predicate well as of as affirmative, has strictly whatever. That assigned a negative, no quantification by the old Logic is merely in view of conversion, vance, it has no other relein anticipation of the converse. and is given solely Now a is which coming term thought without reference to quantitycannot in beof an trudes) the subject affirmation (unless another judgment inbe anythingother than the wholly indefinite "Some" (" may be next to none, or perhaps all). Hence the scholastic rule that the of affirmatives is (in view of conversion) undistributed ; and predicate in thought into I,if the judgments be simple therefore also A converts be swept away by any logical ; and the rule per accidens cannot with words, objectively, to device. For true Logic is not a juggling of what occurs what may be done with them, but a representation see in our subjectively thoughts.6 If this falls, all But the stronghold of Hamilton's doctrine is afa. the others go with it. Let us observe that the doctrine of a quantified either old or new, is applicable predicate, only to judgments in extcn" "
*

"

"

"

"

"

"

Let
term

us

rule requiring that the middle that in the syllogistic note, by anticipation,
be distributed at least once,
"

warned that the predicates of usually affirmatives are not distributed," It should be, are a pure undistributed." ative, negto which is equivalent meaning less than particular." undistributed,"
we are
" " "

122

OF

JUDGMENTS.

for thinking a good reason quantityinto the of conversion, in the old doctrine ; but to in anticipation as predicate is alwaysquantified that the predicate in thought hold with Hamilton is to exclude the judgments in intension from Logic. But,by a curious insists than any other logician, sion, he, more inconsistency, upon intenand ever, expands Logic to embrace it fully. One of the two, howbe givenup. must must we But, if we giveup intension, giveup and then Logic ceases for intension is primary, extension to be. also, have in extension no absolutely meaning Says Mill,"Propositions The Logic of the quantibut what they derive from comprehension. fied and the out of takes leaves them them, comprehension predicate This consequence is certainly sufficient to cast a a caput mortuum"
sion.

We

can

sec

shade
When

of

over suspicion
we

the well-fortified afa.

all trilaterals," arc triangles the whole to cover is it not evident that, ground occupiedby this that "Every triangle is first, statement, two judgments are required: that ?" How trilateral," and, secondly, Every trilateralis triangular is it possible that to be a simplejudgment which is divisible to pronounce of these may into two, and especially when be thought one
"

make

the assertion that "All

without when
one

the

when other,

may
"

may be known be false and the other true?


one

and

the other

unknown,

If "All All

trilaterals is Is it half
a

only one judgment,what judgment ?


support of
afa he says

is

"

all are triangles ? trilateral are triangles

In Hamilton's the

"

Ordinarylanguage fies quanti-

often as this determination of the smallbecomes est as predicate by adding a//, import This it does directly some, or their equivalent it to the predicate; the same or accomplishes predesignations E. g., Directly limitative form. in an exceptive or end indirectly, : as, E. g., Indirectly: Peter,James, John, etc.,are all the apostles.' as, is is all that God is i.e., the ''God alone is good,' good;' Virtue i. e., Virtue is all that is noble ;' On earth there is only nobility,' great.' Now the nothinggreat but man,' i.e., Man is all earthly has always been,as stated by Scheibler : Omnis doctrine of logicians alteram alteram exclusiva resolvitur in duas simplices, affirmatam, been discussed (i, " 12). If it be negatam. This view has already and exclusive propositions are correct,if such exceptive compound, and illustrations, statement that then it appears from Hamilton's own afa is a compound proposition.
* * * * ' i " e

6ioyzV, p. 517.

INNOVATIONS.

123

It may be conceded that that this form afa is familiar in speech, if you please, it is natural, that men make constant of it in reasoning, use that such which
one
or

in forms reducible to syllogistic are easily reasonings and perspicuity both premises are are afa,that brevity

promoted by its use, and hence that it should be included in every of human in this of the forms logical analysis thought. But .Logic cannot proposed analysis stop short of simple and ultimate forms. If it were art teachingns how to reason how to detect an or even in. reasoning, for an then there might be occasion elaboration error and symbolizing of compound forms,though indeed the work would be endless. But as it is on the higherground of a science, ing showone
how
we

do and must
as

it think,

is out

of character to

pound presentcomcan

forms from All A is

the results of

is all

B,

can

be
is A.

analysis.Now provedfrom one


Whatever
can
"

whatever
or

be

proved
"

both of its elements

All A

B, and All B
can

is all B, is A.

which could

B proved from its elements It is not possible that there should be a single instance in from premises with quantified a conclusion, provable predicates, not be proved from if we the same set forth all unqualified,
are a

be

proved from Some Some A is B, and All

be

those which

involved. really

If there could be such

an

instance,

the doctrine of

would be a real addition to the quantified predicate theoryof thought; otherwise not.7 of Mill, De Morgan,Bain, supported by the authority Consequently, and others, to the intrusion of the compound form we object afa,and its train, the simple forms, and reject the doctrine of The among ilton. Quantification of the Predicate," Thorough-going taught by HamWe into are glad to escape from the fearful complications and rest in the comparative which it leads, of the Aristosimplicity telic Logic; and we honor the old logicians in the belief that, during
"

the two

years of their acute and were not allowed considered, surely did not
to belong

thousand

these discussions,
in their

forms

were

system because they


ravage the flock.

the

and if admitted fold,

would

"

6. The

foregoing argument
exclude his forms

is sufficientto refute Hamilton's among


; but
its

trine, docview

and

from

the Aristotelic.

The

taken is

him completeas against

it does not

exhibit completely
us

the ultimate character of afa and


nature

cognates. Let

examine

their It

yet
7

more

We closely.

have

pronouncedthem

compounds.

See Mill's Examination

ch. xxii. of Hamilton's Philosophy,

124

OF

JUDGMENTS.

to say that each results from the accurate perhaps be more and becomes a logical judgments, simple compounding of two simple, mathematical explanation. judgment. This needs some Hamilton of distributed and undistributed predispeakscontinually cates. but only,as The old Logic,too, uses the same expressions, have said, plished accomwe which, indeed,is already precursory to conversion, is thoughtinto the predicate. We the quantification soon as as of a purely have denied that the predicate logical judgment has, simple that it is absolutely or can whatever, have,any quantification affirming thrust in indefinite. We now add,that a quantitative predesignation moves a predicate by the compounding of two simplejudgments reupon fers the judgment from the logical or whole,and transqualitative whole. mathematical it to the quantitative or Hence, if we view the judgment in reference to its origin, we may call it compound, or must no longer compounded; but if we view it in its own sense, we but a simplemathematical call it a logical, judgment (i, " 13). For, consider the meaning of "all" in the predicate.It is not, it but is always the cannot divisive, be,the distributive, exemplar all," whole. cumular indivisible, "all,"a mathematical total, E.g.,"All are men bimana;" this is the distributive "all," meaning that all,

would

"

each,and every
us

man arc
"

is in the

say "All

men

has the mark, bimana. But let or class, all bimana;" this does not mean "Every man
men nonsense. are

is all every

bimana," nor

All
is

every It

bimana," nor
means
"

"

Every

man

is

bimana,"which

"All

men

(asa

matical, mathe" "

in
"

collective whole) all bimana Thus all are total, (as ditto). is never but cumular,and enforces the the predicate distributive, of the also subject
a

all "
a

to

be cumular.
a

So also the total

predicate
" "

of

is negative

not mathematical,

distributed total ; and

some

in the

the

the

is a mathematical whenever predicate part. More generally, is designated, both terms uals, individare quantityof the predicate and the judgment is mathematical. The effect of thus quantifying is to transmute the judgment from the qualitative the predicate to ilton's whole,in which it is simple. This shows that Hamquantitative distributed predicate is a complete misnomer, and the fact
" "

is fatal to his doctrine.8


8

To

avoid

future forms

Hamiltonian

by their
esteem

we author,

will note we misapprehension, that, though denying to the the rank and importantposition assigned to them in Logic occasion to have for the sake of brevity, in syluse them, may logizin
use

Also
a

we

shall be free to
to

his nomenclature

and

valuable contribution

the

of appliances

technical

which notation, logic.

we

PART

FOURTH."

OF

REASONINGS.

I. THE

SYLLOGISM.
treatment

"

1.

The

and logical be
first

natural

of

definition

which ascertained, its

fixes be

that its subject requires its relations to superior fixes its

notions; then
kinds.
let general,

that

subdivisions
is

which ascertained,
its denotation.

First its connotation


us

then settled,

Thus,

in

We
one

refers

before us. now proceed with the subject defined thought in a general have already to be the bringsense ing notion in or under another. This duplex definition obviously of thought, to the two the intension and the extension. quantities The distinction
in

between
we

these found

is

we thorough-going;

met

it be

at

the

outset

concepts ;
notion of
a

that

cjiven judgments may


same
as a

construed

in either

quantity ;

and may

we

shall find the be viewed every

to

be true of

reasonings. As
of

every
a

either

complement

marks,
as

or a

as

kind

thing,so
into
a

either

marks bringing-in
a

reasoningmay be viewed notion, or as a bringingthe notion


that intension and
tension ex-

under

genus.

But

let it be remembered that

and always coexist, the


one

transmuted thought is readily

from

into the other.

or by judging. thought is either by conceiving and judgment are Now let it be again observed that conception kinds or species and the same not two of thought, but one thing in a different form, or viewed different aspects or under phases. Every judgment, and every judgment is an explicit concept is an implicit the definition givenabove of thought is equalconcept. Consequently ly the definition of conceiving and of judging. There are, however,two rect, dithe immediate kinds of conception, or

"We

have

also said that

and Second.
are are

the mediate It is the

or

indirect.

The

first has

been

treated

in Part

comparison of two notions by which they when The second occurs we or immediately conjoined, disjoined. and rethrough ignorance unable to make a direct comparison,
direct

126

OF

REASONINGS.

sort

to

medium, i. e., some


each of the two

third former

notion,which
enables
us

pared combeing directly see

with
or

to

their agreement

and consequently to conjoin them. This or disjoin disagreement, Immediate is mediate conception. has received no specific conception when the unqualified word is used. and is always understood name, is called reasoning.This,then, is the logical inition defMediate conception : Reasoningis mediate conception. in this view. I have the notion man Let us exemplifyreasoning I am On comparing these, and the notion free-willed. unable to decide to that concept. By the prinwhether or not this mark belongs ciple of

the Law

of
or a

Excluded

Middle

am

constrained
cannot

to

believe

that it either does

does not ; but medium


of

which, I

termine. deimmediately
sponsible, re-

So I seek and I
see

comparison.
man

I take the notion

that directly
that

the notion

involves this notion


the notion

likewise responsible, free; and thus I see the notion the

the

notion

involves responsible
man

that the notion

involves view.

as

one

of its marks

free. This is the intensive would the matter extensive quantity, not to decide whether or directly
I know that the class free-ag cnts

But

proceedrather in be expressed thus : I am able unis a kind of free-agent. man contains under it the species sponsible reunder
it
man

If 1

agents, and
able
one now

that this contains

and

so

am as

to

contains think that the class free-agents

under

it

man

of its kinds.

In

exact

accordance

with

we this,

now

observe

that there

are

two

the immediate and the mediate rect. indior or direct, judgments, Immediate It judgment has been considered in Part Third. but issues in the explicit also is the direct comparisonof two notions, Mediate judgment ocdeclaration that theyare conjoined or curs disjoined. to judge this agreement or disagreewhen, not beingable directly ment, of comparison, and explicitly seek a third notion as a medium we

kinds of

state

that each of the other notions


we are

does

or

does

not

agree

with

this

third;and thus
or

enabled to conclude

do

agree with each judgment has received


not

whether they do explicitly other. This is mediate ate judgment. Immediand is alwaysunderstood no name, specific
is used.

when

the The

word unqualified

Mediate

judgmentis

called

ing. reason-

then,is: Reasoningis mediate judgment. definition, logical It is quite evident that there is no essential difference between diate memal, and mediate judgment ; the difference is merely forconception and is usually ment neglected.Also it is evident that a mediate judgwill exhibit three propositions. when Let in words expressed

128

OF

REASONINGS.

Another definition is : A clearly larly will hereafter more appear. is an inference is derived from by which one proposition Syllogism contained in the others. the one others conjointly, two beingvirtually with this definition : A SylloAristotle opens his Prior Analytics gism i n enunciation is an from (Aoyoe, oratio) which, somethinglaid down and admitted, somethingdistinct from what we have laid down, follows of necessity." the import of this last phrase, follows of Let us consider at once of a syllogism The essence not in the truth of consists, necessity."
" "

"

the but

laid down, propositions


in the

nor new

in the truth of that which

is

inferred,

judgment,the truth of be denied without which cannot cepted acimpugning those we have already of the syllogism, for true. In other words, the essence and all is the necessary of the that is actually declared by it, consequence flows from the necesconclusion from the premises. This necessity sary the syllogism to which character of the primary laws of thought, quently alone it is ultimately and by which conforms governed. It is frein the conclusion by the addition of "must." For expressed example, productionof
a

and

distinct

"

Since all metals And Gold

are

fusible,

gold is
must

metal,

be fusible.

The

common

then,between distinction,

demonstrative

and

moral

or

lies wholly in the matter, not at all in the form. probable reasoning, is in all cases The form, or rather the process, by which we the infer, i.e., apodicdemonstrative, equally same, and is in all cases, if correct,

tic, necessary.
affirmation of necessary sequence being the essence, it follows that the syllogism is really indivisible act only one judgment,a single The

thought. Though apparently complex, though in a certain sense three judgments, it does not affirm either of them taken including but only the necessary dependence of one the others. on separately, It is a judgment concerning the relation of one judgments, affirming easilybe expressedin a singleproposition ; may sequence, and That is is it fusible inference from the an e. g., gold judgments that of the metals, and that they are all fusible. is one of this doctrine is that Logic does not conAnother cern consequence of the several propositions. itself with the truth or falsity One all may be false, the antecedents, the consequent or but,havinggranted also be allowed, if the reasoning is sound,the syllogism must regular.

of

THE

SYLLOGISM.

We
is

the consequent note that the antecedents beingtrue, however, what measure of doubt belongs to the antetrue. cedents, Also, necessarily of doubt, no no justthat measure less, belongsto the more, may, But should the antecedents
is

consequent.
be established

be found

it does false,

not

follow that the


as

consequent is false ; it
true

upon

some

other
were Italy were were

simply unproven, and may antecedents. For example,


"

The The
.'. The

natives of Athenians Athenians

Greeks of

natives

Italy ;

Greeks.

Grant

these

and the consequent must antecedents, both antecedents


we can are

be

for it admitted,

follows of

But necessity.1
not

false, obviously yet the


other antecedents.

consequent is
Before

for false,

prove

it from

two additional remarks may be entering upon another section, is an expression of one of the units of which appended. A syllogism elaborate argument is only the sum. The longest chain or the most most complex net-work of reasoningmay be stated in syllogisms linked together. The links are quitesimilar. When we thoroughly

understand
true

one, in its characters


an

and
not

kinds,we

understand

all. of

It is

that

does argumentation

usually present the

form

logisms syl-

; its form ; but

steps are much

abbreviated

by elisions and

variations from

elaborate it may be,stillit can be resolved into its and these stated in regular form and consecution. elementary syllogisms, however

and conversation, common-place thinking of our if closely even witticisms, lightest analyzed and many will be found to resolve into syllogisms. stated, fully is but a repetition of one The other remark made. In previously the study of reasoning not advancingbeyond complete we, of course, are system. The function of reasoningor mediate judgment is clear and distinct, to ascertain their to make our solely conceptsmore in the hierarchies and thus to fix their places true relations to each other, and more of our which therebyapproximatemore thoughts, of knowledge. the perfection to completesystems,
our

Moreover, much

of

bk. ii, ch. ii;and, Logic^ p. 322. Priora, per contra,Esser,in Hamilton's treated Aristotle as by Syllogism, of Logic. The student in the Prior Analytics, read by every should be carefully best text of the Organon is probably that of Theo. Waitz, accompanied by his valuable that the of Latin commentary is but be to it isagogue Porphyry regretted ; been included. has The 1844), not, at least in the edition I am using (Leipsic, and must not be neglected. translation and commentary of St. Hilaire is classical, For English readers Owen's and Poste's text, and notes translation (Bohn's ed.), excellent. and De arc translation, notes on 1866), Soph.(Macmillan, See Anal.
It may be well to say here that the doctrine of the
'

130

OF

REASONINGS.

"
our

2. The
treatment

next

step in the

treatment

of the

like syllogism,

that in

is to view it as a mathematical or proposition, it thus and The obtained sever whole, by partition. quantitative parts cated. by dissection are to be examined and named, and their relations indiwill then proceedto consider the kinds of syllogism. We has been anticipated, consists of three propoThe as syllogism, sitions,

of the

two

of which

are

called the
are

and antecedents,

the

the third, the

consequent. The
in the together
two.

antecedents To

also called the conclude

and premises,

consequent, the conclusion.


So also the word
"

is to (con-cludere)

last proposition notions


"

which

shut up stood apart in the first

ing collecta signifies syllogism(aw-Xiytiv)


"

Aristotle describes a conclusion as a together perfect ; as when is an The example in the syllogismof the extremes." following of extension, quantity
"

All Men

are are are

Persons Men
;

=M:
=

""

P M
P

"

Major
Minor
=

Premise Premise

;
;

All Slaves
.*. All

S: S:

"

Slaves

Persons

""

Conclusion.

There

are

here

only three

terms to

or

notions, Slaves, Men, Persons."

"

It is evident that these stand and

each other in the relation of whole

Slaves being contained under Men, and Men part, under Persons. Persons,then,is the term of widest the symbols) ; Slaves,
extent.
term

being contained
extent

(asin

the

of least extent

; and

Men

of intermediate
is found in each

This

M, which

is called the Middle

Term,

of the which
;

but not in the conclusion. The other two premises, terms, called the Extremes, are both found in the concluare sion together separately they are called the Major Term and the Minor Term.
we

Hence The is The

may

define
Term

as

follows,
"

Middle

(M) is the
the

one

with

which

each

of the extremes

compared in
Major
Term

premises. It
term

is also called the

Argument.

(P)is the

of

the greatest or quantity, greatest

whole. The
Minor

It is always(in the Predicate of the conclusion. extension)


Term

whole. The is The is

It

the least or (S)is the term of least quantity, is always(inextension) of the conclusion. the Subject
is the

Major

Premise

the Major Term. premisecontaining

It

placedfirst. usually
Minor Premise

is the

the premisecontaining

Minor

Term.

It

usually placedsecond.

THE

SYLLOGISM.

131

Examples in
transmute
All Slaves

the

quantityof
into this

intension

will

now

be
one

given.
other,
"

We

the
are

one

above
;

and quantity,
=

add
" "

human
are

Silver is Metallic ;
; Metal /.

S: M: S:

M
P P

\ "" =

All the human


/. All

Personal

is Positive ;

"

S:

"

M:

"

Slaves

are

Personal.

Silver is Positive.

"

The

for expression, awkward.

the sake of form

and
"

somewhat the

By

"

Positive

the graphicnotation'2 on the right, in the conclusion. form This condensed is copulaof the extremes like that standing read exactly justbefore it. The extensive syllogism be expressed in a similar way, only the copulas of course can read in the direction that the copula When inverted. we are points, is contained under ;" when we read it should be read i.e., extensively, in the direction opposite i.e., intensively, to that the copula points, ** it should be read comprehends" In changing the extensive syllogism into the intensive, the middle of the extremes continues but the relative quantity term intermediate,
"

is permitted to be brevity, is meant In electro -positive. long pointeddash below is the

is in

inverted;the greatest part in extension (P) becomes the least part and vice versa. This is in accord with the law that extension intension,
and

intension

are

in inverse

ratio.

In the

example
is

"

Silver

and comprehends Metallic, the


term

this

S Positive," comprehends

obviously

whole, and greatest


is

Hence, in intension the major and the minor is the the Subject of the conclusion, term
P the least.
of the conclusion. And

Predicate

placethe the order of the premises is transposed. We major premise first, have, then, for changing a syllogismof either quantityinto the the followingRULE : Transpose the premises, and invert in other, thought the meaning of the copula; i.e., instead of "comprehends"
to

hence,since it is usual

think

"

is contained

under"

and

vice

versa.

For

example.
"

All Metals In Extension

are

Positive elements

; 5)

-J

Silver is Is

a a

Metal ; Positive element.

"

:M

"

:i

( .*. Silver

Aristotle's definition of the terms


it will his

of

is syllogism

so

that general

which quantity, he recognized both. followers,

apply to

either

renders it I

unlike that, probable

"

he says in the first part call," that which its is both itselfin the mid-

of the Prior

the Analytics,3

"

middle which

term

another

and another

in it ; and

by

lies in position

See Part

3d,iii, " 3.

Ch. iv.

132

OF

REASONINGS.

die. which middle

The

extremes

I call both I define the

that which

is in

and that another,


as

in

another

is.

major

extreme

that in which the middle."


from
ours.

the
4

is ; the minor

extreme,
of

as

that which

is under

Aristotle's method is thus


:

differs the syllogism stating


or in,

It

P inheres
M
.'. P

is predicatedof,all M ;
is is

inheres in,or inheres


or in,

of,all predicated

S ;

predicatedof,all S.

It will be observed first. This

that here the

stands major premise (inextension)

dle bringsthe middle term, as he says above,into the midto state the preferring position.Soon after his day logicians, in their natural form with the subject first, propositions transposed in in order to keep the middle term the middle position. the premises,

We

have,then,the

"

(
Ancient order
"

All S is M All M All

is P ;

( /.
This order
was

S is P.

thought it in this reto placethe major premisefirst, more spect important returning and these high authorities determined to Aristotle, subsequent in method of the in exour tension, Consequently stating syllogism usage. from the time of Aristotle until the only quantity recognized the middle term does not have middle place. recently, There however, no valid reason why the major premise seems, natural to begin should have precedence. It is said that it is more
our

observed

until the time

of

who Bocthius,

statement

with

the

greatestwhole.
a

of actual, expression practical

It may be so, but in the often find the order of we reasoning conclusion

all the first as

the inverted, completely propositions


a

and the or or thesis, problem, qucesitum, in reversed order ; as, Silver is a positive element ; for it is elements." Is not this quitea and all metals are positive unless a more ? If so, then unquestionably, order of statement
" 4

being placed premisesfollowing


a

metal,
natural satis-

Anal

Pri. i, iv.

'O /t""roc.nal al aKpat.


we or

The

middle

term

is the

tween bridge beare

them.

when Properly,
or

inquire after the

meaning
mean

of ye

thing we

ing seek"

the Ex.

mean

middle

term

notion.
our

E. g., " What

by

this service ?"

26. xii,

Meanness,as appliedto
Can To
you
save

using means,
mean

has

acquired a

bad

sense.

imagine I
my

so

could prove,
my

life by is
a

changing of

love ?

"

Dryden.
had sense,

The

monkey using the


term

cat's paw,

proverbial specimen. Barringthe

the middle

is the

cat's paw. logical

THE

SYLLOGISM.

133

reason factory

can

be

adduced, we

are

in viewing the justified


matter

proved ap-

order of the and custom.5 Kant takes


a

as merely a arbitrary, premises

of

tion conven-

and the syllogism. reasoning and so determinin rule, Reasoningis bringinga case under a general the asserthe major premiseis a rule, tion it. In the syllogism The minor the Sumption (Obersatz). of a generalcondition, somewhere or that the condition of the rule, premiseis the cognition under the condition takes place other, ; or, is that which brings a case of what of the rule, the Subsumption (Vntersatz).The nexus somewhat
different

view

of

is subsumed

under

the condition,with
a

the assertion

of

the

is rule,

the Conclusion
that
a

Hence (Schlusssatz).

certain

of its condition
the conclusion
a

syllogismis the cognition is necessary, through the subsumption proposition understand under rule. Hereby we a givengeneral in thingsindividual, but as not as manifested priori,
as

maintained,and universally
And universal

necessary under a certain condition. and is determinable in that all stands under the universal, this,
of necessity.8 itself of rationality or laws,is the principle

"

3.

In

proceedingnow

to

the consideration

of

kinds,we

notice,

into deductive and inductive. reasonings truth from consists in drawing a less general Deduction a particular or Induction consists in rising truth antecedently known. from a general of a general It is rule or law. facts to the determination particular which has justbeen given of reasoning evident, then,that the account and the syllogism relates exclusively to deductive thought. Many writers on Logic, induction as a kind of reasoning accepting opposed the inductive process to syllogistic to deduction, attempt to subject forms and commendable. The results are not laws. nor profitable

the first,

common

division of

Likewise

in the

the following
:

order of the

of propositions

the involved

syllogism

is

reversed completely
"

Qui melior
Non

servo,
cum

qui liberior sit


se

avarus,

In triviisfixum

demittit ob assem,
quoque
; porro,
"

video ;
metuens

nam

metuet qui cupiet


non

Qui
The

vivet liber mihi


may be

erit

unquam."

Hor.

16. i, Epist.

argument

re-ordered

stated thus

Whoever The
.'. No
6

is fearful is not free

miser is fearful miser 56-58. is free

Sumption. Subsumption.
Conclusion.

See

Logik, ""

134

OF

REASONINGS.

tioned They are menhere only to say that it is at least very questionable whether be viewed the inductive process can of reasonas a ing properly species not under the definitions of reasoning have at all, we certainly it will be understood that by reasoning given. Without presentdiscussion, the deductive process, and hold that the syllogism mean we to it. and its laws pertain exclusively is a general Since the sumption of a syllogism since the or rule, major premisecontains notions of wide,often of absolute, generality, of the reader, the questionmay have alreadyarisen in the mind Whence are they obtained ? To say they are the conclusions of prior be true,but is an insufficient and wider reasonings cases may in most to these. recurs as What, then,is the question answer, for the same tion ultimate source We of these generalities? answer, it is either intuithat induction. or By the former we know, for example, Every of gas is in the that "The volume change is caused;" by the latter, inverse ratio of the pressure." Sciences whose deductions are wholly

An

examination

of

such

views

must

be

deferred.

"

from

intuitive

truths

are

called

or priori, are

pure,

or

demonstrative

sciences; those whose


inductions
The
next
are

deductions
a

from

both

intuitive truths and

called

inductive sciences. or or posteriori, empirical,

to be noticed is into intensive and syllogisms examined in the precedThis has already been sufficiently extensive. ing in order there because introduced needful to general inition, defsections, and to a completeview of the relations of the dissected parts. We for all, and once of now are briefly preparedto make an estimate, the importanceof this distinction. Hamilton, to whom debted inwe are for introducing it into the logical literature of our language, insists it is all the two at that strenuously great length -important, of thoughtyielding distinct kinds of reasoning. Reatwo soning quantities and more in intension, natural form he says, is the simpler of relieved a radical it he claims to have reasoning ; and in introducing in the presentlogical defect and vital inconsistency system." refuse to the modes We thus distinguished the title of cannot

division

of

"

kinds ; but how


consists

much

in

this

case

is it worth ?

The

external

ference dif-

premises. But the order of the wholly in transposed thereon premisesbeing merely conventional, any distinction founded and artificial, and hence goes is entirely not real and natural, arbitrary for nothing. It is merelya convenient way by which we dicate agree to inThe other difference named is intended. which in quantity the rule is in the inverted meaning of the copula. This is not an ex-

136

OF

REASONINGS.

and

believe

that

only

he

who

is

ridinga

hobby

would

find

it

faulty.
From these considerations
extensive is of very small, if of syllogisms the emphamoment, and certainly sis very far from deserving any, logical with passive givento it by Hamilton, and repeated sequacity by so shall keep it in view onlyfor the sake writers. We subsequent many and in illustrations use indifferently of more either completetheory, quantity. The following divisions of the syllogism determined are simplyby The general the kind of its propositions. division is into The and the Conditional shall for syllogism. We Categorical time continue to treat of the former exclusively. The consideration some of the latter is postponedto a subsequent Topic. into be s ubdivided Categorical syllogisms variously may The latter are deferred to a subsequent 1. The Simpleand the Compound. for the present. Topic. The simplewill occupy us exclusively
" "

between

and

may intensive

we

conclude justly

that the distinction

2. The

Total
one

and

the

the Universal and or Partial,

the Particular.

is particular, is particular, the syllogism proposition versal, uniWhen all three propositions conclusion. are havinga particular of the proposition the syllogism is universal. The quantity

When

any

determines
3. The

this kind.

one Negative. When premiseis negative, is negative, the syllogism The quality conclusion. a negative yielding of one ing premisedetermines this kind. The two latter kinds,dependand for call remark further at on no quality, quantity present. illustrated by an example which They may, however, be here jointly and one conclusion has one a negative, yielding premiseparticular,

Positive and the

which
No

is both.
murmurs
are

prayers
murmurs

(E)
(I)

^~^

Prayers.

Some Some

sighsare sighsare

/.

not

prayers...

.(0)

~"'S

are Finally, categorical syllogisms

to the relative divided, according

of position

the middle the

term, into four

Topic next in the firstfigure. are Under the present Topic it remains to consider the Canon, and General Rules of the categorical syllogism.

under

sidered Figures.These will be conAll examplesthus far given following.

the

THE

SYLLOGISM.

137

"

4. The

judgment whereof
that the antecedents the three

the

the consists, syllogism essentially mined consequent,is deterhowever, thought. Since these, the

judgment by

necessitate

primary laws of because of their wide generality, not readily are applicable, logicians of sylhave sought to express in a single CANON the principle special logism of the three primary that is only a special statement a Canon laws as governingthe syllogism, and which be used as an may easy and direct test of its validity. The results of these attempts are not of the Canon the several forms quate being each inadevery satisfactory, will here state some of We ; but they are nevertheless useful. the most noteworthy:
1. "Part
are

of

a as

spoken of
is

Remembering that marks partis part of the whole." as parts of a concept,and species parts of a genus,

of thought, to both quantities and obviously applicable the logical and the mathematical. Its generality, ity, brevto both wholes, and simplicity render it perhaps It is, the most useful form. ever, howbeing applicable inadequate, only to affirmative syllogisms.A modified form, applicable is What whole, is : only to the logical of a whole may said distributively be said of a part." If the reader will applythese forms to either of the foregoing affirmative syllogisms, the meaning will be sufficiently obvious ; and it will also become dent eviis only the essential judgment of the syllogism that the Canon in second intentions. generalized this axiom
"

2.

"

Contentum

contenti

est contentum

continentis"

"

Leibnitz.

wise Like-

applicable only to
3.
"

affirmative

syllogisms.

A transest etiam prcedicatum lation prcedicati subjecti." of Aristotle's first antipredicamental lowing rule (Categ. iii).The folof this excellent form : a free rendering as may be regarded
Prcedicatum

4.

"

Whatever
term
or

is universally affirmed predicate

or

denied

of

any

middle

ed containsubject under it." Burgersdyck. Applicable, however, only in extension. de quibusdam et singulis. 5. valet etiam Quicquid de omni valet, de singulis valet.1'' de quibusdam,nee nee Quicquid de nullo valet,

part is also affirmed

or

denied

of any

"

"

These
out

are

the famous

"

Dicta

de omni

by
6.
"

the Latin
Nota
notce
seems

from logicians
est nota

drawn as Aristotle, 8. Part 1st,i, the Prior Analytics,


et nullo"

of

ipsi" This
7. under
"

ipsius ; et to especially adapted


the condition See

rei

repugnans

notaj,repugnat rei

the intensive
of
a

syllogism.

What

stands under
"

that stands also rule,

the rule itself." Kant.

" 2, last paragraph.

138
"

OF

REASONINGS.

8.

In

so

far

as

two

notions

(notions proper,
do
or

or

either individuals)
a common

both

agree, or,
so

one

the agreeing,

other does not, with do not

third other."

notion, in
form
9.

far these notions


"

agree with

each

This is Hamilton's
we
"

will

a Supreme Canon for the "Unfigured Syllogism," consider in the sequel. briefly

What

worse

relation of
and
a

and predicate subsists subject

between
one, at

either of two

terms

common

third term, with

which

is positively that relation subsists between least, related,


terms

the

two

Supreme Canon for the Figured of statement and absolute Syllogism." He claims for it perfection of syllogisms it being the principle intensive and extensive, generality, and negative, positive involving any of the eightHamiltonian
"

themselves."

This is Hamilton's

"

judgments.9
10.

distributed or replaced by an equivalent, by its unby any of its parts." We progenus, or, if distributed, pose it be and a more this, general truly principle, more believing to of the actual process of thoughtin reasoning than some of expressive in the preceding. It is simple and self-evident. For convenience will call it the Canon of Replacement. Its view of the we reference, is somewhat It considers the Sumption as declaring syllogism peculiar. relation between two notions ; the Subsumption as declaring that a other notion is equivalent some logist to,or a part of,one of these ; the syljudgment as being the substitution of that for this ; and the Conclusion as setting forth the result. Thus, to take an old standard are mortal;" but "Socrates is a man," i.e., he is example,"All men "All men" So, replacing by this part, one, a part of "All men." have therefore "Socrates is mortal." This Canon will apply not we in the logical but also to those in the only to all reasonings whole, mathematical whole. For example:

"Any

notion

may

be

is

equal to equal to

B ;

B is equal to C ;
.\

is

C.

This

most

simple and
said could
not

most

common

mathematical any of the

Dr. Reid

be

to subjected

which syllogism, approved logical

See also the Table of the Logic, pp. 604, 605. p. 584 ; and Discussions, F orms Part in 3. Eight Propositional " 3d, iii, of this Canon, it seems that Hamilton's Notwithstandingthe high pretensions* own No. li a and b,No. v a, No. vi b,No. vii a, Negative Moods" (Logic, p. 679), No. viii 6,No, xi a (Ferio\and No, xii b are in direct violation of it.
" "

See

THE

SYLLOGISM.

139

canons,

and

hence very

condemned

the whole

science and

art

of

is Logic,

simplecase when referred to the Canon of Replacement. Moreover, judgments often undergo easy modifications which" difficultto express in strict syllogistic form and bringunder comare mon but which this Canon and justifies. at once rules, logical explains For an example we take the famous logical puzzleproposedby the which not in a page they solve, Port-Royallogicians, very clearly, and a half of discussion ; which Jevons says cannot be provedby the rules of the syllogism ;" and which most other writers omit to notice.10
a obviously
"

The

divine law commands


is
a

us

to

honor

kings;
Louis XIV.

Louis XIV
/.

king;
us

The

divine law commands too

to

honor to

Its solution
seems

by replacementis

obvious

call for

remark,and
once

to be the actual mental

process

by

which

any child will at

acceptthe conclusion.

"
test

5. Aristotle's dicta

are

in directly applicable onlyto syllogisms also because the


as application a

the firstfigure.For this reason, and


in is,
some

cases, somewhat

have logicians confusing,

resolved the

of the syllogism into a series of GENERAL RULES which are principle to all figures conform ; must applicable ; to which all sound reasonings and which, being quitesimple and appliedin succession, render the and sure.11 They are as a syllogism process of testing easy, quick,

follows
1. A

syllogism has three,and


two

only three,terms.

For

if there

be

the four, is
a

A good syllogism have no common term. can premises is a quadruped a a triad, tripod. The following really ; verbally
:

Quaternio Terminorum

Feathers
/.

Light is contrary to are light ;


are

darkness

Feathers

contrary

to

darkness.

10 n

an

to
one

ch. ix ; and Jevons' Lessons in Logic, Penser, pt. iii, p. 158. Hamilton with (Logic, sq.)reduces the six or eight Rules to three, p. 215 it seems and with a sacrifice also, as sacrifice of their generality, acknowledged of 1st and 2d is their stated in first Rule His our merely me, perspicuity. See UArt de

compound
the
easy
to
canon

sentence.
as

from
and

test

of the
canon

back

the

why condense them ? The very intent is to evolve needed for a ready as statements are explicit simple, many of any syllogism. Of course, we may condense them validity much without displaying ingenuityor obtainingany itself,
But

advantage.

140

OF

REASONINGS.

2. It has

three,and only three,propositions. For three


three without only,
: following

terms

give
have

three
more

and pairs,
in the
All

repetition. Apparentlywe

beingsthat have nerves All self-moving thingshave Worms self-moving are


.'.

are nerves

sentient

=A
=

=A
=

Worms

are

sentient

is good, and the form logical shall hereafter reasoning ; but we find that it is a Sorites, into two of three proporesolving syllogisms sitions The each.
3. One

premise

at least must

Ibe affirmative.

For if the middle


cannot

term

agrees with neither of the other two, we whether or not they agree with each other.
No marble
is sentient
are

infer these

From

through it premises,

=E

Some

statues

not

marble true

=0

we

get

no

conclusion ; however
not No
man men

any statue

sentient.
is

be, they do not prove The following, a conclusion : however,yields


of

it may

destitute entirely
are are

religious feeling
are

=E
=

Many .*. Many

not not

true true

believers in God believers in God


not

who

tirely en-

destitute of

religious feeling

=0

the negativeparticle an affirmative, premiseis really which thereby becomes being treated as belongingto the predicate, and constitutes the subject of the conclusion. to infidels," equivalent But the minor
"

4. If

one

premise
term

For if

one

is

the conclusion be negative. must negative, be denied finally to denied to the middle,it must

is

the other term


Few All

which
men men

agrees with the middle

by

Rule

3. =0
=

E. g.

weep

feel

We

cannot

it may

true obviously is essentially Few it. do not yield be,these premises ative, negconstrued givesus a negative and rightly a yielding sumption,

"Some conclude,

who

feel weep." However


" "

conclusion negative

thus,
"

Sumption,
Now Hence

Most All
. .

men

do not feel

weep

=0
=A

subsume,
we

men

conclude, Many
term must in
a

who

feel do

not

weep
once.

=0

5. The

middle

be

distributed

at least

For

if in dif-

each

premiseit

is used

sense, partial

it may,

in

denote each,

THE

SYLLOGISM.

141

ferent

and objects,

so

be

in violation of Kule
Some Some of of
our our

1.

to two terms, making four equivalent From these premises.


use are

in

all,

citizens citizens

refined

profanelanguage gentlemen
the middle

=1 =1

we

can

conclude

for nothing,

different groups of persons. This Undistributed Middle. Sometimes for

refers to entirely evidently of fault is called the fallacy logical it is not quiteso very obvious;

example,
"

valid

This
.'. This

has three terms syllogism has three terms syllogism is a valid syllogism

"A
=

=A

Here hence

the middle
is not

is in each

case

the

of predicate stated

an

and affirmative, conclusion the same, is


a un-

and distributed;
when

therefore the

proven. makes the

Even
is not

the

of portions

the middle

are

clusion con-

which competent unless that fact be declared,


a

virtually
z=I
=A

portion
paper

total.
is

For

example,
"

Some

currency
notes
are

legaltender
currency
we

Government

paper

From and

these
think

no

conclusion is
"

competent ; but

may

happen to know,

it thus,
a

All of

certain

Government
.*. Government

notes

is legal tender portion that are portion notes tender are legal

=A
=

"A term

If, however, the undistributed


the
sum

middle
more

be

so

that quantified
is

of the two

is portions

than

the

whole,a conclusion

competent. This Hamilton


the Middle
Two Two
.*. Some

calls the "Ultra-total


"

Quantificationof

Term."

For

example,
are

thirds of mankind thirds of mankind heathen


are

Asiatics heathen

=1
=1 =1

Asiatics

are

Asiatics
are

"

mankind '
"

(At least one


One other

half

perhapsall are. ) Asiatics,

heathen

example will
have
are

suffice:
never

Very few men Nearly all men .'. Many who are
The old

far

far from

=0 prayed 1 from being saints =0 being saints have (notnever)prayed..


=

Logic makes
that the

no

it is manifest

for this exception to provision rather is mathematical reasoning

the rule ; and than

logical.

142

OF

REASONINGS.

6. Ail extreme For if

particularin
is
to

premise
cannot

must

be

so

in the conclusion. all ;


we

only some
part

wo premised,

conclude

cannot

argue

from

whole.

The

violation of this rule is called the


or

of Illicit fallacy to the according example:

Process.
term to

It is called Illicit Major which the fault attaches.

IllicitMinor,
is
an

Here

ous obvi-

All birds are A


.*. A

winged
a

=A =E

bat bat

is not

bird

is not

winged
is not winged,"

=E

The

majorterm,

"

distributed
an

in the (i. e., is particular)

since premise,

it is there the

of predicate

tributed but it is disaffirmative, it is there the

in the conclusion, since (i. e., is universal) of


a

icate pred-

the

Hence there is an illicit proposition. negative, process of is an illicit not quite so obvious, major term. The following, ess procof the minor
Persons Good
.*. Good

term

without

are imagination

not

true

poets

=E
=

are logicians are logicians

often without
not true

imagination

poets

=E

There and

are

two

useful rules which

are

deduced
we

from

those

preceding,
nately. co-ordi-

might

be

appended as

corollaries ; but

will state them

7. From be premises they be 00,

two 1

particularsthere
is
no

can

be

no

conclusion. for
a

For

if the
5.

1,there

distributed term

middle,Rule

If

Rule 3. If they be I O or OI, are premises negative, of O ; if this be taken the predicate term there is but one distributed, conclusion for the middle term, then illicit major,since the negative the major term ; if it be required by Rule 4 distributes its predicate, both
not
so

taken,then undistributed middle,Rule


Some Some students
row

5.

E. g.

well

=1 =1

study well (No conclusion.)

Some Some

students
are

are

not

card-players

=0 =0

not

church-goers (Xo conclusion.)


waltz students
-

Some Some

students do not
"

=0
"

Germans

"

are

(Nothingfollows.)

144

OF

REASONINGS.

II. FIGURE

AND into

MOOD.

to the position Figuresaccording In the First Figureit is the subject of the major of the middle term. of the minor. In the Second, it is the prediand predicate premise, cate is of In the the of both premises. Third,it subject both. In of of the major premise, and subject the Fourth, it is the predicate

"

1.

are Syllogisms

divided

the minor.

Thus

This last line is


of
"

useful

mnemonic, without other. meaning. The


Aristotle from

tion no-

of Rhetoric, Figure"was borrowed by Figures from of speech. On literal forms which the plain, are departures there ought to be some standard form from which this analogy, one all others are departures, and thence properly called Figures. Such First Figure, which is the pure type of standard form is the misnamed deductive Each

argument.1
figures may
claim to have its

It is

Canon. totle's Arisspecial dicta de omni et nullo are specially adapted to the firstfigure. to adapt them in turn to so as easy to modify the phraseology
But this cumbers
us

of these

each of the others.


one,

with four

canons

instead of

and to

no

advantage.

We

will, then,let them

that the last three may be rules governingthe figures, There are, however, special deduced from rules of the syllogism, to which it is well to give some the general tention. atshow subsequently illustrated by They follow,
an

and go canonless, reduced to the first.

example.

'formulas' vocat), syllogismorum, (Appuleius quce dicuntur figuras lul. Pacius putat, quia figuris ab Aristotele appellatas esse geometricis adscriptis ab eo illustratisint. Equidem hanc vocem tarn a geometrispetinon syllogismi de termmorum ordine tam accipiendamputaverim, quern o-^^/ia ipso appelquam lari licebit, etiam si de geometricis non figuris cogitetur." (Waitz,Com. on Organ., maintains the opinion of Pacius. 26 b 33.) But Hamilton, per contra,
,

FIGURE

AND

MOOD.

145

CONSPECTUS
Example.
1 (subprce). Fig. No
man

OF

FIGURE.
Special Rules.

is

perfect
are arc men

Mnjor premise
Minor

must must

be

universal.

(Elseundistrib. middle.) (Else illicit major.)

.Some saints
.". Some

premise

be affirmative.

saints

not

perfect.

Fig.2 (prce prce).


No is a perfect-one
saints
are are men

man.

.Major premise
One

must be

be

universal.

(Else illicit major.) middle.) 4.)

Some
.*. Some

premise

must

negative. (Elseuudistrib.
is

saints

not

perfect.(Hence

the conclusion

always negative,Rule

Fig.3 (sub sub). No man is perfect.


Some
.'. Some
men are are

saints
not

Minor

promise

must

be affirmative.

(Else illicit major.)

saints

perfect.Conclusion

must

be.particular. (Else illicit minor.)

4 ( Fig. prce sub). is No perfect-one Some


.*. Some
men arc are

man.

.If either
If

prem.

is neg..

maj. must
be

be univ. univ. be

(Else ill. maj.)


mid.

saints
not

maj. prcm.
min.

is aft". rnin. must , is

(Elseuudis.

saints

perfect.If

prem.

conclu. must aff.,

partic. (Else ill. min.)

These

examined ; but grounds should be thoroughly need be retained in memory. All have only those of the firstfigure reference to extension. To adapt them to the intensive syllogism, it is needful only to change the word "major" to "minor" and vice The symbolic notation of the example versa, wherever they occur. for each of the four figures; above (in extension) is the same the notation graphic is different for each of the
Perfect
Men

rules and

their

figures ; thus

^-,S

(Fig. 1.)

I Saints
insist that the variations logicians to display of the syllogism are simplyserving arbitrary, by figure in all possible the middle term They endeavor to positions. in either of the last three is distorted and unnatural, prove that reasoning and that the firstonly is the natural order of thought. Kant in exfollowed by Hamilton in a littletract on the question, himself, in the firstfigure is actually tenso,contends that all reasoning ; for, of the others, the mind interpoin one when perforce it is expressed lates and thus the of of one the converse at least tally menpropositions, which alone is pure and natural. This reduces it to the first figure, We is possible to prove. to conceive, but perhapsimpossible readily

"

2.

Quite

number

of recent

10

146

OF

REASONINGS.

that a reasoning and which in the first is orderly however, grant, figure natural will, when reduced to another, awkward, and appear distorted, unnatural. illustratesthe Indeed,the examplegivenabove sufficiently fact. But it
are seems

that the

same

is true

of

the second

and

third ;

in one which naturally the other or reasonings appear of these two figures, and that these, when reduced become to the first, will briefly harsh and disordered. We consider these two, deferring of the fourth figure. until later an examination It is hardlyto be questioned that the natural order of predication is that which predicates a greaterof a less, as a genus of a species. than to say How better to say Some much scents are pleasant" "Some pleasant thingsare scents." Now there is nothing in the of a negativeproposition nature that determines the relative extent of its two terms ; but if we happen to know that one is wider than the other, make that the predicate we naturally ; and if it be the will naturally middle term, the reasoning cause befall in the second figure, then it will be the predicate in both premises. E. g. :
"

that there

The

true
was was

were apostles a

not

thieves ;

Judas
.'.

thief ;
a

Judas

not

true

apostle.
"

No thieves were the major premiseto By converting but sacrificethe smooth natural we get the firstfigure, as givenin the second figure.

true

apostles,"

order of statement

On
terms

the other hand, if what


of
a

we

know

to be the narrower

of the two

is the middle term, the reasoning will negative proposition fall in the third figure, because then it will be the subject of naturally both premises. E. g. :
The The
/.

Some

apostles sought no temporal reward ; zealous in their work ; were apostles zealous persons did not seek temporalreward.
minor
were

the By converting

zealous persons premise to "Some but manifestly lose the smooth we apostles," get the first figure, of the givenexpression.

uralness nat-

So, then,we
reasons

conclude

with Thomson second


an or

that,since in
third

some

cases

ural nat-

the prescribe

the distinction of these is not


of the mental

and reject the first, figure but a true exvariation, pression arbitrary

act.2

See

Outline, " 95.

FIGURE

AND

MOOD.

147

Let

us

append

that while either of the four forms


in the firstfigure, it
seems

may

be concluded

proposition suited to especially

of the

be establishing general propositions ; the universal affirmative A can proved only in this figure. In its two affirmative forms the predicates wholes than the are always thought as greater subjects.But sometimes mine detera a previous thought, special purpose in view,may the greater whole the subject, and this also to prefer to mate us will often throw in the second the reasoning third figure rather or than the first. The second figure, whose conclusion is always negative, and seems adapted for proving differences in things, especially is its principle that if one obscure thought. Hence term clearing contained under and another excluded from a third, they exclude :is called the dictum de di verso* The third figure, whose each other conclusion is alwaysparticular, seems specially adapted for bringing and thus provingan in examples, universal stateto some ment. exception is that two terms Its principle which contain a common part, if other contains a part which the does not, one partially agree ; or de excmplo. E. g. : differ. This is called the dictum they partially
" "

Tweed Tweed
/.

was

not

an

honorable

man

; ;

possessed high intellectual culture


one

Some

at

least of

high culture
"

was

not

honorable.

This

conclusion
are

is the

of contradictory

All of

high

intellectual culture

and overthrows it. Hence the third figure is well honorable," suited to disprove A, and also E. The middle term in the example is individual. Such a case can occur only in Fig.3 ; for in either of the others the middle term is once at least a predicate, and an individual cannot become This a predicate. alone establishes, not but the merely the naturalness and propriety, of this figure. Moreover remark that while the middle we necessity, is essentially, term of comparison, it is and hence always, the medium that it is necessarily of the first figure only in affirmative syllogisms of intermediate extent. of the logicians But some referred to above, Bain and Bowen, involve in their objections as and to the second

Says Whately (Loyic, arguments used in the process called p. 101), "The in referred to this figure. The phrase be most injiniti will, general, easily is applied to a series of arguments in which we one by one certain go on excluding or certain classes of thingsfrom that whose real nature suppositions seekwe are
Abscissio

ing

to

ascertain."

148

OF

REASONINGS.

third

the figures

notion

that the middle


is
mere

term

intermediate
is meant

extent.
"

This

confusion

ought to be always of of thought as to what


unsound/

by

are middle," and their objections

subdivided into Moods, of the syllogism are figures and quality of the premises. The upon the ground of the quantity and qualconclusion need not be taken into account, since its quantity ity determined are by the premises. The method for determining

"

3. The

four

the moods Relative

is
to

as

follows

A, E, I, 0. exhibited in the following scheme combinations, possible

and we quality, quantity taken These, as premises,

four propositions, recognize at a time, yieldsixteen two


:

But
not to

not

all these combinations

will

i. c., they do conclusions, yield

of valid syllogisms. Those representthe premises

bracketed
"

are one

be eliminated
"

as

violative of General

Rules
with

(i, " 5). Eight


the General
must

half

remain

as

since they accord valid,

Rules. be

In

reference to

IE, we

may

remark

that its conclusion

negative,

by

of this conclusion, the major term, is therefore predicate distributed ; but the major premise I has neither tenn distributed, illicit which violates Rule 6, giving major. in which of the four figures Let us now each of these eight inquire valid combinations We Rules ("1), and applythe Special may occur. find that EA in each
in which

Rule

4 ; the

and El accord with allthese

and therefore rules, in the scheme.

can

appear

of the four

figures,
as

indicated
are

The

figures

the others

we Upon counting,

appear find there are

can

ascertained and indicated. similarly


of the

nineteen valid Moods

Syllogism.
Let
us

"
now

4. The
annex

exhibits four firstfigure


to

moods, AA, EA, AI, El.


of the conclusion

each
a

of

these the symbol these containing

it necessitates,

and
as

coin

word that

the

name

of

mood,

thus:

order, Barbara, Celarcnt, Darii,Ferio.

three vowels

in their

"

Major

remotior
not

their

in secunda minor figura qui mcdio proprior, appellatur qui Aristotle b relations With est ab eo." of the terms, the (Waitz,26 37.) the Cf. fixes Elem. figure. Trendelcnburg, arbitrary position, " 28. terminus

FIGURE

AND

MOOD.

149

The
and

moods the
names

of the
of

other the

three

are figures

treated in the thus coined


are

same

way,

nineteen

moods

arranged

in the

MNEMONIC following to memory commit carefully

HEXAMETERS, which
:

the learner should

Fig. 1 Fig.2 Fig.3 Fig.4


2
"

Barbara,

"

; Celarent, Darii, Fevio que priori* 1 secundcc ; Cesare, Camestres, Festino, Baroco Tertia Darapti, Disamis, Datisi, Felapton, habet. addit Quarto,insuper Bocardo,2 Ferison Bramantip, Camenes, Diraaris, Fesapo, Fresison.

or

Dokainok.

1
"

orFakofo.

These

names

of the nineteen valid moods

By applyingits name and and the quantity its seen now directly,
and the method

convenient. exceedingly indicate its figure, at once to any reasoning, we of each proposition, and also, will be as quality
are

relation to other moods


of reduction.

be reit may duced, Moreover,they constitute a test ;


to
we

which

for,since these
form syllogistic
at
once

are

all the valid


none

moods, whenever
names

have

simple

to which

of these

is

know we applicable,

that the

is reasoning

false. here that had valid


we we

It may
into account

be well to mention in the developing

taken should

the conclusion have found


in

moods,
; in

Fig.1
EAO

two

AAI others, viz.,

and

EAO

Fig.2

two

viz., others,

and

AEO

; and

valid,indeed,but
in each is saL

AEO. These are Fig.4 one other,viz., clusion superfluous ; for it will be observed that the conparticular, althoughthe premiseswarrant a univer"

in

They

are

called the

Subaltern needful

moods,"

"

or

Moods

of

ened weak-

conclusion."
In

It is not

to take them

into consideration. four

it will be seen that each of the notingthe conclusions, is proved in Fig.1. Its four moods, however, are

judgments obviously

reducible to two, the third and the firstand second. Thus


:

fourth

being unessential

varieties of

Barbara

or

Darii.

Celarent

or

Ferio.

AllMisP;
All
.*. All
or or

NoMisP;
All
or

some some

S is M ; S is P.

some

/.
or

No

S is M ; S is P,

Some

S is not P.

Here

is

one

and positive

one

form. negative

Since

all the

other

moods

shall find, the other of be reduced to one or may, as we theyare the two fundamental forms of all reasoning.

these,

150

OF

REASONINGS.

Again, in noting
seen

the

conclusions

throughout it

will be

further

that
A E I 0
1

"

is

proved in
" "

1 3 3

"

figureand figures figures figures

in 1
"

4 6 8

mood, whose moods,


"

initialletter is B.
" "

C.
D.1

"

moods,

"

"

"

"

moods,
3

"

"

"

F.3

Except Bramantip.

Except Baroco
is the

and Bocardo. to

the proposition A Hence, says Aristotle,


and

hardest

establish

the easiest to

overthrow In

; and

is the

easiest to
are more

establish and
overthrown easily

the hardest to overthrow.


;
more particulars

universals general, established. easily

"
two

5. We

are

now

to consider REDUCTION.

It is A

kinds.

then, Ostensive First,


be The
in

Reduction.

stated as of usually syllogismin any


to
one or same

mood, except the firstfour,may


other of

reduced ostensively
in each
name

the
as

these.

initial consonant

is the

that of the mood moods equivalent


and

Fig.1

to which
same

it reduces.

Or, more
must

generally,

have the

initialletter. We

except Baroco
their alternates ing substitutit.

Bocardo,
and
one

or,

consider them rather, The of the reduction

replaced by
is

Fakofo for Other


s

Dokamok.
or

by accomplished
direct
us

both

premisesan
of
a

immediate

inference from
in

consonants

in the

name

mood

doing

this.

indicates that the

symbolizedby proposition
simply.

the vowel

that precedes

it is to be converted p

k f
m

is to be converted per precedingproposition in Bramantip, accidens. after conwhere it shows that, verting (Except simply,a universal is warranted by the premises.This is of per accidens, which reduces quantity.) justthe reverse indicates conversion by contraposition. indicates infinitation. indicates that the premises to be transposed are (mutari). indicates that the
consonants

The

b, d, 1,n,

merely for the sake of remark An exceptive conversion premiserequiring


then proposition,

but are inserted significant, for metrical quantity. or euphony, If in a given syllogism is needful here. the
r,

t, are

not

in order is not

to

reduction

is

an an

individual individual

the reduction

practicable ; for

This consideration makes clear, not be converted. cannot proposition other than the first, but their necesof figures sity, merely the propriety since many individual propositions of our involving reasonings in the firstfigure. cannot be expressed

152

OP

REASONINGS.

This process mood


was

seems
was

to be

simpleand sufficiently
in the

obvious.
more

But

the

Bocardo the

famous of

schools, and, even


of

opprobrium
"

the

scholastic system this mood


once

Baroco, reduction. Says


that it
no was was

than

Hamilton,
looked
to

So
as an

in fact, was intricate,


a

considered

upon
find

into which trap, Bocardo


was,

if you

got, it
Middle

exit.
to

duringthe
as a

ter easy matAges, the name

given in
"

Oxford
a name

the Academical stillremains

for refractory dents, stuor Career, Jail,

of the ancient logical reliquc arose what somegloryof that venerable seminary." Perhaps the perplexity from of rethe process being considered and named as a kind duction. of the present day continue to speak of it Many logicians But obviously it is not a reduction at all, "indirect reduction." as and to call it so is mere confusion. It is, as indicated, already only of of it indirect in these the the when test occurs an validity reasoning

which

moods.
tested

And

it may

be well to add This


is

that all the other moods

can

be

by the same process. by Schuyler.5


the test, even is
in the
case

but uselessly ited exhibelaborately

But

of Baroco

and

Bocardo, is

of

no

and value,

We superfluous.

have

inherited it from

tical practhe old

who, as has been said, logicians, supposedthat these two moods could reduced. In this they were not be ostensively mistaken. Mark Duncan, in 1666, showed that by as as 1612, and after him Noldius, early of contraposition the use Baroco could be reduced to Ferio, and Bocardo Noldius proposed to call the former Facrono, to Darii. and the latter Docamroc. Whately called attention to this method, but
did not observe
a

defect in the

name

Facrono,and rendered the other


letter. Hamilton that recognizes
duce re-

defective

by omittingthe
may

terminal

the reduction

be

made, but blunders sadlyin the attempt to


have

Baroco, which
Fakofo
and in

his editor admits.6 We


as or alternates, as

proposedthe
to

names

Dokamok
an

and have already substitutes,

exhibited admission

example the
we

reduction
not

of the former

Ferio.
of the

The
monic mne-

of these substitutes would could then

affect the metre

and lines,

dismiss

from

technical

Logic this
a

and operose, indirect, The Latin mnemonic of

useless practically

test per

impossibile.
marvel
ing mean-

it must hexameters, calls them


ever were
"

De Morgan ingenuity.
than

are confessed, full more magic lines,

be

of

"

any others that cabalistical verses," and

written."
"

Hamilton
on

calls them

says

that

taking them
6

their

own

Logic, pp. 75-77.

Logic, pp.

313-14.

FIGURE

AND

MOOD.

153

there ground,

are

few
were,

human
so

inventions
as

which

genuity a higher indisplay the figures,

They
invention

far

relates to the firstthree

referred to as the author of the Hispanus, already native of Lisbon,became a prepositional symbols. He was Pope in 1277, and died the same John XXII Greek year. The corresponding lines are much less ingenious, of the moods the names in them as mark onlythe order,the quantity, of the propositions, and the quality of reduction, not the and, indeed,not even indicating any method moods. of Nicephorus Blemthe invention equivalent They were Patriarch of Constantinople. had Which midas, who was nominated in this invention is uncertain. It is curious, however, to note priority that these two attained the two highestplaces, the one in logicians the Roman, the other in the Greek had hierarchy ; but as the one so hardlybegun to reignwhen he was killed by the fall of his palace, the other, the nomination,did not enter on the office at all. declining The several works of the Pope and the Patriarch were for turies cenmany the text-books on Logic, the one in the Latin, the other in the Greek But

of Petrus

schools.7
it may

be asked, properly why should we have reduction ? does not become more Reasoningcertainly cogent by being reduced to the firstfigure and permore elegant ; but,says Bowen, it becomes spicuous. That depends on whether a given case naturally belongsin the first figure. If so, then this is true. But if the case longs benaturally in some other figure, then its reduction to the firstrenders it less awkward and obscure. The answer more or more usually given that the assumes the validity system of reduction is a method for testing of reasonings.As the dicta of Aristotle arc directly applicable the first to instead for the other of other dicta only figure, inventing reduce them to the first, and then apply the dicta de omni we figures, et nullo. Thus we become assured of the validity of our reasoning, and any fallacies in it, which might otherwise escape notice,become It views at once is clear, but unsatisfactory. apparent. This answer it is not worth Logic as an art. If such be the objectof reduction, hour's study; for in actual argumentation this test is never an used for lack of it. The and the uninitiated never err by the initiated, mind ease an practically argument in its familiar grasps with more condensed modes in them of presentation, and sees and more clearly its validity in these prothan when or certainly expressed invalidity, very

Hamilton's

Logic, p.

308.

154 lix scholastic forms.

OF

REASONINGS.

The

answer

we

would

to give is prefer

as

lows: fol-

that

same.

the purLogic is a science. The system of reduction serves pose of showing that all reasoning is governedby the same principle, take natuthese processes of thought, whatever shapesthey may rally and in all cases and the are one spontaneously, fundamentally We thus enabled to comprehend in a single are ments grasp moveof intellect which

otherwise

would

seem

multifarious end

and

plexed. per-

We Our
not

attain that clear

unity which

is the

of all science.

is improved by practice it is knowledge. skill,

such

but investigation,

its direct

is object

"

6.

In the mnemonic

moods hexameters,

of the

same

occur figure

together. We present on the opposite page a scheme in moods moods are equivalent arranged together. Equivalent
reducible The
to

which
are

the those

each

other.

Their
are

names

have

the

same

initial letter.

three methods

of notation

also exhibited. others the

This

scheme

brings

: following facts, important among of moods mood not are Equivalent merely reducible to the same in any a syllogism Fig.1, but are reducible to each other. That is, in any of its equivalents. This mood by reduction be expressed may is evinced by the symbolicnotation being similar for all equivalents. be reduced to another mood not equivalent. But a syllogism cannot Thus well as the order of it appears that the variation by figure, as is in a sense external ; whereas the unessential, accidental, premises, of the and quality variation by mood, which depends on the quantity it would is essential and internal. Hence to be logiseem cally premises, under it accurate to consider the syllogism more as containing under which we find varieties moods, the equivalents being the species, and then we which have in figure, reach the individual moods also might be formed of those A subspecies received proper names. tion. moods requiring only simpleconversion in order to reducequivalent and appear in each of the four Such are absolutely equivalent, figures.They constitute groups in the scheme. that is, initialletter, Moods which have the same moods, equivalent in B conclude A ; those conclude the same formal judgment. Moods

several to light

in

conclude

E; those

in D

conclude

I; those
A

in F

conclude

O.

The

are exceptions

Bramantip and

Dokamok.
are

The

linear and

circular notation

symbolic.
cases

different circular

diagram may

be made

for each individual


so
as

mood, the relative positions


to express the individual

of the circles being varied

in most

FIGURE

AND

MOOD.

155

SYNOPSIS
Equiv. Moods.
IIs-

OF

EQUIVALENT
H"

MOODS.
Linear N.
Eu"-

Graphic N.

Circular

N.

Fig.1.
"

Barbara

C, ^-:M,^-:F

4.

Bramantip

Fig. 1.
"

Celarent

2. Cesare 2. Camestres

"

"

4. Camenes

(Rejected)

"

3. Disarais

"

3. Datisi

"

4. Dimavis

CRejected)

"

3. Dokamok

Ditto

(Bocardo)

3.

Darapti

C,

"

:M:

"

,r

Fig. 1.
"

Ferio

2. Festino

;H-;M,^,r
:^-:M,
"

"

3. Ferison

,F

"

4. Fresison

(Rejected) j

"

2. Fakofo

(Baroco)

"

3.

Felapton Fesapo

C:H*:M:"~,1
o

$"r)

"

4.

(Rejected)

156

OF

REASONINGS.

differences. which The

no tion, Tins,however, presents advantages.The linear notais not thus variable, is on this account rather to be preferred.

graphicnotation

is not

but symbolic,

consists

of arbitrary

signs.It
whereas the mood. may express the

expresses all the accidental variations in external form, linear expresses only the internal, essential feature, i. e., used graphic, In
in the

The

scheme the

to

express

extension,

copula pointsto the in intension to the subject the copula of the predicate, ; in general, In comparing the sevconclusion alwayspoints to the major term. eral of moods in case ing containnot forget, must we notations, especially and therefore interchangeable. indifferent, m, that C and F are what calls the disgusting Hamilton rules Arnauld, after detailing them and proposes to superfor reduction," superfluous, pronounces sede Rule them General follows If the : for Reduction,as by one do not appear in the order required of the syllogism terms by the this order by any legitimate them first figure, make assume conversion, also transposing, if need be,the premises.
"

also intension.

extension

gitimac preparedto examine the Fourth Figure. Its lehas been disputed it to be awkward, Feeling by many logicians. it as an various reasons. encumbrance, assigning they reject of toleration, Hamilton a monster undeserving hotlydenounces it as

"

7. We

are

now

"

far less of countenance and


is

and

favor."

He

argues

that it is unnatural while the conclusion

to the quantities is violative of the order of thought, other in the same and syllogism that his assumptionthat the To this we to no first, object, purpose. in intension is grounded solely are which, premises upon their order, and hence indicates nothing inherent in the we repeat,is arbitrary, that such alternations of quantity secondly, reasoning. We object, in the other figures, often to good purpose, are occur very frequently essential (i, and in some seem cases " 3). If so, we may grant they it. Indeed, in Fig. a ground for rejecting occur 4, without furnishing stand apart. Every logical cannot these quantities has been said, as judgment, every reasoningis in both at once, and their alternate predominanceis not, in any importantsense, a change of thought. have stood Other logicians thought so well of Fig.4 that it has withthese attacks and taken deep root in the literature of Logic,
one

in because the premises are useless, from in extension, and that passing

intension

of these

Logic, p. 303.

FIGURE

AND

MOOD.

157

that every elementary if it treatise must giveit place. Yet,truly, could be discarded without marring the symmetry of the science,
so

without

great
decide

the loss of any essential doctrine or stride towards And it would simplicity. the

form, this would


seem

be- a

not

difficult to

it is that question. The chief reason given for retaining this fourth variation to exhaust the possible forms; Figurerequires used or awkward. 4 is essential to completeness, that Fig. however rarely is essential. But this is true only if the order of the premises We have decided that the order is not essential, tional. beingmerely convenexhaust the forms ; and It follows that the firstthree figures that the fourth is the firstwith transposed premises, contrary to agreement, and hence ought not to appear. which is The advocates of Fig. 4, however, point to its conclusion, and claim that it implies an not that which Fig.1 should give, tial essendifference. The
is reply
not

difficult. Let
S M is
is

us

consider the form

M; P;

.'. P

is S.

that the conclusion is not the one which the mind see readily It strongly inclines to conclude S is to draw. is naturally disposed conscious of a revulsion. P ;" and in concluding P is S,"it is fully Here
we
" "

This which

it is that should

seems

so

awkward, and

violative of that directness

The explanation is simplesyllogism. draw the conclusion S is P," and so does mentally that the reasoner P infers by conversion that in Fig. 1 ; and then immediately reasons and almost unconsciously. But a slight This is done tacitly, is S." reflection on the process leaves littledoubt that the judgment S is the premisesand the expressed between P"is mentallyinterpolated clear. A concrete conclusion. example will perhapsmake this more
" " "

characterize the

Brama-

All All
.*. All .'. Some

kings are
men are

men

mortals ; mortals
are

Direct

"

(an

kings are
mortals

Indirect"

tip.

;) tacit interpolation. kings" by conversion per accidem.


"

the transposition of the similar, entirely and the simpleconversion of the conclusion being all that is premises to present them Celarent and Darii. as May we not rather requisite order of the premises beingnonsay, they are Celarent and Darii (the Camencs and Dimaris
are

with essential)

the conclusion

simplyconverted.

158

OF

REASONINGS.

both Fesapo and Fresison are each reduced to Ferio by converting the conclusion intact. This reduction does not require leaving premises, of the premises. It is not probable, the transposition however, that the mind performsthis double conversion when reasoning tacitly
in these moods.

It would
us

rather

seem

that this-'process is similar to

the above.

Let
Fes-

illustrate:
No

Direct" Indirect
"

ghostsare angels; All angelsare spirits ; ap(e- .*. No ghostsare spirits ;) tacit interpolation. not ghosts by conversion per .'. Some o. are spirits
"
"

accidens.

This

major term. But this the mind and instantly the given quantity restores feels, by The case with Fresison is precisely the same. converting per accidens. These two moods, then,are illegitimate. in concluding that the three legitimate We therefore justified are 4 are those of Fig.1 stated irregularly with moods in reality of Fig. and having an indirect conclusion which is an premises, transposed
an

conclusioninterpolated

is

illicit process

of the

immediate

inference from

the actual and

direct conclusion.

The

two

moods Consequently '^Illegitimate are, of course, to be condemned. f rom its should be with all its moods, rejected Fig.4, usurped place forms should be classed with in the logical system,and its legitimate forms of the syllogism. the irregular is not recognized The fourth figure nor by Aristotle, by any of his tributes earlyfollowers. Averroes, in his Commentary on the Organon, atflourished a thousand its introduction into Logic to Galen,who But a criticalexamination of the extant logical years previously. discovers of trace of the physician it. The Spanish no writings Moor appear
is therefore in any
extant

believed to

have

been

mistaken.

As

it does

not

treatise of earlier date than the

Commentary,

its

is altogether uncertain. We conclude, however, origin may confidently but in early in ancient, times. that it did not originate mediaeval,

this discussion of the simpleAristotelic syllogism, concluding it ever will consider a chargethat has been standing we against back to whom it may be traced. since the days of Sextus Empiricus, contained in one or both premIt alleges that the conclusion is already ises

"

8. In

; that what

is to be

provedis
hence

therein assumed

to

be true ; that

the
can

is begged,and question

make

and

real progress in and to the whole syllogistic theory, frivolity


no

by means knowledge. Thus


that

we syllogism it imputesuselessness

of the

pronounces

its pre-

100

OF

REASONINGS.

speakingof the usual order of the which he calls "the in the formal syllogism, synthetic propositions of petitio this order the objection On principii order," he says, if not unrefutable, stands hitherto unrefuted, tertains against Logic." He enthe odd fancy that the objection be got rid of by merely can in a different order, the propositions puttingthe conclusion writing and insists that it is the true first. This he calls the analytic order," much like a solemn joke. Could he This seems order in thought with think that the difficulty might be obviated by a juggling really his conclusion, if a speaker starts with stating order of words? an Truly, But it in words. he cannot be said to have already admitted ? Else has he not already thought it in a premisenot yet expressed his conclusion be a conclusion ? Bo wen, not seeing the joke, how can adoptsand expands this replyof Hamilton as a serious and sufficient replyto the unrefuted if not unrefutable" objection.18 All logicians We must help ourselves in this matter as well as we can. be nothing in a valid conclusion admit that there can freely i. e., in both premises, both in the premises, that is not contained taken together. The conclusion of a syllogism consists merely of a which statement of the relation of two notions, succinct and explicit relation is thought in their comparison in the premisesthrough a It is universally ate third notion. allowed after Aristotle that a medireasoningis not three successive judgments as appears when written out to the eye, but that it is a single act of mind, a single sion judgment. Hence to admit that the premisescontain the concluis pretty much the same thing as to admit that the conclusion contains i.e., previousitself. But to say that it is contained already, ly, of a reasoning. The in the premisesis to mistake the nature but not chronological, antecedents. arc as premises logical, premises if the comparisonbe onlyapparently Now and not really mediate,
Will Hamilton

help us?

In

"

ia

"

"

if that which

stands for the middle


we

term

is in fact identical with

one clusion con-

of the extremes, it is evident that


is

have but two

terms, and

the

of this known relation. This is fallacy. merely a repetition But if the Herein is no This is to beg the question." progress. that through which the relation of the medium be distinct and really then this is not to other two notions is ascertained, beg the question,"
" "

and there is progress.

12

13

Appendix to Logic, p, G23, 228 Logic, p. sq.

See also

Discussions, p. 004 (Am. ed.).

FIGURE

AND

MOOD.

161

Let

us

remember
can

that the

and premises the other.

conclusion It is
a

are

correlatives,
common case

that neither that


never
a

exist without

very

mind

may

be in full and familiar

of possession

two

truths, but,

the consequence has never been havingthoughtthem together, and is to this mind unknown. It may have occurred thought, utterly as a which (qucesitum) question ; but these two familiar propositions, necessitate it, not havingbeen broughttogether, not premare together ises, and the qucesitum is not a conclusion. For example, everybody knows that young infants cannot have no words, nor talk, signsof that are not merelyinstinctive effects. Again, doubts no one thoughts that such infants really think. Yet many brought persons have never these truths together as premisesof a conclusion. They may have in their own minds the fact that can be inferred, ing questioned but,beit was a question the apart from these truths, merely. But bring truths thus together
:

Infants have But infants

no reason

language ;
;
a

and is it not
new

seen instantly we

that there is involved in this statement and explicate


be done

truth which

may

state

thus apart,

.'. Some

can reasoning

without

language.

any one say that nothingis proved here ? Is there not a step forward in knowledge, advance from the known an to the unknown ? would persons, in view of this simplesyllogism, say, Why, of but I never of it. The two that, course, I might have known thought

Will

Many

contain the conclusion, but this is not premises together ;" they do not assume question it, a new they produce it, from But either alone.
one

to

"

beg the

truth distinct

it is said in
in
one

form the

of the indictment

that the conclusion is

and that in stating it we premisesalone, said in this proposition in the as merelyrepeatwhat is already apart, in this form has example quoted above from Mill. The objection been greatly confirmed by the view that Arnauld takes of the syllogism. He says that every valid syllogism is governedexclusively by this principle One of the two antecedents must contain the conclu: sion,
"

contained

of

and the other show


a

that it contains it."u

and excellent view very ingenious which Arnauld intends the statement.

This is very true, and in of the syllogism the sense in But it is not,as .is an claimed,

14

chs. x, xi. pt. iii, Port-Royal Logic,

11

162

OF

REASONINGS.

it be fairly construed to nor can principii, acknowledgment of petitio sustain the charge. The conclusion is contained in the premisein notion is contained under one that any single broader, the same sense its genus; but observe that the other premiseis necessary to show of a general which on this. I may have a good clear conception rule, ing sufficient grounds I have accepted as true, but know nothuniversally it is applicable. We to which whatever of a multitude of cases may these unknown cases, but I say that the rule contains or includes them to appear by bringing not conscious of that until it is made am and then I progress in knowledge. in as minor premises, All men when are mortal,"have we not we illustration, say "Plato is mortal." Not at least, said, by implication virtually already unless we have also said, or know, or thought that Plato is a man," Plato" may be a statue, or a book, or which is the minor premise. For
" " "

town,

or

what

not.
can

I must

firstthink that

"

Plato is

"

man

before,

under

objection through the unwittingmental supply of the obvious minor premise. 'Yet a reasoning excmplum was found needful by very similar to our The at Lystra.15 Paul and Barnabas people there knew very well should be worshipped." St. Paul sup"No man the major premise, plied with you ;" also are men, of like passions the needed minor, We obvious that and the conclusion contained in these two premises was so in which each premise left unexpressed.But let us take a case it was
"

The bald truisms say he is mortal. in logical to the sake, exampleslend countenance simplicity's this.rule. I

for usual,

is

: questionable

No

murderer

hath
are

eternal life ;
;

All warriors
.'. No

murderers

warrior

hath

eternal life.

Here

we

have

major premisewhich
and

some

the minor; admitting


minor. may

Hence,

who many in the estimation of either class

persons would admit


one

would

deny,while it would deny the of these premises

the affirmation of the conclusion. involving is merely to expand Whately says that the objectof reasoning in those and unfold the assertions wrapped up, as it were, and implied Elsewhere he speaksof geometry as being with which we set out." I suppose this is tantamount all wrapped up in its definitions and axioms. that the conclusion is virtually contained in to the statement to Whately's the premises. I do not object metaphor,but say that be affirmed without
"

15

Acts xiv.

FIGURE

AND

MOOD.

103

knowledgethus wrapped
actual

unwrapped. But is knowledge edge virtual or potential knowledge, knowledge at all? Is not real knowlindeed,be said to exist when only that which is actual ? Can it, not present in mind ? Only in that very shadowy and questionable to exist shape in which potential energy is said by the physicist stored up in an inert, A keg of powder contains in it inactive mass. but a spark is needed to realize it. So, i.e., potentially, an explosion, if the major premise contains the conclusion, it certainly needs the has stored minor to bring it about. A boulder on a mountain-top But it stays there of potential quantity energy. up in it an immense down minor the steep, until some starts it rolling very ineffectively have any experience of its force. and this is necessary before we can arises to a questionclearly Very often,in our search after truth, the have at hand the major premise, establish which we but,lacking unable to reach a conclusion. minor, we are utterly Why is this if in affirmed the conclusion? the major we have already Why affirming and state it as established ? For instance, mer astrononot explicate an it,
observes
a new

up is merely virtual or its wrapping must be

and potential,

come to be-

comet, and
He knows will not

at

once

asks whether that


a

it will return
ing mov-

againto
in
a

our

system.

full well
return

celestial body this

orbit hyperbolic conclude

; but from

he

can

and

pains he is finally

the one in nothingrespecting With minute to establish a minor. sets to work patiently and three or four pointsin the comet's orbit, determines
to affirm that its orbit is

major alone ously question. He labori-

enabled

Then, hyperbolic.
now

but not

till then, the that this comet


is
to to deduce
serve

questionresolves
will not
a as

into

the

established
our

conclusion

return.

largepart of
an

vestiga inthoughtful

search minor

establish, tions proposiin order under familiar general rules, premises


effort to

rather or after,

therebynew

truth.

Is it not
of
new

to deduce the consequences progress in knowledge for one facts and laws obtained by observation and induction ? Is not

movement
an advance,

from

the obscure and


?

confused

to the

clear and

distinct

an

of the true relations of our discovery intuitive ideas and their systematic arrangement something gained, by deductive inference, something new ? All this is accomplished reaches too far to be and by it alone. The objection to the syllogism addition

Is not

sound.

Were

it so, then Euclid and


the the form

Newton
of the

labored in vain.

Let us,

glanceat finally, eagle of

argument that assails the


was

syllogism. The

Libyan

fable

slain

by

an

arrow

164

OF

REASONINGS.

feathered been The he

from

its
to

own

wing.
the

So

the

armory
of

of its

the
own

logicianhas
destruction.
own

imagined

contain

fatal weapon

has empiricist

seized the

but, sheathing his syllogism,


his

sword,
suicide.
His

tries,like Giant
he is
uses

Despair, to get syllogism to


:

captive
the

to

commit

Plainly
argument
Any
The The

the follows

prove

syllogism useless.

as

reasoning principii;
Aristotelic Aristotelic

that

proceeds

upon

the

assumption

of its conclusion

is

io petit

'"

as syllogism,

is admitted

by all logicians, proceeds thus

V.

syllogism is

confessedlypetitio principii.
kid in its mother's milk. itself
true.
a

Surely proved
and

this is

seething the
has be
not

But

if it has
ing, reason-

its conclusion therefore may well

true, then

this

syllogism is

false A

proved
stillthe

its conclusion We

diction self-contra-

dismissed.

remark, however, that, granting

this for

reasoning to be sound,
the minor

syllogismdoes
notice above.
far from

not

commit

suicide,

premise

is

false.16
must

At

risk of which

we being prolix,

another It is

phase

of the

tion objecthe
we

Mill

confuses

with

the

said,very ingeniously,
deduced itself.
is from Thus

that

quite often
not must

the

conclusion, so

being
"

premise, is actuallyrequired to
do
we

deduce mortal is mortal

the
"

premise
Plato
in order

know

from

"

All that

men
"

are

that
"

mortal," but
that it is

first know that


to
"

Plato
are

to

know

reallytrue
that the

All

men

mortal."

The

sumes asobjectionhere falsely

attain
cases

universal

proposition we
If this
were

must

first know

all

individual

it includes.

true, then
But

few, very few,


true.

universal

propositions would
a

be

possible.
as

it is not

We
an

obtain

universal

such proposition,
not

the

one

cited,not
an

from from

spection inhaps per-

of all cases,
a

by deduction, but by
most, from
to
a

induction number.

singlecase,
of

or, at

very

limited

Once

in

possession

it, we
and it,

proceed thereby

bring
new

other

cases,

hitherto

served, unob-

under

draw

conclusions. specific

18

We

should

perhaps

note

that

the

usual

vague

and

inaccurate Its proper

sense

of the will

phrase petitio principii has


be

been

accepted

in this

reply.

meaning

examined

hereafter.

FIGURE

AND

MOOD.

1C5

"
of

9. Praxis.

Supplythe

conclusion

to each

of the

of the name premises. Prefix to each syllogism and Bocardo To Baroco If not in Fig.1, reduce it thereto. apply order of the propositions The regular also the test per impossibile. the one the major premise, is preserved taining conthroughoutthis section, in extension, of the conclusion viewed the predicate being

pairs following its mood (" 5).

stated first.

1. Whoever

possesses

prudencepossesses

all

virtue;
"

Whoever
2. Prudence

virtue must possess prudence. Aristotle. possesses one the benefit of individuals ; has for its object

3.

4.

5.

6.

7.

8.

9.

10.

11.

prudenceis a virtue. No good action results in evil ; results in evil. Some alms-giving the intellect; All abstract studies strengthen the intellect are profitable. Exercises that strengthen of perfection No science is capable ; All science is worthy of culture. No vicious conduct is praiseworthy ; is conduct All heroic praiseworthy. All prideis inconsistent with religion ; Some by the world. prideis commended No duty involves loss; is occasionally To givefreely a duty. virtue a good in itself; account All true philosophers do not account virtue a good in itself. Cicero. The Epicureans is free ; No one governedby passion Sensualists are governedby passion. All good reasoners are candid;
But
"

Some
12. True

infidels are

not

candid.
of

1 3.

14.

genius; have proved true poets. Very unwise men No virtue is a natural quality ; God for its author. has Every natural quality Some kinds of anger are not unrighteous; Every kind of anger is a passion.
poets
are men

1 5. Some

of

our

tax-laws

are

All
1 6. No

measures oppressive

measures ; oppressive should be repealed.


"

truth is worthless ; Many truths are misapplied.

166

OF

REASONINGS.

17. Some

of the truths

human affecting
conduct is

conduct

are

speculative;

All that affects human


18. No

19.

animal are principles are Nearly all animal impulses All expedient acts are comformable to nature; Nothing conformable to nature is hurtful to society.

moral

important. impulses; of action. principles

in Supply, of its mood.

the

Prefix the following, proposition. any lacking Write the linear and graphic notation of each.

name

20.

21.

22.

23.

24.

orbits ; elliptic Therefore the orbits of the asteroids are elliptic. An inflated currency enables many persons to make rapid fortunes ; one hence, since this is promotive of national prosperity, way is to inflate the currency. to promote national prosperity He that is always anxious is never are happy; but covetous men alwaysanxious. infliction of nature; therefore natal deformity an Disgraceis never is no disgrace. the rod hateth his child ; hence no lovingparent He that spareth All
move

bodies planetary

in

25.

26.

27.

prejudiced person can of our reliable judgesare partisans. be a justjudge, none the heart is a blessing Whatever purifies ; But there are afflictionsthat purify the heart. attain high public Sometimes honors; but bad men very bad men are alwaystruly contemptible.
no

spares the rod. Since every partisan is

and prejudiced,

28.

All
are

men

are

liable to

sorrow

; hence

some,

at

of those who least,

29. There

of continuous prosperity to grief. boasting may come virtuous men who are necessitarians; it follows are practically abolish the disthat while all necessitarians speculatively tinction

between
in 30. The

vice and

some virtue,

who

do this

are

theless, never-

virtuous. practice, of mind

connection

with
are

matter

is

but incomprehensible; credible which


are

being most beyond our


31. Not

there certain,

thingsvery

comprehension. ruinous;hence policy.


a

but every one is impolitic; every war ruinous procedure in some is, cases, good virtue is

is

32. No

contraryto the love


is

of truth ; but there is

love of

peace which

opposedto

love of truth.

168

OF

REASONINGS.

the mind, but wearies the flesh ; so that study strengthens what wearies, strengthens. merit in a rival ; 48. Every candid man acknowledges does not do so ; Every learned man is not candid. /. Every learned man 49. It is characteristic of theft to get, though not by gift, something for nothing ; this gambling does,and thus is akin to theft. 50. A true evolution is caused whollyfrom within ; but since very few been if any, have exempt from adventitious causes, beings, scarcely any, perhapsnone, have been evolved. of moral order is wrong; 51. Any disregard is doubtful ; moral order whose moral quality Every action disregards is not doubtful as to quality .'. Any action doubtful as to its moral its moral quality. 47. Hard
52. All

do

not

but all wish strive,

to

succeed ; hence

not

all strive

who
53. What

wish to succeed.
is not in is Scripture
not
on binding are

conscience ;
found

ecclesiastical canons Since many be disregarded.


54. Few
men men are are

not

therein, they may

; unworthy of respect entirely

Most
.*. Some

unlearned
men are

unlearned

worthy of respect.
content ;

55. No

one

is rich who

is not ;

No
.*. No

miser is content
miser is rich.

56. Some

57.

of political ; ignorant economy Congressmen are evidently Such are unfit to legislate ; for the position sent to Congress. unfit are Hence some persons inherit the kingdom of God ; Flesh and blood cannot Its heirs are human beings;

.*. Some

of

us

shall not

retain these vile bodies.

58. All
so

59. No

imprudent acts are not vicious ; all are, however, foolish ; and is not alwaysvice. folly impenitentsinner can hope to escape the wrath of God, yet
the most
not

even

hardened

wish
can

to escape ;

Hence
60.

all who of the

desire it

hope
few

for salvation. ;

Scarcely any
Yet
of the

ship's company
crew

could swim

numerous

only a
z

perished ;
some

.'. Some

could not
x

swim

who

nevertheless survived.
; hence
x

61. Some

is y ; every y is not

is not

z.

FIGURE

AND

MOOD.

169

62.

Bacon
we

was

notable

statesman

and
are

as

lie also
of

was

great

philosopher,

infer

that is of

great

philosophers
use

statesmen.

63.

Whatever

practical
is is
of

is

worthy

attentive
;

study

Syllogistic
.'.

reduction reduction Greeks

no

practical worthy
the
of

use

Syllogistic
The The
ancient

not

attention.

64.

produced
were

greatest
Greeks
;

masterpieces

of

art

Laceda3monians

ancient

.*.

They
All

produced prisoners
are

such

masterpieces.
from

65.

restrained
;
are

enjoying

the

common

right

of

personal
But
.'.

liberty
on

sailors

shipboard personal

not

prisoners

They
Whatever The
use

enjoy

liberty.
should
;

66.

causes

intoxication
causes

be

prohibited

of

wine

intoxication be without
is
not

/.

The No The

use

of

wine

should is

prohibited.
a nervous

67.

sentient sensitive sensitive


man

being
mimosa mimosa

system
;

sentient
no nervous

.'.

The The

has will

system.
his

68.

of

strong
resists
not

conquers
;

passions,

and

so

does

he

that

successfully
.*.

temptation yield
are

Whoever All But rational

does

to

temptation
for

possesses
their

powerful
;

will.

69.

beings
that
suffer

accountable

actions
;

many that
is

suffer

punishment
are

are

irrational accountable

/.

Many
Suicide

punishment
one

not

for

their and

actions.

70.

simply
some

form
or

of

voluntary
has is
not

death;
embraced
to

voluntary
many roes he-

death,
and

in

form
;
so

other,

been

by
be

martyrs

suicide

always

condemned.

170

OF

REASONINGS.

III. QUANTITATIYES. mathematical been or judgment has already quantitative considered at some to consider specially length. It is now requisite whole. in the quantitative Quantitative judgmentsare very reasonings

"

1. The

common.

We

sometimes

reason

with

them

and alone,

in other

sonings rea-

with qualitative judgments. In neither they intermingle is such reasoning case governedby the rules of the Aristotelic syllogism. The old Logicdoes not, I believe, these judgments recognize these reasonings not as distinct in kind, and governed nor ; certainly all to be reduced to the Aristotelic laws. It would require by special This is in most, and broughtin subjection to its rules. syllogism, That but requires more or less violence. cases possible, perhapsin all, is to say, the unitythus attained is not the result of analysis, showing in form and that ultimately these kinds of reasoning are one principle, of the 'other, and the one into the mould but is attained by pressing it into an unnatural form. of Logic beBut the object ing thus forcing of thought in its original to exhibit the essential nature forms, and treat these judgments or reasonings it should recognize in the whole apart from others, and assign their special to them quantitative in these quantilaws. Pure Mathematics proceedsalmost exclusively tative forms ; and the anatomical sciences, which are all essentially ences sciof dissection and naming,deal, with quantities at least, primarily and sections, and kinds. and not with qualities tal fundamenas Logic, these processes, exhibiting should explain to all, the native manner of thought in all its forms. exists between When between two two or individuals, equivalence the copula means is individual, aspects or thoughts of the same is the same matical as,"and may be expressed equal to" or by the matheof London is ( ) sign of equality.E. g., "The population of London is (=) one double that of New York ;" "The population with such propositions million." The principle governingreasoning is the axiom thing are equalto Things that are equal to the same of Replacement (i, each other." The first part of the Canon " 4), notions replaceeach other,"will be found to Equivalent may be more and hence is preferable to the generalin its application,
" " = " "

QUANTITATIVES.

axiom.

The

form typical

of
A

this

is of equivalence syllogism
U;

the

: following
=

B
.'. A

C;
C.
as

concrete The The


.'. The

examplein
of density of density

this form

is

follows

density of the human


water

is

the human

of water body is the density ; of air taken 817 times ; the density of air. body is 817 times the density term

It will be observed And this

that the middle occasion


to

here is the

standard

of

ure. meas-

function of logical of all sorts. standards of measure They furnish the media through which cannot be immediately enabled to compare which we are quantities the pound, the atomic weight the bushel, compared. The yard, of hydrogen,the thermometer, barometer, electrometer, etc.,supply in calculations. with middle terms our through which to reason us of equivais an example of the negativesyllogism The following lence, the only formal variation of which it is susceptible :

gives

remark

Selfishness is not

the

essence

of virtue ;

Duty

is the

essence

of virtue ;

.*. Duty is not

selfishness.

We

remark

that all the terms In

in this

examplehappen to particular

be

then,abstract notions as well as concrete general, may whole. be thought in the quantitative differen inIn the equivalent the order of premisesis obviously syllogism, is also indifferent. order of subjectand predicate The and the of thought, the subject be made either term That is, may without other change. The distinction of major other the predicate, that of major and minor premises, and minor terms, and consequently The does not exist, the terms tion proposiequivalent being equivalent.
abstract.
-is always and

then,has

no no

sion, only simplyconvertible. The doctrine of ConverIt follows, that also, placerelative to this syllogism.

Figure is
to

of

moment,

and

is to

two, the
is

every

term

and the positive always total. Questions concerningdistribution

Moods are duced redisregarded. of negative ; for the quantification

be

and

non-distribution These

cannot, then,arise.
render the
treatment logical this of this

eliminations

syllogism

exceedingly simple. Perhaps


from

from

its clear intuitive exactness, that

within

the grasp

of immature

minds,

even

well as it is, as simplicity is elementarymathematics children often being able

172
to

OF

REASONINGS.

in the logical reasoning the unit form requires whole with the Aristotelic syllogism as more mental discipline and maturity.Hamilton declares "mathematics impetuously exercise." not a logical It would perhapsbe wiser to hold is no substitute for Logic," that and to with Coleridge Mathematics studies as the consider mathematical to preparatory proper discipline studies." logical It will be well to observe that the distinction taken between logical and mathematical is not identical with the familiar distinction reasoning between moral reasoning Moral reasoning, and demonstration. in the quantitative often occurs better called dialectics, whole, and is less uncertainty. then mathematical, or yet it always involves more is in many not quantitative Demonstration but cases or mathematical, The difference between these is that alwayscarries with it certainty. dialectics involves to some extent matter, and hence empirical any is exclusively from falls short of certainty demonstration ; whereas and carries their necessity intuitive principles, along with it. This in the nature then,is not grounded on anythingpeculiar distinction, of the reasoning employed,which in all cases carries with it justthe that belongsto the premises, to certainty but it same approximation of the premises is found in the nature themselves. According to its definition by Aristotle, demonstrative reasoning, producing scientific of the certainty a conviction knowledge in the strictest sense, requires laid down.3 of the propositions His scholastic followers devised the Demonstratio :" as a specimenof the following syllogism potissima
3 " "

apprehendit quitethoroughly ;

whereas

Omne Omuis ergo, Omnis

animal homo

rationale est risibile;


est est

animal

rationale;

homo

risibilis.

Here

is

in completeidentity

the terms, and

the

whole ; but construed in the mathematical readily is empirical, and not not hence a intuitive, priori, In moral reasoning demonstration. we proceedfrom

reasoningmay be its major premise


it falls short of

what

is

granted

See in

Article 1st, Education, Discussions,

Exercise
2

of Mind."

The

distinction

that the mathematical


in
a

sense. specific Aristotle treats of demonstration Anal. Post, i,2, 1. in the his illustrations in from chs. i-xiii, drawing lytics, especially pure
3

as an Study of Mathematics for Aug. 24th,1850. See also an article in the Athenaeum and drawn between mathematical logical reasoningimplies is not logical. The latter term, unfortunately, is used thus all reasoning of course, logical. In its generalsense is,

"

On

the

Posterior

Ana-

mathematics.

QUANTITATIVES.

173

firstbe allowed. must In demonstrative by the disputant ; the principia reasoningthere is no concession ; or rather there can be no nishes furdemonstrative, disputant. Pure mathematics,which is strictly the clearest illustrations of quantitative reasonings.

"

attention to pure mathematics, and therein of quantitative to synthetical geometry, to observe the application
2. Let us,

then,turn
to

our

logical We some use soning, reaqualitative geometry process. in general, of the genus, when it has proved of triangles or as it afterthat the three angles wards are equalto two rightangles, together of triangle the equilatthis truth to the several species eral, applies
find that makes
of
"

and reasoning,

ascertain how

and truly

best to

exhibit its

the

the scalene. isosceles,

We

that find, also,

it sometimes

uses

but that by far the greater part of its mediate syllogisms, comparative inferences are in equivalent syllogisms. itself magnitudes, supplies Geometry,which is the science of spatial of definiwith a series of technical terms tions at the outset by means our complex notions of various magnitudes. It then analyzing these. Thirdly, it states indiscrim concerning laysdown certain postulates certain axioms. These
are,

however,

of

two

kinds:

the self-evident properties judgments,stating synthetical lines cannot Two of spatial enclose magnitudes,such as straight a space" (Ax.x); and, 2d, Certain analytical judgments,such as ing to the same are "Things equal equalto each other" (Ax.i). Accordaxioms proper, and must to Kant, the first are be assumed geometrical

1st,Certain

"

evident before any of the more intuitively complex relations be known can They constitute the ultimate premby demonstration. ises from which the science proceeds, and are, therefore, its peculiar basis. Those of the second class express general of equality conceptions and inequality relative to magnitudes. They are derived from the to quantity, and, corresponding PrimaryLaws of Thought as applied and general rules of the qualitative to the Canon syllogism, govern, in whole.* inferences in the quantitative a mode our similar, entirely to regardthese analytical It has,however, been usual for logicians
as

Axiom

1st of Euclid

(givenabove) is
statements
are

the Canon
same. are

of mediate The Canons

inference.

Nos. 6

and

are

merely modified
3,4, 5, which

of the
from

other

Nos. 2 and

deducible

it,

of immediate

analytic axioms, inference,

to "complex conceptions" " 5). E. g., As from "A corresponding (Part 3d, ii, horse is an is we is and "Whatever strong," immediatelyinfer animal," young A young The sums horse is a strong animal,"so under the axiom of equals b+d. can are we d,that a+c equal," 6,and c immediatelyinfer from a
" " = = =

174

OF

REASONINGS.

in geomeultimate premises with the synthetical, as axioms,together try, of the in to a demonstration, place and, analysis exhibiting logical
one
or

the other

as

the
are

E. major premiseof nearly every syllogism.

g.

Magnitudes which Magnitudes equal


equal to
.'. They
are

to

are equalto each other ; equal to the same the adjacentexterior and interior anglesof a triangle are

the

same

equal to

each other. the


; two

Magnitudes equal to
The three interior
and
.'. They
are

adjacentexterior

and interior

anglesof
the

are triangle

equal to each other


interior

anglesand angles ;

rightanglesare equalto

adjacentexterior

equal to

each other.

All this is very true in this way

and

and operose. formal,but very prolix

Much

Mill exhibits the

of the analysis logicians may be found in Mansel's Aldrich.0 it is very possible Now to exhibit an of arguments in the analysis whole in the same logical making one of the dicta of Aristotle manner, the major premise of the syllogism result ; but both process and would be cumbersome and artificial. It is far simpler, and clearer, natural to treat geometrical treat qualitative more as we reasonings axioms as canons reasonings.Let us take the analytic governing the process, and developthe demonstration the form and authorizing If you ask me to jusby a direct chain of quantitative syllogisms. tify I justify Aristotle's dicta, as by deducing it my Canon, I do it, from the Primary Laws. The above syllogisms would then reduce to the one : following
a
.

and

similar

of Euclid's Proposition analysis v, bk. i;6 lastic from certain old schosame proposition

The

interior

anglesof
equal to

are triangle

equal

to

an

adjacentexterior

and

interior

angle;
But
.*.The

these

are

two

interior

anglesare

right angles ; two to equal rightangles.


more or

The
as

is expression

rendered

facile by the

use

of

lettered
a

figure,
verbal

is

customary,whereby

two

three letterstake the

placeof

of exhibiting the logical of a part. This method description analysis of a geometrical and more shorter, proof is not only far simpler, to me to correctly direct than the usual way, but it seems represent

the actual mental

process, which

the other does not.

lagie, pp. 162, 163.

"

Note Appendix,

L.

OF

REASONINGS.

to properly

be viewed

as
"

the

The copulainterpreted.

same
"

demand

A is a kind might be made to bring A is contained under B," or of its marks of B," or A has for one or B," under the rule species and what follows is the predicate.Then, that the is is the copula, and so ad infinitum. the demand might be repeated, upon the result, in the equivalent misnamed So far of quantitative reasoning degree, the positive degree.
" " "

comparativedegreehave for their copula ematical is greater is less than," for which the maththan,"or its correlative, of form The typical sign of inequality may be substituted. the syllogism is :

"

4.

in Propositions

the

"

"

A"B;
B
/. A

"C;
" C.

these propositions, we Simply converting

invert the

meaning

of the

copulaand

read

B is lessthan A ; C is lessthan B ;
.*. C is lessthan A.

The The
.*. The

planetJupiteris greaterthan
earth is greaterthan the
moon

the earth ;
;

planetJupiteris greaterthan
class of

the

moon.

The What

axiom
is

governingthis
a

syllogisms may

be

stated thus:

than greater
was

What

is greater than the thing.10 still greater said in " 1 respecting the elimination of Conversion,
is to

of comparison. syllogisms for the simplicity of this reasoning. We cannot, however, say as much the premises authorize more than the strictly For,be it observed, cal logiconclusion states. This excess is usually thus : expressed
to
.*. By
so

and Figure,

Mood

be

also applied

much

the

more

is A

greater than C.
may

This
to mean

sort
"

of

argument
a

is called

which fortiori,

be understood thus expressed


:

by

stronger reason,"and the conclusion


Therefore
a

A fortiori

is greater than C.

Such
we

conclusion define the

can

be reached
a

may
10

argument
this

only in the to be a fortiori

affirmative mood mathematical

so

affirma-

In pure

mathematics

of equivalence. We syllogism it Propositions and vii, xvi,xvii,

is used but rarely as compared with the syllogism by aid of however, that Euclid demonstrates find, others of his firstbook.

QUANTITATIVE

3.

177

live

contain less (ormore) than the premises sometimes between the reasoning whole truth. Logicians distinguish and that a majoread minus ; but the distinction is ad majus, a minore since one is simplyconvertible into the other.11 superficial, examine miscellaneous examples.Our Let us now some analytically above may be analyzed thus : typical syllogism in syllogism which the
A B
.*. A

is is is

as as as

much much

as as as

B C C

much

(and more); (and more) ; (and stillmore).

Here

is

simpleconcrete example:
The The
.*. The

higherthan the man ; higherthan the tree ; is stillhigherthan the man. spire
tree is is spire
as

This may

be re-dressed
The The of height of height heightof

follows

than the the tree is greater the the


is greaterthan spire

heightof the man the heightof the tree


that of the
man.

.'. The

is still spire greaterthan

These

propositions may
The

be further
as

thus analyzed,
as

heightof
we

the tree is

muck

that of the

man

etc. (andmore),

Very
the

often

do not
may
The The

need the

conclusion pleonastic
:

; in which

case

argument

be resolved thus
sea
ocean

is broader
is
as

than the lake ;


as

broad

the

sea

(and more);

.*. The

ocean

is broader

than the lake.

Here The

the second

contains premise then

which surplus
into

is elided in

thought.

syllogism may
term

be construed is
as

Barbara, by takingfor
It is evident that

the middle this treatment

"what

broad

as

the sea."

judgments as compound,and views the that both kinds of judgments of degree as complex. Also, reasoning the judgmentsare in the same occur reasoning.Sometimes may as : triplex,
considers the
A B

includes B ;

includes C ; .'. A includes C.

11

De

Morgan givesa
authorities.

more

elaborate

our

common

See bis

of this argument analysis Formal Logic, pp. 20-22. 12

than others of

178 The

OF

REASONINGS.

firstpremise says three


is compounded
u

things.
"

It says that
"

"

is
"

greater than

B," which
it says,

of,1st,

as

much

coincides A partially that 3dly, Not only do both kinds of judgments of degree occur but qualitative judgments also often combine reasoning, For example,
"

as," and,2dly, more is the same with,"or


with

;" also
as, B."
same

in the

tive. quantita-

The
The
.'. Our

sun sun

is

star

about revolving

remote

celestial centre

is the centre

of

our a

its secondaries system, controlling


remote

system revolves about


"

celestial centre.

The

form

is
M M
.'. S

is contained is the
same

under
as

P..

Qualitative.
Quantitative. Qualitative.
cases.

S
P

is contained

under

The

Nothing is in reasoning than to have the minor premise declare more common of notions, of which then replaces the one simply the equivalence The equivaother in the major premiseto constitute the conclusion. lence in such cases, however, amounts to identity, and should be read
"is the
same

Canon

of

is Replacement

well suited to such

as."
a

We
whole

append
to the

singleexample
:

of

from reasoning

the mathematical

follows as part,
A
A
.*. A

minute

is a part
a

degreeis
minute

of a degree; part of a circle ;

is a part

of a

circle.

"

5.

It is

cases, the

manifest how sufficiently syllogism quantitative may

in readily,

be

converted

largenumber of into qualitative.

when this cannot be done without however, many cases and some perhapswherein it is whollyimpracticable. great violence, often be readily On the other hand, qualitative as syllogisms may There
arc,

transmuted all. The of origin

sometimes sometimes not at quantitative, by violence, of this change may have been the frequentpracticability so they not recognizing attempts of recent logicians, many

into

the fundamental
to

distinction of these two

wholes,to reduce

all propositions

therebyto modify,or equations, proposing


Aristotelic system. The

the whole

rather to sede, superbest illustration of this perhaps

is Hamilton's

been already

the Canon of which has "Unfigured Syllogism,"12 whatever given in i," 4. He says that any syllogism

18

See

Appendix

to

his

Logic, p. 626

; cf. Discussions, p. 604.

QUANTITATIVES.

1*79 find

may

be

transmuted the

as

in the

example,and following
: to
f
an

adequate

in expression

form unfigured
reduced

Darii, Fig. 1,
All
are patriots

Unfigured Syllogism.
All and patriots who
some some some

brave

\
"

brave

men

are

Some Some .*.

who who

flee are flee are

patriots "/"
brave.

"]

Some

flee and

are patriots

( .'.Some

who fleeand

brave

men

are

equal; equal; equal.

quantified predicate. has Hamilton overlooked "This form been while, by logicians, says, in fact, it affords a key to the whole dently mystery of syllogism."Eviinto the quantitait is only a forcing the qualitative reasoning tive and the awkward, in order mould, needlessly making expression The innovation and the to avoid even the mere appearance of figure. claim have been received with a justcoldness by all except the most
devoted followers of Hamilton.

It will be observed

that the

change involves

the

"

6.

It is needful

to

before closing, that there observe,

is another

which cannot be regarded either qualitative one as judgments, These are causal judgments.Besides the two modes or quantitative. of thought we have discussed, there is that in which we think events, another. With such one as about, or determining causing, bringing and effects. The judgments we syllogize, pursuinga train of causes class of form elementary
of this

stands syllogism
A B
.*. A
causes causes causes

thus

B ; C ; C.

This is not been

violence to any of the forms we have but logically it is quite similar to the quantitative considering,

reducible without

syllogism.The

copulais
or

"

causes"

this and, in converting,


is
no more

is to

be

changedto
of
cause

the notion of effect.

Obviouslythere
that which

tant imporfacts

in life reasoning

in science than

follows the chain

and

human effect, fixing But the

the or explaining responsibility,


does
not
seem

of nature.

logicinvolved
has been

to

call for

special

discussion after what


to

said of similar forms.

remark,however, that the copulais often absorbed A governs A excites B," etc. as A lifts B," B," These,for be allowed to stand in the place of the more simplicity's sake, may
" " "

It may be well in verb forms,

formal

in the of
one

maintained copula, providedthe causal relation is continuously which was the effect reasoning.Just that event, and no other,
must

be the

cause

of the

next,and

so

on

in

chain

throughout

the series of

propositions.

180

OF

REASONINGS.

"

7. Praxis,

Name

the class to which

each

of the

rea" following

if need proposition.Re-dress, soilings belongs. Supply any lacking the several sylloand exhibit the copula. Explicate be,analytically, gisms

that may
as

be involved

in

one

example.

Construe

two

or

three

: qualitative

1. The

favorite

of pupil

the

Academy

was

Aristotle:

Aristotle became
.-. Plato's

the head

of the rival

Lyceum

favorite became of Athalie the author the

his rival.
was

2. The

author
was
was

the

French greatest

dramatist ;

Racine
:.

of Athalie ; dramatist.

Racine

French greatest

3. The

And
4. John

of death is sin ; sting of sin is the the strength knew


more more more

law.

"

Cor. xv, 56.

than Peter ; than Mark than Mark.


;

Peter knew
.-. John

knew

5. Aristotle lived after Plato ;

Plato lived after Socrates ;


.-. Aristotle

lived after Socrates.


of the Southern

6.

is one Virginia The Southern


a

States ;
of the Union ;

States

are

part

is .*. Virginia

part of the Union.


the life, including
most
are petty,

7. All the vexations of this


numerous as

not

as

its duties ;

Its duties
/.

are

its delights ; less than its delights.


;

The

vexations of life are Red

8. Lias lies above

Sandstone

Red
/.

Sandstone

lies above

Coal ;

Lias lies above is


more

Coal.13
than rubies; precious gold;

9. Wisdom

And
/.

rubies than is of

Wisdom A follows

value than gold. yet higher

10.

B;

BfollowsC;
/.

follows C.

13

This

example

is

given by Whately
See Fowler's
about

without

remark.

It has been

sore

ble trou-

to his successors.

Deductive

Logic, pp. 168-70,for what


compare Dr. McCosh's

the head
summary

of Lincoln
treatment

College, Oxford,thinks of it in his Logic, p. 144.

it ; and

QUANTITATIVES.

181

11. If God
more

clothe the grass of the clothe you? Matt, vi, 30.


so
"

field,

shall he

not

much

12. The

orbit of Venus this within

is within that of the

earth;

And
.*. The

that of

Jupiter ;
that of
a

orbit of Venus radius

is within

Jupiter.
bisects the chord and

13. The

to perpendicular
arc.

chord

tlie

subtended A

For

in the

I) C and B
are

C, A
D
is

gles trianright-angled C is equalto C B, and D C other,

since all radii

equalto
A

each

is

common

if two
nuse

equalto B D ; for have the hypotheright-angled triangles and a side of the one equal to those of
are

; hence

the also
14. The The
.'. The

other,the third sides


the rest syllogistically
is under

of

equal. (Prove the Proposition.)

dome

the

sky;
;

altar is under altar is under

the dome

the

sky.
heaven of heavens
cannot
"

15.

Behold, the heaven


how 27. much

and

contain
1

less this house

that I have

builded.

thee; Kings viii,

1 6. To

To

self-denial is to practise is overcome temptation


is
a

overcome

temptation ;
Satan ; also.

to conquer

.*. Self-denial

mastery

of Satan

17. If two

lines cut each other, the straight vertical or oppositeangles will be equal. For the angles 0 E A and A E D are together to two right equal since the angles which one line makes with anangles, other straight of it side are together equalto tw7o right angles upon one ; and the anglesA E D and DEB are togetherequal to two for the same therefore the two right angles angles reason; C E A and A E D are together A E D equalto the two angles and Take DEB. the common angle A E D, and the away remainingangle C E A is equalto the remainingangleDEB. In the same be demonstrated it can that the angles manner C E B and A E D are equal. Therefore if two straight lines, etc. Q. E. I)." Euclid, Prop, xv, bk. i.
contain

18. Cocoanuts

milk;
cocoanuts ;

These
/.

barrels contain

These

barrels contain milk.

182

OF

REASONINGS.

1 9.

Pilate's The

dictator

was

the with

servile

mob

;
"

multitudes who thus

cried

one

voice,
the
masters

Crucify
of

him

;"

/.

They
For

judged
were

were

the

judge.
to

20.

if, when
death saved
of

we

enemies,
;

we

were

reconciled

God

by
shall

the be

his his

Son life.
"

much Rom.

more, v,
10.

being

reconciled,

we

by

21.

It

were

better
is

to

have

no

opinion
of

of for

God the

at

all
is is

than

such

an

opinion
other of the is

as

unworthy
;

him;

one

unbelief,
the

the

contumely
"

and

certainly
xvii.

superstition

reproach

Deity.

Bacon,

Essay

184

OF

REASONINGS.

formal statement would a logically Besides, of almost any thoughtintolerably render the expression prolix.Brief is not only more and but often more forcible, expression pleasing do not clear. Unnecessary words but obscure,thought. elucidate, It is

to the wise is

sufficient."

best,then, to

use

no

more

than

are

needful

to

convey

the

it is customary and distinctly. For these reasons thought clearly such as are to abbreviate expression. Essential propositions, greatly in various elided ; others are compounded or condensed are obvious, state the thoughtsentire, ing accordnor, indeed, ways, so that theyrarely The Enthymeme is the usual form of brief to their actual order. statement so we frequently ; and since reasonings appear in this guise, section to its consideration. will devote the rest of this prefatory in that case, It is customary, then,to abridgesyllogisms ; and since, is in the mind is some only,such statement part of the reasoning plete incomcalled an which is thus defined : An v (f Ouyuw), Enthymeme We two or one syllogism, judgmentsbeing unexpressed.1 may, four of orders viz. : then,distinguish enthymemes,
1st. The

major premisebeing unexpressed. E.


minor
is not

g., Sinus

is

fixed

star ; therefore it is self-luminous.

2d. The whatsoever 3d. The without has

unexpressed. E.
of faith is sin.

g., Prayers are

often

sinful ; for

conclusion

faith it is

unexpressed. E. g., Enoch pleasedGod ; but to please him (=2 whoever God impossible pleases

faith).
Only
one
"

stone proposition expressed. E. g., if we see on a tombThe memory of the justis blessed," is the impliedsyllogism often occurs manifest. This form of texts, in the use sufficiently If some and in witticisms. proverbs, pithy sayings, me one, seeing is hard," I am vexed, should say, The way of transgressors sorely for the implied concludes me a transgressor, indignant, syllogism yet since it has an undistributed middle, when Falstaff, falsely, running
"

4th.

from which
to
"

the

battle-field, says,
a

"

The

better In the

part of valor
same
scene

is

discretion,"
ply re-

also is Prince

major premise.
Hal,
"

he
is

in exclaims,

Lord, Lord, how

this world
we

!" given to lying

another
1

major premise conveying what

call "an

insinuation,"

though an ancient view and generally This, acceptedin Logic,is not the enter matof a peculiar thymeme of Aristotle. With him the enthymeme is a reasoning from likelihoods and signs, Anal. Prior. See t " t IKUTUV wv. (rvXXoyKTjuoc rj o7//"/ ii, 27; Rhet. i,2; also Hamilton's Logic,Lect. xx; Discus"ians^ p. 153 sq. (Am, ed.) Appendix,note F. ; and Hansel's Aldrich,
"

COMPOUND

AND

DISGUISED

FORMS.

185

is often indirect, question i.e., a premisefrom which the doubtful proposition follows, a very of answer, since it furnishes also the ground of the mode satisfactory violates Ans., "Whatever opinion. E. g., "Is smuggling a crime?" of John is crime." of society the rights Again, when the disciples rectly indi?" he replied asked our Lord, Art thou he that should come not, however, in words,but in by givingthem a minor premise, and simply hour he performed many In that same acts. miracles, the

conclusion.3 implied

The

answer

to

"

"

called their attention


wife

to

them.3
of
a

The

message

to

Pilate from

his

furnishes

an

instance

a singleword, "just" suggesting

while major premise,


"

the conclusion is stated in the form


do of

of

an

tation exhorceeding suc-

Have

thou

sentence

nothingto conveyed a hint


which
Paul

with

that

just man."
for the

The

arguments

of the

premiseson
Caesar."

that conclusion closed his

rested.4 A

proof of each minor premise


with, I appeal
zen citi" "

may

stand alone.
unto

speech before
this minor
"

Festus

The

major to

is, Every Roman

Caesar is entitled to certain immunities." One unto appealing thus standingalone Aristotle calls an of the propositions enthyme'Adararov and matic an as sentence,6 quotes the following example: opyrjv

p) 0uAarre,Qvrirbs"v.
immortal
"

This But the

may

be

rendered, O
a

"

mortal,

cherish not
is rendered,

hate."

more participial phrase,

strictly
to

Being

and mortal,"

this constitutes

minor

the

mainder, re-

which

is the conclusion.
an

So it seems,

in the

common

ical log-

view,to be rather
The

enthymeme

of the firstkind.

than of the frequently major premise is omitted more any ily readother propositions, because it contains commonly a general rule, and understood fullyadmitted; whereas the minor premise is tablished quitecommonly a questionof fact which needs to be stated and esin order to be subsumed. E. g., A certain celestialbody
"

exhibits

proper

motion

among
famous

therefore it is the stars,

member

of

the solar

system."
of
a

The

Csesar consists

series of

speech of Antony over the body of enthymemes,the conclusions being only


an

This suggested.7

When

we

high art before permit the hearers


is
to

audience draw

whose

favor

is doubtful.

to

the conclusion, they then


a

feel the argument than convincing the

be

somewhat

their own,

often feeling

more

logic.

Hen.

4
6

IV, act v, Matt, xxvii, 19. Rhet. ii, 6. xxi,

sc.

4.

Luke
Acts

18-22. vii, xxv, 11. 2.

Julius

act iii, sc. Ccesar,

186

OF

REASONINGS.

is one syllogism, Epichirema, or reason-rendering to both,a supporting attached to either premise, or reason.

"

2. An

that has That


is

to say, it is

thymeme.
A

called another

"

an

having for a premisethe conclusion of an ensyllogism This enthymeme may, of course, be expanded into a syllogism whose premise is the conclusion of another is syllogism episyllogism" One whose conclusion is the premise of
"

is called

protyllogism"E.

g.

Episyllogism. Vice
is odious ; is
a

Prosyllogism.
(
"

Whatever
Avarice

enslaves is enslaves ;
is
a

vice ;

Avarice
.'. Avarice

vice ; for it enslaves ;

-|

is odious.

( .*. Avarice

vice.

The
reason

of thus, in propriety
or reasons

is apparent. propositions of returning the same over ground ; By so doing we avoid the necessity not so likely and by clearing doubts as we to excite we are go along, of suspense. in the hearer the disgust that comes of Cicero pro Milone, though not formallyan oration The epiand an analysis chirema,may be viewed as one on an extended scale, of it stated thus
:

the progress of an in support of its doubtful

some argument, offering

It is lawful to

one slay

who

lies in wait for

us

ing ; for this is accord-

to natural law ; moreover,

the laws of all other nations many wherein precedents

permit
our own

it ;
law

we and, in addition,

have

has

it. justified

Clodius

did lie in wait

for Milo ; for the known

renders

his equipment of probable ; again, the known such a design ; and, finally, it also evinces
to

of Clodius hostility dicates indeadlyweapons

murderous

acter char-

of his attendants

this purpose.

It Therefore,

was

lawful for Milo

slayClodius.
effort of
an

It is

more

common

for

the whole
of the

advocate

to

be directed

to

the establishment
no

minor

premise,and

long speeches

object. This suggests that the arrangement of criminal courts our to, or rather presents the parts of,a corresponds syllogism.The judge expoundsthe law,which is the major premise in the case, and, being fully n o established, requires proof. The that The accused prosecutorendeavors to prove the minor premise, If conclusion Not guilty," is guilty," which no the jury decides. Now the judge, the minor is established. follows. But if Guilty,"
have often

other

"

"

"

COMPOUND

AND

DISGUISED

FORMS.

187

in

draws pronouncingsentence,formally Thus


:

the

which conclusion, logical

the sheriff practically executes.


The law says The The
:
"

Whoever
: :
" "

isguilty of murder The

shall suffer death.

The The

prosecutor proves

is guilty prisoner of murder.

judgeexpounds. Yea. jurydecides,

judge sentences

death. lieshall suffer Therefore

The

sheriff executes.

heap)is a chain of enthymcmes,holding " 3. A Sorites (o-wpoc^a and episyllogism. the relation of prosyllogism It is called throughout It can, of course, the chain syllogism (Kettenschluss). by the Germans
be
in expressed

either

the quantity,

intensive

being the quantity


forms.

mon com-

form.

We

givean

illustrativescheme SCHEME

of the two

OF

SORITES.

or progressive Aristotelic form,

The

The '

Goclenian

or regressive form,

in intension.

u
is
c

in extension.

Resolution.

Resolution.
a

A B
.

is B ; is C ;
is
c.

is d ; E ; E.

is not

E ;

is not

b is not

C
.'. a

is D ; is d.

D is not
.'. A

C is D ;
.'. c

B is C ;
e.

A
e.

is B ;

is not

is not

.'. b

is not

.'. A

is not

E.

Example.
Some The The The Some
who
are

Example.
are on

prosperous
are

avaricious ;

No

discontented
men

men on

are

happy
intent

men men

; ;

avaricious intent
on

gain; gain are discontented;


are

intent

All

intent

gain are
are men men men are are

discont'd
on

All avaricious Some Some

men

gain;
men men.

discontented who
are

not

happy
are

prosperous prosperous

avaricious
not

prosperous

not

happy.

happy

Notation

in

depth.

Notation

in

breadth.

A,

"

,0:

"

,C:
"

,D: -H

:E

E: -H

:D,

"

:C,

"

:B,

"

,A

Other

notations

in breadth.

188

OF

REASONINGS.

The

difference between

these two

forms

is

questionof
We

order of

and therefore premisesmerely, of the syllogism, the case to use second have for extension ; but it is
an

non-essential. the first form

may agree, as in for intension and the

agreement, nothingmore.
and progressive, Till Kant's

Logicians
which

which disputed It is merely a


was

should

be called words.

strife about

sive. regresthe Aristotime,

and the Goclenian progressive. Kant regressive, reversed this. Afterwards, Jacobi restored it, followed by Krug and The other German influence of Hamilton, who follows logicians. Kant, has fixed in all recent Englishtreatises the names as we give If we them. of intension, regardthe Aristotelic form as expressive it ascends from fact to law, and might properly be called the ascending form. If we of extension, regardthe Goclenian form as expressive law to fact, it descends from and might properly be called the form. descending The following should be particularly observed : points
1st. The

telic form

called

Sorites has as many middle terms, and hence resolves regular into as many it has premises, less one. as syllogisms, is the onlymajorpremisethat is expressed 2d. The firstproposition ; all other premises minors. are 3d. Each unexpressed major is the conclusion of the precedingsyllogism.

4th.

Only Only

one

premise may

be

and negative,

this must

come

last in

intension and 5th.


one

firstin extension ; else illicit process. and this must premise may be particular,

be the first middle.

in intension and the last in extension ; else undistributed We also remark that in the scheme all the

in Fig. 1. are syllogisms in the Sorites cannot other ever, occur throughout. One step,howfigures be in Fig.2 or Fig.3, but only one, and it must be either may the first or the last. This, against Hamilton, is established by Mill.8

But, in apparent contravention


two

of this and

others of the above


into
a

rules,
chain.

sorites of different form


forms word may
"

may

be connected

continuous

The

The
name

also of

Schuyler's Logic. and is Sorites was originally, special fallacy.


"

be found

in

still retained

as, the

vol. ii, Logic, of Hamilton, p. 619; and Mill's Examination p. If Sir W. found Hamilton had in any other harshly, sq. says : of logical writer such a misuse language as he is here guilty of,he would have writers." roundlyaccused him of total ignoranceof logical
8

See

Hamilton's

226

rather Mill,

"

COMPOUND

AND

DISGUISED

FORMS.

ISO

"

4. Our

usual way

limited space will admit of in which actual arguments are

only

few

examplesof

the

abbreviated, cumulated,and

compounded, and
Such the

of the fact that all may be resolved into simplesyllogism illustrations are needful, however, in order to confirm

emphasisthat the precedingstatements, and show with practical the unit of elaborate reasonings. is truly simplesyllogism The student of Logic should exercise himself in the reduction of select he will find In most statement. cases arguments to syllogistic to discern and this no easy task,a nice discrimination beingrequisite eliminate the merely rhetorical elements,and to bring out all the rather than expressed. He of which is often suggested much proof, the ultimate and then to is advised to begin by stating conclusion, If a premise seek for the premiseson which it immediatelyrests. and requires proof,regard it as a conclusion from priorpremises, backwards Thus trace the reasoning until the premsearch for them. ises
are

reached but its

with

which For

the the

argument
conclusion
may
occur

commences, is often in

not

its statement, and first, of statement.

proof.

stated order

these

last ; they primal premises

any

"

It will often

happen that

the

same

assertion will have been

proved

by

into the truth of the premarguments, and then the inquiry many ises will branch out accordingly.In mathematical other strative demonor

this reasoning admits of no certainty truth admits

will of

course

never as if,

take

since place,

absolute
same

increase ; and

is often the case, the

of several different

we demonstrations,

and discard the rest. and clearest, the


are premises

But

in

simplest (wherein probable reasoning


"

select the

there is often a cumulation of absolutely certain) conclusion, i.e., each proving it arguments, each provingthe same these we to be probable, and from estimate the aggregate probability."
not
"

who "Whatcly,
out

makes of
an

these

remarks, suggeststhat the student draw


in the form

the

course

argument

of

tree,or

sion, divilogical

thus

Z Conclusion,

is

X, proved by

190

OF

REASONINGS.

Our He

firstexample is drawn
an

from

Austin's Province

ofJurisprudence.
he
is
a

states

argument
"

of the intuitional school of

which moralists,

combats,thus
and

No

opinion or
is held
or

sentiment felt

which

result of observation

Observation by all mankind. and induction, to the same lead different men as to applied subject, But the judgments which are passedinternally different conclusions.
on

induction

the rectitude which


our

or

of actions, or pravity
are excite,

the moral with


not

sentiments
all
men.

or

ings feel-

actions

moral

sentiments

alike precisely or were feelings

quently, Conseour

gotten by
excite of

ductions in;
nor

from
were

the tendencies
or

of the actions which

them

these sentiments

impressedupon our moral our Consequently,


inscrutable facts." This

then

feelings gotten by minds by human


sentiments
are

inductions

and others,

and example. authority ultimate


or

or are instinctive,

of a prosyllogism and argument consists substantially the catch-words brevity


of the

an

epi-

which, usingfor syllogism,


be stated thus
Pro.
:

terms, may

Inductions
Moral
.*. Moral

are

not

held

by

all men

alike ;

^entimentsare
sentiments
are

held by all men


not

alike;

inductions. instinctive ;)

Epi. (Allsentiments
Moral
.*. Moral

not
are are

inductive
not

are

sentiments sentiments

inductive ;

instinctive.

The
28-30.

second "And

example we
we

take from
that all

the

to Epistle

the Romans

viii,

for good to together them that love God, to them who are the called according to his purpose. whom For he did foreknow, he also did predestinate to be his conformed to the image of Son, that he might be the first-born brethren. he did predestinate, them Moreover, whom many among he called, them he also justified he also called ; and whom ; and whom them he also glorified." he justified, and stated so nearly This is evidently in or a polysyllogism, sorites, is needless. Another form that redressing strict logical premise, quite thus : he gloriAnd whom at the close, obvious,is to be supplied fied, work whom all for to are things together good." The they they formal conclusion then would be : he did Therefore,whom strictly work together all things for good." foreknow, they are they to whom it might read thus : More freely and fully whom Therefore, stated, he did foreknow, predestinate, and call according to his purpose, they,
" " "

know

thingswork

192

OF

REASONINGS.

and if he be not creature."

of kin to God

by

Ins

he spirit,

is

base and

ignoble

(What
Man
.'. Man

is of kin to beasts

and

not to

to

God
;

is base

and

ignoble ;)

is of kin to beasts

and

not

God

is base

and

ignoble. ;) ignoble

But

(Who thus deny man's kinshipwith God make man (They that deny a God must deny this kinship ;) man's .*. They destroy nobility.
likewise magnanimity and destroys take an example of a dog, and mark the

"Atheism
nature

; for

and

he finds courage he will put on when man, who to him is instead of a god,or melior natura,
is

human raising what a generosity himself maintained by a


"

which

such manifestly than assureth

as

that creature,without could


never

that confidence So
man,

courage of a better he resteth

nature

his own,

attain.

when

himself upon divine force and faith which human nature and
as

and favor,gathereth a protection in itself could not in

obtain ; therefore,

atheism
nature

is in all respects so hateful,


means

that this, human

human The

of the

to exalt itselfabove

depriveth frailty."

it

above

might perhapsbe
it would
seem

construed that the

as

in part an

argument from

analogy; but

rather to illustratethe wide

example of "a dog" serves of the major premise. universality


by
melior natura, gathers a
tainable strengthunat-

Any being,even (Man, maintained


nature /.

dog, maintained

in its own; and favor, has the confidence of by divine protection


own ;) a Deitygathereth a

better

than
on

his

Man

resting

force and faith otherwise

beyond

his reach.

Now

: by complex conceptions

depriveshim to exalt himself above human means frailty ;) of reliance God man on (Atheism deprives ;) of the exalt himself, to and man means .*. Atheism deprives human nature. magnanimity and the raising
on man (Whatever deprives Eut,furthei;

(Whatever deprivesman

of

reliance

God

of this

so

stroys de-

of the

means

to exalt himself

is hateful

;)

Atheism
.*. Atheism

does this ; is hateful.

One

Whately's annotations on this Essay is as and indistinctly we imperfectly may understand


of of the Eternal

follows

"

ever How-

the attributes of
"

God the

"

Being who

made

and who

'

mind

of this universal

the proof frame,'

governs all things of the existence of a

COMPOUND

AND

DISGUISED

FORMS.

193

of Being possessed very


same

them
on

is most
we men

clear and

full ;

in fact, the being,


one other. an-

evidence How do
we

which

believe in the existence of


exist
"

know

that

not merely beings is, (that

for that is not what we form havinga certain visible bodily chiefly but rational agents, such as we call men) ? imply by the word 'man' not mind is by the immediate evidence of our senses Surely (since not but by observing the things an the objectof sight), performed,
"

manifest result of rational contrivance." The


Pro.

is an equivalent prosyllogism syllogism.


The
exists is the argument exists is the
same

"

proofthat man (The proofthat God .'. The proofthat God


man

founded

on

observed

contrivance

argument ;)
man

exists is the

proof that

exists.

Epi. (The proofthat


"

exists is most exists is the

clear and full ;)


man

The
.'.

The

proofthat proofthat
fourth

God God

proofthat

exists ;

exists is most

clear and full.

from the Appendix to we example, take, of the first part of Paley's Whately'sLogic,an analysis Evidences, somewhat the statement. modifyingand condensing
a

For

and

final

The

ultimate conclusion is established thus


attested by credible miracles religion The Christian religion is so attested ; is from God. .*. The Christian religion
A

is from

God

Of
was case

these two denied

the premises, unbelievers

minor

was

while admitted, whereas


to at

the

major

by

in ancient

times;

presentthe

is reversed.

Paley's argument, therefore, goes


:

establish the

minor,as

follows

All miracles attested by persons


"

who

pass

their lives in

to have witnessed them, claiming and sufferings labors, dangers,

"

in support of their and

statements,
new

"

who

submit

to

rules of

conduct,

"

in consequence
are

of their

belief, by
such evidence ;
:

worthy of credit ;
are are

Christian miracles
.*.

attested

Christian miracles The Christian

worthy of credit

i.e.,

is attested by religion
"

credible miracles.

The
is

major of this syllogism,That a story so attested is credible," supportedby two arguments: 1st,That it is a prioriimprobable
since no sufficientmotive storywould be thus attested, be supposed. 2d, That it is a posteriori since no improbable,
a

that
can

false

13

194

OF

REASONINGS.

storywhatever lias ever been so attested ; and hence, false storyof miracles ever has been so attested. no by subalternation, The proof of this last proposition bifurcates; viz., concerningsuch it is proved to be, cited as parallels, stories as have been,or are likely
other miraculous

either, They
as

"

are

not

so

or attested,"

without being explicable,


etc. hallucinations,

lous," miracuThey are not properly of the narrathe veracity tor, questioning

"

The

pointsof

the minor

premise of

the

are syllogism

established

by
"

series of

arguments :

That

"

"

witnesses for Christianity suffered is proved: early since their doctrine to suffer, 1st. A priori:they were likely foolishness. and regarded as an was offence, 2d. From profanetestimony. Christian testimony. 3d. From is proved: That they suffered in support of their statements 1st. By that they had nothing else so to support except the claims of the new religion. of historians, both Christian and profane. 2d. By the testimony and That they submitted to new rules of conduct,

the

"

That

this

was

consequence

of their belief of

their

are story,

proved. similarly
"

That

the miracles thus attested


is
: proved

are

what

we

cles call Christian mira-

that the original unlikely story should taken its place. have been lost and a new one 2d. By incidental allusions of ancient writers,showing the to be the stories of these witnesses and of our Scriptures
1st. A

priori: it

is

same.

3d.

By

the inherent

of our credibility
a new

historical It will be
of
reasons

Scriptures.
that much

This last is of the prove

by supported
is

series.

seen

argument
the
same

number a cumulative,

point. Also each subsidiary argument can obvious major. an supplying

being cited to that in the latter part of the analysis be turned into a syllogism by easily

"
to be to

5.
a

stated Arguments are frequently

in what

at firstglance appears

be

rule.

discovers but which a slight inspection single simplesyllogism, essential deviation from compound, or at least to involve some will state and analyze We a few examples. representative

COMPOUND

AND

DISGUISED

FORMS.

195

it is evident that compound proposition, and that the statement there must be at least one compound premise, involves two or more syllogisms.E. g. : When
a

conclusion

is

The

triumvirs

were

ambitious
Crassus

;
were were

CaDsar, Pompey, .'. Caesar, Pompey,


Here
are

and and

triumvirs

Crassus

ambitious.

three syllogisms involved in one If statement. obviously there are stillthree. But substitute for the major term we friends," if we then there is but one, since the substitute founded the empire," simple. change makes all the propositions involves a surWhen the conclusion is simple, plus a compound premise
" "

of matter.
Whatever The
.*. The
moon moon

E. g.

revolves about
alone

the earth must

present phases;

revolves about

the earth ;

makes

phases.

This about
not

compound
the
moon

minor

premise resolves
which the revolve about
it would

into

"The

moon
"

revolves What
no

from earth,"

conclusion the

and follows,

is
clusion con-

the

does not

from earth,"

which

is

competent,since
is contained

more syllogism

in the

give illicit major. Hence in this premisesthan can be collected in


other
cases was a compound yield given. E. g. :

the conclusion. But


a

compound exponible premisein

may

which conclusion,

then collects all that


comes

Justification
Our
.*. Our

by faith alone
is

hope highest highest hope

justification ; comes by faith alone.

This may evidently in Barbara be resolved into two simple syllogisms and Celarent. But this is not requisite pound ; for we may treat the comin such case if simple. as propositions An has the effect to distribute, rather to totalize, or subject exceptive the predicate of the proposition, for one of its elements is negative. The following therefore, though it may be construed as reasoning, AAA, Fig.3, is sound :

Only they who Only they who .'. Only sufferers


This conclusion

fail are

scorned

"

afa

fail suffer
are

=afa ^afa

scorned
on

is sound

condition

that the middle


it is
so

is distributed

in both but in

that premises. In the example, common speechit is usual to make

the

sufficiently plain; distribution more fully

is

196

OF

REASONINGS.

appear fail are

by

sort

of

thus: "They, reduplication,

and

who only they,

scorned."

In the

example also following


its solution is
more

all three

are propositions

compound

but exponibles,

intricate.

all are Except the evil-minded, But none are truly happy save
.'. There
are

truly happy ;
the content alone ;
are

none, except the content, but those who

evil-minded.

we this, Redressing

have

as

follows
are

All but the evil-minded None


.'. None

but but

the content the content

are are

truly happy truly happy ;


any but

the evil-minded.

This

is compound throughout, but the matter Camestres, into four syllogisms, the conclusion doubly so. The whole explicates the following conclusions : yielding

is in form

1. The 2. The 3. The 4. The

non-content non-content content content


are

are are.

not

non-evil-minded.

evil-minded.

non-evil-minded.
not

are

evil-minded.

When

we we

the syllogisms givingthe explicate find


an

two

affirmative conclusions,

undistributed which

middle. evolved
to

the compound Nevertheless,


is

conclusion effect of
case,
an

from

they are is subject exceptive

competent, because

the this

totalize the

predicate.In

all contained therefore,

for

we

clusion. premisesis collected in the conSuch an intricate form and analysis less. needare, however, quite will be sufficient; Any one of the simpleelementary syllogisms from its conclusion immediately the infer, by infinitation, may

in the

other three.

"

6.

There

is

class of

which,from syllogisms disguised

ous the vari-

these have received, treatment to have seems unsatisfactory been the bane of writers on logic.The premises stated. arc irregularly in order to indeed,but require, They consist of simplepropositions the substitution of equirules, logical bringthem under the common pollent of inferences. In else one or more or subsidiary propositions, and
some cases

the resolution

is obvious ; in others difficult. We


a

cannot

do better than to examine

few

characteristic

examples.

COMPOUND

AND

DISGUISED

FORMS.

197

The

from following
9

Arnauld

is

by pronounced

Jevons

to

practica be im-

The The
.*. The

sun

is

Persians Persians

thinginsensible ; worship the sun ; worship a thinginsensible.

Here Canon

are

five terms

is obviously yet the reasoning very good. The


" "

the conclusion being obtained applicable, Replacementis directly tributed in the minor premise, the sun by its undisby replacing, declared in the major premise. as a thinginsensible," genus, rules the resolution is very simple. But even under the common logical the major premisewe ceptions, From infer, by complex conimmediately may and a thinginsensible," They who worshipthe sun worship Barbara. then have a perfectly we regular would hardlypuzzle a tyro : The following of
" "

Whoever
A

probes
is

wound

is

on

the verge

of crime ;

wound

probed by
on

the healer ;

.*. The

healer is

the verge of crime.

For the
"

The An

minor, substitute the active form immediately inferred, passive have againBarbara. healer probesa wound," and we immediate inference in opposition is as an example involving
:

follows

That And
.'. It is

riches

are

often
true

bitter

curse men

is true ; desire riches ; is often


a

yet it is also

that most
no

false to say that

men

desire what

bitter

curse.

The

which syllogism

is

in this disguised slightly


is often
a

is the
bitter

Darii following
;

They
Most
.*. Most

who
men men

desire riches desire what desire riches ; desire what is often


a

curse

bitter

curse.

This the

inferred by complex conceptions ; major premiseis immediately then its contradictory, conclusion, by opposition ; for if E is false,
true.

I,is

recall we Finally, solved by the Canon a false syllogism on


must

the

be invalid.
The

He

" 4) as directly example formerlycited (i, of Replacement. Aldrich (p. 99) pronounces it the ground that it has five terms, and therefore is evidently is wrong ; the reasoning very good.
commands
a us

divine law
XIV is

to

honor

kings;
Louis XIV.

Louis
.'. The

king ;
commands
us

divine law

to

honor

Lessons in

Logic, p.

158.

198

OF

REASONINGS.

It is

evident sufficiently
true

that the middle

term

here is

"king." This,

then,is the
in
a

form
All

that

of the major premise, which, being redressed subject : as gives equipollent, may be accepted
of those whom
a

kings are
XIV XIV
is

the divine law

commands

us

to

honor

Louis
.'. Louis

king;
whom

is one

the divine law commands

us

to

honor.

The
form

conclusion
for
"

of this

is merely a similarly Barbara,again, equipollent


us some

The

divine law commands


will to

to honor

Louis
more

XIV."
: satisfactory

The

treatment following
is
a

readers be

Louis XIV
"

king ;
and
we one
we inverting, get: are thinkingof is Louis XIV).

whole to the quantitative by transference XIV (i. king e.,the we : by complex conceptions,get This
is Louis

"

Whatever .But

commands

us us us

to

(Whatever commands
commands

.*. Whatever

Now,

The

divine law divine law

commands commands

king commands to honor all kings commands to honor all kings commands to honor all kings; us
honor this
us

us us us

to

honor

Louis XIV
this

to honor to

king;)

honor

Louis XIV.

.*. The

to

honor

Louis XIV.

and named certain described, Logicianshave distinguished, of which be fairly included of arguing, modes account some may under the present topic. ad rem is the direct proof of the main The argumentum pointin question. The reductlo ad absurdum indirectly proves an assertion by proving of its contradictory. the absurdity It is much used in geometry. It called argumentum The refutation of is sometimes per impossibile. also be accomplished assertion may an by an inverse treatment,10 by its In hear the true. discussions we sometimes proving contradictory

"

7.

remark, "Your
be

argument

is illogical or a premiseis reasoning of false. The argument from effects is very similar. In a question the passage of a law for the suppresmere expediency -as,for example, sion of intemperance we might argue from effects, and, showing that resulted in evil and that they had actually to be evil, they are likely pediency similar cases, we in analogousor entirely might thus prove the inexif posof such a measure. Questions of duty should always, sible, in w ithout but be determined a priori, regardto consequences ;

shown, then

proves either its

too

much."

If

an

absurd

quence conse-

"

"

10

See Part

3d,ii, "

8.

200

OF

REASONINGS.

"
a

8.

Praxis.

State of each of tliefollowing exampleswhether sorites.


case

it is the

or a simpleenthymeme, or an epichirema, In form. implied in full logical syllogisms the pro- and epi-syllogism. distinguish

Write
an

out

of

epichirema,

1. Blessed 2.

are

the

merciful; for they shall obtain mercy.


be
a

Cunning

cannot

virtue ; for

no

virtue

degrades.

3. It is I ; be not 4. 5. 6.

afraid.
"

7.

8.

See Hamilton's Cogito, Metaphysics, ergo sum. p. 258. should be moderate ; for excess will cause disease. Every man have no friends. Kings,havingno equals, leans, sinners above all the GaliSuppose ye that these Galileans were ? because they suffered such things I tell you nay. also it often yields is to it, as Will,since it often combats desire,
not

desire. flesh of ruminants horns


as

9. The

is

good

for

food, and

these

since animals,

they have
10.
to

and
he

cloven
is

hoofs,belongto that class.


liable
of
straint re-

Man, inasmuch
desires and
in

and is, selfish, naturally moreover, which have no limits or power passions

11.

needs the restraints of law. themselves, is preferable to Occasional turbulence, being the less of two evils,

rigid despotism.
12.

What
no

if

rule

never

and is,

a principle alwaysis,

law

admitting

exception?
man

13.

wise
and

is

never

this is because the most


man

because he is never surprised, disappointed; he forms no expectations that are not placed

upon
14.

certain basis.
to

Suppose a
"No

say, "I

dislike all

find foreigners;"

premise

which, with
15. Whatever

this

would saying,

authorize the further

assertion,

foreigner ought to
tends
to

be liked." the inind from

1C.

of a low pursuits deserves to be promoted. This classical learning nature does, since it cultivates a taste for intellectual enjoyments. because the writers had The Scripture narratives are trustworthy, cere sinof knowing the facts;also, were the means they evidently

withdraw

and

candid ;

the narratives and,besides,

are

consistent.

17. All true of

friends to religion, are being the basis religion patriots ance national prosperity; but, since their lives are not in. accordwith its

it follows that precepts,

some

great statesmen

are

not

friends to

religion.

COMPOUND

AND

DISGUISED

FORMS.

201

18. Lithium

is

an

element; for it
an

producesan

therefore is alkali,

metal, and hence


19. I will not

element. it is

do this act, because

unjust;I
so

know my

that it is

just, un-

because
me
so

my conscience tellsme because the act is wrong. observance

; and

conscience tells

20.

When

the

of the first day of the week


of

as

a was

religious
duced, introa

festival in commemoration it must innovation. lead to have In

Christ's resurrection

attracted much

attention ; for it was would

ing strik-

this case, since attention the


an

naturally
it

the truth of inquiryrespecting have been exposed as story would surely been
one.

the resurrection,

imposturehad

Put

the

climax logical following

in the Goclenian
:

form, and write

the
21.

notation and graphic circula^ linear,


The

prudentare temperate;
The

temperate are
constant
are

constant

The

unperturbed ; without sorrow The unperturbed are ; Those without sorrow are happy ;
.*. The

prudentarc
its

85. happy. Seneca,Epist.


"

Put
22.

the

in following

form, and opposite


;

write

the notations

Nothing which
What A has
no

is indissoluble is mortal

of parts is indissoluble composition has no composition of parts spirit ; A thinking substance is a spirit ;

The

mind
.*. The

is

substance thinking is not mortal.


a
"

mind

Plato. either form


a

State each of the


23. We
must

as following

sorites in regular
; for
war

increase the income-tax


cannot

has become

necessity,

and

we

onlyby
country
24.

be raised which can without money, go to war of the But the only tax which the resources taxation.
can

bear

is the

income-tax, since it will fall

on

the

richer part of the A

population.
hold the
must

demagogue must
favorite with he follow.

peoplein contempt ; for,being a


know how
to manage

them,he

them

fore ; theremust

understands

their

weaknesses,and

his

contempt

202

OF

REASONINGS.

25.

Riches

are

for

therefore

and spending for honor and good actions spending, be limited by the worth extraordinary expense must
"

of the occasion.
26.

J3acon, Essay
nor

xxviii.
a

That and But

defalcation is fraud, and neither


no one

therefore

crime, no
should go

one

this who

any

other

crime

acts

with

good

intent

should

deny, unpunished. be punished.


the
quence consea

will

Now, all generous


to

conduct

is of this

and character,

it is generous
are

credit

failures in business many who of free credit ; so that not every one

freely. But

fails is

faulter de-

the Analyze the following arguments, stating sorites : or as syllogisms 27. No

results either

as

ple sim-

imitates the natural action of the nerves effectually of the muscles in exciting than electricity the contractility transmitted alongtheir trunks ; and it has hence been supposed is the real agent by which that electricity philosophers by some But there are many the muscles. the nerves act on objections the rest, to such a view ; and this very important one among trunk which that electricity may be transmitted alonga nervous while the tied tightly round it, has been compressedby a string checked by nervous power is as completely passage of ordinary had been divided. this process as if the nerve Carpenters

agent

more

"

Physiology.
28. We
are

not

inclined to attach much


inductive method

of the

value to that analypractical sis which Bacon has given us in the It

second
rate
we

book

of the Novum

Organum.

is, indeed,an

elabo:

and
are

of that which analysis.But it is an analysis all doing from morning tillnight, and which we continue
correct

to do

even

in

our

dreams.

"

Macaulay.
the reason,
common.

29. Our

intellectual part being common,


we are

in respect of also,

which

rational
reason

is beings,

This
us

being so,
to

mon com-

also is the what hence


same a

which

commands there is
a

what

do, and
also ; of the
a ner man-

not
we

to
are

do ; this

being so,
and

common

law

all fellow -citizens; and

hence

members

political community :
state,
"

therefore the world

is in

Marcus

Antoninus. all laws

30. The

which general object


is to

have,or ought to have,in


may

mon com-

of the community; and, augment the total happiness

to exclude, as therefore,

far

as

that tends be, everything

to

COMPOUND

AND

DISGUISED

FORMS.

203

31.

chief. happiness ; in other words, to exclude misBut all punishment is mischief ; all punishment is in itself an of utility, if punishment evil. Upon the principle ought at all to be admitted,it ought only to be admitted in so far as it promises to exclude some greaterevil. Jeremy Bentham. Because the greatest part of men such as prefer their own are is sensual that good which even private good before all things, that
"

subtract from

before whatsoever

is most

divine ; and

for that the labor of doing

with good,together

the

from pleasure arising

the contrary,
one

doth

make

men

for the most

part slower

to the

and
can

proner

to the

other than

them that duty prescribed

by
that

law
men

prevail
do

with sufficiently
for the benefit of which may

them
men more

; therefore unto it hath

laws

make
wards, re-

seemed

allure unto

alwaysneedful to add good than any hardness


"

de-

terreth from
evil than any
6.

and punishments, which it,


sweetness

thereto

deter from more may allureth. Hooker, Ecd. Pol.,

bk.
32. How
33.

I, x,

did the barbarians

reason

in Acts

3-6 xxviii,

Prove

that O cannot be a premisein Fig.1 ; that it syllogistically be the major in Fig. in Fig.3. the minor cannot Also 2, nor be negative. Also prove that in Fig.2 the conclusion must that in Fig.3 the conclusion must be particular.
out

involved in the following and syllogisms irregular : compound forms,supplying any inference that may be lacking
34. The
to establish a republic. endeavoring A republic is a representative government ; French .*. The to establish a representonce more are endeavoring ative
once
more are

Write

the

French

government.
35.

The

/.

is merely a purchasing value of money ; power Interest on money is only a reward of abstinence ; Interest on money is not the value of money. Now
no

36.

for chastening

the

present seemeth

to

be

37. I
38.

the grievous ; nevertheless afterwards it yieldeth of righteousness which them exercised thereby. unto are 11. ffeb.xu, because I have nothingto give. givenothing, solely
"

but joyous, fruit peaceable

None There

are are

happy but
many
rich

the
men

virtuous;
who
are are

rich
men

not

virtuous;

.*. There

are

who

not

happy.

204

OF

REASONINGS.

39.

Whoever

says, I love
not

is God, and hateth his brother, whom


seen

liar; for he shall he

that loveth love God


40.

his brother

he ?

hath

seen, how

whom
out

he hath not

They
He

are

of the reach ;

of their enemies

who

cannot

be robbed

of what
cannot

they love
be robbed

of what

he loves who

loves God

alone ;

They who love God alone are out of the reach of their enemies. 41. Every good pastor is ready to givehis life for his sheep ; Now pastors in the present day who are ready to give their lives for their sheep are rare ; in the present day scarcely are /. There any good pastors. 42. The Gospel promisessalvation to the faithful ; faithful ; the world condemns are Many whom the world condemns. salvation to many whom ,\ The Gospelpromises is happiness 43. Every one desires happiness ; hence ; but virtue (alone)
/.

every
44.

one

desires virtue.
"

Arist.

Eth.,bk. iii.
masters

to obey their servants obligates Christianity thingsonly which are not contrary to the law

in those
;

of God ;

But

unlawful

trafficis contrary to the law them obligate


so

of God in
an

Therefore

it does not

to

serve

unlawful

ness, busi-

but forbids them


45.

to do.

to fear; not subject are guilt thence it is that conscious hypocrites are alwaysshy and timid, and self-possessed. while the innocent are unsuspecting and Lord Derby are eminent statesmen; 46. Gladstone, Disraeli, But they are also eminent authors ; is not inconsistent with statesmanship. success cases /. In some literary one to sacrificeis greaterthan all others save 47. (The commandment ;)

Only they who

are

not

conscious

of

To
/.

love is

more

than sacrifice ; commandment. greatest


"

To love God No
man

is the

See Mark

28-34. xii,

48.

is to be

tur pro

alicno
our

punishedfor the crime Legal maxim. delicto) ;


"

of another

(Netno puni-

Nearlyall of
.*.

miseries
our

are

entailed
are

on

us

by the

crimes of others ;

Few,
A

if any, of

miseries

49.

true

his chief places philosopher excellence ;

punishments. in happiness activity ;

moral

lectual and intel-

But
.-. A

there is

no

excellence without

true

50. Put

in his chief happiness places philosopher activity. Cicero's episyllogism ("2) in form, and name

moral

lectual and intel-

the mood.

COMPOUND

AND

DISGUISED

FORMS.

205

What

names

may

be

given
in

to

the

following

reasonings

51.

From For with hence

given
a

point

line

only
drawn,
be
a

one

perpendicular angle
which

can

be would

drawn. make and

if

second

could line that all be

be would formed

the

it

the

given
to

right
the
same

angle
side

by by
But

definition,
the

equal

on

first
of

pendicular; perthese be

for

right
part
which
in

angles
of

are

equal.
hence

one

angles

would
to

the

other,

part

would

equal
52.

the

whole,
Demetrius
35-41.

is

impossible.
Acts

Those clerk Those

used
in used

by
vers.

xix,

23-27

and

by

the

town-

53.

by

our

Lord

in

Luke

xiii,

15-16;

and

in

John

x,

34-36.

54.

Those That

used used

by by

Paul

in

Rom. in

v,

7-10.

55.

Eliphaz

Job

iv, 17-19.

206

OF

REASONINGS.

V.

CONDITIONALS.
have been considered.
is
now

"
common

1. Thus

far

forms onlycategorical

The
to

stated.

conditional doctrine respecting logical whether it may be inquired Subsequently


to

forms

be

this -doctrine needs

and modification, A

what

extent.

is

A conditional judgment absolutely. categorical judgment predicates tion. which constitutes a condiaffirms relatively to some prerequisite the condition Its forms are primarily as according distinguished in a disjuncof an antecedent clause, or tion, by means implied expressed
or

both.
f

Thus

:
e.

Categorical
f

Judgments

-j
( Conditional
"

Conjunctive ; e. Disjunctive -J ; e.

g., S is P, and S is not g., If A is B, C is D.

P.

( Dilemmatic
By
Boethius conditional
" "

; e.

g., C is either D or non-D. g., If A is B, C is either D

or

non-D.

synonymous

with

this,having been
here. Each

is used as to put together) (con-dare, and to place under), hypothetical (vTro-nOevat, after him, is adopted usual with most logicians
"

of the three forms

of

conditionals, then,is also called ge" "

is the The word a hypothetical. nerically supposition (sub-ponere) " Latin congener of with it. The dilemand synonymous hypothesis," matic proposition, is also called because of its compound character,

the

conjunctive-disjunctive proposition.1
2. A

of which, involves two clauses, one conjunctive hypothetical the condition, is regardedas the subordinate member, and expressing is called the antecedent, the reason, the protasis to stretch (Trpo-reiveii', is regardedas the the conditioned, before).The other,expressing clause or member, and is called the consequent, the apodosis principal is to give back). Usually and formallythe antecedent (cnrottiSorai, written first, but inversions are quitecommon.

"

with as hypotheticalspecifically, conjunctive." synonymous Hence the dilemmatic he terms a hypothetico-disjunctive proposition. See Logic, also Bain Whately, and, indeed, except Mansel,all the Oxford p. 167. logicians, and others, and "conditional7' as specific. as use "hypothetical" generic,
uses

Hamilton

"

"

"

208

OF

REASONINGS.

to be strictly must make a complete Disjunctive judgments, logical, the disjunct members must exhaust the divisum, disjunction ; that is, be exclusive of each other. They are therefore contradictoand must ries. The characteristic of contradictory is that the oppoopposition

sites cannot
true

both be true
must

and

cannot

both

be false : i.e., one

must

be

and

one

be

false ;

either denies the other, hence, affirming other. The form


is that

denying given; i. e.,

and

either affirms the

already

Either C is Either all Either the

D, or

C is non-D.
are wars are

wars

or some evil, is guilty, or prisoner

not

evil.

he is not

guilty.
have
a

When

the division is

more

than and

dichotomous,we
e.

series of

terms, exhaustive disparate


C is either D, Bodies
are or

coexclusive ;

g.,

either

E, or F, or or or solid, liquid,

aeriform.

must dictories Disparates alwaysbe reduced,for logical treatment, to contraby groupingthem into two opposed members ; e. g., Bodies
are

either solid either

Angles are
Less than

or (liquid rightor (acuteor or

aeriform obtuse
=

=)

fluid.

) oblique.

all the members

of

series disparate

will not

yielda

junctive dis-

judgment,since they are not exhaustive. Thus, to say "Birds shot either sitting for they or are flying is insufficient, may be shot the extreme which are contraries, running or swimming. Hence of a disparate cannot terms a disjunctive series, yield judgment. Thus
"

we

cannot

and

" Men are say therefore the statement

either black

or

white,"for
nor

some

are

red,

is neither true

We also be

said above

in that,

the strictness, logical


is

coexclusive. the

This

true,but often

logical. members must disjunct make we an imperfect

division wherein constitute


or

not but intersect, are and coexclusive, species communicant perfect species.Such a division will yieldan immon, judgment,which, as it is very comincomplete disjunctive

it is needful to take into consideration.


Jack is either
one or
a

E. g.,

fool

or

knave.

We may

affirm he be both.
may

must

be

the

other,but
not

it is also true

that he

These

terms, then,are
thus
:

contradictories. The

ment judg-

be formulated

Either C is D,

or

C is T.

Here

and

stand

for

communicant
one

species.The
true, and both

principle governing
may be
true
:

this form

is that

must

be

CONDITIONALS.

hence,denying one
As

affirms the other ; but

lows. folaffirming one, nothing

for conwe venience, will, subcontrary opposition, and disparate these from contradictory disjunctive distinguish judgmentsas subcontrary disjunctive judgments. Disjunctive judgmentsfrequently appear in the form this is the law of
"

Either C is

D, or

is N.
was a a

Either Richard Either the

III

monster, or Shakespearewas
or

wrong. I mistook

has patient
was

Either Cassar

fever, Brutus or ambitious,


of the

the doctor errs,


was

or

him, etc.

criminal.

Here
a

the matter
not

judgment is
"

between

distinct. Such opposed clauses is entirely for there is no immediate disjunctive, directly position opis mediate ; the opposed clauses. The opposition
he

thus,
Either Richard
Bat If he
was

III
a

was

monster,

or

was

not

monster

not

monster, Shakespearewas
III
was a

wrong

Either Hence,

Richard

monster,

or

Shakespearewas
members that

wrong.

The

not between then,is declared, alternative,

are

directly
of

but opposed, the other.

between

one

of these and

the necessary

consequence

ways aland disjunctive are judgmentsalways affirm, conjunctive C is neither D nor never E," negative. If we say positive, The boy is neither smart nor e. g., good,"this is not to declare an but is merelya double denial. alternative, Both
" "

indicates, propositions " 4. Conjunctive-disjunctive are, as the name compounds of the two precedingforms, and hence involve no new

havinga disjunction conjunctives in both ; or, inverting the in the antecedent, in the consequent, or or in both in one or formula, having a conjunction they are disjunctives to greatapparent members. Their forms,which are certainly subject and intricate. If there numerous as are variations, usually represented is called dilemmatic but two are members, the proposition disjunct They principle.
may
as

be defined

double a (oi-Xe/i/m,

assumption) ; if three,trilemmatic
more

; if

four,te-

tralemmatic ; the

or

if

than

two,
is

ever, howpolylemmatic.Ordinarily,
to applied

"dilemmatic" adjective

all indiscriminately.8

when speakingof will use onlythe adjective we confusion, to certhe use of the nouns tain dilemma," trilemma," propositions, restricting etc.,
8

To

avoid

common

"

"

forms syllogistic

hereafter to be described.

14

210

OF

REASONINGS.

In

"

we

gave the
If A is

abbreviated following

form

as

representative:

B, C

is either D

or

non-D.

This may

now

be

expandedto
is

"

If A

B, either
"

C is

D, or

C is non-D.

Again expandingwe

have

Either if A

is B, C is D ;

or

if A

is

B, C

is non-D.
same

This

now

appears

as

double

hypothetical having the

dents antece-

and

disjunct consequents.
consider that
a

Now of two

let us

difference in the matter, or

in the

quality

is D," and "C is non-D," makes them distinct e. g., "C clauses, E is F ;" C is D," and clauses, quite properlyrepresented by for a partial does not modify, except in ultimate analysis, identity of such propositions. the logical treatment Also, that a separate of contradictory and be representation subcontrary opposition may that every formal statement omitted,with the understanding sents repreeither. Again,since disparate members be must always grouped, for logical treatment, into two contradictory members, and since the forms trilemmatic and polylemmatic and subject of this nature to are this rule, they also may be omitted in a classification of forms that is grounded on the relation of clauses rather than of terms. Under these provisos exhaustive we an may represent very easily of the conjunctive-disjunctive statement forms,thus :
" "

1,Simple,(a)" Either if A
"

is

B, C

is D ;

or

if A

is

B, E

is F ;

having antecedents
is

identical and

consequents disjunct.
identical.

"

(b""Either if A
"

having
if A

B, C is D ; or if E is F, C is D ; antecedents "disjunct and consequents


B, C is D ;
if E is H.

2, Complex,

"Either
"

is F, G or and consequents disjunct. having antecedents disjunct

is

The
1

are following

concrete

examplesof
was

these forms
or

(a)
"

If Socrates

was

innocent, Anytus
we

either deceived
a

If

we

go

to war,

must

either contract

debt

or

perjured. increase taxation, or

demnify in-

ourselves at the
1

enemy's expense.
a

(b)
"

If

man

is either well

or

illdeserving, he is he
at

moral
or

agent.
the

If my

king is moved, or if I am checkmated piece,


knew prisoner he
was

is

covered,
move.

if I capture

attacking
or

the next

If the

the consequences

of his act, he

was

criminal ;

if not,

insane. is

Either

popular, popular, compulsion is unnecessary ; or if it is unwill be resisted if the compulsion people are indifferent, ; or compulsionwill be fruitless.

if education

CONDITIONALS.

211

now described, "Upon the basis of the conditional propositions have in tererected a system of syllogizing minology logicians corresponding with the categorical system. There are then four kinds of conditional syllogisms. The Conjunctivehypothetical which has for its is one syllogism the minor premiseand conclusion major premisea conjunctive proposition, being the affirmation or denial of its component clauses. It

"

5.

claims

for

its

canon

the Law

of

Sufficient

Reason, modified
its

thus

for Every assertion must have a ground or reason is explicated into two axioms, as follows : the reason 1. Asserting asserts the consequent.
2.

This support.3

Denyingthe consequent denies


converse

the

reason.

The

of neither

axiom, it

is

is said,

true.

Denying the
other
reason.

son rea-

does not
not

deny
reason

the

and consequent,

the consequent does affirming


some

affirm the

; for it may

follow from

To

do either is

to this. fallacy.But we shall find exceptions The double axiom The form givesrise to two so-called moods. the conjunctive in these two moods, and their names, syllogisms
as

of
are

follows
MODUS

: f

If A But
A

is B ; is B ;

then

C is D ;
But

POXENS

""

C is not
is not

D ;

"" MODUS

TOLLENS

( .'. C (constructive)
If the PONENS.
"

is D.

',

.'. A

B.

(destructive).
TOLLENS.

wealth peopleare industrious,


are

increases ;
is not

They
.'. Wealth

industrious ;
is

',

Wealth
.*. The

increasing ;
"

increasing;;

peopleare

not

industrious.

is always universal and affirmative. or major premise, sumption, Either or both of its clauses may be particular or negative ; but the total proposition affirms that the antecedent necessitates alwaysuniversally the consequent. It will be observed the minor that the the both major premiseonly is conditional, conclusion
in

The

premise and
the
also

each

of

the

two

moods ise, prem-

If beingcategorical.
were

both or conclusion,

it and
not

the minor be

the hypothetical,

reasoningwould
are

conditional,

but

This categorical. the

paradox will be illustrated subsequently.

In but
terms

may

only three propositions, there may be four terms, as in the given example. All the in the major premise. Hence, unlike the categorical occur logism sylthe minor premiseintroduces no new term, and the conclusion have nothingin common with it.
3

conjunctive syllogismthere

See Part

ii, 1st, " 7.

212

OF

REASONINGS.

There
1. The

are

three RULES

deduced

from

the

axioms,as follows

clause subsumptionin Ponens agrees with its corresponding differ in quantity. but may in quality, with its corresponding 2. The subsumption in Tollcns disagrees clause in both quantity and quality. with conclusion in Ponens 3. The agrees, and in Tollcns disagrees, its corresponding clause in both quantity and quality.
in Tollens are because the only true denial is by disagreements the Let us, however, remember that when the logical contradictory. in the previousexample,contradiction is as subjectis individual, In illustration of the rules, we merely a changeof quality. may take

These

the

: following

If any
are

nation prospers

all

are

benefited ;

SA11
or or

Some

prospering ; are prospering ;


prospers ;
or

Some That

are one are

not

benefited ; benefited ;

"" TOLLENS.

This

one are

is not

.'. All

benefited.

.*. None

prospering.
The principles.

one or both,do Negativeclauses, is in strict conformity : following

not

offend these

If A

is not is not

B, then
B ;

C is not C is D ;
is B.

D ;
"

PONENS, asserts,
"

TOLLENS, denies,

.'. C is not (constructive.)

D.

! .'. A

(destructive.)

If

we

we contrapone a majorpremise,

shall find that this reduces the

moods which

the

one

to the

other.

Hence the

might be the two evolving


In reductio
ad

inferred from moods.

the same, they are fundamentally of the two axioms common origin

it is quite usual impossibile the tollent mood g., "If


we

to

state

the argument

and then hypothetically,


as

to

be left God
a

unexpressed;e.
liar." of

applies, perhapsso obviously have not sinned, we say we

make A

in detail might be urged against this objections will be content at present with pointing scheme of syllogizing. We clause is the infinitated out a couple of exceptions.Whenever one form of the
as other,
"

number

If A

is B, A

is not

non-B," or

if the antecedent of each


iom ax-

is the sole condition

of the

then the consequent,

converse

is true ; i.e., denying the antecedent denies the consequent, and the consequent affirms the antecedent. As these forms affirming

CONDITIONALS.

213

not

unfrequently occur,
rlo,
"

the rules should

providefor

them.

For

am ex-

If silver is

If not If he

even
was

it cannot legal tender, then one was saved,

be refused all
were

in payment of debts.

lost.

saw present, you certainly

him.

If force is

If the scheme

expended,an generated. is perfect, will hold. no exception

equivalentforce is

of the hypothetical the following as the Canon (i. gives e., or more : "Two syllogism propositions beingthought as conjunctive) in quality, indeterminate but as in quality the mutuallydependent, of quality in the one infers a determination of the determination in the other." But this, the other hand, on corresponding quality is true, in its full generality, ry only of the above exceptedcontradicto-

Hamilton

forms.

Hamilton Nevertheless, the simplifies been whole

goes

on

to

"

say,

This

Canon

bodies em-

and which
the Mill

have

syllogisms, mystery of hypothetical and confused mutilated, strangely implicated, by


then
a

He logicians."
"

says, in
our

There

is

proceedsto an of great quantity


and
a

elaborate criticaldiscussion. intricate and obscure

author's Lectures

their

to appendices, relating

speculation, junctive dis-

like a true critic, hypothetical propositions."But Mill, others to reconstruct as proceedsonly in the tollent mood, leaving they may.

and

is one Disjunctive hypothetical syllogism havingfor its and having the disjunction major premisea disjunctive proposition, resolved in the minor premiseand conclusion.6 Accordingto Hamilton, it is governedby the axiom of Excluded that Middle,7 affirming
6.

"

The

of two

contradictories

one

must

be

true

and

the other false.

The

5 Examination vol. ii, Logic, of Hamilton, p. 602. p. 225. Our Hypothetical and Disjunctive be reduced to the class of Syllogisms may Conditional Syllogisms. The Hypotheti.cals should be called, as by they were Boethius and others, the in co-ordinate to of contrast Conjunctive, species Disjunctive. of the species, as a name Hypothetical, ought to be abandoned. The Conjunctive inasmuch are as solutely negationor affirmation is not abconditional, but and left of the is one made quality asserted, proposition alternative, inasmuch dependent on another. The Disjunctive are as is a notion conditional, not absolutely asserted as subjector predicate of another or others, but alternatively with some of but with some a conjoined of part, part, given plurality only the affirmation of it with one the negation of others." notions, part involving 7 of (Hamilton's Logic, p. 600.) Rather, Duality. See Part 1st, ii, " 4. 8
"

214

OF

REASONINGS.

members wo being contradictories, disjunct may, through affirming the and This vice two other, versa. moods, each of yields one, deny which is double. and forms : names They have the following
MODUS PONENDO TOLLENS.

All To. PONENS.


"

men
are are

are

either

Some

not

or justified justified;

under

condemnation
are are

;
"

Some
.*. Some

justified ;
not
or

Po. TOLLENS.

.'. Some

under
or

cond.

under

cond.

This
.*. He

one

is not

under

cond.

That
.*. He

one

is under

cond.

is

justified.

is not

justified.

is universal not the clauses) sumption (thewhole proposition, The conclusion must the subsurnption and affirmative; vary. may and the opposite the subsumption, have the same as quantity quality. We shall hereafter find that every contradictory disjunctive ment judgEach be reduced to two conjunctives. conjunctive yields may with the four two moods, and the four forms thus arising correspond forms. disjunctive Negativeclauses call for no modification of the rules ; e. g. :

The

Either C is not Po. TOLLENS.


"

But

C is not

D, or D, /.

E is not E is F.

F ;

If the

three or more major premisepresents they must be reduced to two terms),


can

members disjunct

parate (disan

contradictories before

inference

be drawn.
Sciences

E. g.
are

either pure,
is neither
a a

or inductive, nor an

mixed

To. PONENS.

"

Astronomy

pure

inductive science ;

.*. Astronomy is

mixed

science.

formal

illustration is

as

follows
or

Either C is D,

E is

F, or

G is H.

Now

let the first clause be Then


:

of contradictory

the other two

taken

gether. to-

To. PONENS."
"

C is not

D ; /. Either E is

F, or
:

G is H.

Neither E is
"

F, nor

G is H

.'. C is D.

Po. TOLLENS.
"

C is D ; .'. Neither E is F, nor G is H. Either E is F, or G is II ; .'." is not D.

216

OF

REASONINGS.

is not wealth,or fame," the disjunction pleasure, be valued on other grounds. We for virtue may should exhaustive, members the disjunct further ascertain whether are or contradictory viewing the only subcontrary.We may then proceed to syllogize, either as a conjunctive or as a disjunctive. proposition the conjunction members If we on reason lookingon the disjunct is governed by the principles the syllogism as a explained singleclause, it must
ensure

either

in

"

5 ; thus : Either if A
is

B, C

is D ;

or

if A

is

B, E

is F ; E is F ;" TOLLENS.

POXENS."

But

A is B ;

j
E is F.
"

Neither
/. A

C is D, nor

/. Either C is

D, or

is not

B. deceived.
_,

PONENS.

"

taughtfalsely, they were either deceivers or apostles They were neither deceivers ) They did teach falsely ; f either deceivers deceived. nor .*. They were deceived. or .'. They did not teach falsely.
If the

in each mood. The one plainly hypothetical syllogisms, the tollent form, is givenby Kant and Wolf as a dilemma, and latter, But certainly it is by Wallis and Mansel as a disjunctive syllogism.7 neither ; for it simplydenies the antecedent throughdenyingthe consequent; resolves one. This it neither introduces a disjunction nor is sometimes tollent form called cornutus, or the horned syllogism because in the sumption the disjunctive members are (bi-cornis), opposed like horns to the assertion of the adversary ; with these we in order to throw it from one side to the other in the subsumption, is very Such a syllogism toss it altogether away in the conclusion. abused for the purpose of deceiving, easily through a treacherous appearance of solidity, and from terrifying a timorous opponent by its horned aspect." Krug. It should be remarked that while the particleseither or" are disjunctive the corresponding negatives neither nor" are not so, but total. They do not exclude one condition of or on are conjunctive but they exclude both or all. They directly the inclusion of the other, and consequently deny both clauses, deny the existence of the disjunction.
are
"
"

These

"

"

"

"

This is not

but annihilation. resolution,

He says this form of reasoning is sometimes Aldrich, p. 109, note. duces called a Dilemma, but it is a perversion of the Dilemma proper, and introis distinction whatever no as disjunctive syllogism, ; being merelya common shown in fact, the enumeratio,not ch. 19). It is, the comby Wallis himcelf (iii, of Cicero. plexio,

See

Hansel's

CONDITIONALS.

21 7

involved disjunction in the conjunctive-disjunctive lookingon the conjunctive proposition the syllogism is disjunctive, and governed statement a single as clause, in " 6. by the principles Thus, adoptingfor convenience explained

Now,

on

the other

hand, if

we

reason

on

the

the abbreviated

form, we
If A

have
is

B, either
D ; is F.

C is

D, or

E is F ;

To. PONENS."

But
/. If A

0 is not

is

B, E

If Socrates But Anytus


.*. If

was was was

innocent, Anytus
not

was

either deceived

or

perjured ;

deceived ;
was

Socrates

innocent, Anytus
was

perjured.

have another To. Ponens. we perjured, If the disjunct members contradictories, they are, as in this example, also two forms in Po. Tollens, or the destructive mood. yield The conjunctive-disjunctive now explained, syllogism, although it is not properly has a dilemmatic dilemma at all. The a sumption, full of confusion here, and often mistake it for the dilemma. are logics and rejects the Thomson, following so names it, Hamilton,distinctly

By denying that Anytus

dilemma

proper

from

Logic.8

is a conditional syllogism (or trilemma,etc.) " 8. The Dilemma and a disjunctive conjunctive etc.) premise, havinga double (ortriple, is dilemmatic, or conjunctivopremise. Neither one of its propositions treated conjuncIn the conjunctive-disjunctive tively, disjunctive. syllogism first the disjunct in the as section, explained part of the preceding members of the sumption are either affirmed both togetheror denied both together. But they may be affirmed or denied disjunctively. is declared in the subdisjunction existing It is therefore in the conclusion. and (in the complex forms) sumption, which then appears merely needless to state it in the sumption, is distributed; Thus the dilemmatic as a double proposition conjunctive. in the of its essential features, the conjunction, one appearing in the subsumption the disjunction, sumption; the other, appearing In this
case

the

and

conclusion.
as

It

the definition above as indifferent, indeed, is, called the

plies, im-

sumption and which the subsumption conjunctive ; but it is usual to place the double and call it the sumption or major premise. first,
to

which

of these shall be

'

So also Bain.

See

Logic, p. 121

sq.

218

OF

REASONINGS.

The follows
1.

Dilemma
:

presents three distinct and


If A But
.'. C is

inconvertible

forms,as

Simple constructive
PONENS.
"

is

B, C

is D ; and is

if E is F, C is D ; is F ;

either A D. is

B, or

2.

Complex

constructive
"

If A
But

B, C

is D ; and
is

if E is F, G is H ;
E is F ;

PONENS.

either A

B, or

.'. Either C is

D, or

G is H. if E is

3.

Complex

destructive
"

If A But

is

B, C
A

is D ; and

F, G
F.

is H ; H ;

TOLLENS.

either C is not is not

D, or

G is not

.'. Either

B, or

E is not

It will be observed between

that the

subsumptionin
components
the conclusion

each form
of the

declares

junction dis-

certain of the

; that in the simpleform in the of

is

conjunctive sumption and that categorical ;


components
must

between complex it declares a disjunction the sumption. A single concrete example from Demosthenes It is in the complex constructive form, as

the other

de

Corona
:

fice. suf-

follows

If JEschines

joinedin is unpatriotic ;
either he

the

he publicrejoicings,

is inconsistent ; if he did not,he

But

.*. Either he is

he did not or did, he or inconsistent,

is

unpatriotic.

The

form

of the

sumption in
If A is

this

examplemay
if A is not

be

thus expressed

B, A

is

C;

and

B, A

is D.

Here

the firstterm

of each of the clauses is the same, the negative. Nevertheless,

and the antecedents form is

differ onlyby the


to corresponding

complex,
other

No.
in

2 ; for

the clauses all differ from

each

either in matter There dilemma.


cannot

or

quality.
a

be both

constructive simple

and

Denying
If But
.'. A

the consequents in No. 1

destructive simple gives


a
"

AisB,Cis
is not

D;

andif

E is

F,C

is

D;

C is not D ; B ; and E is not F.

in Tollens. syllogism This,however, is merelya double conjunctive has an appearance of being than the last, much The following, more the It is given as to No. 1. simpledestructive form, corresponding and copiedfrom him with approbation such by Fowler, by McCosh :
If A But
.*. A

is B, C is D ; and either C is not B.

if A

is

D, or

E is not

B, E is F; F;

is not

CONDITIONALS.

219

But and and

if this then then

be

examined,
in its

it will

be

found
to treat

to
a

be

No.

contraponed,
in

treated
to treat

Tollens.

Now,

proposition
is to

Ponens,
the
same

contraponed
in

form

in is the

Tollens,
same.

do

thing.
cannot

The be

reasoning
as

both

cases

Hence
it

this the
same

form
as

accepted

additional

to

those
a

given,

being

No.

1, and
of

only slightlydisguised by
the letters
in

rearrangement,

after is
a

sition, contrapo-

alphabetical
is to
or

order.
a

Truly, simple

it

simple
may

structive de-

dilemma,
either
are

which

say the

that

dilemma

appear these is the

in

the

destructive same,
are

in

constructive
not

form

'; but, since


; and

essentiallythe
which
we

we

should

reckon

both

this

statement

now

supporting.
"

Whately,9
destructive is true
a

endorsed

by Mansel,10
that
"

says,

There
is

cannot

be

simple
This

dilemma,"
his

the
of

destructive the

always complex."
"

under

definition

dilemma,

as

syllogism having
more

conditional
and

(i. e., conjunctive) major


a

premise
But the

with

than
of
narrow.

one

antecedent,

disjunctive

minor."

this

limitation is too be

the

major
The
not

premise

is

purely arbitrary,and
under the proper

definition there may

truth

is that

definition

either,but

both. In

disputation
If
to

an

adversary
meet

is sometimes another with

"

caught
an

on

the

horns

of

dilemma."
is said Athenian
; for if

he

it

by

opposite conclusion,
thus engage you
son

he "An

"rebut mother

the said what

dilemma."
to

Aristotle
son,

illustrates in

it: fairs afdo the

her

Do will This

not

public
if you

you

do

is will
to
me

just, men
hate
enter

hate the

; and

what

is

unjust, the gods


:

you. into

rebutted
; for

by

following retort
is

ought
love
are

public affairs
what is
constructive

if I do

what will each

unjust,men
me." Both

will

; and

if I do

just,the
form,
The

gods
and

love

these
an

in the

complex
Cynic,
who

is followed

by

implied categorical syllogism.


the

first dilemma

originated with
himself
from

Antisthenes with

proposed

by

it to

excuse

meddling

politics.
10

ch. iv," Logic,bk. ii,

5.

Note

in

Aldrich, p.

108.

220

OF

REASONINGS.

"

9. Praxis.

to Specify

what

class each and

of the

following judgments

its members. form, logical distinguish belongs, put into which of the five forms does it fall ? Contrapone If conjunctive, is it contradictory four or subcontrary, examples. If disjunctive, If reduce to If mediate or immediate? a dichotomy. a polytomy, is it simpleor complex ? Formulate with letters. If the dilemmatic, is defective, proposition say wherein.
it in

1. Wherever

there is

smoke, there

is fire.

(Wherever

If in any

place.)
2. If
a

administered, government is well constituted and skilfully

it is

promotiveof

the I

and industry

wealth
err

of its
is

subjects.

3. If I err, it is because

am

human

; for to

human.

(Isthis
if not.)

good reasoning?)
4. I will not 5. Until the 6. Is any let thee go, unless thou

bless

me.

(Unless
=

nightcome,
or

we

must

work. pray.

you among 7. Lear is at the hut

let him afflicted, the

8. 9. 10.

palace. (Real difference.) Hiawatha left his hut or wigwam. (Nominal.) If the rebellion be not crushed, the king will be dethroned. If virtue is voluntary, then vice is. Aristotle, N. Ethics, bk.
"

iii. the

11. Punishment

is intended

either to repress crime

or

to

reform

criminal.
12.

(Perhaps both.)
threats could

Neither
were

nor flattery

prevail.
do the works
same

13. If ye 14.

Abraham's the
n sun

seed, you
and
moon

would

of Abraham. the tides line,

Whenever
are

attract in the
=

at

maximum.
a

15. Either if this is

If at any time.) judgment,itaffirms or denies; or if it is

(Whenever

tion, ques-

it does neither.
16. Ye

shall not

eat of

neither shall ye touch it, lest ye die. it,

17. If ye eat,ye shall die. Though ye eat,ye shall not die, (Though if. The concessive clause, introduced by " though," =even etc., of grantsthe protasis clause. principal
not
a

sentence

whose
"

is denied by apodosis

the

The

above

means

It is not

true,or

it docs

18.

that if ye eat,ye shall die.") follow, Though deep,yet clear;though gentle, yet not dull. been decided whether the
or war

19. It has not 20. 21. 22.

will continue

or

not.

If Caesar Those The


sun

he will either rule lives, slew Caesar round


are

ruin.

who
moves

either

or patriots parricides. moves

the

or the earth earth,

round

the

sun.

CONDITIONALS.

221

23.

AlthoughHomer
of

sometimes

nods,nevertheless is he the greatest


P.

poets.
amat
woman

24. Aut

aut

odit mulier ; nihil tertium.


loves
or

"

Syrus.

("A
25.

hates; she

never

thirds

it.")
them. destroyed

If you

have

(Si non
26.

the poor, you have occidisti. From Arnauld.) pavisti, failed to nourish
"

If the heart is
is solitary

the right, either


a

actions will be. beast


or a
"

27. The
28.

29.

Polit. i,cap. 1. god. Aristotle, The world will not be happy until either kings become phers, philosobecome or kings. Plato's Repub. philosophers have worn he must made by the prisoner, If the foot-marks were
"

shoes too
30. Virtue
31.

small for his feet.


"

Ad

ybs.

is teachable

if it is
or

If

man

is either well

knowledge. he illdeserving,

is

moral

agent.

32. If the

be of the sides of a triangle one square described upon equalto both the squares described on the other two sides of

it,the angle contained


33.

by

these two

sides is

rightangle.
"

bk. i. Euclid,Prop,xlviii, There could be no choice, were


nor nor grief, pain, Invades thy bounds. This elegant rose, had

there

no

difference.

34. Nor

anxious

fear,
it less, awhile.

35.

I shaken

Might have
If
an

bloomed

with its

owner

kind

is merely a proposition, example among the following gize syllofrom the it. If an incomplete syllogism, completeit and name If a conjunctive-disjunctive, and mood. or a dilemma, classify

and formulate it,

with letters.

If

defective, say wherein.


the

36. If any

that objection

established
37. Mahomet
was an

a change in urged would justify be maintained. laws could reasonably no laws, either an enthusiast or an impostor; can

be

He
.*. He

was was

enthusiast ;
an

not

impostor.
"

Gibbon.
are

38.

Corn be

will be
so.

dear

if the crops

bad, and they


time

seem

to likely

39. A

government
of
a

cannot

be

at the

same

and the despotic

censer li-

free press ;
;

But
/.

the

The

a free press Englishgovernment permits Englishgovernment is not despotic.

OF

REASONINGS.

40. If

man

cannot

make
an

progress towards

must we perfection,

believe

41. I

divine. or brute, already incapable be accused of actingcontrary to my law, with justice, could, only if I maintained that Mursena purchasedthe votes and was that he did not buy But I maintain in doing so. justified the votes; therefore I do nothing contraryto the law." Cicero pro L. Murcena, cap. iii. (Ramus cites this as bad reasoning. him
to be either

Was
42. Unless

he

?) right
can a move

matter

of

its firstmotion itself, But


matter

must
move

have been itself;

given it by
43. If

being. spiritual
be

cannot

etc. therefore,

pain is severe, it will slight ; hence it should


suppose knew
can none

brief ; and

if it last long, it will be

be borne

patiently.
not not

44. If the

system of the universe is


that the Creator did

the best

we possible,

must

or prefera better,

that he But
we

that he could or better,

not

create

better.

45.

by for we should theresuppositions, his power. Therefore his intelligence, limit his goodness, or the system of the universe is the best. (Thomson, Outline, wise. p. 150, like" 109, quotes this as a trilemraa. McCosh, Logic, Are they right ?) of mental discipline or as Whether Logic be regardedas a means But it is it ought to be studied. a practical guide in reasoning, entertain neither of these both. Hence
man were
"

what ?

46. If this

wise,he would
were or jest

not

of Scripture speakirreverently would


;
not

in

jest ;

and if he

good,he

do

so

in earnest;

But
/.

he does

it either in

in earnest

Either he is not

wise,or

he is not

47. If the books

in the Alexandrine

good. Librarybe
is
no

in

conformitywith
; if

the doctrines of the then also

Koran, there
be burned. is worth

need

of them

adverse,

they should

48. If classical education fitted to

either it must the cost, mental

be

nently pre-emi-

developthe

valuable exceedingly be

information.

furnish powers, or it must But neither alternative can is not


a

and so classical education maintained, is givenby Bain,Logic,p. 122, as (This

worth

the cost.

dilemma.

Right?)

49. If any

have been

lishment the estabtheory could be framed to explain satisfactory of Christianity by human causes, such a theorywould such theoryhas ever been proposed before now ; but no be framed. can proposed; hence,none

224

OF

REASONINGS.

57.

If

Christ among there and


faith if

be you be
no

preached
that there

that
is

he
no

rose

from

the
of then

dead,
the
is

how dead

say
?
not

some

resurrection the

But risen your

if

resurrection be
not

of

dead,
is
our

Christ

Christ
is also

risen, then Yea,


and
we

preaching
found that the he dead
:

vain,

and

vain.
we

are

false
raised rise

witnesses

of

God,
,

because he dead

have
not

testified up,
if is is
so

of

God that

up
not.

Christ For

whom the
not

raised
rise

be

if

not,

then

not

Christ
;

raised

and
your

if

Christ Then If
most

be

raised,
also life

your

faith
are

vain

ye

are

yet
Christ
we

in
are

sins.

they
this

which

fallen

asleep
in

in

perished.
of all
men

in

only
1
"

we

-have
xv,

hope
12"

Christ,

are

miserable.
58.

Cor.

19.

system

of

government

which
must

extends
one

to

those

actions either
to
a

that

are

performed
divine

secretly

be this
;

which
or

refers the

regular ments punish-

providence
of another

in

life,

to

rewards

and

world
of

Every
which
.*.

perfect
are

system

government

must

extend

to

those

actions

performed
of
a

secretly
can

JSFo

system
to

government

be

perfect
in

which this

does

not to

refer the Divine


wards re-

either

regular

divine
of
5.

providence
another

life,

or

and

punishments
See

world.

"

Wai-burton's

Legation.
59.

vi, "
of what

There
we

are

two

kinds
and

things
we

we

ought (From

not

to

fret
form

about,
a

what
"

can

help,
either

cannot.

this,

dilemma.)
them latter sin and
;

60.

We the

must

gratify
will

our

vicious
us

propensities
iu must sin and

or

resist the

former

course

involve

misery,
fall into

requires
misery,
or

self-denial

therefore

we

either

practise

self-denial.

ANALYSIS

OF

CONDITIONALS.

225

VI. ANALYSIS

OF

CONDITIONALS.

two declares a relation between proposition categorical condition beingexpressed.A conthat is, ditional no terms unconditionally; involves an or proposition hypothetical express condition. in the conjunctive The latter is a complex sentence, consisting, form, and of a subordinate the apodosis of a principal or clause, consequent, antecedent or condition. the protasis or clause, and syllogisms whether conditional propositions The question may is forms discussed much to categorical not be reduced by or may proposition "By Kant and his followers the hypothetical logicians. of judgment essentially form tinct disis described as representing a the latter being thoroughlyassertorial, from the categorical; assertorial only as in its constituent parts, the former problematical each in itself them. Two the relation between judgments, regards combined in a single truth ; and thus be hypothetically false, may form. The hypobe reduced into categorical cannot this combination thetical of distinct from in like manner, is a form reasoning syllogism, and not reducible to it, the categorical, beingbased on a different law of Sufficient Reason,a raof thought, namely,the LogicalPrinciple rationati ad negationemrationis valet tione ad rationatum,a negatione consequential whether the categoris not properly But observe that the question ical the and hypothetical forms are convertible. as Theory of Logic not be done with with what may or Thought has no concern may this : Does forms. The question thoughthypotheticall proper to Logic is differ from that categorically expressed ; and if so, expressed what is the specific difference ? In other words, Are there two processes and do kinds of of thought, two we there are as propositions, how we to explain need a distinct system of syllogizing necessarily ? We to think when matter is thought hypothetically propose now its process is one ; of thought, show that while there are two spheres has essentially that all reasoning the same form, havingthe Aristotelic for its formal unit. We syllogism propose to discover the true rela-

"

1. A

See Kant's

Log'ik, " 25

and

"

76 ; and 15

Hansel's

Aldrich, Appendix,Note

I.

226

OF

REASONINGS.

and hypothetical and to show that they categorical thinking, do not differ logically, but only psychologically. will first pointout a psychological In attempting tion distincwe this, of thought; then consider the propositional between two spheres of hypotheticals, and the syllogisms therefrom ; and then use arising advert to the common doctrine of conditionals. Herein we logical that thought hope to confirm the generaldoctrine of this Treatise, is of only two kinds,immediate and mediate,and that of the logical is ultimately latter the Aristotelic syllogism the universal form.

lions of

thought as or categorically. Ideal thought considers its matter firms and afas merelylogically possible, in a supposititious that is, mode. This matter hypothetically, exist ; but thoughtposits ence, merely its ideal existmay or may not really and, limited only by self-contradiction, proceedsto evolve logically
2.
or

"

Thought

is either of the

real

of the ideal.

Real

considers its matter

and existent,

affirms

denies of it

conceivable
to

be

the real,

So even is known when the matter consequences. mind choose rather to view it ideally, thought may
one

it from transferring readily


Plato is
a

sphereto

the other.

Thus

when

I say

man,

therefore he is
a

mortal,
But when
I say

I think the matter

and draw real,


If Plato be
a

real conclusion.
then he is

man,

mortal,

I think the matter


and fact,
on

this

without regard to ideally, making a supposition I reason statement to an ideal hypothetical equally

conclusion. What

ideal from real thought is precisely what distinguish distinguishes from categorical hypothetical judgments. Thus far we and "conditional" have used the words changeably "hypothetical"as inter" But the former is opposed to categorical synonymous. in the characteristic that it formallyexpresses a condition of the presses thought; the latter in the other characteristicthat it exprincipal and not real declared fact. The ideal, supposititious thought, words should be used accordingly. It is manifest that the distinction between and categorical thetical hypobut psychological. judgments as real and ideal is not logical, it is shown This will still more that thought in plainly appear when the real and in the ideal sphereis logically the same ; that is, governed into the same laws, assuming the same forms, analyzing by the same and hence indistinguishable on logical principles, grounds.
"

ANALYSIS

OF

CONDITIONALS.

227

These

two

mental the two that

by
It would

pressed exare formally -moods, the real and the ideal, grammaticalmoods, indicative and subjunctive.

constructed and expressing by a language scientifically these moods would always of the speaker, the mind accurately refined But perhaps in all of the more be sharplydiscriminated. notablyin our own, there has been a strong tendencyto languages, obliterate the subjunctive forms, and to substitute the indicative to all which In hypothetical are propositions, express ideal thought. the indicative has largely usurped the place of the ideal, essentially
seem

subjunctive. It is quite common of mood as expressive


past tenses
with
never

grammariansto characterize the subjunctive for its But this is inept, doubt or uncertainty. is entirely sistent conexpress doubt, and its presenttense
for in this case,
or so

the doubt full conviction,

far

as

the

pression ex-

formal impliesit, being altogether actual. It should be observed

and rhetorical,

not

that the real and

ideal

arc

modes

of

of intellectual apprehension ; whereas belief and doubt are cognition, of self-consciousness. These coexist with cognitions, modes feelings, in psychological from them but are very widely separated analysis. tion distincIf,then,they are not to be made the basis of a psychological the less should they be made modes much of thought, between distinction. Any uncertainty basis of a logical attendinga premise do modifies in no way whatever the character of our reasoning. We
not
are reason one

way

when
cases

we

are

in

doubt, and

in another

when

we

certain.

In all

tion the deducapodeictically, reasoning proceeds


more so

is necessary, not

in

demonstration

than

in

dialectics.

An

and attaches to the conclusion, in a premiseis carried along, uncertainty fects The doubt afwithout being itself increased or diminished. Hence must the doubt. we the reasoning not the reasoning, nor of certainty, aside any consideration of the feelings here set entirely have care not to confuse doubt, etc. ; and especially degree of belief, real and ideal. these feelings with the intellectual moods The indicative deals with mood, then, properly
as

the

real.

It declares

facts concerning

influence of
to the
a

doubt,taken

ideal subjunctive, ideal which subjective


of
"

perhapsunder the itselfto express, what properly belongs upon deals with thought. The presentsubjunctive is objectively contingent.It expresses a supposition
facts. It has moreover,
one

fact, the ideal ;


"

which

The past tenses fact, the contingent.


come quendij

a may or may not become of this mood have,in usus lo"

to express

supposition contraryto fact,

an

not ideal,

228

OF

REASONINGS.

real distinction between The psychological but unreal. contingent, in language.3 embedded and ideal thought is thus profoundly taken It will be useful to illustrate this matter divisions, by some In the developmentof view. in a grammaticalrather than a logical forward in have moved of the subjunctive the tenses our language,

time, so

that

the present tense usually The

expresses

future

perfect time ; the im-

tense, presenttime, etc.


ceased to express
it may
"

present tense has not, however,


the book be in this room,

presenttime.

E. g., "If

be said, it would commonly now is equally If the book is in this room," etc., which, though indicative, ideal and contingent. however, the step forward in time Considering, of ideal subjunctive find three phrases as we thought: established, be found."

Perhaps more

1st. The

and apodosis future; both the protasis contingent lyingin the future ; beingsuppositions ideal and
from

(a)Future

the

of standpoint
be

the

; present

e.

g. :

If he repent, he should Should he come,


you
to

forgiven.
be welcome.

he
wax

would

Only were
I tell you

should fat,

I love you

more.

if these should that, would

hold

their peace,
out.

the stones

immediatelycry

(b)Future
2d. The
ideal

from

the
if you

of standpoint
were

the past ;
would

e.

g. :
it.

I told you

to do

I this,

reward

and

unreal present ; it

being impliedthat
e.

neither the

nor protasis

exists ; really apodosis

g. :

If he Were The

were

the
moon

I would if I could. I would tell him. here, be answered. should it definite, question would be always full if it were self-luminous.
were

If all the year To

playingholidays,
as

play would

be

tedious

as

to

work.

"

Shakespeare.

3d. The both

ideal and

unreal past ; wherein

likewise the real existence of


;
e.

is impliedly denied and apodosis protasis


If he had been

g. :
been

present, I should have


held out, the

seen

him.
not

Could

the fort have


your

citywould

have

taken.

joinedwith mine, once pledgeappeared the token ; These follies had not then been mine, had not been broken. Byron, My earlyvows Oh, had
As

fate been

this

"

Too

deeply to

be

uprooted

or

disturbed

by Schelling's Philosophy of Identity,


of being and thinking. ideal,

the declaring

absolute

of the real and the identity

ANALYSIS

OF

CONDITIONALS.

229

Besides there
are a

these fundamental number


of mixed

forms

of the pure and strictsubjunctive, forms,as follows :


e.

Past tense
Had Were

combined

with the present ;

g.

he been

these his

he were now prudent, living. he had companions prudent,

not

lost his life.


e.

with Subjunctive protasis


If this be

indicative

apodosis ;

g. :

judged treason, still will

I maintain it.

The

same

in the concessive relation


hand

(see v, " 9, Ex. 17);


shall not
be

e.

g. :

Though
The
same

joinin hand,the

wicked

unpunished.
a

in the iterative

to relation, equivalent
centres

rule ; general
and
be
moon
an

e.

g. :

If

the (atany time,or whenever)


same

of the

sun

be

in the

line with

the centre

of the

there earth,

must

eclipse.

The

with subjunctive
If love be

the

imperative ; e.
rough with

g. :
"

rough with
of

you, be

love.

Shakespeare.
be made

If thou be the Son

God, command
the

that these stones

bread.

The

with subjunctive
Had you
seen

potential ; e.
it
was

g. :

{he citybefore
and

razed,you
foreseen

might

have

thoughtit

destruct in-

could not

have
an

its fate.
e.

construction comparative
He

with

of the apodosis ellipsis ;


of note.
"

g. :

brags as (hewould brag]if he

were

Shakespeare.

of these mixed examination forms must be omitted ; we Any special being mixed, the principles onlyobserve that, governingtheir elements

govern

them.

conjunctive then,is an ideal form of speech hypothetical, either the contingent is a subthe unreal. The protasis ordinate or expressing clause related to the apodosis, in the contingent forms,either antecedent condition. This indicates a double as a qualifier or as an in thought. They are that is made forms of these hypothetical use either propositions a containing qualified term, or they are propositions first inference. We will consider the an declaring qualified propositions. Looking on the contingent forms,we observe that very often the sole purpose in the mind of a speaker usingthis form is to declare an ideal truth. It is a mere not intended to offer a one proposition, reason, but to state a judgment. In such case, since the mind passes from the ideal sphere and vice versa, these to the real, readily prop-

"

3. The

230

OF

REASONINGS.

be easily reduced to categorical forms. The generally but rather as a qualification, being viewed,not as a condition, protasis or we limitative, explicative " 2) thus : may redress the four forms (v, ositions may
1

(a)If

house

be

undermined,it will
undermined

fall

i. e., A

house

will fall.
;

(b)If (c)If (d)If


What the

voluntary, vigoris not a virtue i.e., Vigor is not voluntary virtue.


mere

virtue is

written ; rhyming is poetry, poetry is easily i. e., Poetry that is mere is written. rhyming easily
carbon will

burn, the diamond

will burn

i.e.,The

diamond,being carbon, will burn.


states

the categorical ideally, equivalent This difference is psychological states as a real fact. and grammatical, is grammatically not logical.The hypothetical a proposition The generality of a universal sentence. a complex,but logically simple, statement must not be confused with the ideality of a hypothetical
" A house undermined and " An will fall," jurious insay deed,if it be unintentionally committed, is not a crime,"the

here hypothetical

When

we

former

is stated

as

real

if not fact, having a potential,

an

existence. actual,

not ideal. The latter is both general and ideal. general, Each of the foregoing and examplesmay be taken as a general rule, stand as the major premise of a syllogism, it may be viewed as a or Other cases are specialized times somestatement,and used as a minor. and fitted to become The following, onlyparticular, only minors. cited by Fries3 as an example of a hypothetical not reducible to it : or form, is general as we categorical particular according interpret
If Caius is

It is

he disengaged,
as a

is

writing poetry.

It may

be construed

universal statement

meaning
"

whenever Caius,

is writing disengaged, poetry,


"

thus But

iterative relation, At any or every expressing also be construed as a particular it may statement
is Caius, being disengaged,
now

time

that,"etc.
"

meaning

writing poetry.
in jeder behauptet,

Es ist sogar fehlerhaf t, indem man Systemder Logik," 62. ein die Subjekthat,konne man hypothetischen Regel,die nur Caius verbinden Wenn frei z. eine kategorische B., Regel ; der ganze Yordersatz dichtet er.' Im Allgemeinen, wenn so
'

"

beiden Pradikate
von

in

mit dem
so

Pradikat

verbunden

und

nicht

nur

sein Pradikat

der

geht diese Veranderung gar nicht. Noch willkiirlicher sind die hypothetische wenn Regel zwei verschiedene Subjekte hat." 239. Aldr'wli, p.

ist, Subjekt Grund im Satze ist, die Veriinderungen,


oder sein

Geschaften

See

gel's also Man-

232

OF

REASONINGS.

are proposition,

obscure

or

unknown. kind.
arc

But

this does not


is

differentiate

these that
four

as propositions of conjunctives

of another four
terms

Nor

it,as Thomson

always causal.
very is

says, Attributives of
as reason

terms,

as

the first example

above, are

common,
no

well

as

reducible causals of three terms.


in

Moreover, there

why,

from the atthe causal should be distinguished tributive Logic, deduction all judgments are thought judgment. In logical and far as they are and conceived in so cause effect, attributively, and consequent. stand to each other in the relation of reason thought, becomes reason. cause subjective Objective

deductive

forms claring deas contingent propositions inference. The conjunctive an as a whole, affirms proposition, of which the two subordinate propositions relation between it cona sists. It expresses a judgment respecting judgments. It is logically is the subject, and the protasis The apodosis the a simplesentence. is so named, and The protasis ten writpredicate.5 (jrpoTtiveiv} usually shall hereafter show, it is in reality we as because, first, a premise,

"

4. Let

us

consider

now

the

and hence And

the

antecedent. logical declares ? This relation is conjunctive The proposition declares consequence.

what

is the relation the

invariable. that
one

It is the relation of

judgment is consequent on, or follows from, another. Let it now be particularly observed that the affirmation is not onlysimple, 1. c., this relation is affirmed unconditionally. but categorical; E. g. :
If virtue is

knowledge,it is
its

teachable.
we

Now
That

of this proposition strip


virtue is teachable is
an

and dress, hypothetical


the

have

"

inference from not

judgment

that it is

knowledge.
sented pre-

This

is

But purelycategorical. in its hypothetical dress.

less is it

when categorical

The

relation of the clauses is real.

The

that of

invert this statement. generally logicians here which evidently which something is said,
If is reliable, the history

But

the

is subject, properly,
e.

is the apodosis; the better

g. :

latter

days are

days.

days being the better days,and we say quite of history. on, or follows from, the reliability in conjunctives would be a littleplainer This relation of subjectand predicate if and stated thus : the usual form were, as in the present sentence, inverted,
Here
we are

talkingabout

the latter

simply that

its truth

is conditioned

C is D, if A The flowers will

is B ;
sun

bloom,if the

shines.

ANALYSIS

OF

CONDITIONALS.

233

That the
the other
as

having one conjunctive proposition,

clause

as

its

subject,

the relation of consequence, is a and declaring predicate, look into the will perhapsbe a littleclearer if we simplecategorical lies its material truth of the proposition, and consider wherein matter When or we falsity. say
If
man

is

he responsible,

must

be

free agent,

we

do not affirm the


are

of reality
as

his

of nor responsibility
we

his free agency.

These

treated here

ideal.

But

do

affirm the real

connection,

the necessary coexistence of the two. Indeed, the force of the word " " of this consequence. in the example is to declare the necessity must the dependence of one of the two That the conjunction on clauses, the

other,is all that is affirmed is still more


that the truth of this affirmation is
E. g.
from

manifest

when

we

sider con-

entirely independentof

the

truth of the clauses.


If the Koran
came

God, Mohammed
is

was

the

prophet of
we

God.

The

truth of this statement

indisputable, yet
be
one

hold

both

of the

considered apart,to be false. clauses, A false


is hypothetical

having a false condition. is false, that the protasis but that the This,however, does not mean is false, that the givencondition is not the affirmation of consequence Hence it would perhapsbe better to say that a false hypocondition. thetical is is one as affirming inconsequent.E. g. : consequent what
If Moses
was
a

said to

he lawgiver,

was

very

meek.

Here

we as

may
a

to separately judgments is judgment respecting

admit

each clause

be true, but the sition propofalse ; the one does not

follow

from

the other.

The

concessive

nies dea protasis, clause, granting

false ; but it the hypothetical the consequence, thus pronouncing also. E. g. : does more, it denies the apodosis

(Ifour
"

outward
our

man

the perish,
man

inward

man

must
man

fail.)
is renewed."

Though

outward

the inward perish,

Since,then,the
lie

truth

and

acceptanceof
of this

conjunctive proposition
declaration
as a

wholly in the

correctness

unconditional single

of sequence, it is manifest

that the statement,

whole, is a simple

affirmation of this relation. categorical In the previous section it appearedthat the conjunctive hypothetical its firstprepositional in makes simplyan ideal statement,and use it and the correspondthat the sole difference in thought between

234

OF

REASONINGS.

the latter real; judgment is that the former is ideal, ing categorical It now a difference that is non-logical. thetical hypoappears that the same in its second prepositional the relation of as declaring use, in that regard, and real. In a former chapter inference, is, categorical it was or judgment in the categorical pointedout that the syllogistic Aristotelic syllogism declares consequence. in Wheresimply and solely then is the distinction between this and the hypothetical expression None of inference ? in that the this, beyond syllogistic appears inference is from is that the matter whereas judgment pronouncedreal, in the hypothetical judgment the inference is from matter that is is psychological, ideal. This difference, not So we logical. repeat, find no ground to justify discrimination between we a logical far, then, and hypothetical categorical thought.6 in our Before advancing remarks arc worthy of place. two analysis, of the conjunctive First: We have pointed and predicate out the subject ; where is the copula? Many logicians call the conjoining particle, united with the verb to be" the copula. Thus, say they,in conjunctives
"

it takes, among
"

these others,

forms
-

"

If

then

is

;"

When

then is
"

;"
:

"

Where
is

there is

."
"

In disjunctive

these forms
or

Either
a

or

is in

;"

is either
of

."
"

This is

and confusing,
us

Logic an improper, use


of

the word

copula." Let
the conditional the

rather say that the appearance


is

the

copula in
that it is

forms

impliedby

but inadmissible, grammatically and and disjunctive illative conjunctive

particles.7
We the that while remark, secondly, the
common

characteristic of above
not

and syllogistic as judgments is, conjunctive it is affirmation of the sequence of dependence,

the indicated,
to peculiar

at all

It may
"

be well to note

that immediate

e.

g., is merely conversion


7

If

then ignoranceis degrading, per accidens. is the most

are expressed ideally; easily somethingthat degradesis ignorance."This

inferences

junctive, grammatical characteristic of the conis classed as a conjunction. But it was ing a transitive verb,havoriginally for its object the clause following it. As explained sions Tooke, in Diverby Home it is the the second of of Parley, imperative, Anglo-Saxongif, singular, person is the verb gif to Its then, grant," m eaning, give. original allow," admit," an, "in case to "provided that," "suppose,"but is now equivalent that,""should it that." be proved that," it follows from Thus : or
" " " "

The

word

which ""/","

usual

If That

man

love me, he will the conclusion

keep
must

my

words.

and is, grant this premise,

follow.

ANALYSIS

OF

CONDITIONALS.

235

Like other things, propositions. respecting propositions the attribute of attributes. In a conjunctive have a variety propositions is that of being an inference from another proposition. predicated them
to

be

But

this is
That

onlyone
the whole

of many

attributes that may


a

be

E. predicated.
axiom.

g.

is greater than God

part is
an

mathematical
now.

My

belief is that with


that

time is

eternal

It is obvious

propositions may

be either term

of

predication.

of a merelyas a qualifier thoughtwe use the protasis such from evident that we may reason it is quite of the apodosis, term form not. or judgments as premiseswhether reduced to categorical The only difference is,that when the judgments are in hypothetical reduced to catein the ideal mood; when form, we are then reasoning real. "When, on the other hand, we think the matter as we goricals, son we view the propositions an inference, as declaring may likewise reaits and this judgment being categorical, from them as premises, the copula understand We is real. matter ?/" as representing may form, thus: from" and presenta typical "follows

"

5. "When

in

"

C is D
E
.'. E

if A

is B ;

is F if C is D ;
is F if A is B.

The

order of nature

if it tends to promote

It must
if
.*. It must

product of benevolent design, good ; and beneficent author, have had an intelligent it is the product of benevolent design; and beneficent author, have had an intelligent
is the moral

if it tends to promote

moral

good.
the usual ing form, the follow-

is Barbara. This,evidently,

Returningto

: example is only in part hypothetical

If the

using of

credit is

demand

for

goods,

all forms
But
.'. If

of credit affect prices;

exchange are a form of credit ; for goods, the using of credit is a demand bills of exchange affect prices.
bills of

is Barbara. This,also,

We

call attention to its easy solution

by

the

Canon It is

of

Replacement.
discover
no new

and principles, hence need no new These examplesmay properly be rules or forms. called Conjunctive and so distinguished from Hypothetical Syllogisms,
we so manifest, then,that, far,

236

OF

REASONINGS.

forms purelycategorical difference. logical

the

; but

the

difference is

not evidently

Let The

us

at

once

extend

the

view

to

other

forms

of

hypothetical.

pounded which,as we shall hereafter show, is comdisjunctive proposition, and of conjunctives, therefore subject to the same ment, treataffirmation, may, however,be considered as a simplecategorical of alternatives. either predicating or a mark alternatives, predicating of disjunctives, formed So, then, we may have Aristotelic syllogisms and such are true Disjunctive Hypothetical Syllogisms.E. g. :

Memory
.*. In

Also it is either

philosophic ; or spontaneous ; voluntary this case, what is either voluntary or spontaneous is also either circumstantial or philosophic.
or

is either circumstantial

of disjunctives. It is consists partly following Aristotelic ; but its reduction to strict logical terminin form, thus deevidently its mood, is quitea complex process. Its solution by replacement

This

is Darapti. The

however, obvious is,


Desires But
are

and
or

easy

either spontaneous
is

whatever
are

voluntary has

moral
or

.*. Desires

either spontaneous,

voluntary ; quality ; they have moral quality.

Since the Dilemmatic


it is subject to junctive, it. involving syllogisms
If
a

is a compound, a conjunctivo-disproposition
the
same

view, and

we

may

have

Aristotelic

E. g.

But

unselfish use of despotic entirely power, be cither a saint or a philosopher; he must saints and philosophers are rare ; ruler makes
an

.'. Those

rulers who

so

conduct

themselves

are

rare.

There

are, of course, If matter is

E. Enthymemes, comprising hypotheticals.

g.

there must be a higher essentially inert, moving power, and this impliesa governingwill.

we So,also,

may

have

E. Epichiremas, comprising hypotheticals.


a

g. :

If government has
and

rightto
use

enforce the

without
a

this it could not

then it has

rightto
cases

laws, subsist, its own force against military


be

citizens,

for in extreme

this may
a

requisite ;
civil war, inaugurate of such
use

If so, then government has since civil war of


is the

rightto
to the

result likely

/.

If

counter military power, has a right to enforce its laws, civil war it has the right to inaugurate of revolution. for the suppression state

rightof revolution ;

ANALYSIS

OF

CONDITIONALS.

237 of conditional
:

series of may be

formed syllogisms hypothetical into abridged


a

tions proposi-

Sorites.

E. g.

If the If If
.*. If

the word of God, they should be clearly are explained Scriptures ; studied ; they must be diligently they should be clearly explained, order of men be devoted to them ; must an studied, they must be diligently the Scriptures the word of God, an order of men be devoted to them. must are

This is

purelyAristotelic reasoning. Had


But
.'. An

we

affirmed
;
"

"

the

are Scriptures

the word

of God
to

order of

men

must

be devoted

them

the forms

would

be

mixed, the last step being the so-called hypothetical

construct in the ponent mood. we a Finally, syllogism, may of disjunctives, Sorites consisting wherein the reasoningis strictly of this character, is partly and involves a Aristotelic. The following : prosyllogism
"

Every
A

science is either pure

or

inductive ; of the necessary forms

since it treats pure science, either of is either

thought or of imagination, mathematical or logical ;


science is either exact
or

mathematical science of

worthless ;
exact

The
.*. It is

is neither logical nor probabilities


or

either inductive

worthless.

The

in reasoning

all these

cases

Hence of sequence alone. under its moods, and is subject to its Canon
cannot

tion affirmaupon the categorical it is strictly of Aristotelic form, comes


turns

and

rules.

then, Logic,
of

these distinguish

as

kinds

of

as reasoning,

different forms

thought.
viewed as declaring ence, inferan conjunctive proposition, within itself a reasoning. The affirmation of sequence implies is a characteristic common to it and the syllogistic judgment. The is a condition or logical antecedent of the apodosis protasis ; in other is a consequent, conclusion. and the apodosis or words,it is a premise, is thoughtthus,or merely as a qualiNow, whether a conjunctive fied be ascertained onlyby considering the proposition, can, in general, of course, undetermined. and the context. In pure Logic it is, matter
6. But

"

the

Let

us

illustrate:
If air is pure, it is wholesome.

in the minds This,probably,

upon

mere

is testimony,

persons who do not receive it direct induction from observation or exof most

238

OF

REASONINGS.

is and, though capable of being construed syllogistically, perience, with them a simplejudgment,not expressive of any reasoning ever, whatwith but equipollent
"

Pure

air is wholesome.

But

in this

example,
If the
moon

has

no

it has atmosphere,

no

twilight,

there would

seem

by

the

cessitated reasoning implied being ne; the apodosis under some such as : rule, protasis standing general

to

be

Atmosphere
The

is essential to the

phenomenon

of

twilight.
as

thus impliedmay reasoning

be

in expressed
no

full

follows

(An
now, it follows that The

orb

that has
has has
no

If the
moon

moon

atmosphere has no atmosphere, twilight.


no

twilight ;)

We

ideal minor an have, then, in this given condition or protasis the apodosis. It is manifest, ideal conclusion, an yielding premise, that the contingentconjunctive therefore, hypothetical proposition inference is a simpleIdeal Enthymeme.8 an declaring in the ideal sphere been indicated that we may reason It has already and that the principles of thought as well as in the real, are precisely the
to
same.

We

may

pass from
case;

the

one

to

the

other; from
the
if real,

the real
we

the ideal in every


In

from

the ideal to
have
a

have

ground.
from

the last
and

example we
We

ideal minor

conclusion.

may

and major real, pass to an transfer a reasoning tally toreadily to say

the real to the ideal.


If all and then
men

Thus, it is easy and proper


are a

mortal,
man,

If Plato is Plato

is mortal.

This

throughoutall of

its

is purelyideal. propositions

A varied but

quitecorrect

view of the

is conjunctive hypothetical sequence.

as

follows

: quence se-

It is

under
affirm
an

Upon the existence (inthe Form 1 a) of would come the ideal minor which, as a generalrule, this major ; is only to affirm indirectly the sequence unexpressedpremise. For example :
If virtue is

merely an depend?

affirmation of necessary

But upon
an as

what

does this

unexpressed major, a To specialcase.


prove
it is to establish

to

knowledge,it is teachable.
that is merely to say that you admit this
"

Do
of

you

admit

this ?

Yes.

Then

All forms
a tal men-

knowledge are teachable." judgment,which,taken as

Hence
a

affirms hypothetical conjunctive necessitate the consequent.

major,would

240

OF

REASONIXGS.

In

this last may,

example

all the

Clauses

however, be

middle requisite from remote so logically links must be

terms

are

given.
plete com-

each

other that

several, perhaps many,


the chain.
media
are

intermediate
not

to supplied

This it may

obvious.
us.

be easy to It is the part of the


at the

do, unless

the unexpressed writer nish to fur-

speakeror

these to chief

He

may,

as laydown outset, preparatory,

his

with his ideally connectingit at once premise hypothetically, and then proceedto supplythe media, E. g. : ultimate conclusion,
If the desire for distinction is
an

essential stimulus

to

industry,

then communism

is

to antagonistic

the progress of civilization.

Here

arguments might be needed


a

to establish the it

long series to we also, might say,


"

show

that

and haps antecedent, pernecessitates the consequent. So, Euclid


is

If the tenth

of proposition

true,then

the

one an

hundredth actual

is true

also." of the is
matter

As

example
He

before

us,

we

will quote

speaking contemptuouslyof the Art of the syllogism, saying, God has not been so sparing make to them barelytwo-legged to men creatures,and left it to Aristotle to make them rational." He then tries to show that logical than useless, forms are worse beingconfusing.The passage is curious effort to overthrow that which it uses, and therefore unwittingly an as acknowledges. He says, To infer is nothingbut by virtue of one laid down true." true to "draw in another This he as as proposition shall be punished illustratesby the following in another example: If men
" " "

passage from of Logic and

Locke.9

world,then
"

men

can

determine

themselves."

lie then

remarks,

What

is it that shows

the force of this


a

and consequently inference,


of all the intermediate

the reasonableness

of

but it,

view

of the connection
. .

ideas that draw

in the conclusion ?

The
.

the mind, seeing

connection

punishment in another world and the idea of God's punishing;between God's punishing of the punishment and the justice of the punishment; between justice and guilt; between guiltand a power to do otherwise;between a
power
to

there is between

the idea of men's

do otherwise

and

freedom

; and

between
men

freedom and

and

self-

the connection between sees determination, Now, I ask whether the connection
more seen clearly

tion. self-determinabe
not

of

the

extremes

in this

simple and

natural five
or

and jumble of perplexedrepetitions

than in the disposition six syllogisms?" It is

Essay on

tJw Human

" bk. iv, ch, xvii, Of Understanding,

Keason,"

ANALYSIS

OF

CONDITIONALS.

241

in decrying form logical very clear that, and natural" way, he has developed the
a

stated sorites, progressive


is needless.

so

showing us the simple enthyrnemeinto hypothetical form that redressing nearlyin strict logical
and
all the

"

plied imfounded on or reasoning in the form of is thoughtstrictly in the contingent hypothetical is ideal, the Aristotelic syllogism.The only distinction is that one add that viewed as a conditional proposition, the other real. We now it differs from the categorical only in that the apartfrom its ideality, latter does not express a condition. But, in fact, tion proposievery logical is its premises is a conclusion conditioned on ; so all reasoning conditional reasoning.The conditional character may not appear in but it belongsto all thought. It adheres to every possible expression, is b ut then this not or intuitive; judgment except the primitive is susunconditional neither requires nor ceptible thought. A judgment truly of proof; it cannot appear as the conclusion of a syllogism. is a conditional judgment in Or, in other words, every syllogism the antecedents and the conclusion the consewhich the premises quent. are and conditional, So, then,the distinction between categorical accident of expresis a mere with Aristotle,10 which did not originate sion, and ought to be dismissed to have been introduced, ought never and hypothetical from Logic. The distinction between categorical is of no moment, and belongsto psychology, logical propositions ought also to be discarded.11

It has

now

been

shown

that

10

See Part
"

" 7,note. 3d,i,


or

ing, in themselves, Logic knows nothpropositions, is not that conceived as and takes no account Logic may be held true the the clusion, conneither nor In premises Logic guarantees contradictor}'. reasoning, the former ; for a syllogism but merely the consequence of the latter from the on assertion of the truth of one is nothingmore than the explicit proposition, contained. of other propositions hypotJicsis beingtrue in which that one is implicitly and false A conclusion may thus be true in reality (as an assertion), yet logically
11

Of the truth

falsehood

of

; all in

(asan inference).
In
a

certain ; and

sense,

inference is hypothetical, all logical hypothetically therefore,

of Logic stands opposed to absolute or necessity hypothetical have well denominated scholastic philosophers recent The more simple necessity. the necessitas The and these two species the necessitas consequentice consequentis. of one former is an ideal or formal necessity thought ; the inevitable dependence necessary

the

upon

The latter is a real or material nature. of our another, by reason intelligent the inevitable dependence of one thingupon another because of its necessity; to all legitimate come* The former is a logical common nature. own necessity, 16

242

OF

REASONINGS.

the ideal unreal are now expressing junctive These are to be considered. always in the past tenses of the subIn usus the meaning which they convey is mood. loguendi, (hence unreal ") of the thought,and, always to deny the reality to be of these also two thus,is alwaysindirect. There seem uses, either indirectly to declare a fact, to declare an inference. or indirectly the firstuse : We at once exemplify

"

7. The

forms hypothetical

"

"

Were

he

king,he

would

tyrannize.

That

is to say, he is not

king,and
not

he does not
not

tyrannize.

If it were

so, I would

say it.

That

is to say, It is so, and


as a

I do say it. Thus

the ideal

case

supposed

is denied

its statement

which is to posit its opposite. The apodosis makes fact, which is thus indirectl of,the real fact, contraryto, or in spite

declared.13
is not the ultimate purport. Yet supposition these propositions more indirectly, quite another meaning. convey to be a rhetorical or grammaticalor linguistic device for They seem saying something emphatically, quite aside from and beyond what

But

this denial of the

the words
to state

directly express,
Were he

and

which

it would

perhapsbe

difficult

For example: directly.


I here, would tell him.

It is

clearly impliedthat
seems

he is not

here,and I do
the

not

tell him.

But

to state

these patent facts is not


to

purport

be to declare my
to
some

myself in
but and my of is
now

reference

matter

I mean quiteemphatically,
in this
case

sayingthis. The state of mind, perhapsto justify in question.So, briefly, and indirectly, to affirm that I am so disposed
my circumstance
one

objectof

determined action now,

that

no

whatever

prevents

except the obvious


mind
is

of the absence

of the

object
there
so

action'; my
no

made fully

up, all of to

and questions settled,


me

other external fact I know


you

hinder

from

thus and it.

know

what

I think

and

feel and

will about

Observe

quence, whatever

modalityof its objects. The latter is an extrathe syllogistic and wholly dependent over logical necessity, inference, the modalityof the matter This ancient on modern distinction, consequent. ophers philoshave not only overlooked but confounded." Hamilton,Discussions, p. 146; 12 The past tense of the subjunctive in the subordinate clause of a categorical force of denial. E. g., "I would I were propositionhas the same a boy" implies that I am not a boy.
be and above
"

the material

ANALYSIS

OF

CONDITIONALS.

240

that the denial

more implies

than

we

first stated.
not

In full it should This


is

be,

here,and therefore only I do the sole condition, the only reason why I do then,there is none. present disposition,
Just
so

"

He

is not

tell him." tell him.

not

In my

in the former

example,
He
is not

"

If he

were

he king,

would

nize," tyran-

the does
not

meaning
"

is

"

thus tyrannize,"
If it
were

king,and for this reason only he his tyrannical indirectly tion. disposideclaring
so, I would not

Again,

not

say

affirms my it,"

fulness. truth-

In the

following example,
"Were this beam rotten
not

rotten,it would

serve,

we

think of the beam


to chiefly

as

and

but unserviceable,

and

affirm the suitableness of all its other

primarily unnamed ties. qualimean

In the trite

proverb
were

If wishes
we

horses, beggars would ride,

say

but

about beggars' vain longings, somethingindirectly, rhetorically, still more in the application of the saw, we mean to rebuke indirectly, In Macbeth's speech, aspirations. extravagant
If it were It
were

done done

when

'tis done,then 'twere well

quickly,
outlives
must itself,

he

means

not

but rather he

That a deed merely, his to justify means


How Were

giveus

pause ;

hesitation.

Gay'scouplet,
!

happy

could I be with

either,
away

t'other dear charmer

is

of palimpsest

enamoured

distraction.

So far the unreal

present.

In the unreal

we past,

have,
here, my
brother had not

If thou

hadst been

died,

which

of confidence in superhuman a indirectly strong expression and love. Might, could, or should,in the apodosis, modifies power the meaning,referring the matter to possibility, or ability, duty. of this sort must be treated logically with reference Propositions to the primary, fundamental,unexpressed meaning,and not to the ostensible ideal statement,nor of fact, which are secto the negation ondary, tent, indirect inand, taken apart from the primary though more are to be viewed senseless. They are, then, and interpreted generally as simplecategorical judgments. to the unreal proposition Turning now an inference, we declaring find it presents a further peculiarity. Let us recall that the denial of

is

244

OF

REASONINGS.

consequent or conclusion denies


necessitate it.
Whoever

an

one antecedent,

or

more

of the

that premises

Then
so so

let us
must

consider the

: following

talks

be crazy ;

Diogenes talks
.*. Diogenes must

be crazy.

in mind may for emphasisperhavingthis reasoning prefer, haps, to state it indirectly. denial of this a By expressing ideally mental conclusion, he denies ideally the fact of his minor, a denial of a grantedfact, and hence professedly he and thus indirectly false,

Any

one

affirms his conclusion.


Were

Thus

Diogenes not

crazy, he would

not

talk

so

meaning,But since this by this device,

he does talk so, therefore he must


custom

mean we language, of our words. Our reasoning is consciously and intentionally and goes to establish its opposite. unreal,

of

be crazy. So to declare the posite op-

For further illustration we

renew

our

old familiar
is not
a man.

example,

It Plato be not

he mortal,
a

Here

the matter

is stated in
were

as ideally

mere

as contingency, formally

But questionable.

If Plato

not

mortal,he would

not be

man,

the matter

is stated

as

unreal, absolutely therebydeclaring emphatically,


that
a

without

sayingso,

Since Plato is

man,

he must

be mortal.

But

in the

the fully,
No

affirmative example we following express stated : purposedconclusion being distinctly


rain has fallen ; for if there

ourselves

more

had,the ground would

be wet.

It will be observed
as

that this is in the

is

manifest sufficiently
.

the reductio essentially following examples:

ad

absurdum,

If

were ignorance

Were

all the prosperous

would

be

'twere folly to be wise. bliss, then discontented some happy, happy. (Seeexample in iv, " 3.)

These the

and absurd. evidently self-contradictory is true. of their antecedents contradictory Again :


are

conclusions

Hence

Were

not Christianity

from been

it would Were

not

have

God, accompanied by
of

credible

miracles;

its miracles

unworthy
have

credit,
attested in the
manner

they would
in which

not

been

it has been

proved they were.

(See the argument

in

iv, " 4.)

ANALYSIS

OF

CONDITIONALS.

245

The

formal

indirect

called the method of absurdum, appropriately is conveniently, and usually stated demonstration, elegantly, For
an

reductio ad

in these ideal unreal forms. We

instance refer to
of the
as

iv, " 8, ex.

51.

conclude when

that the ideal unreal form inference


or

an declaring

offered

hypothetical sition, proporectly. indireasons proof,

it denies an conclusion, By an ideal denial of an unexpressed but unquestionable which denial, unexpressed premise, being absurd, of that conclusion.13 affirms the truth and reality impliedly

commonly known in (v, conjunctive Logic as the hypothetical syllogism " 5). Aristotle ignores In one all forms of the so-called conditional syllogism. placein
8.
now

"

It is needful

to

revert

to

the form

his

however, he describes the process Analytics,


denies that it is a

now

known
was

as

the

right. nevertheless introduced into Logic by his Conditional syllogisms were in the Lyceum, Theophrastus, immediate successor were accepted by his rival Eudemus, and were adoptedby the Stoics. They have received the of nearly in one all logicians down to the pressanction, way or another, ent endorsed and time. were Especially they by Boethius, developed and his great authority has given them a permanent placein Logic. Stillthere has been a continual wrangle about the details of the system, their be to a dissatisfaction, sidered conbetraying deep although right of m odes has been special reasoning hardly questioned.The of Aristotle have generally felt it needful to admiring commentators for the hiatus which his disregard of them makes in his Analytics apologize however,Saint-Hilaire, who, in his translation of the ; excepting,
contained in this and the development of the matter of the views presented throughout this general sion. discussections, previous nor, indeed, is found in in hint to be Arnauld a our sentence a Hardly Logics. single speaksof the enthymemic character of conditionals. Mansel (App. to Aldrich, p. 240) writes two in sentences which the doctrine glimmers. The most statement explicit I have encountered is from Titius (Ars Cogitandi, ch. xii), follows : Conas ditionalis seu nihil aliud est quam hypotheticus enthymema vel sine majore vel minore." e st "Syllogismusdisjunctivus enthymema sine majore." "Sequitur nullum circa syllogismos vel formam condipeculiare concludendi fundamentum tionales occurrere, nam materiam adeoqqe argumentationesirnperfectas, sylloviews were worked out before this gismorum regulariumillicontinent." My own new. caught my ey-e, but it seems they are not altogether
seen a
"

but syllogism, hypothetical

lie syllogism.14

13

1 have

nowhere

14

Anal. Prior, i,32, 7.


be ; and
essence

"

If

because

man

it exists, should be

is necessary
;

that

animal

should

animal
must

exists,
the

that existing, be. necessarily subsist


as we

there

But have

this is not said

do propositions

not

then,because man for yet syllogistically inferred, they should,"


essence

240

OF

REASONINGS.

Emboldened Organon,insists that they are therein recognized.15 by let us question admitted silence of Aristotle, this generally their title, did his work onlyby half. and judge whether the Stagyrite of modern A number writers on Logic,recognizing hypothetical in modes various distinct of endeavor as syllogisms reasoning, ways to show that they may be reduced to Aristotelic forms. But are they recall that deductive inference is of two kinds, at all? We reasonings
mediate
and

immediate. notions formal

In mediate

inference middle

we or

determine medium.

tion the rela-

of two
is the

the througha third,


of expression

gism sylloa

this mediate Now

process, and

hence

middle

so hypothetical syllogisms, contain Therefore they are not syllogisms, middle term. no called, of reasoning the following not expressive at all. Inspect :

term

is its essential feature.

If law

MODUS

PONENS.

Cut
.*. Our

our prevails, rightsare law does prevail ; are secure rights

secure

;"
=

Major
Minor

Premise. Premise.

Conclusion.

There

is
are

no

term

here with which the

the two

terms

found

in the conclusion

premises. There are in all four terms, and all found in the so-called major premise. The so-called minor duces introwith the conclusion, no new matter, and has nothing in common in the true syllogism. as occurs necessarily
dont on a davantage les syllogismes hypothetiques, voulu faire honneur eleves Theophrasteet Euderne. encore Les syllogismes a ses sont Aristote les de conce qu' vention. hypothetiques appelle syllogismes d'hypothese, II en avait traite tout au long dans un ouvrage que le temps nous a ravi, mentionne dans le Premiers Analytiques, mais que lui-meme i, 44, 4." Logiquc then d' Aristote, Ix. St.-Hilaire discussion. to a proceeds Preface, See,also, p. tome top. i,8, 9. He has against him, however, Waltz (see Comment, on Anal. iv, Prior, i, thorities, (seeDiscussiojis, 44) and Hamilton p. 151). For references to other auand Hamilton's Grote's note. see Logic, Aristotle, p. 613, note; p. 243,
"

compared in

16

"

Aristote

n'a pas

omis

In the passage
some

above of
as

referred

to

Aristotle by St.-Hilaire, but Hypothesis,


more

promises

to

treat

at

future time
never

Syllogismsfrom
there
are no

was can

realized,

extant

references to it.

probably the treatise Against St.-Hilaire it

the various by Syllogismsfrom Hypothesis Aristotle meant all and forms what are now known of the Rcductio ad impossibile, not at as thetical Hypohistorical fact the that stated, Syllogisms.Moreover, already Theophrastus of the term which was firmative, categorical," changed the Aristotelic sense simply "afis evidence that he, and not his to the sense opposed to "hypothetical," of the inventor the the point was system. I have not seen master, hypothetical but the change seems to indicate that Aristotle had no such opmentioned, clearly posed need for one to mark a new term, and that Theophrastus found a special be

proved

that

"

distinction.

248

OF

REASONINGS.

of
"

more

than
of

part; if so,
debtors

we

can

conclude
me.

the Here

of reality
it appears

part only
that the

Some

my

will repay

is conjunctiveproposition hypothetical

the

ideal

entbymeme; that

is not a syllogism at all, nor pressive exconjunctive syllogism of reasoning inference of any kind ; that it merely reiterates or the enthymeme as real ; that it indicates a transfer from the ideal to the real on unexpressed grounds; that it is simply a formal mode in whole of announcing the ideal premise established as real, in or of the conclusion. The reasoning part, and the consequent reality in the two is purely and is duplicated Aristotelic, implied enthymemes. A second view considers the conjunctive as proposition merely an In this view affirmation of sequence, its second use. prepositional consists of three propositions. The conjuncthe so-called syllogism tive affirms the necessary coexistence of the other two judgments, or, it affirms only a consequence from One of to the other. one better, these affirms categorically in whole in part, of one the existence, or

the so-called

fact.

The

other infers the existence of another


x

fact.

is ; but

if

is, y

is ; then y is.

againis an enthymeme. In this view,however,the enthymeme in the two lies solely but is strengthened categorical judgments, by a distinct affirmation of their necessary sequence. The reasoning, then, lies not at all in any inference from the hypothesis to the assertion, but wholly in the relation of the two categorical judgments as premise and conclusion. This reasoning is purelyAristotelic. A third view is that the conjunctive affirms indirectly proposition In this view the so-called hypoan unexpressedmajor premise.18 thetical of affirms the three real propositions a categorical syllogism Aristotelic syllogism. It is not now directne or an enthymeme, unless the inof the major be held to bestow this character, and not the it as a special form slightest ground appears on which to distinguish of reasoning. mode or
It follows
in

Here

that the axiom The three

of Sufficient Reason10

is

an

entire superfluity

Logic.
and

Primary Laws, and


every be
case

the rules evolved from


of the axioms of

for them, -are all-sufficient;

of
on

violation

Reason

the enthymeme, developing rules of the Aristotelic to be a violation of one another of the general or that the Platonicosyllogism. Hamilton latterly suspected Leibnitzian Law Mansel of place in Logic,and out was definitely

Consequentwill

found,

18

See supra foot-note 8.

"

Sec Part

1st, ii, " 7.

ANALYSIS

OF

CONDITIONALS.

249

reached

this conclusion. the realm of

There

can

be

no

doubt
it
was

that it should

be

to relegated

whence Metaphysics,

drawn.20

that disjunctive and explicitly other are compound conditional propositions merely enthymemes. We here speak of disjunctives as compounds, and it is easy to show that they are so. two members Every disjunctive havingsubcontrary consists of two hypotheticals, which may be explicated thus :
9. to
more

"

It remains

indicate

C is either D

or

E ;

c.
e.

g., God

is either loved
is not is not

or

feared ;

yieldsIf
and

C is not

D, C

is E ;

g., If God

If C is not

E, C

is D ; e. g., If God

he is feared ; loved, he is loved. feared,

When

others

is contradictory, in "God is cither trustworthy as opposition the two or untrue,"the analysis yieldsfour hypotheticals, being:
If C is D, C is not
If C is E ;
e. e.

the

g., If God g., If God

is

he trustworthy,

is not

untrue

E, C

is not

D ;

is untrue, he is not

trustvvorth}'.
or

quadruple proved hypotheticals of it is is true it. Moreover, tive easy to show that the so-called disjuncis merely a reiteration of the enthyrneme expressed syllogism by another Thus of these constituent or one : hypotheticals.
it follows hypothetical,

Now, the disjunctive proposition beingmerelya double


that what has been
of

C is either D

or

into explicates

(What

is not
D

D
;

is E

;)
"and"

(What

is not

is D

;)

If C is not

If C is not

E ;

then C is E.

then C is D.

These
Modus

two

in simple syllogisms

Barbara of

or

Darii

correspondto
is not D

the

T.ollendo

Ponens.
is D is not E

In

case

have also : we contradictories,

(What

;)
"and"

(What
then

is E

;)

If C is D ;

If C is E ;

then C is not

E.

C is not

D.

These Modus

two

latter

in syllogisms

Celarent

or

Ferio

Ponendo

Tollens.
or

condenses

It appears, then,that involves in one compound statement and


or as

correspondto the the disjunctive osition proptwo


or

four

hypothetical enthymemes ; is merely a restatement


either
as

that the

pretendeddisjunctive gism sylloof of these enone thymemes explication some

ideal

or

real.
; also p. 251.

20

See Hamilton's
p. 235
; and

note

Logic, p. 62, and note ProlegomenaLogica, p.

See also Hansel's

Aldrich,

193.

250

OF

REASONINGS.

pound, acknowledged comand is obviously made and the dilemma up of conjunctives cate It is needless to trace the principle through these intridisjunctives.

The

is proposition conjunctive-disjunctive

an

forms.

It may

be

well,however,

to

observe

that the former


of which

is has

one proposition, disjunctive merely a disparate E. g. : form. been reduced to the conjunctive

member

Man If
man

must

bo either

capableof

progress,

or

brute,or
a

is incapable of progress, he must

be either

brute

divinity. or a divinity.

gisms, Ought not, then,these conditional forms,these pseudo-syllofrom to be banished Logic? By no means ; for they are true, of expressing and hence modes and very common thought, natural, and treatment. call for logical common Nothing is more analysis his premise than for a reasoner to state hypothetically at the outset

"

10.

and

conclusion. he is

This

he does E. g. :

for the sake

of

and clearness,

to

sho\v

whither

tending.

If the

was prisoner

sane, then he is

for responsible

his act.

His

first

argument
As

may

be

to

show

the

of necessity
next

the
to

herein declared. this antecedent


his agent, in medical

he counsel, accusing

endeavors

sequence establish

minor, perhaps by showing the deliberation of the his motives, etc., etc. ; and, it may be,he brings consistency,
evidence. "When the he closes by argument is complete,

: categorically declaring

The

was prisoner

sane, therefore he is

for responsible

his act.

to call the various conditional place, proposes for argumentation." forms preparations Again,many of these conditional forms present exceedingly densed conof which the mind darts with expressions reasonings through and unless the thinker is familiar with their analysis, he is rapidity, in the more in danger, intricate dilemmatic especially forms,of paralogism, of Hence these were or being imposed upon by sophism. with the Greek Sophists, favorite forms and indeed are still preferred On the by all who wish to make the worse appear the better reason. other hand, their condensation and givesto a justargument weight, and rhetorical force. They should, then,be discussed, not only logical of analysis, but also because of the practical as advantage subjects sulting re-

Hence

Hamilton,

in

one

"

from It is The

their close examination.

however, that their nomenclature clear,


of misapplication the terms
"

unfortunate

changed. syllogism," major and


"

ought to

be

ANALYSIS

OF

CONDITIONALS.

251

minor and
true

premise,"
methods of

"

mood,"
reduction

etc.,

etc.,

and

the but

attempt
distinct with
and

to

enunciate those and


of

rules the

parallel Logic logical


for

to,

from,
confusion

syllogism,
so

has

filled
in

centuries

error.

But false

deeply
and

rooted

literature,
that the

so

widely

spread
can

is

this

system

terminology, authority.

needed

correction

be

made

only
It

by
is
of
a

the

highest

great
any
adduced

satisfaction,
of

however, conditionals,
of

to

say
so

that
far from

the

omission

by
for

totle Aris-

treatment

calling
and the
it

apology,
ter characof

may of

be

as

an

evidence

the should
is

profound
respect clear,

thorough
silence be

his

Analytics.
when imitate
its

Logicians significance

the
and

master,
modest To immediate these

and

not

would

well

to

it.
There

sum

up and

are

but

two

kinds

of

deductive

inference,
is

the

the The

mediate.
various forms

The
of

analysis
conditional ideal distinct

of

Aristotle

limited

to

kinds.

propositions enthyrnemes.
from

are

tially essen-

hypothetical
such

conjunctives, reasoning
and all

or

There

is

no

thing

as

conditional

categorical
conditional.

but The

all

conditional called of

is

categorical, syllogisms
;

categorical
not

is

so-

conditional kind
do

are

syllogisms
of the

at

all,

nor

inferences real.

any

but

are

mere

reiterations distinct the

enthymeme
of rules

as

They
but

not,

therefore,
take their

require places
of

system
Aristotelic

and

forms,
which is

rightly
exhaustive

under

system,

an

analysis

deductive

thought.

PAET

FIFTH."

OF

FALLACIES.

I. DISTRIBUTION.

whose consequences have been " 1. The Primary Laws of Thought, in expoundedin the foregoing pages, are derived from, or formulated accordance
are

with,the ultimate
their is,

constitution original
contradictories the human
are

necessary ; that be doubted or cannot mental

The}7 inconceivable, they


It follows that

of

mind.

by questioned

mind.

with them are equally processes and results in strict conformity in the same in But these Laws not necessary sense. arc necessary
sense

be obeyed. Mental processes do they must perforce conform to them. not necessarily They declare how we must think, Our thoughts if we think consecutively ; but they are not inviolable. in their course, like the planets, determined not are by inexorable The forces. planethas no choice. Laws of thought arc impressed mental constitution just as laws of health are our impressed upon The latter constitution. we or consciously unconsciously may upon our physical is but the inevitable consequence disease ; the disregard, but only to incur the deadlier conformer sequence we may likewise disregard, of error and folly. its posiA System of Logic, a Theory of Thought, is complete on tive do and must in showing how we ly think,if we think correctside, But this cannot and fruitfully. at the be, without contemplating of incorrect thinking. of error, and modes time the possibility same that every notion has its opposite, of Relativity Law The declare^ the notion of error, that the notion that the notion of truth implies the notion of incorrect, of correct, regulated thought implies lated unreguand of the were same white, shade, thought. If all objects ContraHence the scholastic maxim would be distinguishable. : none the
that riorum law eadem
est

scientist. We

cannot
one

consider

the

observance other.

of

apart from

its violations ; the

the implicates

When

is exhibited, bad reasoning must be conceived as at good reasoning least possible, else the good cannot be conceived as good. According
"

DISTRIBUTION.

250

is a reasoning, reasoning is one and in a certain old kind of syllogism, sophlsticus syllogismus and ignorance, book the fruits of demonstration opinion, are, science,
to

old

definitions," says De

Morgan,

"

bad

the latter derived demonstration." been necessary, in at the incorrect.

from

bad

what demonstration,

we

would

now

call no

it has Hence, all alongthrough the presenttreatise,

to glance showing the methods of correct reasoning, been the rules have frequently Examples violating side fixed on the positive given. But as our view has been steadily of the theory, has been very incorrect thinking, the negative or side, completionof our task imperfectly developed. To the satisfactory and systematic view it is needful now that we take a comprehensive of the violations of the Laws of Thought. If any further justification treatise needed for adding to our were in the valuable practical it might be found discussion of Fallacies, a to a habit the studyof them. It contributes greatly results following of clear and logically that one be familiar with consecutive thought, it is inwith the slips to which clined, the various dangersthat threaten it,

with harmless But and

the

snares

which

environ

it.

Error,seen

to be

error, is

; it is

error,

thus

only when in the guiseof truth that it is dangerous. skill in detecting abounds, and a practical disguised,
is of inestimable value. So
is important

exposingit

this

sidered, con-

confine itself to very simple that,while Logic might justly illustrations of the violations of its rules, it is customary to extend the examination many
to

quiteintricate
be

and

difficultcases, and

to

consider

varieties of

error.

Moreover,if it can

shown, as

thinkingare

at bottom

cious progress, that all kinds of fallait violations of established logical rules,
we

that the Aristotelic will go far to confirm the doctrine of this treatise, is the unit of all mediate thought. syllogism

"
them
name as

2. Bacon

was

the

who first philosopher


sources

attempteda systematic
error.1 He
made of

enumeration
a

of the various

of human four

quaint classification into

of "Idols"

the significant genera, under of illusions, described in the sense 'an image), (a^oc,
"

in

I do find, in a magic mirror. He therefore, presented says : this enchanted glass false appearances, of several distinct four idols, or subdivisions." These he sorts,every sort comprehending many
as

if

enumerates

follows

Novum

Organum,

lib. i ;

Summary

of Part

ii;

Aphorism

38 sq.

254

OP

FALLACIES.

Idola
common

tribus ; Idols

of the nation

or

to which, from tribe,


are

certain

weaknesses

of human of the

nature, we
den of
or

Idola

specus ; Idols

cave,

liable. universally which, from the peculiar


in different

and (I.positions
manners.

circumstances

mislead them individuals,

Idola

fori;
as

Idols
usage

of the

or forum, publicassembly,

bar, arising
wise other-

from

the current than

of words

which

representthingsmuch
false

are. they really

Idola and

theatri ; Idols of the


erroneous

which theatre, of

systems of philosophy

have introduced.2 reasoning The intellect, son therefore, by mixing with pure reamay be perverted individual propensities or our our affections, gregarious ; the false the imposing delusions of reinvolved in language, or ceived suggestions declares that the doctrine concerning theories. Bacon these of nature relation to the interpretation Idols bears the same the as doctrine bears to deductive Logic. sophistical paralogisms concerning Whcwell, however, thinks that his precepts concerningthese Idols methods "have little to do
the fori,

with
snares

Natural
of

And Philosophy."3

moreover

the

class Idola

with language, corresponds prettynearly

Aristotle's Fallacies in dictione.

"

3. The

next

most
uses

notable

attempt

at

classification of include

error

is
of

that of Mill.* lie

the word

"fallacies" to

all kinds

intellectual error, and discovers five genera : 1. Fallacies a priori; Errors in simpleinspection, from arising
"

natural

prejudices.
Observation
;
"

2. Fallacies of

Errors
or

in the

ground

of

induction,

from arising

either mal-observation
;
"

non-observation

of the facts.

3. Fallacies of Generalization

from arising

Errors in the process of induction, of drawing conof the legitimate mode misconception clusions facts.
"

from

observed

4. Fallacies of in against
5.

Ratiocination ;

Errors

in

argumentation, provided

Syllogism. Fallacies of Confusion ; Errors arising from evidence beingconceived in so indistinct a manner not to produce any clear consciousness as of the means by which the conclusion is reached.
"

the rules of the

See Hallam's

Literature

admirable

chapterxx, Part

ch. iii, the of Europe,Part iii, "" 58, 59. Read, also, the of in on Port-Royal 3d, Logic, "Sophisms common

Civil Life."
3

ch, xv, " of Discovery, Philosophy

20.

bk. Logic,

v, ch. ii.

250

OF

FALLACIES.

presenttask,then,is to trace This we may do, for they are

the

fallacies

common

to

every

science.

limited

in number.

the common fallaciesthat belongto no investigate We limit our attention to formal shall accordingly

must logician particular sphere."


8

The

fallacies; material

fallacies are may

excluded.
to

We

shall consider
in order
to

matter
a

only in

so

far

as

it

be needful

it inspect

to discover

fault of form.

But
of

then,indeed,we
fallacies

shall undertake
as

show
are

that
at

nearlyall

the kinds

classed usually and we rules, syllogistic


as

material

bottom

formal, violating
scholastic

shall

adopt the

old Aristotelic and All

classification
are speaking,

sufficient to this end. fallacies.

fallacies, logical properly

formal

is commonly described as "any unsound of mode " 5. A Fallacy which conviction,and to be decisive our arguing, appears to demand of the question in fairness it is not." in hand, when Says Kant : A is false in form rational reasoning which while valid in appearance is a fallacy. Such a reasoning is n. paralogism if we ourselves deceived are Let us by it. It is a sophism if we seek to deceive others." define more and say that any violation of logical lacy. law is a falwidely, This falsuin).We agrees with its etymology (fallere, may
" 9

have
tur.

fallacious definitions and As for Kant's

classifications as

well

as

the

non

sequi-

psychological ; one of not the least moment Although, and is by the influence of Hamilton, it has crept into our language, writers on erence by nearlyall subsequent Logic with humble defrepeated
in and Logic, used little

it is not subdivision,

but logical,

elsewhere.

to these

shall make we great authorities,


a

bold to discard
sense. logical

and it,

and sophismsin distinguish paralogisms

useful

Fallacies, then,are
1st. of ^uponinspection

of two

kinds

or Paralogisms;

those whoso alone.

violation of
This

law logical

is manifest

the form

accords

with pretty nearly

meaning of the word as used by Aristotle. It is so used by De is best fitted to Morgan, who says: "Paralogism, by its etymology, offence against What the formal rules of inference."1 an signify here we call paralogisms formal are by Whately as distinguished and by Mill as fallacies of ratiocination." fallacies,"
" "
B

the

De

ch. Sophistici Elenchi,

ix.

The

full titleof this

which treatise,

is the last of

the series

the ORGANON, but printedby Waitz as the final section of constituting the Topica, is as follows : ITept$e T"V "ro$"i"mfcwv iXsy^wv Kai ~Cjv tyaii'OiJ.evdii' St d\\' OVK i\tyxwv Hiv ovruv iXtvj^uv. TrapaXoyivn"v 9 lo Formal Logik," 90. Logic, p. 239.

DISTRIBUTION.

257 law logical is not fest mani-

2d.

Sophisms ;

or

those whose

violation of

of the form but requires consideration a alone, upon inspection of the language, of the matter to discover it. These correspond in or
and to Whately's"material fallacies," general
to Mill's "fallacies

of"

confusion." "rather
and
not to
seem

"It
to

answers seem

the purpose of some and to be philosophers

persons," says
not to

totle, Aristo

be, than

be

; for

is seeming but sophistry

unreal of

the the

a person sophist That reality."

who
is to

uses

the semblance
is
a

and philosophy, without philosophy wise


man

say, he

counterfeit

(So"/"oc,

clever, cunning).11 above,subdivided by Aristotle into two of the Scholastics, follows are as classes,12 which, in the terminology in voce or (a)Those in dictione, (01 rt]v \i"tv) ; the formal napa
as

Sophisms are,

indicated

fault

therebeing concealed by ambiguity of language. Generally, other. fore, they disappear by being translated from one languageinto anThey correspondto Bacon's Idola fori,and to Whately's " fallacies." Of them Aristotle makes a selection rather semi-logical than
a

'

for division,
we

it is far from in detail.


or dictionem,

of exhaustive,

six

sequently, classes, which, sub-

treat extra

Those (b) fault

in

re

t"w (oi

rijQ Ae"wc) ; the formal

as therefore, Generally, lying concealed in the subject-matter. in whatever language to the thought, adhering theypersist, expressed. fallacies" of Whately. Of them to the non-logical They correspond considered. Aristotle selects and treats seven kinds; subsequently
"

It is needful to forewarn
a

the reader that fallaciessometimes

view one aspect, It becomes, in such another under another.

double

or

manifold

present them under one class, bringing


case,
a

matter

of doubt

or

of choice to which often

Very

the

same

shall be referred. a given species genus even be individual fallacy may, with equalpropriety,
can some

referred to different

regardit as

and sometimes we species, if For instance, not. or fallacy


a

choose whether
one

to

on expatiates

the distress of
we tyrannical,

and country, suppose


him

hence
to

argues
assume

that the either that

government is
"

must

Every coun-

11

Sir Thomas
a

with
cause

fonde

(Works,p. 475)thus caricatures him : argumente, prove unto a symple soule that two
More

"

woulde, Sophyster
were

egges

three ; bemake three.

that ther is one, and

that ther be twayne,

and

one

and

twayne

to soyle hys fonde argumente, symple unlearned man, though he lacke learnying the two and byd / and to eat hath yet wit enough to laugh thereat, egges himselfe, tak and eat the thyrde." the Sophyster

Yt

17

258

OF

FALLACIES.

try under

which constitutes the fallacy of undistributed tyranny is distressed," that middle; or "Every distressed country is under a tyranny," false premise, which, though materially a nevertheless, yields,
a

good argument, and is not a fallacy. The foregoing distribution of fallacies, well as the detailed statement as is substantially that of Aristotle. He has been followed hereafter, for two thousand ification closely by logicians years, the onlyconsiderable modwhich we adopt. Attempts beingthe scholastic terminology, at an improved classification have been made, but no one has been partment but in the degenerally approved. Mill's arrangement is masterly, of deductive fallacy he adheres quiteclosely to Aristotle. We have herein, treatment, to present. In the special then,nothing new that the several we hope to show, by a more thorough analysis, classes are amenable to the laws of the syllogism, and hence are strictly
a

formal from

fallacies. The exhaustive. The

classification and

treatment

arc,

however, far

No 'one can forecast the ground is boundless. devious intricacies, the perplexities, the incoherences, the entanglements to the human possible understanding. On se fait une idee precise
"

de

metis Pordre,

non

pas

du

desordre"

"

6.

Paralogisms,

as

we

have

termed

them,

were

not

treated

as

ter, by Aristotle or by the scholastics. The masand his devout disciples until very recent times, were so perfectly familiar with the laws of thought and their application, that the idea of an the formal of a proposition structure or open offence against committed and maintained seemed to being unconsciously syllogism and absurd. But it is different with us. them impossible Palpable all laws violations of syllogistic of comlaws,though they are mon-sense, merely of fallacy whatever. are as as frequent any other species The slipshod ers, judgments and crippled arguments that every -day talkand teachers, sometimes and even are content legislators, preachers, of their utter inconsequence, need to be to use, unconscious greatly and spreadout in thin transparency. But broughtinto the sunlight has read the preceding it were who one superfluous pages, for him than barely indicate these bald simplicities. that we more in the logical A paradox, When this sense, is a self-contradiction.13 is manifestly to A fallacious non-A, we have a formally equivalent
=

class of fallacies either

13

This is the

sense

in which

I understand

Aristotle in

to general

use

the word.

See De

ch. xii. Soph.

DISTRIBUTION.

259

attribute. Is such an error judgment or a contradictory thus When a remark," beginswith a preliminary speaker
"

possible?
" "

to what

he

is about

to

we say,"

are

reminded
"

of

referring ward schoolboy backmarch


to

in his

and progress,"
course,
are

of the
mere

captain's forward

the

rear."

These, of

blunders.

Fallacious definitions and

divisions have been

in A

illustrated under their topics. sufficiently How Immediate inferences are sometimes fallaciously drawn. often, if not orally, is this error the silence of thought, committed : All is B ; all B therefore,
of money

amount large

that to possess a me is to be wealthy I ; then,in the haste of talk,

is A !

You

agree with

may

afterwards
a

say you

justnow

admitted

that to be

wealthyis

to

of money, and, unchallenged, draw a false conamount clusion. large is or is not good whether a man of determining The difficulty is a commonplace of moralists and satirists. Society, however, applies, without hesitation, a very beyond doubt,good men simplerule. Since, it concludes, do good deeds, that he who to itself, quitesatisfactorily does good deeds is a good man ; whereas selfish prudence dictates a / of action almost as imperatively virtue itself. We virtuous course as possess
are
more

liable to
we

this

error

are, when

consider

the

universal affirmatives many matter, simply convertible ; as, " Coin is


so

because

metallic does
come

money." Moreover,though "All seed however true it may not follow, logically
from

come

from
"

it plants," All

be,that

plants

seed."

the fallacy of usingthe contraryof a proposi-/ opposition, logical has already instead of its contradictory, been noticed.14 Importion, tant it is maintained,as errors practical may arise from this. When that this in some creeds, Every dutiful act is meritorious," popular No dutiful act is meritorious," should not be met by the moralists with In
" "
"

for

of
are

two not

contraries

both
may

dutiful acts
at least not not

prove

easily proved; not the other, to popularapprehension. That a thingis not white does ,/" this fallacy thus broadly it black. Nobody can commit
so." This
in the intricacies of
an

may be

be

false,
"

but

with

"

Some

stated;but
many

argument, and in the confusion


is fatal.

of

words, it often lies in wait and


"

that
not

Some

are," my
it

;" for,unless

opponent ought not to be the same Some," both


"

Again,when triumph with


"

I affirm

Some

are

may

be true.

Yet,if palm.

he

the the people, frames an extended artfully reply, will very likely not strictly givehim questions personal,

arbiters in all the

"Part

"8. 3d,ii,

260

OF

FALLACIES.

Paralogisms violatingthe
illustrated several could but
in

law

of

syllogism
the

have

already been
Rules.15
If

ciently suffithe

connection
a

with

General

propositions of hardly
ever

syllogism were
but
lurk

fullystated,these
almost

paralogisms expression
The is

occur;

since

always

the

fallacymay partial, remedy


is is

unseen

in

the

unexpressed thought.
false
;

obvious Another

complete
is to

statement.

paralogism
false,or
of
a

regard
the

the

conclusion
is

as

because

premise
the
one

because
from

argument
of God

unsound

also, to Thus,
if

infer
some

truth argues

premise
the

that
of
a

the
from
to

conclusion.
its

for

existence

lieved, being universallybe-

another
an

might perhaps
of
some

be

able

refute

the

argument

by

ducing pro-

instance

nation
;

destitute

of such

belief,the
then
to

dictory contra-

of

the But had

minor many

premise might
the

the

argument

ought
God,
term
true

go

for

nothing.
refutation would be

think

otherwise, and
of the
a

consider
in

that

this

disproved
an

existence of

which
:

they

guilty of
Whatever

illicit process
is

major
must

; thus
; ;

universally believed
of of
a

be

The
.*.

existence existence

God God

is not is not

universally believed
true.

The

Others, again, from

being already
of
a

convinced would infer

of

the

truth
truth of

of

the

first

conclusion, the existence


which would
What The
.'.

God,

the

the
:

ise, prem-

be

the
is

fallacyof

undistributed
is true
;

middle

; thus

universally believed
of of
a

existence existence

God God

is true is

The

universally believed.
one

If

these

two

fallacies the

were

put in hypothetical form, the


of

would
quent, conse-

proceed
the

from

denial from These


under

the

antecedent

to

the

denial
to

of

the

the

other

affirming the
two
a

consequent

the

affirmation have been


to

of ready alrespond cor-

antecedent.

conditional

which fallacies,

pointed out
middle.16

previous topic,are, therefore, found


those
of illicit

with respectively

process

and

undistributed

16

See

Fart

4th, i,"

5.

16

Whately, Logic, p.

191.

SOPHISMS

IN

DICTION.

261

II. SOPHISMS

IN
are

DICTION.
an require inspection fault. They all logical repeated ambiguously,

"

1. The

sophismcein

dictione

those that

of the

arise

languagein order to detect the formal from of expression.A term ambiguities


to

though identical
two

eye and

ear, must

be counted

for twice,
a

it represents
term

different

concepts. A

such syllogism containing

is,

in thought, risively a therefore, Quatcrnio terminorum,or, as it has been deanimal a called, quadrupes logicum (see logical quadruped, is the vice of all fallacies General Rules,No. 1). This,fundamentally,

ambiguityis in the middle term, the fallacy with that of undistributed middle ; for while corresponds very nearly in ambiguous middle the extremes are compared with two different terms, in undistributed middle they are compared with two different
parts of the
We adhere
treats enter to
same now

in dictione.

When

the

term.
must

and Aristotelic ground, upon the consecrated the time-honored terminology. Aristotle enumerates of these

and

six kinds

of sophisms,

which

we

adopt the following

scholastic

designations.
first
a

"

2.

The

or class, jffiquivocatio, Homonymia


or

is / (o^uww/jm), words in two

ambiguity in
different
senses.

term, single

the

use

of

word
we

or

If this is the middle

term,

have

the
"

sophismof

a quaternion.For example, ambiguousmiddle,formally

All criminal actions should Prosecutions


.*. Prosecutions

be

punishedby law;
actions
;

for theft

are

criminal be

for theft should

punishedby

law.

The

doubly ambiguous,both ''criminal" and "actions" The phrasein one premise being used in different senses. deeds ; in the other, a legal signifies highlyinjurious process. Again :
middle
term

is here

Finis rei est illius perfcctio ; Mors


.*. Mors est est

finis vitae ; vitte perfectio.

Here

the

ambiguity may
upon the

be thrown

If perfectio.

we latter,

either upon the finis the or upon have ambiguous major. The follow-

262

OP

FALLACIES.

ing example is one


is taken from

givenby

Aristotle
of

the
is
a

Euthydemus

redressed by Poste. It (ch. iv), The middle term, Plato, " 12-18. has learned
to

y/m/z/iamroc,
term

is

schoolboywho ambiguous.
o

spell. The

minor

6 JpafJtp,aTLKOQ

tTTlffrf]fJl(t)V'

.'. o

[ia.vQa.vwv
these would
of
course

Such
scorn

obvious which

cases

as

deceive

no

one.

The

ever, howlogical examples are often treated overlooks, in actual discussions are the fact that premises often very wide not stated at all, and the conindeed, clusion perhaps apart, one or the other, and also remote well tion, so an ambiguitymay very ; escape detecand lead to error. "Whenever the premises we can bringtogether and conclusion in the form of a compact syllogism, the sophism of is usually manifest. We must recollect, equivocation quite too, that a
"
"

with

series of

arguments
link.
If
an

is like

chain,which
is
"

is not

stronger than

its

weakest
cannot

ambiguous term
One said." may

somewhere, the lurking

chain

be

depended on.
has been

truth

in what

observe, There is a great deal of Yes, maybe it is all true, except one

essential minute

is most point. The sophistry dangerousthat lies hidden in do not, in general, and come neglected points. Burglars
"

batter down

the front been

door; but climb

in at

some

window

whose

"

does not kindle a tar neglected. An incendiary of the hall, but leaves a lighted barrel in the middle candle in the thatch or in a heap of shavings." is so prolific of false doctrine as this. Are mere Perhaps no fallacy ? Men imagine," that their words, then, so dangerous says Bacon, of language; but it often happens that have the command minds this rule is often misAnd rule. languagebears rule over their minds." in ambiguities, abound and no proespecially, Livinglanguages, cedure is safe that has not providedagainst and that does not them, The onlyremedy is an exact definition close watch upon them. lieep of terms. Whoever would and a consistent use discuss a subject in tions, writingor speechwith scientific accuracy must set out with definiin which and often state the precise he uses sense common
have fastenings
"

"

words.

It is

one

criterion of The

an

advanced
and

science to have

its terms
the

defined. accurately firstto make the moral

mathematical

and progress in this direction, sciences thus attempted to escape vagueness

sciences \vere physical only in recent times and

have

erroneous

consequence.

264

OF

FALLACIES.

may

attach to

supreme
"

duties ; his enactments

The

confusion of nature

of

is

has no sovereign create these for his subjects. law in the juridical with law sense as in Butler's chapter "The on exemplified
" " "

but legislator;

the

moral

formity uni-

Moral

Government
on

of God."

He

calls the

course

of nature

government

merely ground pain. But have nothingmoral in them ; they may be used for these precautions criminal ends. Guy Fawkes obeyed a law of nature when he arranged his powder-minewith safety to himself.2 for firing is used, The several meanings in which the word inconceivable and needhave obscured greatly, and its confusion with lessly incredible," the controversy between the intuitional and empirical extended, but incredible to the ancients, schools of philosophy. Antipodeswere that inconceivable. the not properly Every child conceives clearly cow or jumped over the moon," and maybe believes it, maybe not. of which is truth is a thing conceivable, the contradictory Necessary i. e., cannot be thought or imaged by the mind. This inconceivable, is incredible ; but it does not follow that whatever is incontradictory ceivable, inconconceivable is incredible. Two contradictories may be equally finiteand infinite space ; but, beinglogical as contradictories, it was be true. must able inconceivone Again,before the coming of Christ, that justice and mercy could consist, but not incredible ; since conceivable also. Now then it has become it is inconceivable clearly that election and free-will can consist ; but these, not being logical
" " " "

the

that it induces

to avoid precautions

nevertheless are contradictories, The mercantile

found

credible.8

commit a fallacy publicfrequently by the ambiguity of the phrase scarcity of money." In the language of commerce, medium, "money" has two meanings, currency, or the circulating investment on loan. In this and capital investment,especially seeking is spoken of, last sense the word is used when the "money market" is said to be high or low, the rate of and when the value of money The consequence of this ambiguityis that as interest being meant. in this latter sense of money the scarcity soon as as beginsto be felt, and the rate of interest of obtaining there is a difficulty soon as loans, is high, it is concluded that this must arise from causes actingupon
"
"

Zo/7/c, p. 617. The troublesome of inconceivable discussed by Mill in are ambiguities his Examination his and in chs. vi ch. bk. v-vii. He argues, ii, of Hamilton, Logic, ; and has failed to dissipate the mists. however,in the interest of empiricism,
8
" "

Bain's

SOPHISMS

IN

DICTION.

265

the

of money in quantity medium must circulating

the other and have

more

sense popular

; that the
to

diminished

in

or ought quantity,

have

been

increased.
no

medium, circulating

for more money, cry then arises for more relieve this increase of which can possibly

pressure.4
A man is justified (Rom. iii, 28) that ently without the deeds of the law,"he is usingthe word justify consistas throughout, meaning treated by God as free from guilt." When then how that by works "Ye St. James says (Epist. see ii, 24), and not by faith only," is justified, he too is usingthe word a man consistently, meaning seen to be justbefore God," which, he says, will see and acthe evidence of works. All candid minds knowledge requires When

St. Paul

concludes

"

"

"

"

"

that and The

in such

case

the two
are

statements

are

not

tory, contradic-

that both

arguments
pun, is

conclusive.5

the logical cation. sophism of equivogenerally Swift's taken from Charles Lamb6 quotes the following, Miscellanies : "An Oxford scholar meeting a porterwho was carrying him with this extraordinary accosts a hare through the streets, tion quesLamb hare ?" is that thine ments comor a wig : Prithee, own friend, the fun of it admirably. The Logic of it and analyzes on this, is quite expandsthus : plain.The enthymeme impliedin the question
or paronomasia,

wig is not one's own Surelythat is not your be a wig. .*. It must
A

hair ;
own

hare ;

Here
as a

are

two

or premises, negative

else undistributed
say that
a

middle,as well

ambiguous middle.
mock As herein
the
reason
"

Still we
on men

argument founded
"

term.

Two
ate

generally quite of the middle a palpable equivocation for a wager, one ate ninetyate oysters
may pun
for he
ate
a

is

nine,but
Here the

other
is

two

more,

hundred

and

won."

famous formally proposed. Virgil's


miseroe nimium vicina Cremonse !"

line,7

Mantua, vae
pun,
as

contains

double

such

untranslatable

of course,

but may kinds

be

similarly analyzed.
It may
be well
to

remark

here,once

for

that all,

most

of

to Humor sort. seems mock are wittyjests logicof some to feeling, or misplaced. Wit primarily exaggerated feeling

relate relates

Mill's Logic, p. 564.

McCosh's

Essaysof Elia,

"

No. ix. Popular Fallacies,"

'

Logic, p. 176. 28. Eclogueix,

266

OF

FALLACIES.

rather to
under

from intellectual in character, and often, is more cognition, and cious, fallaeven a absurdly logical play of thought manifestly Dr. Johnson's lets flya sharp dart of truth. a fishing-pole,
"

rod

with

worm

at

one

end and
"

fool at the the

is a other,"

mock

tion. definibad ple, peo-

Mr. Beecher's
and
on a

jest, People are


a

the good people, Artemus

the

Beechcrs,"is

mock

division.

Ward, travelling

in alarm, "Mister Conductor, railway -car, suddenly cries out you'veput the cow-catcher on the wrong end of this 'ere train ; there ar'nt nothingon airth to prevent a cow from in behind coming right and biting and the folks." Here is a curious mixture of humor here, in the affected alarm at the supposed mistaken sarcasm rangement, ar; humor and the grotesque consequences apprehended; wit in the duction slyassumption Your train runs slower than a cow," implied by the deEven the most ously serithrough the ambiguous cow-catcher." intended form, sophism becomes, when reduced to strict logical be ludicrous sham that wonder could so one we palpably a any of its externals becomes deceived by it. As majestystripped a jest, and contempt. a grave so argument may be exposedto laughter many
" "

"
V

3. The

second

Fallacia class,

differsfrom (a/u"p*/3oXm), amphibolice


in the construction of
a

the last in that the rather than in


will go and
a

ambiguitylies

sentence

term.

return

E. g., How I to-morrow.


of
"

much

is twice two

and

three?

hope

member
a

of the House
rose liar,

Commons,
is

the enemy may slay. other chargedwith havingcalled anI


am

that you

and

said, It

quitetrue, and
6p" bpqi Ti"'
'

sorry for it."

An

example of

Aristotle's is :
TOVTO
o

opt! TIQ
TOVTO
o

b
.*. 6

KIHJV KlbiV

OjO".

The

major premise is ambiguous.


he takes from the the

Is the question by a factoryat work, we shall find iron things. This furnishes the syllogism,
to The The
.*. The

totle example givenby ArisEuthydemus, " 67.8 A disputant says, in reply of the silent possible ? that if we speaking go tools far from

Another

beingsilent

speakingof speakingof speaking of

iron tools is

iron tools is the

the

possible ; speakingof the silent ; silent is possible. (Poste.)


"

In the Nicene

Creed,the words
8

by

whom

all

were things

made

"

are

See Jowett's

vol. i, Plato, p. 205.

SOPHISMS

IN

DICTION.

267

referable either grammatically


Second

to the Father

or

to

the Son. is

In the
a

Commandment,

the clause "of


"

them
"

that hate me"


a

tive geni-

by generation." the hearer has thus two grammaticalrenderings, sentence When a his preference and overlook is likely to adopt that to which inclines, governedeither by
or

children

"

the

other.

This the

was

the habitual

trick of the

oracles.

Thus

the

prophecy of

in Henry spirit
The But duke him

VI:10

yet lives that Henry shall depose,


and outlive,

die

violent death.

But

this, says York,

is

just the

famous

response

of

the

oracle to

: Pyrrhus

Aio,te,JSacida,Romanes redibis numquam Ibis,

vincere posse ;
in bello

peribis.

and Falcompositions and dcatjpcarcc), arise from the confusion of lacia divisionis (avvQeaiQ ay tributed universal with a collective term. Accordingto Whately,when a disof compoit is the fallacy term is afterwards used collectively, sition when collective is term used it is a afterwards distributively, ;

"

4. The

third and

fourth

Fallacia classes,

of division. This is clear, but seems not fallacy and the distinction the meaning of Aristotle, preserving.Aristotle's example is as follows :

the

to

have

been

actly ex-

is

hardlyworth

Two Two
/.

and and

three three

are even (distributively} five ; are (collectively}

and

odd ;

Five is
"

even

and

odd.

The
at
one

sense,

noticed. repeatedly time in its cumular,at another in its exemplar or it gives rise to this sophism. E. g. :
All the
A
.'. A
are equal to anglesof a triangle of an a : triangle angle is equal to two rightangles.

ambiguityof

all" has been

When

taken

distributive

two

rightangles ;

B C is B C

So

"

All these trees make

thick shade

do so, or that each does so. together to the mind, many are presented

either that all mean may AVhen multitude of particua lars


are

"

persons

too

weak

or

too

in-

One

more

notable
two

amphiboly :

be
"

occupiedby
students

students

shall occupy these." student shall occupy Two


10

shall university except nine,they being single." (Old Regulations.) in this university and one room except nine, every Code.) (Revised

"

All

the

dormitories

of this

Part 2, act

sc. i,

iv.

268

OF

FALLACIES.

view of them ; but confine their attention comprehensive to each by turns, infer, decide,and act accordingly. Thus, the his health by successive acts of intemperance, debauchee cause bedestroys of these acts would of itself be sufficient to destroy it. no one thus: I am Others reason not bound to contribute to this charity, conclusion that all to that, to the other,drawing the practical nor nor The said to reason be neglected.11 Owenites thus are charity may the doctrine of human : against responsibility

dolent to take

He But
.*. No

who

necessarily goes or stays is not a free agent ; either goes or stays ; one necessarily every
is free.

one

All such We

are reasonings quaternions. obviously

sometimes

hear

an

argument

great men. very well without would still have been discovered, at most
Newton law
arose

prove that the world could do If Columbus America had never lived,
to

only a

had

never

some lived,

of

any

until some etc. one gravitation, and of Newton. Because of Columbus having the qualities one might have had his placesupplied great man by another
man,

other person would Granted, but probably not

years later ; if have discovered the few

great
and

the

argument
The
term

concludes

that all great

men

could

pensed be dis-

with.

"great men"

is distributive in the

premises,

collective in the conclusion.12

t
*

"
An

5. The

fifth class is Fallacia

or prosodies,

accentus

example givenby

Aristotle is from
v

Homer

KarcLTrvQiTcn

o[i(3p"p.1*
ov

Some

he critics,
TO ov

says,

emend

tbis,speakingthe
affirmative to

more

sharply

(XeyovTEQ
of
"

changing oZvTepov),
"

part," saying naught is rotten the remark that the ambiguitycan writing. This is because in his time
were

instead negative; this by by the rain." He prefaces in speech, but onlyin hardlyoccur the written words
and breathings, of the Greeks
were

not

marked

with

accents

and

hence

times some-

ambiguous to
In

the eye when


us a an

not

to the ear.
a

like

manner

with

ambiguityin
in

written word

or

phrase

is resolved

usually by
"

stress

courteous. gallant',

Not

brave ; and voce. Thus, gallant, the least difference " may either no mean

difference at

or all,

very
"

difference. considerable, perhapsthe greatest,

11

Whately, p.

217.

Mill, p.

570.

1S

Iliad, 23,328.

Dindorf

has

ov.

SOPHISMS

IN

DICTION.

269

thy neighbor," against that we emphasized, meaning convey jury perthe neighbor. We read in the is not forbidden except against the prophet spake to his sons, of Kings,xiii, first book 27, "And The italics indicate Saddle me the ass ; and they saddled him" saying, that the word was by the translators; mistakingit for an supplied so emphatic word transfers the saddle. Jeremy Bentham, it is said, feared being misled by false accent that the person employed to read for him was to maintain a monotone. required The fashion of takinga Scripture text and drawing thence a series word and then on of doctrines by puttingemphasis first on one other anif not dangerous. A wrong is very questionable, emphasismay pervert and wholly confound the meaning. But, on the other hand, we present different by admissible and various emphasisforcibly may in what different lights Observe the sentiment. views of the same thought may be placedby changing the stress of voice on the words of our thou the Son of man with a kiss ! Saviour : Judas,betrayest
If in

reading

"

Thou

slialt not

bear

false witness

the last word

is

the

Bdraycstthou, makes tliou,makes Betrayest


" "

the

reproachturn
rests

on

the

infamy of treachery.
with

it rest upon
"

Judas's it upon kiss ! mark


"

connection

his Master.

Betrayestthou the Son of man,


thou the Son Betrayest
of
man

the Saviour's
turns

personalcharacter.
his of the prostitution

with
to

a a

it upon

and signof friendship

peace

of hate and

ruin.

Any
of such Gesture and which

statement tone
as manner

of
was

something that
meant to

has been

said with

and

may
A person

accompany make all the difference easily who


animus

it is the

suppression of accent. fallacy


a

between

truth

falsehood.
serves

quotes another,omittinganything
of the

to

show

the

meaning;
he
to

or

one

who

out withas

notice puts any word alter its emphasis; or any


so as

of
one

the author who

cites in italics so

to

attempts

to make notes

them
of

imply more

or by italics,

exclamation, or

sertions, asheightenhis own than he would openlyavow, is guiltyof F. acotherwise,

centus.

fallacies. Sarcasm and are jests generally of accent, perhapscannot be assumed ironymay be referred to the fallacy without it. Some be, declines a task as beyond his one, it may and another assures him that his diffidence is highly commendable, powers; and Said Job to his fully by the circumstances. justified shall die with and wisdom No doubt but ye are the people, friends, meaning the contrary. The tones and inflections of his voice, you; feel sure, were to irony. This is very effective, those peculiar we may since it is hardlypossible to frame a reply. We have said that
"

270

OF

FALLACIES.

"

6. The

sixth

Fallacia figurcedictionis (or^j/yua class, was Xt^fwe),


tions; using of words having similar terminaunlike things have names with like inflection.
an

limited

by
to

Aristotle to
wherein

the

cases

The
name

name

of what
an

is not

give ground for in uninflected languages, sophistry.This,however, is hardlypossible is commonly held to include and so at present the species any perversion solecism. For of grammar, example: any
Whatever This
/.
man a man

of

action

(e. g.,

he says, action, may and and ailm^ cuttm^),

terminate

like the

walks
on

on

he

walks

the whole

trampleson day ;

He

trampleson
this
source

the

day.

from the arising of paronyms, use or words, such as a substantive, conjugate adjective, and verb coming from the same These have by no means ilar simroot. artful;""Pity and pitiful;" meanings. E.g., "Artist, artisan, "Presume and What is presumption;""Project and projector;" "imaginary" is unreal,but an "image" formed of wood or stone is real ; To to a knowledge apprehend is to lay hold on, or to come fear or dread. of,while apprehension often signifies
" " " "

Very

similar to

of

ambiguityis

that

Designingpersons are untrustworthy ; Everybody forms designs ; be trusted. .*. Nobody can
Are there

enough to think that strong ? Then the drink,because it is strong,givesstrength they commit double of ambiguous terms, and of supposing that an effect fallacy be like its cause. must They should try strong poison. Fallacies such differences, founded than on hardlybe more says Whately,can because in Greek, a more jests. They are not named by Aristotle, constructed the meaning of paronyms, with regularly language, very few exceptions, does exactly (ra ffvaroL-^a) correspond ; and paronyms
.

peoplein

the world

foolish

were

locus of

i. e., of valid, dialectic, reasoning.14 of

The

literal construction under

is also to be included

metaphors and figuradictionis ;


is
is is
a a a

other
e.

of speech figures

g.

Herod
A fox

fox ;

quadruped

.'. Herod

quadruped. breaks patience


que I'un des que

In
14

Hamilton's givingthis example,


See ch. ii, Topica, ix.
aura
"

down.

Disgusted
bon
ou

Si Ton

demontre

conjuguesest
les autres

est qu'il

mauvais,
"

on

demontre,par

cela meme,

tous

le sont

egalement."

St.-IIilaire.

OF

FALLACIES.

III. SOPHISMS

IN

MATTER.

Sophismata extra dictioncm are those in which we must go and inspect the matter beyond the outer form and beyond the diction, in order to discover the logical fault. They are commonof thought, ly

"

1. The

called "Material
not

and described Fallacies," in

as

those whose

fault does

lie in form

nor

but language,

in the

matter,

meaning by

this

that the form

premisesare false. If so, then their consideration faultless, and, as alreadysaid, they are logically does not belongto our subject. But it is not so ; these sophismsare that we examine the formallyfaulty logically, ; only it is requisite
is correct,but that the
matter

in order

to

discover this.

Of

this genus,

and Aristotle,

after

him

the Latin

enumerated logicians,
accidentis

seven

The
^

Fallacia firstclass,

follows : as species,1 arises, (rrapa.(rvpfitfiriKog), says


TO

of subject and accident, from the equation or Aristotle,

whenever
common.

it is

assumed

that

and subject

accident have all their attributes in

By
not

"

accident" here

Aristotle as (erv^ftfftriKoq opposed to ovaia)

means,

but any suborcalled the accident in Logic, dinate usually notion. and individual is to be Every species part of a general regardedas an accident of its genus in this sense.2 For example, mortal ; but Every horse is (an accident of) All men are (subject) and accident), mortal ; hence (equating subject Every horse is a man, is a horse." But it does not follow that "man" and Every man and An "horse" have all their attributes in common. example from the is
"

merelywhat

text not

is :

"

Since

Coriscus is not
is not

and Socrates Socrates,


a

is

man,

it does is denied

follow that Coriscus of

man,

because

who Socrates,

is merely an Coriscus,

accident

of .man."

examples are,
but
as

the

one

undistributed

middle, the
we

Obviouslythese other illicit major ;


take
a

illustrations of the
of them.
are

present sophism

must

ent differof the

view

Either

premiseof

the firstand
instead simply,

the

major

second
1

supposedto

be converted

of per

accidens.

Soph. ch. v. Aristotle does not consider these sophisms as having false in detail their formal faults. He repeatedly excludes from but exposes premises, the consideration of matter false. or as true Logic * See De Soph.ch. xxiv, where Accidens is discussed at greaterlength.

De

SOPHISMS

IN

MATTER.

273

would This,if legitimate,

giveBarbara

and

gitimate, Camestres, but,being illeAnother

givesrise to
text

the F. accidentis.

example from

the

is

as

follows:
do not know what
I
am

You
I
am

going to
the nature of the

ask

you

about
summum

going to
do not

ask you

about

of the

bonum;

.". You

know

the nature

summum

bonum.

Here

the

of (unknown) subject

with its accident

(summum

the genus (about is equated to be asked) bonum). The example may be viewed as

undistributed middle, or We been


are now

enabled
in
He
He
/. He our

loose lying

as an properly amphiboly. certain sophismswhich have long to classify Logics. The standard example is :

stillmore

who who who

calls you calls you calls you

a a a

man

speakstruly ;
calls you
a man

knave knave

speakstruly.

what is presubject (knave) naming a species mised of a subject This is the best solution I a naming genus (man). have seen, but it is not therebybrought under any Aristotelic class. and concludes it is best considDe Morgan confesses it troublesome, ered But it is clearly Aristotle's F. accidentis. Thus : Equivocation.3 You (subject) are a man (genus) ; but A knave is (an accident of)a Here
is inferred of
a
"

man

; therefore

and (equating subject

You accident)

are

knave."

Or

undistributed middle. else, evidently,. The


name

conversion of A by Boethius4 given to the legitimate confirms this explanation of Aristotle's meaning. He has been very and very variously that practically this so misunderstood, generally of sophismhas long since dropped out of the list. Indeed, species who there are very few logicians treat it correctly, to or seem even understand from this malconversion it. Errors arising have already been indicated in i," 6, on Paralogisms.

Fallacia dicto secundum a " 2. The second class, quid ad dictum simpliciter (TOa.7r\wQ f/fji"ia7rXd"c uAAa Try T) TTOV "/ TTOTE "/ Trpog n arises from the confusion of an absolute statement with a Xeyevdai), statement limited in manner, time,or relation. It is obvious place,

that this includes the correlative Fallacia


secundum

dicto

ad dictum simpliciter

the intent of Aristotle ; was quid. This,beyond question, but Whately,followed by De Morgan, Mill, Bain, and their seconda-

Logic, p.

242.

Tart

SJ,ii, "

7.

13

274

OF

FALLACIES.

identifies the latter with F. accidentis, which, in the Aristotelic ries,


sense,

is

ignored.

It is needless

to

make

of these separatespecies

correlatives.6
The
dum

firstinfers from
to
one

statement

made

under

restriction

(secun-

quid)

made

without
is

restriction

E. g. : (rimpliciter).
be forbidden
;

Whatever The
.'. The
use use

pernicious ought to
is

of wine

pernicious ;
be forbidden.

of wine

ought to
to

Here

the minor
to

premiserefers
used.

wine

used

wine,however
infers from

This

is the

clusion, immoderately ; the contime-honored sophism of

a thingfrom arguingagainst

the abuse of it.


statement is made

The

second

without

limitation

to

one

from what limited, proceeding


The

it may essential,

be,to what

is accidental.6

old standard
What You

example is :
to

.'. You

bought yesterday you ate bought raw meat yesterday ; ate raw meat to-day.7
you

day ;

Here

in the conclusion, of meat with the accidental quality inferred, of rawness added, what in the major is said of it simply; i. e., of the essential substance, without regardto its accidental qualities.

is

The

firstof these cases, when we look into the matter, may evidently be construed as illicit minor ; for what is premised of some, a tain ceruse

of

wine, is concluded

of all

use

of wine.

The

second
"

case

is

a plainly quaternion, havingan ambiguous middle is used in two different bought yesterday" senses, with its accident. only, secondly this class of sophismsmight be included Under
"

; for

What

you
sentially es-

firstsimply or

one

to

be

called

F.

dicto secundum

quid

ad

dictum
a

secundum

alterum

quid. When
which life

it is asserted
to despicable,

that the desire of


answer

sportsman to take who that those, eat flesh also,


therefore
to

life is cruel and from

has been
is to

taken

by
one

others have

cruel and

infer from

case special

another

desires despicable special case, and is the

sophismnamed.8
6

See De
"

Soph.ch.
was raw

xxv.

Hence,perhaps,the
mentioned
same

confusion

with F. accidentis.
as

This piece of It
and

raw

meat

has remained

uncooked, as fresh
it in the
state

time.

when

Reisch it in

1496;
8

Whately found
2C5.

justthe

in

prodigious in Margurita Philosopliica, 1826." De Morgan, p. 251.


ever,
a
"

De

Morgan,p.

SOPHISMS

IN

MATTER.

of the species dangerous sophisms before us are those which do not lie in a single but slip now syllogism, in when from to another in a chain of arguone passing syllogism ment, and are thus committed the One of the by changing premises. conditions oftenest changed is the qualification of time. It is a in political that prices, principle profits, economy wages, etc., always find their level." This is often interpreted if it meant that they as while the truth is they rarely most at their level, are generally are, their as but, Coleridge level," alwaysfinding expresses it, they are which might be taken as a paraphrase ironical definition of a or an
commonest
" "

the Perhaps

and most

storm.

very good rule not to encourage infer of all who solicitalms what is true
a

It is

but beggars,

we

should

not

only of professional beggars.

rule to avoid lawsuits, but sometimes cumstances cirgood general make an appealto law a duty. These may be taken as instances of the error called the misapplication of abstract truth ; vulgarly where that is, is applied true in the abstract, to concrete a principle, true absolutely, and no modifying cases, and reasoned on as if it were circumstances could ever exist. This is to reason a dicto by possibility
a

it is So, also,

simplidterad
and It is
are

dictum

secundum

quid. It is

an

error

very

common

very fatal in

and politics

society.9
orators

by

this

that fallacy

and

devotees the words

deceive

and others,

themselves

while they use deceived,


essence

The faith, liberty, religion.


with

of these

their accidents.

Men

commend

loyalty, authority, noble qualities is confounded to a person a loyalty

which

to a nation ; obedience which has no to a power disloyalty which is licentiousness ; a faith which is rightful authority ; a liberty which is superstition.10 mere credulity ; a religion The gods,say the Epicureans, be invested with human must form, because that form is most and everything beautiful must be beautiful, found in them. form is not absolutely But as the human beautiful, but only in relation to other bodies,it does not follow that it must be

is

in

God, who
The

is beautiful

absolutely.11
cases, to
a requires degreeof

in criminal law, especially secundum

in
A

the stating
man

quid which

many

persons

seems

accuracy absurd.

the

was on a ham acquitted stealing evidence showed only that he had stolen a part being convicted of perjurycommitted "in the

indicted for

the of
a

ground that
ham. other An-

year

1846," the

Mill's Logic,. p. 562.

10

McCosh's

Logic, p. 128.

Il

Arnauld,p.

262.

276

OF

FALLACIES.

judge
read
" "

entertained the

of objection

the counsel that it


ia

ought to

have

as

Such minutiao are in the year of our Lord 1846." and quirksof the law ;" but abundant the quibbles shown that the most
minute caution is
not requisite

denounced

experience
to

has

commit

of secundum quid. through the fallacy injustice that the lacies. falstatement We are recur jests usually palpable againto A servant who was Boccaccio tellsthe following ing roaststory: stork for his master was a prevailed by his sweetheart to upon When the the bird came the table, cut off a leg for her to eat. upon of the other leg. The man what was become desired to know master had but one answered that storks never leg. The master, very angry, before he punishedhim, dumb but determined to strike his servant where took him the next day into the fields, they saw storks standing The servant to turned triumphantly storks do. each on one as leg,
"

his master,

on

which

the latter shouted,and


4

the birds put down


'

their
did he
sumption as-

other
not

flew away. Ah, shout to the stork at dinner


and legs,

said the servant, sir,'

but

you

would

have

shown

his other
can

that what
of predicated

be

yesterday so, ; if you had done of this is in too.' The the leg gist of storks in general can predicated
"

be

roasted storks ;
so

quid.
how

And

when

the

ad dictum secundum simpliciter calculating boy, Zerah Colburn,was asked


a

dicto

many

black

beans

it would if you

take

to

make
A

ten

white

ones,

he

"Ten, promptlyreplied, of its accidents stripped

skin 'em."

worthy reply.

bean

is still a bean.

"

3. The

third

elenchi (TO Ignoratio class, Trapa T^V rov

of the refutation, is ignorance to the wrong en-), answering point, proving of the thesis which one (elenchus) somethingnot the contradictory This supposes a disputant, intends to overthrow. an futation, attempt at con-

and is the view to which is usual


now

Aristotle limited his treatment. the


more

It

title, general the issue, or mistaking proposedby Whately,of Irrelevant Conclusion, where the attempt is to establish a thesis by a include all cases to it,or of something which may be proof of somethingnot sustaining mistaken for it. This latter might well be termed Ignoratio or Mutatio condusionis. something Formallythe fault is either in establishing of that is not the required the else or thesis, contradictory establishing thesis. somethingthat is not the required
to
a

take

wider

view, and under

"

For

discussion of these two

cases,

sec

De

Morgan, p.

252

sq.

SOPHISMS

IN

MATTER.

277

If I argue

the

of utility general

some

proposedmeasure,
we are

and

my

in confutation, proofthat opponent offers, in

not

ed interestspecially

is irrelevant. and his conclusion the true elenchus, he ignores it, I show that it is the proper consequence If,in support of my thesis, the true conclusion, and my conclusion I ignore of previous legislation, is irrelevant. his If it be affirmed that
a man

has

and you attempt to propertyas he thinks best,


he has

of rightto dispose refute by showing

that the way

adopted is not the best ; if one party vindicates, instance of resistance the ground of general a particular on expediency, to government, and you ought not to do evil that oppose that we good may come, you are guiltyin each case of ignoratioelenchi. has committed cious atroan instead of provingthat the prisoner Again,if,
cious crime,you prove that the crime of which he is accused is atrothat the poor ought to be relieved in this instead of proving ; if, to be you prove that the poor ought certainly way rather than that, The of ignoratio in each case condusionis. are guilty relieved, you

of law previous in our courts so called, technically pleadings, special of the varieties of statement out intended to produce, to trial are that the case may not at issue, the real points so made by the parties, elenchi. "A murrer" dethe defence ignoratio be ignoratio nor conclusionis, to the remark is about equivalent "Well, what of that?" it may, perhaps, be no in question, That is, grantingthe statement and, if so, is irrelevant. ground of action, in the construction and prosecution important Nothing can be more of the precise of an argument than a clear and adequateconception swer pointto be proved or disproved. In the speechof Diodotus13 in anto put the Mityhad arguedthat it would be just to Cleon,who but not that, that the question was lenians to death,he reminds him

whether

it Avould be
a
"

for the Athenians expedient

to execute

them.

So

Canning,in
we

says, shall continue the


our success

speechin the House The question is not, as


war war

of Commons assumed

in
my

replyto

ceval, Mr. Per-

by

opponent,whether
it is essential

in the that
our

Peninsula,but whether

to

in the

unchanged."

Thus

it is not

presentsystem of currency remain for the after a protracted debate, unusual,

cooler thinkers to
of the real nature

their remarks with remindingthe audience preface of the pointon which issue is joined ; and the longer

and

more

the heated the discussion,

greaterthe need

for these monitory

exordiums.

when For,especially

the the field of debate is large,

19

bk. iii, TImcydides, year

5.

278 combatants
at all.

OF

FALLACIES.

often goes

joinissue

One
in

the wrong points, or do not joinissue loses the propto the east,another to the west ; one osition
on

and question,

wanders

amidst

crowd

of irrelevant

details;
ticulars par-

another

mistakes

contraries for
some

universals for or contradictories,


of

; and, after

hours

storm, they know

not

what

hav" been

One out a case which his discussing. it has not the least bearingon the quesas readily admits,the more tion overthrown similar collateral a proposition, ; another,having his resound the field; makes over pretendedtriumph yet another, of to the prejudices having been rather shattered by reasons, appeals rational antagonist with his auditory, and, overwhelming his more off the apparent and acknowledged victor ridicule and abuse,comes
in the contest.1* And

has made

they adversary

this reminds
of

us

that the

form

We personalities.
one

mutatio often takes the or ignoratio with warmth, and without understanding dispute
or

another.

Passion

bad

faith leads

us

to

attribute to

our

meaning,in order to carry on the contest to greater advantage. It is a signboth of weakness and depravity the debaters ignore that in almost every dispute the question, In all the and aim their tongues or their pens at their antagonists. of mankind, this tendency controversies that have shaken the opinions radical and rebel, is visible. In politics, the epithets tors, tyrants and traiIn philosophy, and weapons. have for ages been watchwords
what adversary the terms ferent materialist, sensualist, idealist, transcendentalist, are, in difof admiration
are or

is far from

his

mouths, terms
names

the contempt. In religion,


of
scorn

Quaker

and

Methodist

memorials
are

in the

past ;

and

"fanatics" "heretics," "bigots," rush


at the throat

of

our

of pugnacity, crowns display

in the present. We plentiful and the world,delighting in a antagonist, the fiercer and more batant. comvituperative ; reason, did and
not

But

argument, not abuse


What
if Luther

is the touchstone ridicule,

of truth.

This

does not did

Calvin

prove burn Servetus? The


success

absurd things? many of the Roman Church. the authority if What


wrote

This

does

not

prove

Calvinism

to

be

fanaticism.

of

Pascal's

Provincial vituperative

Letters

the is very littleto the honor of their author, for it indicates at once he thus aroused to weakness of those he attacked and of those whom

joinin
truth
14

his

The hostility.

as

Juvenal

satiristsof all ages have of Rome. did for the morality


"The Jew cTesprit, Irrelevant Conclusion. Noodle's

done

as

littlefor

some

Sydney Smith's well-known amusing examples of the

furnishes Oration,"

280 4. The
to

OF

FALLACIES.

"

fourth

givesrise

Fallacia consequents (TOnapa TO class, fallacy, says Aristotle,because the consecution


"

of antecedent

and

consequent

seems

If reciprocal. from

follows whatever

from is

A,

we

imagine that A must follow has a beginning, it need not


Because every
man

B.

Because

be that whatever in
a

has

generated beginningis generated.


not

every

man a

fever
16

is

hot,it

does

follow

that

at firstglance, examples, tive. of converting to be merelythe fallacy seem simplya universal affirmafurther. be Aristotle's meaning. Let us examine This cannot of this falsely inferred conIn another mode he says,17 sequence, Subsequently

who

is hot is in

fever."

These

"

the relation of the

contradictories of the

antecedent

and

to the relation of the directly consequent is supposed to correspond antecedent and consequent. If B follows from sumed asA, it is falsely So in Melissus's argument, if that non-B follows from non-A. is unlimited; so that if the is limited, the generated the ungenerated

heavens

uncreated, they are boundless." This makes it sufficiently is to infer the truth of the antecedent that Aristotle's F. consequentis plain
are

from

the truth of from

and consequent,
an

to

infer the

of falsity

the

consequent
If A
can

the

of falsity
cannot

antecedent. B

When

it is

admitted,
is ;
nor

then B is, we is,


we

say, But

and therefore A is, is not.18

say, But

is

not, and therefore B

16

De De

Soph.ch. v.
Morgan
does
is :
states not

17

Id. ch. xxviii.

18

which His

sion to be simply the affirmation of a concluconsequentis f ollow from the a mere non logically premises, sequitur.

the P.

example

Episcopacyis of Scripture origin ; The Church of England is the only episcopal church
.'. The

church

established is the church

that
"

should

be

England ; supported.

in

he says, as consecutive and without flaw," logicof this, the to was clergy recently imputed by an Englishnewspaper ; which,he adds,Avas hard it is merely a logical the clergy. Truly, insect. on for, being sexipedalian, in well to any and every fallacy But De Morgan's definition will apply equally ; is, in definition of could not have a proper fallacy general. This, fact, logical then, been of the schoolmen, his studiouslypassive the meaning of Aristotle, lowers, folnor who surely Neither De Morgan nor meant to be specific. Hamilton,who omits all mention of this sophism in his Lecture xxiii, to have looked into seems The the treatise De Soplmtid Eknclti. former apparently draws from Aldrich, who misses' the point entirely. Nor is Aldrich corrected by Mansel in his notes. Bain views the examples as merely erroneous conversions (p. 675). No recent writer seems the false reasoning properlyto apprehend the scope of this species ; and if treated at all, is treated entirely out of place. duly included by it,

The

maintenance

of the

SOPHISMS

IN

MATTER.

281

This where

think
cannot

has alreadybeen noticed under Paralogisms, inconsequence is often conBut the fallacy the formal fault is pointedout. cealed continually by the matter, and beclouded by feeling.People and express themselves as if they believed that the premises be false if the conclusion is true. The truth, or supposedtruth, which follow
from
a

of

the inferences

doctrine

often

enables it to

find

sophical philoabsurdity. How many had intrinsic recommendation which scarcely systems any because they were have been received by thoughtful men supposedto favorite view in lend additional support to religion, some morality, other cherished persuasion or some politics, ; not merelybecause their but because its leadingto wishes were therebyenlisted on its side, sound conclusions appearedto them what they deemed a strong presumption
acceptance in spiteof
its gross in favor of its truth ! 19

And,
or cause a

on

the other

hand, a good

bad of

argument falls into

supportedby false premises disrepute.A notable instance is the


cause

and extreme advocates adduce in its Temperance. Its warm and of distorted and disof misstatement amount favor an appalling from facts ; and, again, facts they sometimes unquestionable proportioned totle reach their conclusions by a startling to Arislogicunknown followers. Now the argument for this good and his slow-gaited it does not is very simpleand impregnable cause ; but,unfortunately, furnish material enough for the popularoratory of the day,which, untethered by fact or logic. The revulsions the cause soars therefore, has suffered ought to teach its advocates that a bad argument is worse stantly than no they inargument. For when peoplediscover the fallacy, conclude that because and the counter-fallacy, commit a therefore the conclusion the argument illogical, or premise is false, is false ; and
so

the last state think

of that
hold

cause

is

worse

than

the first.

Whoever

that to the principle steadily but merely unproven. in such case the conclusion is not disproved, An and the prisoneris declared "Not indictment fails, guilty," not provedguilty."But the is an abbreviation for which, I take it, found innocent." True, he is to be people conclude he has been is not proof. presumed innocent until found guilty ; but presumption is he the more The more deliberate and skilful the criminal, likely between this verdict. The vast remove to win unproved guiltand marked. innocence ought to be clearly would should truly
" "

19

Mill's

Logic, p.

560.

282

OF

FALLACIES.

"
vtiv

5.

The

fifth class is Petitio

(TOnapa. principii

TO

kv

upxy
an

XajLr/3dtion assump-

"Petition 1}aiT"~iffdat). Says Aristotle,


a

is (curate)

opposed to the belief of the hearer ; or, still wider,a

proposition

proof assumed without proof." Elsewhere he says that requiring this sophism may be called,81 the Petitio qucesiti, or more as correctly in five ways. The first and begging the question, appears to occur the very thing that should be proved is manifest most way is when the terms the This cannot assumed. are easily pass undetected when
"

same

; but when

synonyms
may

are

used,or

name

and its definition second

or

it circumlocution,

is particular
if
we

to be

A escape detection. proved,and the universal that contraries


are

a way is when inis assumed ; as, for stance,

of a single ence, sciobjects of a single and assume their genus, are objects that opposites, in It appears that what should be proved alone is assumed science. universal A third way is when with other propositions. a company is assumed and the particular what ought is to be proved, ; as when Here it appears of some. to be proved of all contraries is assumed is aswith other propositions that what is to be proved in company sumed A fourth way is when alone. divide^the questionto be we it in detail ; as when and assume cine have to prove that mediwe proved, is the science of health and disease, and successively it to assume be the science of each. A fifth way is when two facts are reciprocally

have to prove

and involved,

we

assume

the

one

to

prove

the

other ;

as

when

we

20

Anal. Post, i,10.


Petitio is principii

blunderingtranslation of the Avistotelic phrase, iv, i, 3, Aristotle defines though of universal acceptance. In his Metaphysics, ciple," prinis produced, that from which anythingexists, is known." in general, as or used for that on which something else depends ; and It is always and properly law and for an original dement. Cf. Hamilton's thus both for an original Reid,p. of the principle not 761. The fallacy before us is the assumption, properlyso form or other,of the qiwstion proposed for proof. originally but, in some called, est Pacius, in his Cvtnmentarius in Organon (in Anal. Prior, ii, 16), says, "Non vel *" ry "PXVi ^ "S*i *n principle iv rrJQ "px$"'^ es^ Prineipi^ ; sed TOV petitio initio fuit in id et quod proposituni disquiest, ejusproblematis, dpxy TrpoKtijikvov, and Hansel's See also Hamilton's 369 sitionem vocatum." Logic, Aldridi, p. ; Appendix^note E. We have rather a startling etymologyof the phrase furnished us by Du Marsias, for its own is worth sake: "Co mot preserving s'appelle Logique, p. 81, which voler vcrs quelque de principe, du mot et du chose, petition grcc Trcro/jcu, qui signifie ainsi faire de latin dire veut mot commencement; line petition principe, principium, qui chose que ce qui a d'abord etc c'est recourir en d'autres termes a la meme mis en question."
a
" "

21

rather

SOPHISMS

IN

MATTER.

283

assume

that the side of


we
"

when side."

have to prove The first two


we

with the diagonal, square is incommensurate is incommensurate that the diagonal with the of these five

modes, they being the

most

portant, im-

proceedto illustrate at some length. of this sophism occurs when a premiseis either the The firstmode in sense the conclusion, else actually or same as proved from it. This named the Hysteron protcron,and the Circle. indicates two varieties,
will
now

The
are or

former
in
sense

wherein (vcrrepov TrpoTepov), the same,


"

the conclusion

and

ise prem-

does not

syllogism ; e.

g.,

The A

schism

in the church."

single proposition doctrine is heretical, for it has wrought a from which is thus a corollary proposition
a

extend

beyond

itselfwould
were proved,

not, by any person


it not

in his senses,

be considered

as

therein

in languagewhich makes to be it seem expressed in abstract that a proposition It is not uncommon two. expressed in conis offered as proof of the same terms crete expressed proposition both take Pretended terms. proof and pretendedexplanation this form ; e. g., The loadstone attracts iron because of its magnetic This is burlesqued by Molierc in the speechof Bachelierus : power.
23

"Mihi

docto doctore
causam

Deraandatur

ct

rationem

quare

Opium quoi respondeo:


est

facit dorraire.

Quia

in

eo

Virtus

dormitiva, Cujus est natura Sensus assoupire."


The

being compounded English language,


well fitted for this form of of

of several

is pelanguages, culiarly make


or e. an

affirmation in words the


same

in words

We principii. petitio and offer as a reason Saxon origin, and vice versa; of Norman origin,
is well

tion explanag., The

bill before education

the House
in

calculated to

elevate

the character

of

the

country,for the general standard


raised
it is

of instruction ladies' reasons."

in all the schools will be

by

it. The

These

"

are

It is

so.

Why

Because

so.

are merelyequipropositions pollent,

and should be
23

from distinguished
then

immediate

inferences.

five modes also of proceeds to distinguish Petitio contrariorum. has reference to the In petitio procedure principii wrong it affects only the contrary and affects the conclusion ; in petitio contrariorum b them. For a parat hemselves and the relation subsistingetween propositions phrase of these five modes, see Grote's Aristotle, vol. ii, 62. p. 33 Le Halade Imaginaire Intermede. : Troisieme
13. viii, Topica,

Aristotle

284

OF

FALLACIES.

This
what

which

but occurs in does not, however, require a proposition, fallacy Bentham calls question-begging ;" meaning,names appellatives under it. The names of poguiseof stating beg the question litical as arc Liberal, Conservative, Democratic, Republican, parties,
"

much

used

in this way

"

e.

g.,

Those

who

favor

the

of preservation

the fundamental
with
even

of our act principles government should of course the Conservative party." These are potent when but laudatory, when more so Rebels,and most vituperative ; as, Radicals, litical poThe word "innovation" catchwords. bad having acquireda

sense, the

admission,which
is has

is

that unavoidable,
to its

new

measure

is

an

innovation Galileo

alwaysconstrued
accused

Aristotle himself
:

disadvantage. of being guiltyof petitio

in the following argument principii


The
nature

Now

heavy thingsis to tend to the centre of thingsto flyfrom it ; that heavy thingstend towards experience proves and that light thingsflyfrom it ;
of

the

and universe,

of

light

the centre

of the

earth,

.'. The

centre

of the earth is the centre

of the universe.

could Aristotle say in the major that heavy thingstend to the centres of the universe,except by assuming that the two centre are How
which identical, is what

he undertakes

to

prove.84

Sophistes, attempts to prove that thingsmay exist which are incorporeal, arc by the argument that wisdom and justice and justice must be something. Here, if by and wisdom incorporeal, of something" be meant, as Plato did in fact mean, a thingcapable in and by itself, of some and not as the quality other thing, existing the conclusion does he begs the question anythingelse, ; if he means This fallacy not follow. might also be classed as ambiguous middle ; in the substance, "something" in the one premise meaning some of thought, whether substance or attribute. some other, object of matter,that It was once an argument for the infinite divisibility have an upper and an must small, every portionof matter, however under surface. Those usingthis argument did not see that it assumed of arriving the impossibility the very pointin dispute, at a minimum
Plato,in
the
"

of thickness ; for if there be will of


course

minimum,
a

its upper

and

under

surface The obvious

be

one

; it will be

and nothing more. surface,


more

argument
than the

because the premise seems plausible identical with it.25 conclusion, though really
is very

84

Arnauld,p.

249.

"

Mill's

Logic, p.

574.

SOPHISMS

IN

MATTER.

285

The

formal

fault of
two terms

Hysteronproteron is
only,
"

of syllogism

the usual and

enthymemic by givingthem in
A A
.'. A

mode

of

pretended logical biped. This is disguised by but two of the propositions, stating
a

that it is

different words.
;

The
A
A
.'. A

forms
is B ;
is A ;

are

these,
"

is B

is B ; is B.

is B.

There We

is
can

no

step forward here;


now

it is

merely"marking

time."

understand

condemns Let principii.


us

in the why Aristotle, of definitions as the premising consider the following:

tinctly dispassage quoted, of petitio this mode

of three sides has its anglesequal to Every rectilinear figure is a rectilinear figure of three sides ; Every triangle has its anglesequal to two rightangles. .'. Every triangle

two

rightangles;

and predipremiseis a definition. Now the subject cate of a defining identical in thought, the latter are proposition in the above example, The pointto be proved, merelybeingexplicit. is that the three-sided figure has its anglesequalto two rightangles, in the major whether it be called a triangle This is assumed not. or and reiterated in the conclusion. The example is obviously premise, in the second of the two cither extreme precedingforms.28 Whenever of an is identical in thought with the middle apparent syllogism but two terms, however much the phraterm, there are of course seology tion, iterainvolves mere change. Such a pseudo-syllogism may and no been progress of thought; the conclusion has already and nothing is proved. It is merely the replacestated in a premise, ment in the following: of a term the reverse; as or by its definition, will be to depress The effect of the proposedmeasure wages and to oppress all needy persons, since lower rates of payment for labor will and a cruel, be caused by it, unjustburden laid upon the poor." The use of a proposition which it is itself dependto prove that on ent for proof by no means impliesthe degree of mental imbecility
" a

Here

the minor

potissima in Part 4th,iii, " 1. 27 I am that this view might be extended to the inclined to the opinion strongly and synthetic analytic judgments of Kant (seePart 3d, i," 12). Perhaps it would be correct to say that any syllogism anaI}Ttical having either premise a mere ment, judgis and is in what contained a unfolding principii, petilio actually name, and that whose those are a nothing premises synthetical judgments, ; only proves of to actual proof. If so, this would amount conjunction distinct facts, modify the defence of the syllogism and facilitateit. (Part4th,ii, " 8)
Demonstratio

36

See also the

"

"

286

OF

FALLACIES.

of comprehendinghow this might be supposed. The difficulty be committed when reflect that we sophism can possibly disappears all persons, even of opinions the instructed, hold a great number out withhow they came Hence exactly recollecting by them. they may be from into them another. one easily betrayed deducing alternately A person of the Scriptures time insist on the divine origin may at one because they contain certain sublime doctrines which could not be discovered by the natural sagacity of the writers;at another time he in the Scriptures, may insist that these doctrines are true because found to be wholly accepted. So which, being of divine origin, are in his Phcedo,demonstrates the immortality of Plato, says Hamilton,*8 the soul from its simplicity demonstrates its ; and, in the Republic, from its immortality. simplicity When the same a are premise and conclusion which are actually thus somewhat from of the firstmode remote each other, this variety of petitio is called "Reasoning in a Circle," Orbis vel circulus principii
in

which

demonstrando,vel
as a

diallelus

The (SiaXX?'/Xwr).
:

form

may

be

resented rep-

pro- and
A

thus epi-syllogism,
is B ;

C is B ;

C is A ;
/. C is B.

then"

A
/. A

is C ;

is B.

Of

course

any

number

of

syllogisms may
steps,the
when
more

and intervene, is likely


a

the

greater

the number

of intermediate
man

the

A escape detection. of his circular movement

walking around
; not
so

hill is

sophism to fullyconscious
meridian line.

he walks

alonga

have only to narrow the circuit we expose this fallacy, and out intermediate exhibit the proposition, when casting steps,
to
comes

Hence,

by
it

round

The

in the same words. again, in following example of reasoning

circle is from

"Whately:29

of matter Every particle gravitates equally. have you for that? Why ? What reason

Because

those bodies which heavier.


are

contain

more

ever particles

more gravitate

strongly ;

that is, are But those which

heavier
more

are

No,
How

but

still they contain

always more bulky. condensed. more though closely particles,


not

do you

know

that ?

Because
How

they are
that

heavier. it ?
matter

does

prove

all particles of Because,

that equally, gravitating the


more

mass same

which

is

specifically

the heavier must

needs

have

of them

in the

space.

46

Logic, p. 372.

89

Logic, p. 221.

288

OF

FALLACIES.

is not at all a being stillafloat. This,then, question in takingthat for grantedwhich Its fault lies solely It would be petitio prindpiito prove to a Mohammedan
of Christ from of authority
nem

formal
is not

fallacy.
granted.

the
not

divinity
the
ad homi-

texts

in the New

Testament,for he does
be
a

admit

the Bible ; but it would


to

valid argumentum

mission of Jesus, prophetic of the Koran for the authority he acknowledges. The phrase the unwarranted petitio principii, assumptionof a principle, and the the i s or begging question, properly specifically applied of the sophism. It is not, however, to this second mode to designate in which anythingis presupposed be understood as if e'very probation If so, it and not proved were worthless. at once to be rejected as
to prove

him

from

the Koran

the

be necessary in every case and these of human knowledge, would


as might be rejected

to

ascend

to

the ultimate

principles

of proof, themselves, being incapable

ing, assumptions. Were this the meanis guilty of whatever.33 A probation there could be no probation which may be doubted on the this sophismonly when a proposition ple ground on which the thesis itself is doubted is assumed as a princiand we thus attempt to prove the uncertain by the equally of proof, Sound probation must uncertain. as are departfrom such principles cither immediatelygiven as ultimate, or mediatelyadmit of proof than the proposition from other sources itself in question.33It is connected are allowed," closely assumptions says Aristotle,that when with the issue, we on deny them, and refuse them as premises, may the pleathat they beg the question." mode of the sophism was of this second Among the schoolmen of their time consisted largely of interest. The philosophy peculiar established by authority, and certain general (principia) propositions derived from intrinsic evidence. Among supposed to be ultimately of the doctrines which with these tenets were were Aristotle, regarded Stultum due only to inspired est dicere Arisa reverence Scriptures. which Others were considered as totelem errare. were propositions
" " 34

unwarranted

83

"The

main

of principles

reason

are

in themselves

apparent.

For

to

make

to take a\v;iy all possibility were nothingevident of itself to man's understanding of knowing anything. And herein that of Theophrastusis true, They that seek of all thingsdo utterly overthrow reason.' a reason Hooker, Ecd. Pol. i,8, 5. 33 Hamilton's that a saltus in probation is a He further observes Logic, p. 371. of of intermediate an case use a propwe ; for, special link, petitio by an ellipsis osition which is actually without its proof. 34 DC Soph. xvii.
'

"

"

SOPHISMS

IN

MATTER.

280

those as rigorous of Euclid. None were in rare ever questioned; perhaps, cases, except, in the nominalist as when, consequently, was controversy,society shaken to its foundations by a moral earthquake. These principia, at the command of every disputant. admitted,were beinguniversally had properly in Barbara The syllogism for its sumption, a principium and an cxemplum for its subsumption. The petitio occurred principii when any one, to prove his case, made it an example under a principle
as

been fully established by having

demonstrations

which without

was

not

among
"

those

and which received,

was

assumed

to bring it under their logical offering empire. Thus, were to argue from must one Every beingvoid of reason perish that therefore the brutes perish, it would be denounced as petitio principii, this sumption not being found among the acknowledged principia. is essential to Again,suppose one to argue that since Entire liberty civil law,being an abridgmentof liberty, and happiness," well-being
" "

is therefore detrimental

and

should

be abolished.

To

this would

be

Of course, if your major is true ; but unless you offer prereplied, liminary the We illustratefurther beg question. proof, by you may the reply of Cardinal Richelieu to an f or who applicant clemency the to il faut vivre." thought reason matter, saying, Mais,monsieur,
"

Said his
a

Grace, Je n'en vois pas la necessite." There is, perhaps, breath of inhumanityin this, it means but logically that the postulate
was

"

admitted by him as Cardinal, and principles that one but not the question. might reasonably beg his life, The third mode of petitio the particular to prove assumes principii the universal. Aristotle himself seems of this when he to be guilty is in accord with natural law,on maintains that slavery the ground that the neighboring the are barbarians, being inferior in intellect,
not

among

the

born bondsmen The the


fourth

of the Greeks.85

and fifth modes

need

no

illustration. special
how of

ing Concern-

latter, however,we

will remark

easy it is to frame

different by the use apparently opposed or terms. For example, "Everywhere the lightof life and truth was for darkness covered the land,and gross darkness the people." lacking, Again, Alexander was the son of Philip;therefore Philipwas the father of Alexander." The last exampleis cited by Dr. Reid as a case of for which Logic does not provide. Truly so ; simplereasoning" it. but,on the other hand, Logic has been careful to provide against
" "

propositions correlative

88

2. Politica, i,

19

290

OF

FALLACIES.

"
TLOV

6. The

sixth class is Non


"

causa

pro

causa,
"

(TOp)

a'inov

u"c

at-

not

for a cause what TiQlvat). We mistake,"says Aristotle,36 for what is not a reason] when an a reason a cause [meaning,

is

relevant ir-

one

of

has been foisted into an proposition argument as if it were His example is a reductio ad impossithe necessary premises." ble that
"

to prove We
assume

Life and the soul

are

not

identical ;" thus

of destruction is generation opposite ; of destruction is a particular Therefore the opposite a particular generation. and its is life But death is a particular destruction, opposite ; and to live is to be generated. This is absurd. is generation, therefore, Life, that the
"

Therefore

life and

the soul

are

not

identical.
"

Q. E. D.

The

absurd

conclusion may
"

be

the contradiction of a justify is mentally foisted into that Life and the soul are identical," premise, stated as the Q. E. D. It is treated as and its contradictory the train,
if it
we were

sequence, and its absurdity premise. But here an unexpressed


a

proper

the the

cause

of the absurd

which conclusion,
or non causa

it is not, and
causa.

so

have

of fallacy
37

false cause,

pro

Aristotle

afterwards the

says

that to detect this this

of suppression
we

does not, then and treated


in
case as

know
cause

must examine whether we fallacy If it the sequence. premisewould interrupt foisted in that it is a superfluous proposition

the

of the absurd

lacy conclusion ; and this is the fal"

question. In

the Prior

he Analytics,

says,

The

most

ous obvithe

of the irrelevance of the thesis to the conclusion

is when

thesis is not
was

connected

by

any

middle

term

with

the

as conclusion,

the sophism of non causa Topica when discussing pro We the to commensuratethis should causa. if, disprove exemplify of the side of the square to the diagonal, we ness appended an argument for Zeno's theorem that there is no such thing as locomotion, therebyto establish a reductio ad absurdum" pretending It is clear that Aristotle intended to designate by non causa pro wish to refute is the cause, the pretencethat the proposition we causa in a reductio ad impossibile, of the false conclusion which in fact flows from the sophism consists in maintaining other premises; that is,
said in the
s

that the conclusion It is


from
a case

is false because

that

particular premise

is false.

An

impertinence.It arises in dialectic disputation the practice of askingthe opponent to grant certain premises. is asked and granted among the rest, proposition unnecessary
of sheer it is selected
"

and afterwards
88

as

the false

assumption.39
S8

De

v. Soph.
89

Id. ch. xxix.


notes
on

Ami.

Prior,

19. ii,

See

Manseljin

Aldrich, Appendix, " 4,4.

SOPHISMS

IN

MATTER.

291

Aristotle does
cases a

not, however, limit the


ad

sophism of

false

cause

to

of reductio

but includes under it all cases impossibile,


to

wherein

by virtue of a premisethat does not of this vice. For instance, necessitate it. He himself is not guiltless he insists that there are three kinds of simplemotion, because body has three dimensions, but hardlymakes it clear how the one follows from the other, i. e., givesus no middle term these propto connect ositions.
exist

conclusion

is declared

He

would

prove

also that the heavens

are

unalterable and there is


no

because incorruptible, motion motion

they

have

circular But

motion, and
has the

contrary to circular motion.


to do

what

of contrariety And
is not

with the

or corruption

alteration of

body ?

rectilinear motion

contraryto circular? This sophism has been misunderstood, or


all recent writers
on

at least

misstated, by perhaps

noticed several Logic. We have already deviations from the Aristotelic sense common more misapprehensions, In this case is of sufficient importance the error less grave. to reor quire that the common view be set aside and the original stored. reone It is needful to explain the deviation and to justify this
statement.

Let Causa

us

first note

distinction drawn determines

by

the old

The logicians.
a

essendi is that which

the existence of

fact. When

rain falls upon the ground,the ground is wet; the rain is the cause The cause of there beingan eclipse of of the ground's being wet.
it and the earth. between The interposes is the cause of our knowing a fact. It has rained, Causa cognoscendi that the ground is wet. Here the same therefore I know thingis the both of the existence of the fact and of my knowing the fact. cause

the

sun

is that the

moon

But The

what

is effect in the firstsense I know


must

may

be

cause

in the other.

E. g.,

ground is wet, therefore


the
moon

it has rained.
it and

There

is an

eclipse

of the sun, hence


causa

be between

the earth.40 The

the

mining, deterthen,is the logical ground ; it is the cause cognoscendi, but the judgment. This we now not the fact, commonly call and use the word cause reason for,or signof,a thing, only in the
of
causa no

sense specific

essendi*1
doubt that

in the title of the sophism Aristotle, the causa intended exclusively under consideration, or reacognoscendi, There
can

be

40

In this

inversion, reasoningfrom

effect to cause,
An

we

should
be

note

that
a

we

are

liable to the causes,


41

of Causes. of Plurality fallacy than any that to other a cause perhaps illative"because" is stillused

effect may have been

due

to

of variety

observed.

The

generically.

292

OF

FALLACIES.

son;

and that his the

understood ancient and mediaeval, so followers,


same

him,
cause

and intended

limitation.43 In

recent

times,the word

have for the causa essendi, logicians becoming used almost exclusively this sophism. commonly mistaken his meaning and wronglyinterpreted sufficient to be the assumption without They define the fallacy of another. Thus, essendi} (causa ground that one thingis the cause of a change in the weather ; is the cause that a change in the moon the heat bad luck ; the dog-star, causes thirteen at table brings Sirius, attributed his that prevails during his ascension.43 Whitcfield once at the last being overtaken by a hail-storm to his not having preached fore thereSince many town. a nation having a heavy debt has prospered, instances These are clearly a national debt is a national blessing.
" "

of the

hoc ergo propter Post hoc ergo propter hoc,or of Cum fallacy duction, bad inof bad generalization or is merely a case hoc.4* This fallacy and therefore, however importantit may be,has no proper placein Deductive Logic. But by our recent writers it is declared to be strictly the non causa pro causa, and is introduced and exclusively it is not only an this title. Now discussed in this placeand under

entire deviation thus to

from
non

the

meaning
pro

of Aristotle and
a

the

scholastics
to

the interpret

causa

causa, but also

blunder logical

the deductive fallacies. On the include the inductive post hoc among be Aristotle's non other hand, however causa lightly may pro causa esteemed,it clearly belongsto the deductive fallacies; its formal vice, since it has Next
error was no

middle

term,

being that
word

it is
cause

to

the

restriction of the

quatcrnioterminorum. in usus the loquendi,


and
"

to the probablydue secondarily was Aldrich, therebyconfirmed. or, at least,


non causa

influence of Arnauld The


we

former

says,

The

pro

causa

is very
causes a

common,

and

fall into it

of the true

of

things.

have
vacuum

attributed

thousand

It is in this way effects to nature's abhorrence


water

through ignoranc that philosophers


of
a

; for

that vessels full of instance,


then

break
a

when

it freezes, which
ure nat-

because

the water
endure

contracts,and
and
so

thus leaves

vacuum,

cannot

;"

on,

of illustrations.45 through a variety

of that which is rendered cause,"but has the generalsense fairly of blamable." with bad the sense a thing ;" mostly something chargeable 43 ^En. x, 273. See Virgil, 4* inNon sic causa cujus est causa. Says Cicero, Causa ea est quae id efficit sed quod cuique efficienter id ei causa sit, debet,ut, quod cuiqueantecedat, telligi
42

aiTiov

is

"

"

"

'

"

antecedat."
48

Port-Royal Logic, pp.

251-56.

SOPHISMS

IN

MATTER.

293 ; sivc sit a

Aldrich
vera

it designates

"

Fallacia
non

non

causa

pro

causa

non

lum Haec

pro vera ; sive a erit. Nullo modo fallacia bene


48

tali pro tali : ut, Cometa


;
nam

si

aliis de fuerit,

fulsit ; ergo Belcausis futurum est.

solvitur

negando causam

falsam ; mclius adducendo

is germanam." Whately, under the influence mainly of Aldrich, He first acceptshis mistaken view,and illustrates at fault. evidently the blunder, it. Then, dissatisfied, he guesses correctly that logicians and reason were confoundingcause ; and proposes to substitute the *" of Undue title Fallacy remarkingthat the varieties of Assumption," this are infinite.47 Verily from a false ; for this is merely to reason in any way. But such is not a logior premise, suppressed disguised cal for Logic has nothing to do with the falsity of the at all, fallacy premises. De Morgan treats the non causa pro causa very gingerly. is of lie says, It the mistake imaginingnecessary connection where
"

there is none, the word."


4

in the way This is wide

of cause, considered

in the widest

sense

of

and the wrong. But right view only. For instance,he quotes the had such a profound knowledge of music

and might enough,truly, his examplesshow that he


statement

include both the takes the wrong that Saunderson

that he
"

could who

distinguish
made this

the fifth part of


statement

note

; and

then

remarks, The
any

one

did not

that know, first,


a

person

who

cannot

distinguish
pends de-

less than the fifth part of intention of upon

note

to

begin with,if he
bound

exhibit the least intonation


to

learning any
the

musical

instrument

in which
over

keep the not so gifted that if Saunderson were by nature, peace ; and, secondly, the defect of than music have supplied would more no knowledge These remarks show that would givehim sight." knowledgeof optics he had only the causa essendi in mind sumption ; for he therein denies the asof the disthe efficient cause criminati that knowledge of music was And recent Englishlogicians so our generally.49 promptly

ear, should

be

48

48

Logic, Appendix, " 4, 4. Formal Logic, p. 268.


Bain

4T

Whately'sLogic, pp.
p.

223-33.

49

makes

the

mistake

(Logic, p.

626

and

675). Hamilton, following

view as a dethe mistaken ductive Krug, misstates the meaning of non the deductive fallacies fallacy.He also wrongly puts post hoc among the title non causa (Logic, pp. 237-39). Mill does not use pro causa, and omits to notice the Aristotelic species.He puts the post hoc in its appropriate place among Fallacies of Generalization.") Minor false inductions. (See Logic, bk. v, ch. v, on all that I have examined,and they are many, blunder along with passive writers, Bequacity.
causa, and
treats
"

294

OF

FALLACIES.

interrogationum (TOra the sophism of many questions.It is the effort epionifiara tt" TTOICIJ'), asked in one. to get a single to several questions E. g., Was answer

"

7. The

seventh

class is Plurium

Pisistratus the
not

to respondent A a single a false position. indeed,but so question, will assert or deny some stated or compounded that a simpleanswer E. g., Did you take anythingwhen you other impliedproposition. broke into my house last night? Are you the only rogue in your family? Have you quit drinking?60 Have you cast your horns? the sophism is sometimes this last ancient example, called the From Several questions Cornutus. put as one should be met at once by of the compound question into its elements."5 the decomposition which has long served as the example," Obviously ; as in the following

tyrantand scourge of Athens either a yea or a nay would the other,


variation is to ask

As

he

was

the one, but

commit

the

"

standard
trem

illustration:

"

Menedemus, Alexino
Nee inquit,

verberare

desiisset?

rogante,Numquid desii." So nee verberavi,

pathe

savans Royal Society Why does not a live a

dead

one

does?

waggish query of Charles II : fish add to the weight of a bowl of water, as This implies which for a time the two questions,
sit?
for

at last solved the

viz. 1st,An overlooked, philosophers puzzled All this


seems

2d, Cur sit?"

notingthe sophism of dialectic disputation the is to be found in the eristicmethod among and answer, the answers Greeks, which proceedsusually by question familiar to readers of being conventionally yea or nay," a method is to entrap his Plato's Dialogues. The effort of the Sophist unwary into which him admission be turned can an as against respondent from borrowed is attribThe following uted, Fries,55 example, paradoxical. to Eubliin its original form, by Diogenes Laertius (vii, " 196), des the Megarian as the inventor:
"

frivolous. quite

The

occasion

Have Must

No. ? you lost ten counters you not have lost what you had
"

at

the

beginningof

the game

and

have

not

now

"

Yes.
counters
now

Have Then

you

ten

"

No. contradicted

you have

lost ten

counters, and have of the ten

yourself.
stillhad

But

he had lost

only two

counters, and

eight.

60

52

63

B1 De Soph. xxx, 3d,i, " 12. from 135. ii, Originally DiogenesLaertius, See the hackneyed story at lengthin Hamilton's Metaphysics, p. 118. ch. 18, " 135. See De Soph,xvii ; and Diog.Laert. bk. ii,

See Part

65

" Logik,

109.

It is cited also in De

xxii. Soph.

296

OP

FALLACIES.

IV. EXAMPLES.
from " 1. Logic, The profession.
to

the time
acute and

of

became Aristotle,

among

the Greeks

themselves

invent

Athenians busied fun-loving especially with which deride the to entangleand puzzles these worthies themselves used the
same

and stately professors;


means

with Many of these puzzles, together have been handed down similar inventions by the scholastic logicians, As satisfactory the centuries to us, discussed at every turn. solutions Rationes." were Inexplicabiles They rare, they received the title of mostly from Diogenes Laertius, by Gassendi,in his \were collected, reviewed Liber de Origins et Varietate Logicce, and are analytically by wit and be of to a mere acuteness, Hegel.1 Appearing generally play marvel at the interest they have excited, and at at their celebrity, we of the most the importanceattached to them by some distinguished have an historical interest ; and thinkers of antiquity. They certainly references to them, the student of Logic literature makes as frequent their acquaintance. to make cannot neglect The disguises which innumerable. It assume are sophistry may in the conditional forms,for these, to lurk most seems securely being the guise often very intricate, most are confusing. Perhaps completedisits great capacity is the dilemma, which, from for entangled and hence is always the favorite form of the Sophists, statement, was and distrust. In some regardedwith suspicion cases, however, very will select and exWe amine simpleforms have provedvery troublesome.
"

to discredit their rivals.

few

of the most

noted

of the

Inexplicables. They are


the matter
to

known

derived generally from names by specific were applied. originally

which

they

"
the

2. The

Achilles

of tenet leading of rest and motion is a necessary result of the conthat the identity trary not serious in this arguhowever, he was opinion. Probably, ment,

to support proposed by Zeno the Eleatic, Parmenides, the unity of all things, by showing
was

but intended

it to retort the ridicule which

had

been

thrown

on

See Gesch. der

xvi, Fhilos.,Werkc, p.

119

sq.

EXAMPLES.

297 in the

the doctrine

his opponents by involving to find in his theory.a absurdities that they professed thus : Suppose that Achilles runs The sophism runs
of

his master

same

ten

times

as

fast has When mile

as run

tortoise this

that

is

one

mile in advance.
has advanced

Now, when

Achilles

mile,the tortoise
has
run

yV
has
so

of

mile

his pursuer

farther;and
Achilles
can

then
never

TV,the tortoise TTJVo of a mile; and


this overtake the tortoise.

advanced
on,

ad

beyond. of a T-J-y infinitum.

Hence

this a sound argument, though leadingto pronounces demonstration falsehood. not canWhately says the pretended palpable

Hamilton

be possibly

exhibited
of

in

Hansel, is
easier.

surrender
:

form.3 This confession, syllogistic says criterion. But the syllogistic nothing is

Thus
space
an

Any
The
.'. This

+ equal to ffc-

-I~

+ TTrinr

is

infinite, being the


this
sum

sum

of

infinite series ;
to be

space space

passed over

by

Achilles is

equalto

is infinite.

The

whole

mystery of this famous


false.

sophism

lies in this: The

major

premiseis
case

The

finite. The is,

infinite series may be, and in this whether space is or is not false, premiseis equally
sum

of

an

divisible ad The
6

says
tacit

infinitum. This is the solution given by Descartes.4 is refuted by Herbart. Mill solution attemptedby Coleridge5 in the lies in the ambiguity of the word the fallacy infinite," nitely is infifalse assumption, Hobbes and as hinted,that whatever divisible is infinite. The argument proves that to pass through
"

finite space
time.

a requires

time

that is

but divisible, infinitely

not

an

finite in-

This

is tantamount

to the solution

of Descartes.

Viewed

as

volving it is not a logical fallacy.Viewed as inpremise, an ambiguous term, it is a quaternion. Aldricli says, "Solvitur ambulando, quod fecit Diogenes." This ley's reminds us that Dr. Johnson, in like view, thought he refuted BerkeZeno and Berkeley affirm that idealism by kicking a stone. contradicts sense. reason practically, Diogenes and Johnson reply,

having a

false

that
a

sense

contradicts
in

reason

; which

is

clcncki.7 ignoratio
1.

Hansel's

note

Aldricli, Appendix, " 5,


4 ; xxxiii,

Cf. Plato, Parmenides, p. 128;

De Aristotle, d'ffle.
4

Soph,x, 2, and

and

Cousin,Nouvcaux

6 vol. iii, Friend, pt. i, Epist. p. 93. Ep. 118. The four principal arguments with which Zeno proposed to

Fragmens, Zenon 8Zc*7w,p.411. 6 Logic, p. 168. ality disprovethe re-

of motion

are

as

follows

298

OF

FALLACIES.

"

3. The

Diodorus

Cronus

is

so

called from

the

name

of its inventor.8

It also
It ranks

the impossibility of motion. to demonstrate professes the Inexplicables, and has probablybeen more high among other
on puzzle

discussed than any


If motion
where is

record.

It is

as

follows
it is, or

a body moves possible,

either in the

placewhere
is not and

in the

place
in the
act
or

it is not ;
move

But

it cannot

in the

place where

it

for there is,

room

nor

place where
suffer where
.'. Motion

it is not, for it is not


it is not
;

there to move,

nothing can

is

impossible.
had
reason

The

story goes that Diodorus


He
sent

to

lament

vention. this brilliant in-

instead

surgeon to reset his dislocated shoulder,who, irrefutable of setting it,set himself to prove by this same for
a

that dislocation was impossible. logic It is a conjunctivo-disthe is quitecorrect. Formally reasoning treated conjunctively in the tollent mood. But syllogism, junctive the major premiseis false. is not contradictory. the disjunction "The First, place where a of the place where it is not ;" but moves body is is contradictory is not contradicted in a placewhere in a place it is by moves it is not,"but rather, it should be,by does not move in where as it is." If so stated, the same conclusion could not be a placewhere denied drawn, for then the consequent could not be totally formally the grounds adduced in the minor. on view the disjunction cannot we as quiring remerelyincomplete, Secondly, and therefore inept;for the tertium quid to complete a it, second member, "moves where it is not,"cannot be accepted in a place at all ; it is a self-contradiction, or a mere jumble of words,a bit of
" " " " " "

sheer

nonsense.

1. Motion

cannot

placeuntil

it has

begin,because passed through an


overtake
at
a

body

in

motion

cannot

arrive at

another

unlimited

number
as

of intermediate

places.

2. Achilles cannot

the

because tortoise,

often

as

occupied by the tortoise


3. The 4. The
arrow flying

moment, previous
of time is

the latter has


moment

is at rest ; for it is at every division

place alreadyleft it. in only one place.

he reaches the

half of

a same

moving
in the

with the

traverses velocity,
a

pointat rest,in whole of that time. a given time,in the other in the For interesting these famous arguments, see historical notices concerning Hist, Phil. 20. weg's of " Laert. ii, 112. "Diog.
one

case, with

equal to the whole ; for the same point, distance when an equal (i. compared, e., the other with a point in motion), in the one
Ueber-

case, in half of

EXAMPLES.

299

the Thirdly,
use

firstdisjunct member
"

is rendered

absurd

preposition in." A body cannot be in situ. This,also, is essentially in a place," a self-contradiction, of words. A body can ing/row an use only be thought as movincongruous it was, through the placeiu which it is, the placein which This objection raised by into the placewhere it is about to be. was and is repeated Gassendi, by De Morgan.9 Mansel considers the disjunction as a third as incomplete, omitting in a place where and of it is, horn, the possibility moving partly the major where it is not ;" and therefore he rejects in a place partly is givenby Hobbes.10 But solution, substantially, premise. The same I cannot understand what is meant by a body's moving partly clearly in a placewhere it is not." Hobbes, however, undertakes to prove small it however with a diagram that a body, quantulumcunque sit, all at once leave the whole of its former so place may be, "cannot to the that a part of it shall not be in that portionwhich is common is left and the other which which is two namely,the one places, This is merely an evasion ; for a part of a body is itselfa reached." body,to which the sophism stillapplies. Or it may be considered as the question an metaphysically, involving attempt to solve the difficulty of matter. the infinite divisibility concerning refers the sophism to F. accidentis" and supplies the omitted Bo wen A moving body,at o.ny one indivisible moment, must limitation thus :
of the
" " " "

rate by an inaccuing thought as mov-

be either where
moment

it is is

or

where

it is not.

Hence, in

any

one

indivisible

motion When

for impossible, here proviso

motion

space.
is

the

italicized is the he

time well as as requires the proposition expressed,

true,the reasoning is sound, and


What it must
of the
or disjunction,
"

conclusion
mean

correct." the second

I
ber mem-

am

still partially in the dark.

does

by

be where

it is not?"

"
of

4. The

is or Reciprocus, Litigiosus,

noted

dilemma The

of which

we

have two

accounts, one

Greek

ia

and

one

Roman.13

latter tells it

his pupil in the and Euathlus, the princeof sophists, Protagoras, law. Euathlus had contracted to pay his tuition fee when he gained

Logic, p. 260. nZo^,p.298. 18 in Waltz's By Suidas,


Tisias ; and
MOV,

Formal

10

ch. viii, Philosophia Prima, pt. ii, "11.

Rhctores
is said

Greed, vol. iv,p. 13, where


the originated

it is told of Corax

and
KCIKOV
13

thence

to have

proverb, KOJCOV

which

in part still survives among v, ch. x.

the

vulgarof to-day.

Aulus

bk. Gcllius,

300

OF

FALLACIES.

his first case. fee


"

But

not

having any
in

case, he

was

sued finally thus


:

for the

who, by Protagoras,
Learn,most
demand
you

court,addressed
men,

him

foolish of young
must.

my

For
; and

if the if in

by decree
you

of the court

matters however that, be judgment against you, by the terms of your favor,

may

turn

out, pay

me

I shall
our

obtain

the fee

contract, for then

shall have

gainedyour

first case."

To in
"

this

Enathltis, provingat least that he


follows
:

was

an

replied apt pupil,

corresponding terms, as
Most

of masters, learn from your own whatever may be sapient argument that, of the court, absolved I must the finding be from For if the any claim of yours. I shall accordingly and if I decree be in my favor, adverse, shall pay pay nothing; nothingby virtue of the contract, for I shall not have gained my firstcase."

The the
case

unable perplexed judges,


sine die. dilemmas
are

to

find

ratio

decidendi, adjourned

The omitted
for pros.

the
"

same.

member the

is

no

decree

and suit,

judges should the result. this was Practically


classed Mcntiens,

is incomplete. The disjunction had no at all." Protagoras ground have quashed the case with a nolle

The

"
to

5. The

of old among

the

and known Insolubilia,

the Greeks

by

the title tyevdonevog, was


six treatises on

also invented

by
to

Eublides.

the Stoic, wrote Chrysippus,

and Philetas of Cos, it it,

is

studied himself said,


states

to

death

in

the vain

attempt

solve it.14

Cicero

it thus
that

"If

you

say you

you

and lie,

say

so

then truly,
same

you

do lie; but

if you
at
once

say

so

then falsely,

speak the

truth.

The

is assertion, therefore,

false

and

true."

15

"The without

"is very obvious. solution," says Mansel,16

No

one

can

lie

lying about something.


But it
seems we are

The
to

statement

'

taken alone, lie,'


"

is senseless."

understand

that

I lie in this

that I lie." Then it would be more to formally logical very statement like all other assertions, offered primarily say that this statement,being,
a self-contradiction, itself, logical paradox, destroying and therefore null. Gassendi puts the sophism thus : Qui juratse \ falsum jurare et falsum jurat, vere jurat." as a
"

true, is

14

Diog.Lacrt, 7,"

196.
16

15

Acad.

30. Qucest. iv,

Note

in

Aldrich, Appendix, "

6,4.

EXAMPLES.

301

Let
of the

us

take tins occasion to

speak once

and again,

more

generally,
a

A paradox.17 logical
in

self-contradiction in terms

of course, is,

blunder
Human Let The There
us

in the following as : dictione, examples


bounded
to

thought is compel them


are

onlyby

the infinite.

volunteer.

crime

of suicide deserves
many kinds
to

capital punishment.
than to know extra
so

of individuals.
so

It is better not

know

much

many

thingsthat
not

aren't

so.

But

sometimes

the

paradox is
say of say
we a or man

and dictionem,

quiteso
be

vious. ob-

Suppose we
true,then he
the truth. follows
is
a can never

that he is
"

alwaysa

liar.

If this be

imply

for this would lie,"

telling

But

since
man

must

that "a

always a
"

think that any man it may say this, it liar" is an impossible conception,

self-contradiction.

Hoc

unum

scio, quod

nihil scio" Socrates is

It is certain that there is nothing certain," to have said. reported said the paradoxical of the Middle Academy. This sayphilosophers ing is the thus uncertain so : Pyrrho, Sceptic, Everything disputed that it is even doubtful whether there be nothing certain." But this absolute scepticism those who doubt that doubt itself is doubtof ing involves also a self-contradiction ; it professes a belief that there is no belief. Of all universal propositions, it has been said, one only latet error: is allowable, In generalibus this denies all others;and looked at,it too commits suicide. then,when closely to the remark that jests once arc more Recurring fallacies, generally
" " "

"

we

add

that the

essence

of

the "Irish

bull" is self-contradiction.

Though perhapsnot half the lies they tell of the Irish are true,yet natural art with them, and not intentional mistake. seems a bulling
Bulls
are

not dead,but in earnest. The Paddy who said he was jests of the isle whose a living was only speechless, felo de se, and typical is not yet full, askingonlyfor non-interference, overflowing cup of woe

and for but littleof that.

"
des.

G. The

Sorites

a heap, is attributed by (vapor),


"

Persius18 to

Chry-

sippusas
the false. No. Do

the

but Diogenes Laertius inventor,

attributes it to Eubli-

It is defined

grees by Ulpian as a sophism in which by very small detrue to the evidentis broughtfrom the evidently ly respondent -of corn make For example, I ask,Does one a heap? grain ? No. And two grains Do three grains make a heap ? No.

17

See Part

ii, 1st, "

3.

"

80. Satires, vi,

"

Bk.

ii, "

108.

302

OF

FALLACIES.

so

on,

adding each
with

time

until single grain,

at

last the
a

is forced to say that the total reached

does make

respondent heap. I then

single grainmakes allthe difference bewhat is and what is not a heap,which is absurd. tween This reasoning, is called by various as appliedto various objects, Besides Sorites, which Cicero renders by Acervalis, have names. we the Crescens, the Superpositus, the Calvus, This last name and others. is derived from the exemplum of Eublides, wherein the series of queshair from a man's head whether pulling one y tions beginswith asking makes him bald. The sophism is used by Horace21 to ridicule the fashion of valuing rites Soauthors merelyfor their antiquity.The name does not occur in Aristotle. After him, it was used by the ancients, of the above but only as a designation sophism. About the middle of the fifteenth centuryit came also to the chain to be applied between which and this ancient sophism there exists no syllogism,28
a
'

chargehim

that saying

20

whatever.23 analogy
In
"

of the sophism, explanation Krug says,24It attempts,from the of assigning the limit of a relative notion, to show, by impossibility continued interrogation, the impossibility of its determination at all. There are certain notions which are conceived only as relative, as portional, proand whose limits we the cannot, therefore, assign by gradual

addition
if that,
a

or

detraction of
cannot

one

denomination.

But

it does not
it is

follow

incapable of any determination, and therefore null." This explanation is adopted Hamilton.25 in The is this to as view, evidently, by form, sophism, of definition. a fallacy
is commonly attributed to the Ignava ratio (ayoe Ao'yoc) Stoics, by whom it was employed in support of their doctrine of fate.26 It is propounded by Cicero27 in the form of a complex constructive

notion

be determined

in this manner,

"

7. The

dilemma; thus:
If it be fated that you
recover

from
a

your present
or

disease,
will
recover

then,whether
and

you

call in

doctor

not,you
not, you

if it be fated that you you

do not
a

recover,
or or

then,whether
But
.*. It

call in

doctor

wi1:!not

recover. recover.

it is fated either that you


a

recover,

that you do not

is useless to call in

doctor.

30

De

23 88

S1 4. Divinatione, ii, 1, 43 sq. Epist. ii, 21 See Hamilton's Logic, 267-69. Logik," 177. pp. This origin is questioned by Hamilton,Logic, p. 331.

a3

Part
"

"

De

4th,iv," 3. Logic, p. 332. Fato,ch. xii.

304

OF

FALLACIES.

of the universe." a8 the whole future history, agency, lie might predict This doctrine excludes human and, if pushed to its logical results, liberty,

does not The

from fatalism. essentially has continually and Necessity controversy between Liberty tated agi-

differ

the

world, in

one

form

or

another, from
more

most

ancient

times.
more

No

controversy is
morals
not

more

none ancient,

none universal, a

has

keenlyexcited
on

the minds

of men,
not

none

has exerted

greater influence

; it has divided

only their

but nations, and has modified only schools, but their manners, opinions, customs, religion, its influence the

and many
names.

government.

Under

Ignava

ratio
a

has

taken

to various matter, and received forms,been applied

of variety

Among
libero

these

are

De

fato,Metens

De possibi(thereaper),

De libus, ris

De providentia, De divinis decretis, De futuarbitrio, De physicaprcedeterminatione, We here etc. are contingentibus,

concerned The "an

with the argument false. The

only in
Let
us

the form examine

givenabove.
it. What
is meant

is subsuraption fated
not is, to

by

event

be?"

essential idea of "fated"

is "inevitable,"

by any precedentor concurrent of my and personally substantially expressed by regardless So our personal argument reduces to :
"

that

modified

events,
effort."

If

an

event

of is, regardless

my

to be, effort,

my and my

effort is useless ;
if it

of is, regardless

my

not effort,

to

be,

effort is useless. of is, regardless


or

But

every event either to be

my

effort,

not

to

be.

.*. All

my

effort is useless.

Now, consider this subsumption. Who


has it ?
effect it

can an

affirm it ?

What I
can

ground
neither

It may
nor

be true of

some

event,as
we

that eclipse,

affect it ; and

of it

or place from other causes wholly or in part upon my of conditions. of causes, an assemblage only real fate is a concurrence of the necessary conditions, and one Supply a new cause, take away if you please the result will be different, to call it so, a though still, fated or necessary result. Fate changes then her decree. Sending for a doctor introduces a new to send may be the neglecting cause;

either take may say that it must at all. But not events depend many effort as a conditio sine gua non. The

omission

of

condition

necessary to recovery.

88

Mill's

Logic, p. 250.

EXAMPLES.

305

If the necessitarian
causes, I

that objects

my

will is itselfdetermined
am

by prior
a

replythat
recover

then,it may
or

be,I

fated to

send for
a

doctor,

and

so

to

may

be fated not to So my

of this

fated neglect,

to die.

consequence effort is not useless, quent. not inconse-

send,and, as

who Zeno, the Stoic, conceded this. he He


was

that pleaded

adopted the argument, once, it is said, undertook his slave for theft, to flog who aptly fated to steal. And I to flog you,"was the reply.
"

miscellaneous examples are Among the following of good reasoning from true premises, and others from some cases false premises, well as fallaciesproper. is sound, If the reasoning as indicate the logical fect, degive the mood; if not, analyzethe thought, of fallacy. and name the species
8. Praxis.

"

*S legitimate argument may fail to win assent; No fallacy is a legitimate argument ; fail win to assent. c an /. No fallacy 2. Whatever ought to be resisted represses the liberties of mankind ; that do so, there are governments ; Among those things .*.Governments ought to be resisted. 3. Every one desires happiness ; Virtue secures happiness ; desires virtue. .*. Every one is wicked ; 4. Idolatry Wickedness should be punishedby law ; should punishidolatry. .*. Law be proven true by its success, 5. Christianity cannot
1. A

for Mohammedanism Nor


can

has succeeded ;

it be proven by its alleged miracles, has its alleged for Buddhism miracles ;
to be true.

.*.

6.

be proven cannot Christianity We ought to giveone day in


if the fourth commandment

But
/.

we

ought thus
forbidden

to devote

The

fourth
are

commandment
to kill ;

is

duties, religious obligatory upon us ; one day in seven ; obligatory upon us.
seven

to

is

V. We

Inflicting ; capital punishmentis killing forbidden to inflictcapital arc .*. We punishment. it immediately Then 8. A king is a man. minants, follows, by added deterthat a good king is a good man.
20

306
9. No

OF

FALLACIES.

moral

But
.-. Some

10. The
were

is an animal impulse ; principle of action ; animal impulses are some principles of action are not moral principles. principles them if the penal laws against would be aggrieved papists enforced ;

But
11.

these

are

not, and hence they have

no

reason

to

complain.

/.

12.

/.

Nothing is better than wisdom; Dry bread is better than nothing; Dry bread is better than wisdom. is a true poet; No person destitute of imagination who are destitute of imagination Some are good logicians. No true poet is a good logician.
But necessitarians ; virtuous men are practically subvert the distinction all necessitarians speculatively between

1 3. Some

vice and virtue ;


/.

Some

who

deny

the distinction man's

are

virtuous. practically

14.

Interference with another

business is

illegal;

interferes with another's business ; Underselling is illegal. /. Underselling mined 15. Pestilence, being "a visitation of God," its event is not detercauses. by physical of its nature, but only in desires to do wrong if cognizant 1 3. No one of ignorance virtue is knowledge, ; and, therefore, consequence ing and is to be attained by education ; for no one desires evil knowPlato's Gorgias. and to do wrong is evil. From it as such, of character might have been inferred from his 17. His imbecility have this failing. for all weak princes proneness to favorites; illud sentit; 18. Quod tangitur a Socrate Columna a Socrate ; tangitur Ergo,Columna sentit. therefore we 19. The right of the government is unquestionable; ought to obey it. is seen by white that does not decomposelight 20. Every visible object and is therefore white ; light, that does not decompose light. A black-board is a visible object
"

/.

black-board form of

is white.

21.

Any
A

government that excludes the peoplefrom


not
so

political
not

power

to violent revolutions ; is subject

democracy does
to subject

exclude the

and therefore is people,

violent revolutions.

EXAMPLES.

807

22. The

are seven planets Mercuryand Venus

;
are

planets ;

.*.

They

are

seven.

23. No

cat has two

tails;
one

Every cat .*. Every cat


24. To

has

tail more

than

no

cat ;

has three tails.

freedom of speech must allow every man whole, for the good of the state ; for it is the interests of the
a on

alwaysbe, on conducive highly


should

the
to
joy en-

community that each individual of expressing his unlimited, perfectly liberty,


and

sentiments

its affairs.

25. Let and

x=2,

y=r3.
add
=

Take

the

self-evident

ax^ax equations

ay^ay,
ax

them
"

and together

transpose terms; this will


a"

give ax"
2
s
=

ay

ay.

Dividingby

a,

we

obtain

x=ry,

or

3. est syllaba
caseum

26.

Mus Mus

rodit ;

caseum Ergo,syllaba

rodit.
"

27. There who

are

many
not

men

know
"

how

48. /SVweca, J2/?^. that reason exceeding clear and rightly, to make a syllogism Logic is ; therefore,

useless.
28. If all
are

Locke. miracles is to be the popishlegends admitted,

to testimony

to be believed ;

But
.*. No

they are not to testimony


but whites

to be believed ;

miracles is to be admitted.
are

29. None

civilized ;
were

The
.*.

ancient Germans

whites ;

30.

civilized. They were Only give me the luxuries necessaries.

of

and life,

I will

with dispense

the

31. Unless

to Logic professes

be

an

instrument

of

the invention,
;

proach re-

that it discovers

But
.'. The

it does not is reproach


"
"

make

nothingis unfounded this profession ;

unfounded.
is conscience ?"

32.

Teacher,
Smart

What

boy, "Don't know" (=" unprepared"). within you that tells Teacher, Why, conscience is something
"

"

"

you

when
"

you

have done

wrong." they had


to

Boy,
"

Oh,

yes ; I had

it once, and

send

for the

doctor."

308 33. All these exercises will

OF

FALLACIES.

me fatigue

This itself is
.*. It

one

of them

34. A

me. fatigue grainof corn does

will

not

make

heap;
grain ;
may

A
.*. A

hundred hundred
is
no

(99+ 1) is made
is not
uncommon
a

by

one

heap.
occurrence

35.

What

be expected; reasonably
;

To
.-. To

is no uncommon in a lottery occurrence gain a high prize be expected. gain a high prize may reasonably

36.

My
The

hand

touches

.*.

My
No

pen touches hand touches


minimum

the pen ; the paper ;

37. The
\

the paper. visibile is the least

magnitude

that

can

be

seen.

and all partsmust part of it alone is visible,

affect the mind

in

order that it may be visible. Hence every part, though invisible, affect Hamilton's the mind. See must Metaphysics, p. 243.
"

38.
J

events happen every day ; Improbable

event happens every day must be a very probable are .*. Improbableevents very probable.

But

what

39. Omnis

equus

est bestia ;

est sequus ; Justus est bestia. .*. Omnis Burgersdyck. Justus 40. Nuisances are punishable by law ; \ A noisydog is a nuisance ; .*. A by law. noisydog is punishable There's ne'er a villain dwelling in all Denmark, 41. Ham.
"

Omnis

But Hor.

he's

an

arrant
no

knave. from the grave

There To

needs

tell us

this.
"

come lord, ghost, my Hamlet, act i,sc. v.

42.

All that Tinsel

is not gold glitters ; glitters ;


is not

.*. Tinsel

43.

gold. the chancel) to Bishop (inspecting


"
"

Curate

(ritualistically inclined),
cross over

I
"

am

sorry to
"

see

that you

have

placeda

the altar

to the signdeeplycut into the wood-work). (pointing observe we have not placedone there, But please Curate, but, the contrary, have taken one away." Punch. on
"

"

44. Tu

qui es ; Quies est requies ; Ergo,Tu es requies.


es

EXAMPLES.

309

45. What

would
an

be the consequence should insurmountable obstacle ?

an

irresistibleforce

counter en-

Ans."

motion. Compound stationary Socrates brother,


son

40. Is Patrocles your

? of my

Yes,he
Then Not he

is my

the half-brother,

mother,but

not

of my

father. and is not, your brother. is, the


same

by

father, good
was

man

; for Cha3redemus

was

his

and mine father,

Sophroniscus.
other than
a

Chajredemus,then,was Yes, than mine.


But the I
am can same

father ?

he who
as a

is other than
stone

father be

father? or,

are

you

afraid you you

Are
I
am.

will prove me so. other than ? a stone not, indeed,

And, being other than


other than

a are

stone,you
not

are

not

stone ; and

being

gold, you

gold.
a

Very
And It At
seems

true.
so

Chrcredemus,being other than


he is not
a

is not father,

father.

father.
is

if Chaeredemus least, than a father, you, my

then Sophroniscus, other father, being fatherless. Plato's Euihyare Socrates,


a
"

demus, "
47. Who Who
/.

26.

is most
eats eats

hungry eats most; least is most hungry ;


least eats most. be for attended, zealously in it we

Who

48. A

school ought to professional

specially prepare
This school is not

for

our

vocation ; it
we

for in professional,

do not

specially pare pre-

for business ;
/.

This school need


is
no

not

be

49. There

rule without

attended. zealously exceptions ;

This statement
.*. This

is itself a rule ;

statement
are

has

i.e., There
50.

rules

exceptions ; without exceptions.


a

Now

is

no

part of time,for

whole

is Time

composed
is either

of its parts, and

time The

is not

former

composed of nows. no longerexists ;


no

past or future.
fore There-

the latter not

yet exists.

time has

existence.

"

Aristotle's

10. Physica, iv,

310 51. The

OF

FALLACIES.

is a square, say 16 inches ; figure 256 therefore containing 16x16 square it in the inches. Cut dividing piecesby and placethe parts in position so as Jines, base is 26 whose the rectangle to make annexed
=

inches

and

altitude 10
=

inches.
2 60

This

tangle rec-

contains 10x26
.-.256 52. The
man
=

square inches. smaller ;

260.

who I
see

is

walkingaway
"

from

me

does not

grow

But
.-. What

what I
see

grows smaller ; Hume. is not the man. have


no

53. If the blest in heaven

desires,
;
are

they will
and But
or so

be

content perfectly

will

they if their desires


have
no

gratified. fully

either

they will

desires,

gratified. they will have them fully be will content. .-. They perfectly 54. Knowledge is power ;
Power
.-.

is desirable ;
is desirable. sui sunt sui ;

Knowledge
Qui
sunt

55.

domini domini
sunt

sui

juris;

Servi sunt

56.

\forever
Mind,
Ideas cannot I have the But
it

juris. boasted that he had The acute metaphysician Bishop Berkeley and irreligion atheism, by scepticism, put an end to of an Eternal of the existence the followingdemonstration Ergo,Servi
" "

sui

of God

:
a

exist without idea

mind

in which

to-daythat I had unless there this would be impossible existed duringthe interval.
same

they inhere. yesterday.


were a

mind

in which

Hence their

there must

exist

Universal

Mind

in which

all ideas have

permanent residence during the interval of their conscious


own a

presence in our thrusts 57. Whoever surgeon Weib Ein 58. nur
59. A

minds.

an injury. person commits an does this ; therefore he commits injury. besitzen ist seiner Leidenschaft Ziel. fries, " 109. zu

knife into another

"

is justified in usinghis magistrate views,because every man religious own opinions.

officialpower to forward his has a right to inculcate his

312

OF

FALLACIES.

67. There is

no

such

thing as
none

void ; for in
; for
as

void there could be

no

difference of up and there are so is

down

or merely a privation

ferences, nothing there are no difin privation or negation.But a void in a Therefore, negationof matter.

in

void,bodies could
nature to do.
"

not

move

up

and

down,

which

it is in their

Aristotle's attended

Physica.
The
court

68. The

doctor who
to
sue

Pat's wife in her last illness afterwards gave Pat


' "

had

him
"

for the fee.

to permission

him. Docthor, didn't ye agree that, kill or cure,' question ye would charge me "Yes." Well, docthor, only a guinea?" did
ye
cure

her?"
"

"No, she's dead."


"

"Well, docthor,did
did

\
69.

ye killher ?" how can ye ask Themistoclis Verum

if ye No, surely." Thin,docthor,


a

naither,

fee ?"
nee

films

Gra?.cis imperat, nee

70.

imperat matri, qua3 et non-dominatur.31 Grsecis, imperat. Dominatur itaque If the wife you espouse be beautiful, she excites jealousy ; If she be ugly, she disgusts ;
Therefore
it is best not
a

imperando cogitat. imperat Themistocli,qui Graecis


de

to marry."

Bias,quotedby Aldus
ashore ;

Gellius.

71. A

sailor is not

board,nor
sailor.

is

sailor a shore ;
or

But
/.

alwayshe

is either aboard
a

sailor is not desire to

72. A

gainby
All

another's loss is

violation of the tenth it


a implies

mandment. com-

since gaming, therefore,

desire to

the expense of another,breaks this commandment. 73. He that is of God heareth God's words; ye, therefore, hear them John viii, 47. not, because ye are not of God.
at profit
"

74. If Abraham

were

justified by works,then
But
not

he had whereof
to

(before God).
God
2. Romans, iv,

(anyone

can was

have whereof
not

glory fore beglory)


to
"

Abraham (therefore,

justified by works).
the
of testimony falsity

75. What

is of less be
are

cannot

Miracles
;
/.

than occurrence frequent established by testimony ; of less frequent occurrence

than

the

of falsity

mony testi-

Miracles cannot
This famous
"

be established
"

by testimony. Hume.
"

Inexplicable is called Dominam, by Plutarch, but and others, Arrian,Lucian,Gellius,


For
a

31

or

Kvpievuv. It is mentioned not fully explained by any.

discussion

of it, see

Butler's Lectures

on

Ancient

i, Philosophy, p. 414

; and

Hansel in Aldrich, " 9. Appendix,

EXAMPLES.

313

76.

is due Exemptionfrom punishment


you maintain

to the innocent

as therefore,

that the

at prisoner

the bar

ought not

to be punished,

it appears

77. In Platon's Ja.


Also
"

that you maintain his innocence. dieses vor : Hast du einen Hund Euthydcmos kommt
er

? Ja.

Hat

Junge?
Hund
em

Ja.

1st

er

der Vater

der

Jungcn?
cm

ist dcin

dcin Vater,und folglich

Vater

Hund.

Fries,"
men are

109. not

Cited also in De

Soph. xxiv.

78. If

known

to be influenced in the performanceof a likely the oaths commonly duty by takingan oath to perform it, if men administered are superfluous; to be so are likely
one

influenced, every
must

should

be made
one

to take
or

an

oath to behave
other

rightlythroughouthis
be the
are case.

life. But

the oaths

of these

either Therefore,

the
man

commonly
be

minister ad-

or superfluous,

every

should

required
"

to

take

an

oath

to

behave

rightlythroughout his life.

Whately.
79. The of justice and the appointments of being variable, principles it follows that the principles of justice nature invariable, are no Aristotle's Ethics, bk. iii. appointmentof nature.
"

80. If the
at
or

favor

of God

is not

bestowed

at

random,
"

on

no

principle
persons

it must all, with

be bestowed

either with respect to men's But


must

to respect

their conduct. his favor


"

God

is

no

respecterof
with

persons." Therefore
to men's

be bestowed

ence refer-

conduct.

81.

82.

of the son was Jupiter the grandsonof Saturn. Two lines cannot straight lines of straight parallel

Preaching. Apostolical Saturn,therefore the son of Jupiter was


enclose
a

Sumncr^s

space

(Axiom x).
an

But

two

infinite

drawn midway Moreover, if a third space. between them, it will divide this infinite space into two equalparts,each of which is one half of infinity.
83.

lengthdo enclose line parallel to these be

infinite

Opium

84.

"

of their patients advise some poison. But physicians of their paadvise some to take opium. Therefore, physicians tients to take poison. I cannot be conscious of a The knowledge of relatives is one." mental act without being conscious of the object to which that of memory lies in the the object act relates. But confessedly is

past,and has ceased


of
not
"

to

be.

Therefore, in memory
am

am

scious con-

an

in object

the

and past,

conscious

of what

does

exist.

See Hamilton's

Metaphysics, p. 146

sq.

314

OF

FALLACIES.

85.

Animal
use

food
it ; and do

do not with,for the vegetarians dispensed food may be dispensed with,for the Esquimaux vegetable may be
use

not

it.
or

But
/.

all food is either animal

vegetable.
who
are

with. All food may be dispensed soldiers should be brought into the field but those 86. No
well

None

to perform their part; qualified well qualified to perform their part; but veterans are

only should be brought into the field. of objects For if attention at once. attend to a plurality 87. We can be nothingbut the concentration of consciousness on a smaller of simulthan constitute its widest compass of objects taneous number N it is evident that, unless this widest compass knowledge, do attend of consciousness be limited to only two we objects,
.-. Veterans

when

we

converge

consciousness

88.

89.

once. complement Hamil tori's Metaphysics, p. 165. In what and how many ate phrasesdoes Hamilton, in his immediis cognizant of extension, that sight demonstration assume to be proved? Ibid., the point p. 385. in an Anabaptist to prove against It would be bad reasoning the in believing that infants are caCatholics that they are wrong pable since nothing is said of it in the Scripture of baptism, ; that we ought to believe only because this proof would assume which is denied by the Catholics. what is in the Scripture,
" .

that total

any smaller number of objects which it can embrace at


on

than

"

"

"

"

Arnauld, p.
90.

251.

How

can

we

conceive
we

God, since

we

can

attribute

no

virtue to

him ?

prudence? But since prudence what need consists in the choice between good and evil, God have of this choice,not being capableof any evil? can ? and reason But intelligence Shall we say that he has intelligence
For shall and
from
to
reason serve

say that he has

to discover to

us

that which
can

is unknown

that which Neither

is known
can

now

there
in

be

nothingunknown
this relates
he has
no

God.

be justice
men

God, because

only to

the intercourse of

nor

temperance, since

since he is susceptible of strength, and is not exposedto any danger. How neither pain nor labor, that be a god which has neither intelligence nor can therefore, desires to

moderate;

nor

virtue ? 32
83

De quoted by Cicero, Cotta,

Natnra

Deorum,

bk. iii.

EXAMPLES.

315

91. That

which
is

has the

use

of

reason

is better than that which

has not;

There
.*. The

better nothing'
use

than the universe ;


of
reason.
"

universe has the does


a

The

Stoics.
of
a

92.

Why
full

when ball,

not sail,

dropped from fall exactly at the foot


are

the mast-head

shipin
to

of the mast, but

nearer

the stern
93.

of the vessel ?

Gold
and

and silver

wealth ; therefore the diminution

of the

gold

silver of the country


of the
not
no a

by

is exportation

diminution

of the

wealth
94. If
man

country.
necessary

be
is

agent determined
for rewards
men were

there would

foundation

and

and pain, by pleasure punishments. These

be

useless unless

determined indifferent
cause

and by pleasure and pain, to pleasure pain could


to observe

and were necessary 'agents, free and were pain; because if men be
no

motive

to

them

the law.
is
a

95. It is certain

that

drunkenness

vice

odious
are

to God

and

man.

It is

certain equally

that alcoholic drinks

destructive to the

and physical of him who moral,intellectual, energies habitually makes of them. I claim, use n ot it is the that therefore, only

96.

97.

from their use, but,as a totally citizen and philanthropist, to exert all his influence to obtain and enforce a law prohibiting the sale of any kind of intoxicatin beverages. the of Biunde,as stated (Analyze argument of Krug and the reply in Hamilton's Metaphysics, pp. 565-66.) Minus multiplied minus cannot plied by giveminus ; for minus multiby plusgivesminus, and minus multiplied not by minus cangivethe same productas minus multipled by plus. Euler's Algebra. See Mill's Logic,p. 575.
man

duty good

of every

to

abstain

"

98.

"

Either God

wills to

remove
nor

evils and cannot


can

or

he

can can.

and will If he If

not;
he
to

or

he neither will

or

he both will and


is not
true

will and
can

cannot, then he is weak, which


and will not, then he
of God. and is

of God.

which malicious, neither will


nor

also is
can,

foreign
he is If he
of

the nature malicious


can

If he

then

both both

weak, and therefore


are or evils,

cannot

be God.

and

which will,

alone is consistent

with

the nature
not
remove

God, then
them ?"
33

whence

why

does

he

83

Be Epicurus, quoted by Lactantius,

Ira

Dei,xiii.

316

OF

FALLACIES.

99.

The

doctrine

of
of

an

omnipresent
is
as

divine
to

power the
say

and

agency
as

in

the

operations
sound

Nature
; for

contrary

Scriptures
expressly,
"

it is to earth

philosophy
forth

the

Scriptures

The

bringeth
100.

fruit

of herself ""Mark,
the ancient
means

iv, 28.
historical
of records and will

If

reconciliation culture
to
manner

between
be

modern be

sought

by

interpretation, it
did
not

attempted
in

prove

either

that

the
is ;
or

divine
to

manifest historical
rences occur-

itself

the
of

related, which

deny
the
away

the

validity

the
not

ancient

Scriptures
is to

that

actual

were

divine, which
books.34
tell its
own

explain

the

absolute

contents

of

these Bible truth


a

101.

Do

but

let

the the

story;
which

grant,

for

the

sake

of

argument,
and it

of the consistent

dogmas
whole. the
a

it asserts
a man

throughout, begins,
as

becomes

When
of

Strauss wonder

does, by
he finds

assuming
its

falsity

its

conclusions,
chaos
of

no

premises

fragmentary

dictions.35 contra-

34

D.

F.

Strauss,
Alton

in

Leben

Jesu,

Int.

"

1.

35

Dean

Locke,

Works,

vol.

i,ch.

xxxviii.

FINIS.

VALUABLE

AND

INTERESTING
FOR

WORKS

PUBLIC

AND
BY

PRIYATE
HARPER "

LIBRARIES,
NEW YORK.

PUBLISHED

BROTHERS,

For

" BROTHERS, of Books suitable for Libraries published by U AKPER be had CATALOGUE, which gratuitously on application may the publishers personally, or by letter enclosing Ten Cent* in postage stamps.
a

full List
see

HARPER'S

to

HARPER

"

BROTHERS

will

send

their

publications by mail, postage prepaid,

on

receipt of the price.

MACAULAY'S of James from New

ENGLAND.
II.

The THOMAS

History
in vols.,

of

England

from New

the

sion Acces-

By

BABINGTON
5
a

MACAULAY.

Edition, Paper
50.

ElectrotypePlates. Edges, only in


and Gilt Sold Sets.

Box, 8vo, Cloth,with

Labels, Uncut $21


25.

Tops, $10 00; Sheep, $12

50; Half Calf,

Cheap Edition,5 vols., 12mo, Cloth,$2


WORKS. New The Miscellaneous
5

MACAULAY'S of Lord

MISCELLANEOUS

Works
a

Macanlay.
50 ; Half

From

Plates. Electrotype

in vols.,

Box,
00 ;

Svo, Cloth, with

Paper Labels, Uncut Calf,$21


25.

Edges,
Sold

and

Gilt Sets.

Tops, $10

Sheep, $12
HUME'S Caesar and

only in
from

ENGLAND.
to

History of England,
of James New

the

Invasion HUME.
G

of Julius New
a

the Abdication

II.,1688.

By

DAVID

Elegant Library Edition,from


00
a

ElectrotypePlates. Edges, and


Gilt

in vols.,

Box, Svo, Cloth,with Paper Labels,Uncut Sheep, $15


C
;

Tops, $12

00 ;

Half

Calf,$25

50.

Sold 00.

only in

Sets.

Popular Edition,

in vols.,

Box, 12mo, Cloth,$3


The

GIBBON'S

ROME.

Historyof
GIBBON. SMITH.

the Decline Notes

and Dean

Fall of the Roman

Empire.
ZOT, and
G

By
Dr.

EDWARD WILLIAM

With New

by

MILMAN,
New

M.

Gui-

Edition, from
Calf, $25
50.

Electrotype
and Gilt

Plates.

Svo, Cloth, with Paper Labels, Uncut vols.,


Half

Edges,
Sold

Tops, $12 00; Sheep, $15 00;


in Popular Edition,G vols., IIILDRETH'S SERIES
: a

only

in Sets.

Box, 12mo, Cloth,$3

00.

UNITED From the under

STATES.

History of
the Continent Constitution.
to

the United
to

States.

FIRST the

Discovery of
the Federal Federal

the

Organizationof
SERIES: of the

Government the

SECOND the End

From

Adoption

of the

Constitution

Sixteenth
a

Congress.

By

RICHARD

HILDRETH.

in Popular Edition, G vols.,

Box, Svo, Cloth,with Paper

Labels,Uncut
50.

Edges,

and

Gilt

Tops, $12

00 ;

Sheep,$15 00;

Half

Calf,$25

Sold

only in Sets.

Valuable

Works

for

Public

and

Private

Libraries.

MOTLEY'S

DUTCH JOHN

REPUBLIC. LOTH
HOP

The

Rise

of the Dutch D.C.L.


a

Republic.
a

History. By
of William with Half of

MOTLEY,
and

LL.D.,
Gilt

With

Portrait

Orange.
75.

Cheap only

in Edition,3 vols.,

Box, 8vo, Cloth,

Paper Labels,Uncut
Calf, $12
Sold 50.

Edges,

Tops, $6 00; Sheep, $7 50;

in Sets.

3 vols., OriginalLibrary Edition,

8vo, Cloth,$10
MOTLEY'S From
"

UNITED the Death With of the

NETHERLANDS. of William full View and

History of
of the

the United

lands: NetherTruce

the Silent to the Twelve

Years'

1584-1G09.

English-Dutch Struggle against


of the

Spain, and
JOHN in vols.,

Origin

Destruction D.C.L.

Spanish

Armada.

By
Gilt Sets.

LOTIIROP
a

MOTLEY,

LL.D.,

Portraits.

Cheap Edition,4 Edges,


Sold and in

Box, 8vo, Cloth, with

Paper Labels, Uncut


Half

Tops, $8 00;

Sheep, $10 00;

Calf,$17

00.
00.

only

OriginalLibrary Edition,4 vols., 8vo,Cloth, $14


MOTLEY'S
JOHN OF BARNEVELD. of Holland. the With Years' The
a

Life and of the

Death

of John

of

Barneveld, Advocate
and Movements
of

View War."

Primary
JOHN

Causes

"Thirty

By
Gilt

LOTHROP
a

MOTLEY, $5 00;
2

LLD.,
Half

D.C.L.

Illustrated.

in Cheap Edition,2 vols.,

Box,

Uncut Svo, Cloth,with Paper Labels,

Edges, and
Sets.

Tops, $4

00 ;

Sheep,

Calf,$8

50.
00.

Sold

only in

OriginalLibrary Edition,

vols., 8vo,Cloth,$7
WORKS.

GOLDSMITH'S
PETER

The F.S.A.

Works From

of Oliver

Goldsmith.

Edited
4

by

CUNNINGHAM,
Half

New and

Electrotype Plates.
Gilt the New

vols.,

8vo, Cloth, Paper Labels, Uncut

Edges,
Uniform

Tops, $8 00; Sheep, Library


Editions

$10 00;
of

Calf,$17

00.

with

Macaulay, Hume,
JOHN DE

Gibbon, Motley, and


WITT. of

Hildreth. of the Administration JAMES GEDDES. of John Vol.

GEDDES'S
De I."

History
Holland.

Witt, Grand
1623-1654.

Pensionary
With
a

By

Portrait.

8vo, Cloth,$2 50.


OF
to

MULLER'S

POLITICAL With

HISTORY Reference

RECENT

TIMES WILLIAM
1876
to

(1816MULLER.

1875).
the Rev. SYMONDS'S

Special
an

Germany.
the Period

By

with Translated, JOHN

Appendix covering
Ph.D.

from
00.

1881, by

P. PETERS,

12mo,
STUDIES SYMONDS.

Cloth,$3
IN
2

SKETCHES

AND ADDINGTON

SOUTHERN

ROPE. EU-

By $4
00.

JOHN

Post vols.,

Svo, Cloth,

SYMONDS'S

GREEK

POETS.
2

Studies

of the Greek

Poets.
50.

By

JOHN

ADDINGTON

SYMONDS.

vols., Square 16mo, Cloth, $3

Valuable

Works

for

Public

and

Private

Libraries.

DU

CHAILLU'S
in of the

EQUATORIAL
with

AFKICA. Accounts of the

Explorations and
of the Manners and

tures AdvenCustoms

Equatorial Africa;
and other
00
;

People, and

of the Chase

Gorilla, Leopard, Elephant, Hippopotamus,


P. B. Du CHAILLU. Illustrated.

Animals. Half

By
25.

8vo, Cloth,$5
DU and CHAILLU'S Further

Calf,$7

ASHANGO Penetration into

LAND.

Journey By
25.

to

Ashango
B. Du

Land,

EquatorialAfrica.
00 ; Half

P.

CHAILLU.

Illustrated. DEXTER'S Last Three


to

8vo, Cloth,.$5
CONGREGATIONALISM. Hundred

Calf,$7
The

Congregationalism of
:

the

Years,

as

Seen

in its Literature
or

with

erence SpecialRefWith
a

certain

Recondite, Neglected, By
THE Sources
II. M.

Disputed Passages.

Bibliographical Appendix.
STANLEY'S Dark THROUGH

DEXTER.

Large Svo, Cloth,$6

00.

DARK
of the

CONTINENT.

Through
the Great
to

the of

Continent;

or, The

Nile,Around
STANLEY.

Lakes

EquatorialAfrica, and
149

Down 10

the Livingstone River

the Atlantic
2

Ocean.

Illustrations and

Maps.
00.

By

II. M.

vols., Svo, Cloth,

$10

00; Half Morocco, $15


FROM
the

DABTLETT'S

EGYPT South
to

TO

PALESTINE.
Observations

Through Sinai, the


of
a

Wilderness, and
with
LETT,

Country.
the

Journey By

made

Special Reference
D.D.

History of

the Israelites.
50.

S. C. BART-

Maps
LIFE

and

Illustrations.

Svo, Cloth,$3
The With

FORSTER'S
Swift Uncut GREEN'S JOHN

OF

DEAN JOHN

SWIFT. FORSTER.
50.

Early

Life

of Jonathan

(1667-1711). By
Edges
and Gilt

Portrait.

Svo, Cloth,

Tops, $2
PEOPLE.

ENGLISH RICHARD

History
With
00.

of the 4

English People. By
00 ;

GREEN,
Half

M. A.

Maps.

Svo, Cloth,$10 vols.,

Sheep, $1200;
GREEN'S
MAKING

Calf,$19
OF

ENGLAND.

The

Making
50.

of

England.

By

J. R. GREEN.

With

Maps.
OF With

Svo, Cloth, $2
ENGLAND.

GREEN'S

CONQUEST
GREEN. NORTH of

The

Conquest
50.

of

England.

By

J. R.

Maps.

Svo, Cloth, $2
OF

SHORT'S
Americans

AMERICANS Their

ANTIQUITY.

The

North

Antiquity. By
JOHN

and Type Origin,Migrations, Illustrated.

of Civilization
00.

Considered.

T. SHORT.

Svo, Cloth,$3

SQUIER'S
Land

PERU. Incas.
to

Peru

Incidents GEORGE

of Travel

and

Exploration
00.

in

the S.

of the

By

E.

SQUIER, M.A., F.S.A.,


Svo, Cloth,$5

late U.

Commissioner

Peru.

With

Illustrations.

Вам также может понравиться